You are on page 1of 118

Upload by CHVF Team wWw.chuyenhungvuong.

net
Lêi nãi ®Çu

Häc to¸n vµ lµm to¸n lµ hai vÊn ®Ò hoµn toµn kh¸c nhau. §ã lµ hai mÆt
kh«ng thÓ t¸ch rêi cña to¸n häc, trong ®ã häc to¸n lµ c¬ b¶n vµ lµm to¸n lµ mét vÊn
®Ò ®Æc biÖt quan träng. Häc to¸n sÏ gióp cho chóng ta n¾m ®−îc nh÷ng ®iÒu c¬ b¶n
nhÊt vµ nh÷ng vËn dông ban ®Çu cña lý thuyÕt c¬ së. Lµm to¸n nghÜa lµ ®µo s©u
suy nghÜ, ph¸t triÓn mét bµi to¸n ë møc ®é t− duy cao h¬n, nhê ®ã sÏ gióp chóng ta
cã mét c¸i nh×n toµn diÖn vµ s©u s¾c h¬n vÒ mét vÊn ®Ò. Vµ hÖ qu¶ tÊt yÕu cña viÖc
®µo s©u suy nghÜ ®ã lµ nh÷ng s¸ng t¹o to¸n häc nh− nh÷ng kh¸i niÖm, nh÷ng bµi
to¸n, nh÷ng øng dông hay lý thuyÕt míi. §ã míi lµ môc ®Ých s©u s¾c nhÊt cña to¸n
häc. Víi tinh thÇn ®ã, nhãm nh÷ng cùu häc sinh tr−êng THPT Chuyªn Hoµng V¨n
Thô – Hßa B×nh ®H cïng nhau x©y dùng nªn tê TËp san To¸n häc 2007 nh»m môc
®Ých ®éng viªn phong trµo häc to¸n ë tr−êng Chuyªn Hoµng V¨n Thô nãi riªng vµ
c¸c b¹n häc sinh cña TØnh Hßa B×nh nãi chung. Tê b¸o ®−îc hoµn thµnh víi sù t©m
huyÕt, lßng yªu to¸n vµ h−íng tíi m¸i tr−êng cò cña nh÷ng häc sinh ®H tõng häc
tËp d−íi m¸i tr−êng Hoµng th©n yªu. §ã còng lµ mãn quµ mµ nh÷ng cùu häc sinh
muèn göi tÆng ®Õn c¸c thÇy c« gi¸o víi lßng biÕt ¬n s©u s¾c!

§©y lµ lÇn thø hai TËp san ra m¾t, nh−ng víi quy m« vµ néi dung phong phó
h¬n rÊt nhiÒu so víi lÇn ra m¾t tr−íc ®ã. Néi dung cña TËp san lµ nh÷ng bµi viÕt víi
néi dung t×m tßi, s¸ng t¹o, nh÷ng kinh nghiÖm, øng dông vµ nh÷ng ph−¬ng ph¸p
häc to¸n. Hy väng r»ng dï víi mét l−îng kiÕn thøc kh«ng nhiÒu, nh−ng TËp san sÏ
mang l¹i cho c¸c b¹n nhiÒu ®iÒu bæ Ých vµ lý thó.

V× kh¶ n¨ng cña Ban biªn tËp cßn nhiÒu h¹n chÕ vµ thêi gian cã h¹n, nªn
trong qu¸ tr×nh biªn tËp, ch¾c ch¾n kh«ng tr¸nh khái nh÷ng thiÕu sãt vµ nhiÒu ®iÓm
kh«ng ®−îc nh− mong muèn, rÊt mong nhËn ®−îc sù th«ng c¶m vµ nh÷ng ®ãng
gãp x©y dùng cña c¸c b¹n ®éc gi¶. Vµ chóng t«i còng hy väng r»ng, víi truyÒn
thèng hµo hïng cña tr−êng THPT Chuyªn Hoµng V¨n Thô, c¸c b¹n thÕ hÖ sau sÏ
tiÕp tôc ph¸t huy vµ kh«ng ngõng n©ng cao vÞ thÕ cña tuæi trÎ Hßa B×nh trong m¾t
b¹n bÌ ë mäi miÒn ®Êt n−íc. Hy väng r»ng TËp san sÏ ®−îc c¸c b¹n khãa sau duy
tr× vµ hoµn thiÖn h¬n n÷a vÒ mäi mÆt. Ban biªn tËp xin ®−îc c¶m ¬n tÊt c¶ c¸c b¹n
®H tham gia vµ ñng hé nhiÖt t×nh ®Ó tê TËp san ®−îc ra m¾t ®óng nh− dù kiÕn. Xin
tr©n träng giíi thiÖu cïng b¹n ®äc!

Chóc c¸c b¹n thµnh c«ng trong häc tËp vµ thµnh ®¹t trong cuéc sèng!

Hßa B×nh th¸ng 1 n¨m 2007


Ban biªn tËp

Upload by wWw.chuyenhungvuong.net
TËp san To¸n häc 2007
Héi ®ång biªn tËp
Tr−ëng ban biªn tËp: NguyÔn L©m TuyÒn Phã ban biªn tËp: Bïi Lª Vò
Céng t¸c viªn: NguyÔn Th¸i Ngäc, L−u Nh− Hßa, trÇn quang thä
ph¹m th¸I s¬n, nguyÔn duy hoµng
Môc lôc

PhÇn 1. S¸ng t¹o to¸n häc


Giíi thiÖu ph−¬ng ph¸p tÝnh mét sè líp tÝch ph©n d¹ng hµm l−îng gi¸c – Cao Trung Chinh………... 1
Tæng qu¸t hãa bµi to¸n - §ç ThÞ Thu Hµ……………………………………………………………... 3
Xung quanh bµi to¸n bÊt ®¼ng thøc thi To¸n Quèc tÕ 2005 – NguyÔn Anh TuÊn……………………. 5
Thö ®i t×m bÊt ®¼ng thøc trong tam gi¸c – D−¬ng ThÞ H−¬ng – NguyÔn Nh− Th¾ng……………….. 9
Mét sù t×nh cê – NguyÔn L©m TuyÒn………………………………………………………………… 12
Sö dông tÝnh chÊt hµm ®¬n ¸nh ®Ó gi¶i bµi to¸n ph−¬ng tr×nh hµm – NguyÔn Th¸i Ngäc……………. 15
Lêi gi¶i c¸c bµi thi To¸n Quèc tÕ 2003 – Hµ H÷u Cao Tr×nh ………………………………………... 17
Sè phøc víi h×nh häc ph¼ng – Vò H÷u Ph−¬ng……………………………………………………….. 20
Ph−¬ng tr×nh hµm vµ sù trï mËt – Bïi Lª Vò ………………………………………………………… 23
DHy sè vµ sù trï mËt trªn R+ – Hå Sü Tïng L©m……………………………………………………. 26
Mét sè bµi to¸n sè häc vÒ dHy tæng c¸c lòy thõa – TrÇn Quèc Hoµn…………………………………. 28
C©n b»ng hÖ sè trong bÊt ®¼ng thøc C«-si – NguyÔn L©m TuyÒn…………………………………….. 30
Ph−¬ng ph¸p sö dông ®Þnh nghÜa ®Ó tÝnh giíi h¹n – Lª B¶o Kh¸nh………………………………….. 35
§iÓm Lemoine trong tam gi¸c – Lª V¨n §Ýnh………………………………………………………... 38
C©u chuyÖn ®−êng trßn vµ elipse – L−u Nh− Hßa…………………………………………………….. 40
Mét sè ph−¬ng ph¸p x¸c ®Þnh giíi h¹n cña dHy sè – NguyÔn L©m TuyÒn…………………………… 41
Mét líp c¸c bµi to¸n bÊt ®¼ng thøc – Nguyªn Minh Phóc……………………………………………. 46
Mét sè kh¸i niÖm vÒ gãc ®Þnh h−íng – TrÇn Quang Thä……………………………………………... 48
Tiªu chuÈn héi tô tæng qu¸t – Bïi Lª Vò – NguyÔn Th¸i Ngäc……………………………………… 52
øng dông ®Þnh lý Stolz trong t×m giíi h¹n cña dHy sè – Ng« NhÊt S¬n………………………………. 55
øng dông cña mét bµi to¸n tæng qu¸t – NguyÔn Hµ ThuËt…………………………………………… 57
TËp d−ît s¸ng t¹o – §Æng Phïng H−ng………………………………………………………………. 59
VËn dông ®Þnh lý s¸ch gi¸o khoa linh ho¹t – TrÞnh Anh TuÊn……………………………………….. 61
Më réng kh¸i niÖm t©m tØ cù cho tø diÖn – Hoµng An Giang………………………………………… 64
Ph−¬ng ph¸p logic mÖnh ®Ò – Ph¹m Phóc L©n……………………………………………………….. 66
PhÐp chiÕu vµ øng dông cña phÐp chiÕu – NguyÔn L©m TuyÒn………………………………………. 69
Mét sè bµi to¸n bÊt ®¼ng thøc chän läc – L−u Nh− Hßa……………………………………………… 73
Sö dông ®¼ng thøc ®Ó chøng minh bÊt ®¼ng thøc – Vò ViÖt Dòng…………………………………… 75
TiÕp cËn to¸n b»ng vËt lý – NguyÔn L©m TuyÒn……………………………………………………... 78
BÊt ®¼ng thøc Schur vµ øng dông – Tr−¬ng Quèc H−ng……………………………………………… 81
Mét sè bµi tËp vÒ to¸n rêi r¹c – Bïi M¹nh Qu©n……………………………………………………... 83
Sö dông hµng ®iÓm ®iÒu hßa ®Ó gi¶i bµi to¸n cùc trÞ – TrÇn ThÞ Linh Ph−¬ng……………………….. 85

PhÇn II. LÞch sö vµ øng dông To¸n häc


Sù ph¸t triÓn cña sè häc – Phïng Ngäc Th¾ng………………………………………………………... 87
To¸n häc vµ tù ®éng hãa – NguyÔn L©m TuyÒn……………………………………………………… 90
Dïng ®a thøc ®Ó ph¸t hiÖn lçi ®−êng truyÒn – NguyÔn L©m TuyÒn………………………………….. 93
CÊu tróc tù nhiªn – NguyÔn Th¸i Ngäc……………………………………………………………….. 95
PhÇn III. To¸n häc vµ ngo¹i ng÷
Häc to¸n vµ ngo¹i ng÷ – Ng« Thµnh Long………………………………………………………...... 97
Ph−¬ng tÝch cña ®iÓm víi ®−êng trßn – L−u Nh− Hßa………………………………………………... 98
PhÐp nghÞch ®¶o – L−u Nh− Hßa……………………………………………………………………... 99
PhÇn IV. Nh÷ng bµi to¸n hay vµ c¸c bµi to¸n tù s¸ng t¹o
C¸c bµi to¸n tù s¸ng t¹o – NguyÔn L©m TuyÒn………………………………………………………. 103
Nh÷ng bµi to¸n hay – NhiÒu t¸c gi¶…………………………………………………………………... 109
PhÇn i

S¸ng t¹o To¸n häc


PhÇn I - S¸ng t¹o to¸n häc.

Giôùi Thieäu Phöông Phaùp


Tính moät soá lôùp tích phaân daïng haøm löôïng giaùc
ThÇy cao trung chinh
GV. THPT Chuyªn Hoµng V¨n Thô, Hoµ B×nh

§Ó gióp häc sinh cã thªm nh÷ng sin 3 xdx


kiÕn thøc mang tÝnh hÖ thèng, t«i xin
2.VÝ dô 2. TÝnh I = ∫ 3
cos 2 x
.
giíi thiÖu mét sè líp tÝch ph©n d¹ng hµm
Lêi gi¶i. §Æt t = cosx ⇒ dt = − sin xdx .
sè l−îng gi¸c th−êng gÆp trong c¸c k× thi
Ta cã
tèt nghiÖp còng nh− thi ®¹i häc. Hi väng
qua bµi viÕt nµy, c¸c em cã thÓ rót ra 1− t2  4 −2 
I = -∫ dt = ∫  t 3 − t 3  dt =
nhiÒu ®iÒu bæ Ých cho b¶n th©n. 3 2
t  
7 1
3 3 3
I. D¹ng ∫ f (sin x, cos x)dx . t − 3t 3 + c = 3 cos 7 x − 3 3 cos x + c
7 7
1. NÕu f(sinx, cosx) lµ hµm h÷u tØ th× ®Æt
x C¸c b¹n hHy tù gi¶i hai vÝ dô sau:
t = tg .
2 cos 3 x + cos 5 x
2. Mét sè hiÖn t−îng c¸ biÖt. 3. VÝ dô3. TÝnh I = ∫ dx .
sin 2 x + sin 4 x
- NÕu f(-sinx, cosx) = - f(sinx, cosx) th× 4.VÝ dô 4.
®Æt x = cost. dx
- NÕu f(sinx, - cosx) = - f(sinx, cosx) th× TÝnh I = ∫ 2
sin x + 2 sin x cos x − cos 2 x
®Æt x = sint.
- NÕu f(-sinx, - cosx) = f(sinx, cosx) th×
®Æt x = tgt. Chó ý: ë ®©y mäi nguyªn hµm ®−îc hiÓu
Qua c¸c c¸ch ®æi biÕn nh− trªn, ta cã lµ trªn mçi kho¶ng cña tËp x¸c ®Þnh.
thÓ tÝnh c¸c tÝch ph©n mét c¸ch ®¬n gi¶n
vµ nhanh chãng. Sau ®©y lµ mét sè vÝ dô II. D¹ng ∫ sin m x cos n xdx .
cô thÓ. - NÕu m hoÆc n lµ sè nguyªn d−¬ng lÎ
dx th× t−¬ng øng ta ®Æt t = cosx hoÆc t =
1. VÝ dô 1. TÝnh I = ∫ .
sin x sinx
- NÕu m vµ n ®Òu lµ sè nguyªn d−¬ng
Lêi gi¶i. ch½n th× chóng ta dÔ dµng sö dông c«ng
x dx thøc h¹ bËc vµ gãc nh©n ®«i ®Ó gi¶i
§Æt t = tg ⇒ dt = ,
2 2 x quyÕt bµi to¸n.
2 cos - NÕu (m+n) lµ sè nguyªn ch½n th× ®Æt
2
2t t = tgx hoÆc t = cotgx.
sin x = . VËy
1+ t 2
Tïy theo tõng ®iÒu kiÖn cña bµi to¸n
dx dt x mµ ta cã thÓ chän lùa c¸ch ®Æt cho phï
I =∫ = ∫ = ln t + c = ln tg + c
sin x t 2 hîp. Sau ®©y lµ mét sè vÝ dô:

TAÄP SAN TOAÙN HOÏC - 2007 1


TRÖÔØNG THPT CHUYEÂN HOAØNG VAÊN THUÏ - HOØA BÌNH
PhÇn I - S¸ng t¹o to¸n häc.

dx
4.VÝ dô 4. TÝnh I = ∫
sin 11 x cos x3
.
1.VÝ dô1. TÝnh ∫ sin 4 x cos 5 xdx . 11 1
Lêi gi¶i. DÔ thÊy m = − , n = − vµ
3 3
Lêi gi¶i. §Æt t = sinx, ta cã dt = cosxdx m + n = - 4 nªn ta ®Æt t = tgx , ta cã
VËy ∫ sin 4 x cos 5 xdx = ngay dt = (1+tg2x)dx . VËy:
( ) 2
(
= ∫ t 4 1 − t 2 dt = ∫ t 4 − 2t 6 = t 8 dt ) I= ∫
dx
=∫
dx
11 12
1 5 2 7 1 9
3
tg x cos x cos x3 tg 11 x
4

= t − t + t +c
5 7 9
= ∫
(1 + t )2 2

dt = ∫ (1 + t 2 ) .t

11
3
dt
1 2 1
= sin 5 x − sin 7 x + sin 9 x + c .
11

(1 + t ) .t

2 3
5 7 9
 −311 − 35 
3
sin xdx = ∫  t + t  dt
2.VÝ dô 2. TÝnh ∫ cos x 3
cos x
.  
3 − 3 −2
8
= − t 3 − t 3 +c
Lêi gi¶i. 8 2
8
sin 3 xdx −
4 3 − 3 −2
Ta cã ∫ = ∫ sin 3 x cos 3
xdx = − tg 3 x − tg 3 x + c
8 2
cos x3 cos x
§Ó kÕt thóc bµi viÕt, t«i xin ®−a ra
4 mét sè bµi tËp ®Ó c¸c em luyÖn tËp thªm
§Æt t = cosx (do m = 3, n = − ), ta cã
3 vÒ ph−¬ng ph¸p trªn.
dt = - sinxdx. VËy
4 4
III. Bµi tËp.
xdx = - ∫ (1 − t ).t dt
− −
∫ sin x cos
3 3 2 3
TÝnh c¸c tÝch ph©n sau:
 2 −4  sin 2 x cos x
= ∫  t 3 − t 3  dt a) I1 = ∫ dx
  sin x + cos x
5 1 cos 3 xdx
3 −
= t 3 − 3t 3 + c b) I2 = ∫
5 sin 2 x + sin x
sin 2 xdx
c) I3 = ∫
5 1
3 −
= cos x − 3cos 3 x + c
3
cos x − sin 2 x − 1
3
5
sin 3 xdx
d) I4 = ∫
3. VÝ dô3. TÝnh I = ∫ sin 2 x cos 4 xdx .
3
cos 2 x
cos 4 xdx
e) I5 = ∫ ./.
Lêi gi¶i. Ta sö dông c«ng thøc h¹ bËc: sin 2 x
1 1 + cos 2 x =============================
sinxcosx= sin 2 x , cos 2 x =
2 2
vµ dÕ dµng gi¶i quyÕt bµi to¸n. Gi¸o dôc kh«ng ph¶i lµ sù chuÈn bÞ cho
cuéc sèng; ChÝnh gi¸o dôc lµ cuéc sèng.
Jonh Dewey

TAÄP SAN TOAÙN HOÏC - 2007 2


TRÖÔØNG THPT CHUYEÂN HOAØNG VAÊN THUÏ - HOØA BÌNH
PhÇn I - S¸ng t¹o to¸n häc.

toång quaùt hoùa


Baøi Toaùn

§ç ThÞ Thu Hµ
Chuyªn To¸n K97 - 00
Sv. Khoa KÕ to¸n – KiÓm to¸n
§¹i häc kinh tÕ Quèc d©n - Hµ Néi

Chµo c¸c b¹n - Nh÷ng ng−êi ®H, ®ang vµ gi¶i bµi to¸n. Nh−ng t«i nghÜ lµ rÊt tù nhiªn
sÏ tiÕp tôc g¾n bã víi To¸n häc trªn con vµ “dÔ lµm”. Chóng ta hHy xem xÐt mét sè vÝ
®−êng ®i t×m vÎ ®Ñp léng lÉy cña nã! Ch¾c dô:
h¼n tÊt c¶ chóng ta ®Òu ®H tõng kinh ng¹c vµ 1. VÝ dô 1. T×m hµm f: [0;1] → R, liªn tôc
th¸n phôc tr−íc c¸c ph¸t minh cña nh÷ng
trong [0;1] tháa mLn: f(x) ≥ 2x. f(x 2 )
nhµ to¸n häc vµ còng ®H tõng hái, t¹i sao
nh÷ng kÕt qu¶ ®Ñp nh− vËy l¹i kh«ng ph¶i do
chÝnh chóng ta s¸ng t¹o ra. Trong khi ®ã, T«i xin ®−a ra 2 c¸ch gi¶i kh¸c nhau.
trªn thùc tÕ, nÕu chóng ta ®−îc ®èi mÆt víi a) Lêi gi¶i 1. Tõ f(x) ≥ 2x. f(x2) suy ra
nhiÒu trong sè c¸c ph¸t minh ®ã th× chóng ta x.f(x) ≥ 2x2.f(x2) , ∀ x ∈ (0;1]
cã thÓ t×m ra lêi gi¶i dÔ dµng trong tÇm kiÕn Thay x = 0 ⇒ f(0) ≥ 0
thøc cña m×nh. Hay ®¬n gi¶n h¬n, nh÷ng b¹n §Æt g(x) = x.f(x), ∀ x ∈ (0;1]
yªu to¸n ®H tõng tham dù gi¶i bµi trªn t¹p ⇒ g(x) ≥ 2 g(x2)
chÝ To¸n häc vµ Tuæi trÎ, ®H cã bao giê c¸c 1
1

b¹n muèn trë thµnh ng−êi ra ®Ò to¸n hay ⇒ g ( x 2 ) ≥ g(x).


2
ch−a? Hay b¹n cho r»ng ®ã lµ c«ng viÖc cña
B»ng quy n¹p, ta chøng minh ®−îc:
thÇy c«, cña nh÷ng ng−êi ®ang nghiªn cøu 1
to¸n häc? C©u tr¶ lêi lµ kh«ng ph¶i! Chóng 2 g(x) ≤ g(x ), ∀ n ≥ 1. V× g(x) liªn tôc
n 2n
ta ®Òu cã thÓ t¹o cho m×nh mét c¸i g× ®ã trªn 1
nÒn t¶ng nh÷ng g× chóng ta ®H biÕt vµ ®H cã, trong [0; 1] nªn lim g(x 2 ) = g(1).
n

vµ c¸i chóng ta cÇn chØ lµ mét chót s¸ng t¹o. n → +∞

T«i muèn cïng c¸c b¹n thö søc víi mét 1


1

⇒ lim . lim g(x 2 ) ≥ g(x)


n

trong nh÷ng ph−¬ng ph¸p - ph−¬ng ph¸p n→+∞ 2 n n → +∞


tæng qu¸t hãa! ⇒ 0 ≥ f(x), ∀ x ∈ (0;1] (1)
1
Khi c¸c b¹n gi¶i xong mét bµi to¸n, b¹n MÆt kh¸c, víi x ∈ [ ;1) ta cã
hHy nªn tù hµo mét chót vÒ c¸ch gi¶i cña 2
f(x) ≥ 2x. f(x ) ≥ f(x ) ≥ ... ≥ f(x 2 )
n
m×nh vµ hHy tù hái xem, liÖu c¸ch gi¶i ®ã cã 2 2

cßn phï hîp nÕu b¹n thay ®æi chi tiÕt ë ®Ò ⇒ f(x) ≥ lim f(x 2 ) = f(0) ≥ 0 (2)
n

bµi. Theo t«i, c¸ch gi¶i tèi −u ph¶i lµ c¸ch n → +∞

gi¶i sö dông Ýt nhÊt nh÷ng d÷ liÖu ®H cã ë ®Ò 1


bµi. Khi ®ã víi nh÷ng gi¶ thiÕt kh«ng cÇn Tõ (1) vµ (2) ta cã f(x) = 0, ∀ x ∈ [ ; 1)
2
thiÕt, b¹n cã thÓ thay ®æi nã mµ c¸ch gi¶i 1
vÉn gi÷ nguyªn. §ã lµ mét c¸ch “tæng qu¸t Víi x ∈ (0;
). B»ng quy n¹p ta chøng
hãa”. §iÒu nµy cã vÎ h¬i tr¸i quy luËt v× 2
minh ®−îc: f(x) ≥ 2nx 2 −1 f(x 2 )
n n
c¸ch lµm lµ tæng qu¸t bµi to¸n dùa trªn c¸ch

TAÄP SAN TOAÙN HOÏC - 2007 3


TRÖÔØNG THPT CHUYEÂN HOAØNG VAÊN THUÏ - HOØA BÌNH
PhÇn I - S¸ng t¹o to¸n häc.

sè f : [ 0;1] → R , liªn tôc trong [0;1], tháa


⇒ Víi n ®ñ lín th× f(x) ≥ 0 (3)
mLn: f(x) ≥ g/(x). f(g(x)), ∀ x ∈ [0;1].
1
Tõ (1) vµ (3) ta cã f(x) = 0, ∀ x ∈ (0; )
2 Cã b¹n sÏ tù hái t¹i sao l¹i cã thÓ ®−a ra
VËy f ( x) = 0, ∀x ∈ (0;1] . V× f(x) liªn tôc mét bµi to¸n nh− vËy. RÊt ®¬n gi¶n: B¹n hHy
trong [0;1] nªn f(x) = 0, ∀ x ∈ [ 0;1] thö tæng qu¸t hãa b»ng c¸ch thay x2 b»ng
mét hµm g(x) bÊt k×, vµ ¸p dông hoµn toµn
t−¬ng tù c¸ch trªn b¹n sÏ thÊy cÇn ph¶i bæ
NhËn xÐt. Tõ c¸ch chøng minh trªn, ta thÊy:
sung gi¶ thiÕt ®Ó cã mét c¸ch gi¶i hoµn
Sè 2 trong ®iÒu kiÖn hoµn toµn cã thÓ thay
chØnh. V× nh− t«i ®H nãi ë trªn, c¸ch tæng
b»ng sè a > 0 bÊt kú, khi ®ã ta cã bµi to¸n:
qu¸t hãa bµi toµn ë ®©y lµ xuÊt ph¸t tõ c¸ch
gi¶i chø kh«ng ph¶i tõ ®Ò bµi. TÊt nhiªn, víi
Bµi to¸n 1.1. T×m tÊt c¶ c¸c hµm sè f(x):
gi¶ thiÕt qu¸ cô thÓ nh− trªn sÏ dÉn ®Õn thu
[0;1] → R, liªn tôc trong ®o¹n [0;1] tháa hÑp h−íng tæng qu¸t cña bµi to¸n, vµ ®Ó cã
mLn ®iÒu kiÖn: f(x) ≥ ax f(x2) , ∀ a > 0. ®−îc mét ®Ò bµi thùc sù tæng qu¸t t«i rÊt
mong chê ë kh¶ n¨ng s¸ng t¹o cña c¸c b¹n.
H¬n n÷a, ta cã thÓ thay ®æi thµnh bµi to¸n Sau ®©y, mêi c¸c b¹n cïng theo dâi vÝ dô 2,
tæng qu¸t sau mµ lêi gi¶i kh«ng thay ®æi. cïng víi 2 c¸ch gi¶i ë c¶ vÝ dô tr−íc, t«i xin
®Ò xuÊt vÝ dô 3 kh¸ thó vÞ:
Bµi to¸n 1.2. T×m tÊt c¶ c¸c hµm sè
f: [0;1] → R, liªn tôc trong [0; 1] tháa mLn 2.VÝ dô 2. Gi¶i ph−¬ng tr×nh
®iÒu kiÖn f(x) ≥ ax α −1 f(x α ), trong ®ã α >1. 1 x
f(x) - f( ) = x2 trªn tËp tÊt c¶ c¸c hµm
2 2
b) Lêi gi¶i 2. Do f(x) liªn tôc trong [0;1] nªn
1 1
f(x) cã nguyªn hµm trong [0;1]. Gäi F(x) lµ liªn tôc trong ®o¹n [- ; ].
mét nguyªn hµm cña f(x) trong [0;1]. 2 3
§Æt g(x) = F(x) - F(x2) Lêi gi¶i. Gäi F(x) lµ mét nguyªn hµm cña
⇒ g/(x) = F/x) - 2x F(x2) 1 1 x
f(x) trong [- ; ]. §Æt g(x) = F(x) - F( ),
= f(x) - 2x. f(x2) ≥ 0, ∀ x ∈ [0;1] 2 3 2
⇒ g(x) lµ hµm kh«ng gi¶m trªn [0;1]. 1 x
ta cã: g/(x) = f(x) - f( ) = x2
Mµ g(0) = g(1) = 0 nªn g(x) = 0), víi mäi 2 2
x ∈ [0;1] 1
n ⇒ g(x) = x3 + c.
⇒ F(x) =F(x2)= .... = F(x 2 ) 3
⇒ F(x) = lim F(x 2 ) = F(0), ∀ x ∈ (0;1) V× g(0) = 0 nªn c = 0.
n

n → +∞
1 x 1
⇒ f(x) = 0, ∀ x ∈ (0;1) ⇒ g(x) = x3. VËy F(x) = F( ) + x3
Do f(x) liªn tôc trong [0;1] nªn f(x) = 3 2 3
0, ∀ x ∈ [0;1]. 1 1 x 1 x
⇒ F(x) = x3+ ( )3 +... + ( n −1 ) +
Nh− vËy, theo c¸ch gi¶i thø 2, ta cã thÓ 3 3 2 3 2
kh¸i qu¸t ®−îc bµi to¸n nh− sau: x
+ F( n ) =
8 3 1 x
x (1 - 3n ) + F( n ), víi
2 21 2 2
Bµi to¸n 2.1. Cho hµm g: [0;1] → [0;1] cã 1 1
®¹o hµm trong [0;1] tháa mLn ®iÒu kiÖn mäi x ∈ [- ; ]. Khi n ®ñ lín, ta cã:
2 3
hµm [g ( x) − x ] ®¬n ®iÖu trªn [0;1], g(0)=0 vµ 8 3 1 1
g(1)=1. T×m tÊt c¶ c¸c hµm F(x) = x + F(0) ∀ x ∈ [- ; ].
21 2 3

TAÄP SAN TOAÙN HOÏC 2007 4


TRÖÔØNG THPT CHUYEÂN HOAØNG VAÊN THUÏ - HOØA BÌNH
PhÇn I - S¸ng t¹o to¸n häc.

⇒ f(x) =
8 3 1 1
x , ∀ x ∈ [- ; ]. Xung Quanh Baøi Toaùn Baát Ñaúng Thöùc
7 2 3
Thö l¹i thÊy ®óng. THI TOAÙN QUOÁC TEÁ 2005
NhËn xÐt. Qua c¸ch gi¶i trªn ta thÊy ®iÒu
1 1 NguyÔn anh tuÊn
®Çu tiªn lµ gi¶ thiÕt x ∈ [- ; ] lµ kh«ng cÇn
2 3 Chuyªn to¸n k97-00
thiÕt, ta cã thÓ më réng tËp x¸c ®Þnh lµ Sv. Líp D2000VT, Häc viÖn C«ng nghÖ
[ −1;1] mµ kÕt qu¶ kh«ng thay ®æi. Thø hai, B−u chÝnh ViÔn th«ng.
gi¶ thiÕt x2 còng cã thÓ kh¸i qu¸t thµnh 1 ®a
thøc. Nh− vËy, ta cã thÓ kh¸i qu¸t nh− sau: Trong kú thi Olympic To¸n Quèc tÕ lÇn
thø 46 tæ chøc t¹i Mexico cã bµi to¸n vÒ bÊt
Bµi to¸n 2a. Cho g(x) lµ ®a thøc bËc n cã ®¼ng thøc (B§T) nh− sau:
tËp x¸c ®Þnh lµ [-1;1]. T×m
hµm f :[−1;1] → R , liªn tôc trªn R vµ tháa Bµi to¸n 1. Cho 3 sè thùc d−¬ng x, y, z tháa
mLn ®iÒu kiÖn xyz ≥ 1. Chøng minh r»ng:
1 x
mLn : f(x) - f( ) = g/(x). x5 − x2 y5 − y 2
3 2 + +
C¸c b¹n hHy thö t×m ®iÒu kiÖn cho g(x) x5 + y 2 + z 2 y5 + z 2 + x2
nÕu ta muèn kh¸i qu¸t g(x) thµnh mét hµm z5 − z2
liªn tôc bÊt k×. + ≥0 (1)
z5 + x2 + y 2
Trë l¹i bµi to¸n vÝ dô 1, víi c¸ch gi¶i
Lêi gi¶i 1. B§T (1) t−¬ng ®−¬ng víi:
tr×nh bµy ë bµi to¸n vÝ dô 2, ta hoµn toµn cã
thÓ thay ®æi gi¶ thiÕt f(x) - 2xf(x2) ≥ 0 bëi ( x5 + y 2 + z 2 ) − ( x 2 + y 2 + z 2 ) +
f(x)- 2x f(x2) = g’(x). x5 + y 2 + z 2
C¸c b¹n hHy ®−a ra mét ®Ò bµi cã c¸c
®iÒu kiÖn r»ng buéc cho g(x) ®Ó t¹o thµnh +
(y 5
+ z 2 + x2 ) − ( x2 + y 2 + z 2 )
+
mét bµi to¸n hoµn chØnh. y5 + z 2 + x2
KÕt hîp c¸c h−íng tæng qu¸t trªn, t«i xin
®Ò xuÊt mét bµi to¸n tæng qu¸t h¬n: +
(z 5
+ x2 + y2 ) − ( x2 + y 2 + z 2 )
≥0
z5 + x2 + y2
3. VÝ dô 3. 1 1
⇔ + 5 +
Cho c¸c hµm sè g: [0;1] → R, x +y +z 25 2
y + z 2 + x2
f : [ 0;1] → [ 0;1] trong ®ã g, h cã ®¹o hµm 1 3
+ 5 ≤ 2 (2)
trªn [0;1], h(0) = 0, h(1) =1 vµ g lµ ®a thøc z +x +y
2 2
x + y2 + z2
bËc n .T×m hµm f: [0;1] → R, tháa mLn: Ta sÏ chøng minh:
3( y2 + z2 )
f(x) - h/(x).f(h(x)) = g/(x).
Mêi c¸c b¹n hHy gi¶i bµi to¸n nµy vµ tiÕp 1
≤ .
x5 + y 2 + z 2 2 ( x2 + y 2 + z 2 )
2
tôc! Sau ®©y lµ bµi tËp ®Ó c¸c b¹n tù luyÖn:

Bµi tËp. Cho f(x) cã ®¹o hµm trong (0;1), ThËt vËy, theo gi¶ thiÕt xyz ≥ 1 ta cã:
liªn tôc trong [0;1], ngoµi ra f(0) = f(1) = 0. 1 1
≤ 4 ≤
Chøng minh r»ng tån t¹i mét sè c ∈ (0;1) x +y +z
5 2 2
x
+y +z
2 2

tháa mLn ®iÒu kiÖn: yz


f(c) = 1996.f/ (c).
Chóc c¸c b¹n thµnh c«ng!

TAÄP SAN TOAÙN HOÏC 2007 5


TRÖÔØNG THPT CHUYEÂN HOAØNG VAÊN THUÏ - HOØA BÌNH
PhÇn I - S¸ng t¹o to¸n häc.

1 1
≤ 4
(3) + z 2 + x2
2x 1 y
+ y2 + z2 ≤ vµ
y + z2 y + z 2 + x2 ( x2 + y 2 + z 2 )
2 5 2

¸p dông B§T Bunhiac«pxky ta cã: 1


 2 2 
2
 + x2 + y2
+ 1 z
 y z 2
 +y +z ×
2 ≤
z + x2 + y 2 ( x2 + y 2 + z 2 )
5 2
 2  
  
Ta suy ra:
 
2

2x 4 1 1
×  2  + y2 + z2  ≥ + 5 +
  y + z2   x +y +z
5 2 2
y + z 2 + x2
   1

≥ ( x2 + y2 + z 2 )
2 + 5
z + x2 + y 2
1 3( y2 + z2 ) + + + 2 ( x2 + y 2 + z 2 )
1 1 1
⇔ ≤ (4)
( )
x y z
2x 4 2 x 2
+ y 2
+ z 2 2

+y +z
( x2 + y 2 + z 2 )
2 2 2
y2 + z2
Tõ (3) vµ (4) suy ra MÆt kh¸c tõ gi¶ thiÕt xyz ≥ 1
3( y2 + z2 ) 1 1 1
1
≤ . ⇒ + + ≤ yz + zx + xy ≤
2 ( x2 + y 2 + z 2 )
x5 + y 2 + z 2 2 x y z
≤ x 2 + y 2 + z 2 , do ®ã tõ B§T trªn suy ra (2)
Còng t−¬ng tù:
⇒ ®pcm.
1 3( z 2 + x2 ) §¼ng thøc x¶y ra ⇔ x = y = z = 1.

y5 + z 2 + x2 2 ( x2 + y 2 + z 2 ) B»ng c¸ch 2, ta chøng minh ®−îc bµi to¸n
2

tæng qu¸t sau:


1 3( x2 + y2 )
≤ Bµi to¸n 2. Cho n sè thùc d−¬ng x1 , x2 ,..., xn
z5 + x2 + y 2 2 ( x2 + y 2 + z 2 )
2

Céng theo vÕ c¸c B§T trªn ta thu ®−îc


( n ≥ 3) tho¶ mLn ®iÒu kiÖn x1 x2 ...xn ≥ 1.
(2) ⇒ ®pcm. Chøng minh r»ng:
§¼ng thøc x¶y ra ⇔ x = y = z = 1 x12 n +1 − x1n
+
x12 n +1 + x2 n + x3n + .. + xn n
Lêi gi¶i 2.
x2 2 n +1 − x2 n
¸p dông B§T Bunhiac«pxky ta cã: + +…+
x2 2 n +1 + x1n + x3n + .. + xn n
( x5 + y 2 + z 2 )  1x + y 2 + z 2  ≥ xn 2 n +1 − xn n
+ ≥ 0 (5)
xn 2 n +1 + x2 n + x3n + .. + xn −1n
≥ ( x2 + y2 + z 2 )
2

Chøng minh. Theo B§T C«-si vµ gi¶ thiÕt


1
+ y2 + z2 x1 x2 ...xn ≥ 1 ta cã: x12 n +1 ≥
x12 n

1 x
⇒ 5 ≤ . x2 x3 ...xn
x + y2 + z2 ( x 2
+ y 2
+ z )
2 2

( n − 1) x12 n
Thªm hai B§T t−¬ng tù n÷a: ≥ (6)
x2 n + x3n + ... + xn n

TAÄP SAN TOAÙN HOÏC 2007 6


TRÖÔØNG THPT CHUYEÂN HOAØNG VAÊN THUÏ - HOØA BÌNH
PhÇn I - S¸ng t¹o to¸n häc.

MÆt kh¸c, ¸p dông B§T Bunhiac«pxky ra  x1n + x2 n + ... + xn n 


 2
 ⇔  2 n +1 − 1 +
 x1 + x2 + x3 + ... + xn
n  n n n
 x n
+ ... + x n 
cã:  2 n
 + x2 + ... + xn ×
n
 n −1    x + x2 + ... + xn
n n n

    +  2 n +11 − 1 + … +
 x2 + x1 + x3 + ... + xn
n n n

 2

× 
( n − 1) x12 n 
 + x2 n + ... + xn n  ≥  x1n + x2 n + ... + xn n 
  x2 n + ... + xn n   +  2 n +1 − 1 ≤ 0
    xn + x1 + x2 + ... + xn −1
n n n

⇔ (5)
≥ ( x1n + x2 n + ... + xn n )
2

§¼ng thøc x¶y ra ⇔ x1 = x2 =…= xn =1


1 Mét d¹ng tæng qu¸t kh¸c cña Bµi to¸n 1
⇔ ≤
( n − 1) x1 2n
+ x2 + x3 + ... + xn
n n n nh− sau:
x2 n + ... + xn n Bµi to¸n 3. Cho sè tù nhiªn n ≥ 3 vµ 3 sè


n ( x2 n + ... + xn n )
(7)
thùc d−¬ng x, y, z tho¶ mLn ®iÒu kiÖn
xyz ≥ 1 . Chøng minh r»ng:
n − 1 ( x n + x n + ... + x n ) 2 xn − x2 yn − y2 zn − z2
1 2 n
+ +
Tõ (6) vµ (7) suy ra: xn + y 2 + z 2 y n + z 2 + x2 zn + x2 + y2
1 ≥ 0 (8)
2 n +1

x1 + x2 + x3n + ... + xn n
n
1 1
Hay lµ: n + n +
1 x +y +z 2 2
y + z 2 + x2
≤ ≤
( n − 1) x1 + x n + x n + ... + x n
2n
+ n
1
≤ 2
3
x2 n + ... + xn n
2 3 n z +x +y 2 2
x + y2 + z2


n ( x2 n + ... + xn n )
.
B»ng ph−¬ng ph¸p t−¬ng tù nh− lêi gi¶i 2
chóng ta cã thÓ chøng minh ®−îc Bµi to¸n 3
n − 1 ( x n + x n + ... + x n ) 2 ®óng víi n ≤ 8. Sau ®©y ta chøng minh
1 2 n
trong tr−êng hîp n = 6 :
Cïng víi n -1 B§T t−¬ng tù kh¸c, céng
vÕ víi vÕ ta thu ®−îc: ¸p dông B§T Bunhiac«pxky tæng qu¸t ta
1 cã: ( x 6 + y 2 + z 2 )(1 + y 2 + z 2 )(1 + y 2 + z 2 ) ≥
+
≥ ( x2 + y2 + z 2 )
2 n +1
x1 + x2 + x3n + ... + xn n
n 3

1
+ 2 n +1
x2 + x1 + x3n + ... + xn n
n
+…+
⇒ 6
1

(1 + y + z )
2 2 2

.
+ 2 n +1
1

x + y2 + z2 (x + y + z )
2 2 2 3

xn + x1 + x2 n + ... + xn −1n
n
Thªm hai B§T t−¬ng tù n÷a, suy ra
n 1 1
≤ + 6 +
x1 + x2 + ... + xn n
n n
x +y +z
6 2 2
y + z 2 + x2
x1n + x2 n + ... + xn n + 6
1

⇔ 2 n +1 +
x1 + x2 n + x3n + ... + xn n z + x2 + y 2
x1n + x2 n + ... + xn n
+ 2 n +1 +…+
(1+ y + z 2 ) + (1 + z 2 + x2 ) + (1 + x2 + y2 )
2 2 2 2
x2 + x1n + x3n + ... + xn n
≤ (9)
(x + y2 + z 2 )
3
x1n + x2 n + ... + xn n 2
+ 2 n +1 ≤n
xn + x1n + x2 n + ... + xn −1n Ta sÏ chøng minh

TAÄP SAN TOAÙN HOÏC 2007 7


TRÖÔØNG THPT CHUYEÂN HOAØNG VAÊN THUÏ - HOØA BÌNH
PhÇn I - S¸ng t¹o to¸n häc.

(1 + y + z ) + (1 + z + x ) + (1 + x
2 2 2 2
+ y2 )
2
2 2 2 3
vÕ tr¸i → +∞ , trong khi ®ã vÕ ph¶i → ,
x2m
≤ 3 ( x + y + z ) (10)
2 2 2 2
v« lý.
ii) Víi n = 1. Khi ®ã (12) cã d¹ng:
§Æt u = x , v = y , t = z th× ta cã (10)
2 2 2

⇔ (1 + u + v ) + (1 + v + t ) +
2 2 ( x 2 + xy + y 2 ) + ( y 2 + yz + z 2 ) +
+ ( z 2 + zx + x 2 ) ≤ 3 ( x 2 + y 2 + z 2 )
+ (1 + t + u ) ≤ 3 ( u + v + t )
2 2

⇔ xy + yz + zx ≤ x 2 + y 2 + z 2 . B§T nµy
⇔ 3 + 4 ( y + v + t ) + 2 ( uv + vt + tu ) +
®óng víi mäi x, y, z ⇒ (12) ®óng.
+ 2 (u 2 + v2 + t 2 ) ≤ 2 (u 2 + v2 + t 2 ) + ii) Víi n = 2. B§T (12) cã d¹ng:
+ 4 ( uv + vt + tu ) + ( u + v + t )
2
( x + xy + y ) + ( y + yz + z ) +
2 2 2 2 2 2

⇔ (u + v + t ) - 4 ( y + v + t ) +
2
+ ( z + zx + x ) ≤ 3 ( x + y + z )
2 2 2 2 2 2 2

+ 2 ( uv + vt + tu ) - 3 ≥ 0 (11) ⇔ 2( x y + y x + y z + z y + z x + x z) ≤
3 3 3 3 3 3

Tõ gi¶ thiÕt xyz ≥ 1, suy ra uv + vt + tu ≥ ≤ ( x + y + z ) + 3( x y + y z + z x )


4 4 4 2 2 2 2 2 2

≥ 3 3 ( uvt ) = 3 3 ( xyz ) ≥ 3, do ®ã (11)


2 4
1 1 1
⇔ ( x − y) + ( y − z ) + ( z − x) ≥ 0
4 4 4

®óng vµ ta cã (10). VËy tõ (9) vµ (10) ta cã 2 2 2


®pcm. B§T cuèi ®óng ⇒ B§T (12) ®óng víi
T«i dù ®o¸n r»ng B§T (8) ®óng víi mäi n = 2.
n, mong c¸c b¹n cïng quan t©m tíi viÖc iii) Víi n ≥ 3 , ta sÏ chøng minh r»ng khi ®ã
chøng minh bµi to¸n nµy. Sau ®©y lµ mét bµi (12) kh«ng ®óng. ThËt vËy, ta cã (12) ⇔
to¸n míi mµ t«i ®H ph¸t hiÖn ra trong qu¸ n n
tr×nh më réng bµi to¸n trªn.  x 2 + xy + y 2   y 2 + yz + z 2 
 2 2 
+ 2 2 
+
x +y +z  x +y +z 
2 2
T×m tÊt c¶ c¸c sè nguyªn n sao cho bÊt
®¼ng thøc (B§T) sau ®óng víi mäi x, y, z n
kh¸c kh«ng:  z 2 + zx + x 2 
+ 2 2 
≤3 (13)
( x + xy + y ) + ( y + yz + z ) + x +y +z 
2
2 2 n 2 2 n

Chän x = 1,1; y = 1; z = 0,1 th× ta cã:


+ ( z + zx + x ) ≤ 3 ( x + y + z )
2 2 n 2 2 2 n
(12) n
 x 2 + xy + y 2   y 2 + yz + z 2 
n

Lêi gi¶i. Víi n = 0 th× B§T (12) hiÓn nhiªn  2 2 


+ 2 2 
+
x +y +z  x +y +z 
2 2

®óng víi mäi x, y, z ≠ 0. Ta xÐt c¸c tr−êng n n


hîp sau:  z 2 + zx + x 2   x 2 + xy + y 2 
+ 2 2 
> 2 2 
=
i) Víi n < 0. §Æt n = -m (m > 0) , khi ®ã x +y +z  x +y +z 
2 2

B§T (12) trë thµnh: n


1 1  3,31 
+ + =  > 1, 49 n .
( x + xy + y ) ( y + yz + z )
2 2 m 2 2 m  2, 22 
B»ng quy n¹p ta chøng minh ®−îc
1 3
+ ≤ 1, 49 n > 3 víi mäi n ≥ 3 . Tõ ®ã suy ra (13)
(z 2 m
) (x + y2 + z2 )
m
2
+ zx + x 2
kh«ng ®óng víi mäi x, y, z > 0 ⇒ ®pcm.
B§T nµy kh«ng ®óng víi mäi x, y, z ≠ 0. Tõ nh÷ng ph©n tÝch trªn ë c¸c tr−êng
ThËt vËy, cè ®inh x sao cho y → 0, z → 0 th× hîp trªn ta ®i ®Õn kÕt luËn: tÊt c¶ c¸c sè
nguyªn n ph¶i t×m lµ n = 0, 1, 2.

TAÄP SAN TOAÙN HOÏC 2007 8


TRÖÔØNG THPT CHUYEÂN HOAØNG VAÊN THUÏ - HOØA BÌNH
PhÇn I - S¸ng t¹o to¸n häc.

Mét. Ngay lËp tøc ta sÏ gÆp mét hÖ


THÖÛ ÑI TÌM MOÄT BAÁT ÑaúÛNG THÖÙC qu¶: x + y + z = 0 ⇒ a 2 yz + b 2 zx + a 2 xy , vµ
víi x = b, y = c - a, z = a - b ta ®−îc mét kÕt
TRONG TAM GIAÙC qu¶ quen biÕt:
a 2 (c − a )(a − b) + b 2 (a − b)(b − c) +
D¦¥NG THÞ H¦¥NG + c 2 (b − c)(c − a ) ≤ 0
Chuyªn To¸n K98 – 01 hai . Mét vÝ dô kh¸c Ýt tÇm th−êng h¬n lµ
víi bé sè (x, y, z) = (1, 1, 1) th× tõ (*) ta
NGUYÔN NH¦ TH¾NG nhËn ®−îc:
MA2 + MB 2 + MC 2 ≥ ( a 2 + b 2 + c 2 )
Sv. Líp 3 CLC, K51 §HSP Hµ Néi 1 1
3
Ta gÆp l¹i mét kÕt qu¶ quen thuéc trong
TÊt c¶ chóng ta ®Òu biÕt ®Õn nh÷ng ®Þnh tam gi¸c. §¼ng thøc x¶y ra ⇔
→ → →
lÝ, nh÷ng kÕt qu¶ lý thó hay nh÷ng chøng ⇔ x MA+ y MB + z MC = 0
minh ®éc ®¸o trong to¸n häc. Vµ liÖu ®H cã
→ → →
®«i lÇn b¹n ®H tù hái v× sao ng−êi ta l¹i nghÜ ⇔ MA+ MB + MC = 0
ra nh÷ng ®iÒu tuyÖt diÖu nh− thÕ? ThËt khã ⇔ M lµ träng t©m tam gi¸c.
®Ó cã thÓ tr¶ c©u hái nµy mét c¸ch thËt chÝnh
x¸c. Nh−ng nh− thÕ kh«ng cã nghÜa lµ chóng ba. Thö víi bé (x, y, z) = (a, b, c) ta ®−îc:
ta sÏ chÞu “bã tay”! Môc ®Ých cña bµi viÕt (a + b + c)(aMA2 + bMB 2 + cMC 2 )
nµy lµ ®Æt chóng ta ®øng ë vÞ trÝ “nh÷ng nhµ ≥ a 2bc + b 2 ca + c 2 ab
kh¶o cæ” thö ®i t×m mét chót g× ®ã, cã thÓ ⇔ aMA2 + bMB 2 + cMC 2 ≥ abc
chØ lµ mét “trß ch¬i” cho riªng m×nh! T«i Ta thÊy l¹i kÕt qu¶ ®Æc tr−ng cho t©m
ph¶i l−u ý c¸c b¹n r»ng, chóng ta sÏ thö lµm ®−êng trßn néi tiÕp tam gi¸c ABC. Nh÷ng
nhµ “khai kho¸ng”, “kh¶o cæ”, “t×m kiÕm” vÊn ®Ò t−¬ng tù ®−îc xÐt cho trùc t©m, t©m
chø kh«ng ph¶i lµ nhµ ph¸t minh bëi cã thÓ, ®−êng trßn ngo¹i tiÕp, ®−êng trßn bµng tiÕp
nh÷ng g× chóng ta t×m thÊy sÏ kh«ng cã g× lµ v. v ... Nh− vËy, víi mçi bé sè (x, y, z) thay
qu¸ míi l¹! vµo (*) ta sÏ thu ®−îc mét B§T. C¸c b¹n hHy
Th«ng th−êng, ®Ó cã thÓ t×m kiÕm, khai thö chän vµi bé nµo ®ã nhÐ!
th¸c ®−îc, ta ph¶i cã mét “khu má” hay mét Bèn. Nh−ng nh− thÕ th× c«ng viÖc cña
m¶nh ®Êt mµu mì. chóng ta ch−a cã g× lµ thó vÞ c¶. Thö lÊy
B¹n ®H bao giê ®Ó ý ®Õn ®iÒu nµy ch−a:
“ Víi mäi tam gi¸c ABC vµ c¸c sè thùc x, ( x, y, z ) = ( a 2 , b,1) xem nµo! Ta cã:
y, z ∈ R vµ víi mét ®iÓm M bÊt kú th×:
→ → →
(a 2
+ b + 1)( a 2 MA2 + bMB 2 + MC 2 ) ≥
( x MA+ y MB + z MC ) 2 ≥ 0 ” ? ≥ a 2b + b 2 a 2 + c 2 a 2 b
Kh«ng qu¸ ®Æc biÖt, nh−ng b¹n thö biÕn §iÒu nµy cã vÎ “ngå ngé”! B¹n ®H nghÜ
®æi l¹i xem nµo! Kh«ng mÊy khã kh¨n, b¹n ra c¸ch nµo kh¸c ®Ó chøng minh ®iÒu “ngå
cã thÓ nhËn ®−îc bÊt ®¼ng thøc (B§T): ngé” Êy hay ch−a?
( x + y + z )( xMA2 + yMB 2 + zMC 2 ) Xin c¸c b¹n ®õng véi bùc m×nh v× c«ng
viÖc khai th¸c cña chóng ta qu¸ chËm ch¹p.
≥ a 2 yz + b 2 zx + c 2 xy (*), víi a, b, c lÇn l−ît Trªn ®©y chóng ta ®H sö dông “c«ng nghÖ”
lµ 3 c¹nh BC, CA, AB cña tam gi¸c ABC. kh¸ cò kü lµ khi thay bé (x, y, z) bëi nh÷ng
§©y chÝnh lµ “khu má” mµ chóng ta sÏ bé sè cè ®Þnh. ViÖc lµm nµy kh«ng ph¶i lµ
khai th¸c. kh«ng cã lîi Ých g× nh−ng xem ra “s¶n

TAÄP SAN TOAÙN HOÏC 2007 9


TRÖÔØNG THPT CHUYEÂN HOAØNG VAÊN THUÏ - HOØA BÌNH
PhÇn I - S¸ng t¹o to¸n häc.

phÈm” cña chóng ta ch−a ®−îc phong phó §Ó ý lµ khi M kh«ng n»m trªn cung lín
l¾m. BC chøa A th×: ( − aMA + bMB + cMC ) > 0
Sau ®©y lµ mét vµi c¶i tiÕn nho nhá nh−ng
(B§T Pt«lªmª) nªn ta thu ®−îc:
sÏ mang l¹i nh÷ng kÕt qu¶ bÊt ngê.
MB.MC MC.MA MA.MB
n¨m. Trong (*) thay − + + ≥ −1 (2)
a b c bc ca ab
(x, y, z) = ( , , ) víi Khi M ∈ BC kh«ng chøa A th×
MA MB MC
M ∈ { A; B; C} còng ®óng. VËy (2) ®óng víi
M ∉ { A; B; C} ta cã:
®iÓm M bÊt kú.
a b c
( + + )(aMA + bMB + cMC) ≥ Ta ®H t×m thÊy mét ”hä hµng” cña B§T
MA MB MC Pt«lªmª: Khi M∈ BC th× (2) vµ B§T
a b c Pt«lªmª lµ t−¬ng ®−¬ng. Nh−ng tiÕc r»ng,
≥ abc( + + )
MB.MC MC.MA MA.MB trong khi B§T Pt«lªmª th× ai còng biÕt cßn
MB.MC MC.MA MA.MB ng−êi “anh em” nµy th× ch¼ng mÊy ai biÕt
⇔ + + ≥ 1. (1)
bc ca ab ®Õn! ThËt ®¸ng th−¬ng!
DÔ thÊy khi M ∈ {A, B, C} th× (1) trë B¶y. Vµ nÕu chóng ta thay
thµnh ®¼ng thøc. VËy (1) ®óng víi mäi M.  1 1 1 
Suy diÔn mét chót, chóng ta sÏ cã ngay ( x, y , z ) =  2
, 2
, 2 
th× tõ (*)
 MA MB MC 
nh÷ng kÕt qu¶ quen thuéc:
ta thu ®−îc:
MA MB MC
+ + ≥ 3  1 1 1 
a b c  2
+ 2
+  . (1 + 1 + 1) ≥
 MA MB MC 2 
ma mb mc 3 3
+ + ≥  a   b
2
  c 
2 2
a b c 2 ≥  +  + 
a b c  MB.MC   MC.MA   MA.MB 
+ + ≥2 3
⇔ ( MB.MC ) + ( MC.MA ) + ( MA.MB ) ≥
2 2 2
ma mb mc

( a MA2 + b 2 MB 2 + c 2 MC 2 ) (3)
n n n n
 MA   MB   MC   1  1 2
 +  +  ≥ 3

  3
 a   b   c   3
(3) vÉn ®óng khi M ∈ { A; B; C} .
ë ®©y ma, mb, mc lÇn l−ît lµ ®é dµi c¸c
trung tuyÕn øng víi c¸c ®Ønh A, B, C cña tam B¹n thö chøng minh (3) khi M ≡ G xem
gi¸c ABC. nµo, ch¾c còng kh«ng ®¬n gi¶n l¾m!
S¸u. Chóng ta còng thö lµm ®iÒu t−¬ng tù T¸m. Trong (*) thay
a b c  1 1 1 
khi: (x, y, z) = ( − , , ) ta cã: ( x, y , z ) =  , ,  ta ®−îc
MA MB MC  MA MB MC 
 a b c  ( MB.MC + MC.MA + MA.MB )( MA + MB + MC )
− + +  ( − aMA + bMB + cMC )
 MA MB MC  ≥ a 2 MA + b 2 MB + c 2 MC
a b c
≥ abc( − − )
MB.MC MC.MA MA.MB
⇔ ( − aMA + bMB + cMC )
 MB.MC MC.MA MA.MB 
× − + + 
 bc ca ab 
≥ ( aMA − bMB − cMC ) .

TAÄP SAN TOAÙN HOÏC 2007 10


TRÖÔØNG THPT CHUYEÂN HOAØNG VAÊN THUÏ - HOØA BÌNH
PhÇn I - S¸ng t¹o to¸n häc.

Gi¶ sö tam gi¸c ABC tï t¹i A, khi ®ã ta thøc néi suy Lagr¨ng cho ®a thøc vµ so s¸nh
cã: MB ≥ c – MA, MC ≥ b – MA. hÖ sè. Ch¼ng h¹n, vÝ dô trªn khai triÓn x2 t¹i
DÊu b»ng xÈy ra ⇔ M ≡ A. Do ®ã a, b, c. Do c¸c sè phøc tÝnh to¸n nh− sè thùc
a MA + b 2 MB + c 2 MC ≥
2 nªn trong c¸c sè phøc còng cã c¸c ®¼ng thøc
nh− vËy. VËn dông tÝnh chÊt “nhHn” (hoÆc
( a 2 − b2 − c 2 ) MA + bc(b + c) ≥ bc(b+c) (4). chuÈn) cña tæng vµ tÝch cho mçi sè phøc
§¼ng thøc xÈy ra ⇔ M ≡ A. t−¬ng øng víi mét ®iÓm. Ch¼ng h¹n tõ ®¼ng
Ta thÊy r»ng (4) ®óng c¶ trong tr−êng hîp thøc trªn ta thu ®−îc:
M ∈ { A; B; C} vµ khi M ≡ A th× cã dÊu ®¼ng (m − b)(m − c) (m − c)(m − a) (m − a)(m − b)
+ + ≥1
thøc. Nh− vËy ta cã bµi to¸n: (a − b)(a − c) (b − c)(b − a) (c − a)(c − b)
Cho tam gi¸c ABC tï ë A. T×m gi¸ trÞ nhá MA.MC MC.MA MA.MB
nhÊt cña biÓu thøc. ⇒ + + ≥1
AB. AC BC.BA CA.CB
( MB.MC + MC.MA + MAMB . )( MA + MB + MC ) Theo c¸ch ®ã, b¹n sÏ thu ®−îc rÊt nhiÒu
Ta thÊy r»ng nh÷ng bÊt ®¼ng thøc thu ®−îc ë ®iÒu thó vÞ.
trªn ®Òu ®óng víi mäi M. Do ®ã khi thay M
bëi nh÷ng vÞ trÝ ®Æc biÖt ta l¹i thu ®−îc kh¸ B. M¶nh ®Êt 2:
nhiÒu kÕt qu¶, tÝnh chÊt thó vÞ.
C¸c b¹n thÊy ®Êy, xuÊt ph¸t tõ (*), mçi
lÇn thay (x, y, z) bëi mét bé nµo ®ã, ta thu
®−îc mét kÕt qu¶ míi. C«ng viÖc cña nhµ
t×m kiÕm lµ ph¶i biÕt ch¾t läc, gi÷ l¹i nh÷ng
g× cã gi¸ trÞ tõ nh÷ng thø t−ëng chõng nh−
tÇm th−êng. Cã mét lÇn lµm thö, chóng ta
míi thÊy r»ng, ®Ó cã nh÷ng g× chóng ta ®ang
®−îc häc h«m nay kh«ng ph¶i lµ dÔ. T«i hi
väng sau bµi viÕt nµy, mçi chóng ta sÏ rót ra
mét ®iÒu g× ®ã cho riªng m×nh ®Ó cã thÓ häc
m«n To¸n vui vÎ h¬n, vµ nh÷ng ai say mª Gäi a’, b’, c’ lµ ba c¹nh cña tam gi¸c
muèn lµm nh÷ng nhµ “kh¶o cæ”, hHy cø b¾t A’B’C’ t−¬ng øng. C¸c ®iÓm M, N nh− trªn
tay vµo c«ng viÖc cña m×nh dÉu biÕt r»ng h×nh vÏ.
chóng ta cã thÓ ch¼ng thu luîm ®−îc g× to XuÊt ph¸t tõ kÕt qu¶ a’.NA + b’.NB +
t¸t. Nh−ng, cã mét ®iÒu t«i tin ch¾c lµ sau +c’.NC ≥ a’.MA + b’.MB + c’.MC. HHy
khi nh÷ng lÇn nh− thÕ, b¹n sÏ thÊy To¸n häc t×m c¸ch gi¶i trän vÑn bµi to¸n sau:
cµng ®¸ng yªu h¬n. Cho tam gi¸c ABC vµ x, y, z > 0 . HLy
NÕu b¹n c¶m thÊy thÝch c«ng viÖc t×m t×m ®iÓm M trong tam gi¸c ABC sao cho:
kiÕm, mµy mß nh÷ng ®iÒu míi mÎ (dï chØ S(M) = xMA + yMB + zMC nhá nhÊt.
cho riªng m×nh) t«i cã thÓ giíi thiÖu víi b¹n Ch¾c c¸c b¹n còng ®H biÕt khi x = y = z
mét vµi m¶nh ®Êt “mµu mì”, phï hîp víi th× ta cã bµi to¸n ®iÓm Toricelli cña tam gi¸c
nh÷ng g× b¹n mong muèn: C. M¶nh ®Êt 3.
A. M¶nh ®Êt 1: Tr−íc hÕt, c¸c b¹n hHy chøng minh víi
M¶nh ®Êt nµy ®ßi hái b¹n ph¶i cã mét vµi x + y > 0, y + z > 0, z + x > 0 th× ta cã :
sù chuÈn bÞ vÒ sè phøc. Trong sè thùc, ta cã xMA + yMB + zMC ≥ 4 xy + yz + zx .S ABC
®¼ng thøc:
Tõ ®ã hHy x©y dùng mét vµi tÝnh chÊt míi!
(m − b)(m − c) (m − c)(m − a) (m − a)(m − b)
+ + =1 VÉn cßn nhiÒu vïng ®Êt míi ®ang chê in
(a − b)(a − c) (b − c)(b − a) (c − a)(c − b) dÊu ch©n c¸c b¹n. Chóc thµnh c«ng!
§Ó cã ®−îc ®¼ng thøc trªn hoÆc nh÷ng
®¼ng thøc t−¬ng tù, b¹n cã thÓ dùa vµo c«ng

TAÄP SAN TOAÙN HOÏC 2007 11


TRÖÔØNG THPT CHUYEÂN HOAØNG VAÊN THUÏ - HOØA BÌNH
PhÇn I - S¸ng t¹o to¸n häc.

Moät Söï
ii) Hái trong ®o¹n [1;32000 ] cã tÊt c¶ bao
nhiªu sè nguyªn a sao cho P(an) chia hÕt
cho 32000.
(C¸c b¹n cã thÓ tham kh¶o thªm ë c¸c sè t¹p

Tình côø chÝ To¸n häc vµ Tuæi trÎ th¸ng 01, 02, 09
n¨m 2001)
Sau nhiÒu ngµy suy nghÜ t«i ®H ph¸t hiÖn
ra mét c¸ch chøng minh, nh−ng kh¸ dµi vµ
NguyÔn L©m TuyÒn chØ cho riªng Bµi to¸n HSG1 (xin kh«ng nªu
Chuyªn To¸n K99 – 02 ra ë ®©y).
Sv. Líp §iÒu khiÓn Tù §éng 1 - K47
§H B¸ch Khoa Hµ Néi II/ T×nh cê.
B½ng ®i mét thêi gian ®Ó råi t×nh cê lËt l¹i
trang s¸ch. ý t−ëng chît lªn, t«i h¹ bót viÕt
nh− ®H ®−îc “lËp tr×nh” s½n. T«i ®H cã mét
Trong cuéc sèng, nhiÒu ®iÒu thó vÞ lêi gi¶i míi cho Bµi to¸n HSG1, nh−ng tÊt
nhiªn lµ víi “phong c¸ch” hoµn toµn kh¸c.
®«i khi ®Õn víi chóng ta mét c¸ch nhÑ
nhµng, man m¸c … Lµm cho ta thªm yªu Lêi gi¶i ®ã nh− sau.
®êi, yªu cuéc sèng! Bµi viÕt nµy t«i xin Ta cã P(x) = x3 – 9x2 + 24x – 27
tr×nh bµy “mét niÒm vui nho nhá” mµ t«i = (x - 3)3 – 3(x - 3) – 9
t×nh cê cã ®−îc khi ®ang “th¶ hån” víi ⇒ P(3x+3) = 9(3x3 – x – 1).
nh÷ng bµi to¸n hãc bóa. Th©n tÆng tíi c¸c Bµi to¸n quy vÒ viÖc chøng minh: Víi
b¹n, ®Æc biÖt lµ c¸c em líp chuyªn To¸n mçi n, tån t¹i bn ∈ N* sao cho Q(bn) chia hÕt
K01-04 – nh÷ng ng−êi rÊt t©m huyÕt víi tê cho 3n. ë ®©y Q(x) = 3x3 – x – 1.
b¸o nµy. Hy väng r»ng qua bµi viÕt nµy phÇn Ta sÏ chøng minh ®iÒu nµy b»ng quy n¹p
nµo sÏ gióp Ých cho c¸c b¹n trong qu¸ tr×nh theo n. Víi n = 1 chän b1 = 2.
häc to¸n. Gi¶ sö kh¼ng ®Þnh ®óng tíi n.
Ta cã Q(bn+Q(bn)) =
I/ Thö th¸ch. = 3(bn+Q(bn))3 – (bn+Q(bn)) – 1
N¨m cßn häc líp 10 Chuyªn To¸n, cã hai = (3bn3 − bn − 1) + 9bn Q (bn )(bn + Q (bn )) +
bµi to¸n khiÕn t«i rÊt tr¨n trë. §ã lµ hai bµi
to¸n thi häc sinh giái Quèc gia, võa quen l¹i + 3Q 3 (bn ) − Q (bn )
võa l¹: = 3 3bn Q (bn )(bn + Q (bn )) + Q 3 (bn ) 
Chän bn+1 = bn+Q(bn) th× Q(bn+1) chia hÕt
Bµi to¸n HSG1. Cho ®a thøc P(x) = x3 – 9x2
cho 3n+1. Tãm l¹i ta cã ®iÒu ph¶i chøng
+ 24x – 27. Chøng minh r»ng víi mçi sè tù
minh.
nhiªn n, tån t¹i sè nguyªn an sao cho P(an)
chia hÕt cho 3n.
Véi vµng ®em ¸p dông cho Bµi to¸n
HSG2 nh−ng … kh«ng thµnh c«ng! T«i
Bµi to¸n HSG2. Cho ®a thøc P(x) = x3 quyÕt ®Þnh quay trë l¹i Bµi to¸n HSG1 víi
+153x2 -111x +38. môc ®Ých më réng nã vµ ®H ®−a ra ®−îc bµi
i) Chøng minh r»ng víi mçi sè tù nhiªn n, to¸n tæng qu¸t sau.
tån t¹i Ýt nhÊt 9 sè nguyªn a thuéc ®o¹n
[1;32000 ] sao cho P(an) chia hÕt cho 32000. Bµi to¸n A. XÐt tËp hîp c¸c ®a thøc cã d¹ng
T = { P(x) = ax3 + bx2 + cx + d / a ≠ 0,
b ≡ 0(mod3), c ≅ 0(mod3), a + c ≅

TAÄP SAN TOAÙN HOÏC 2007 12


TRÖÔØNG THPT CHUYEÂN HOAØNG VAÊN THUÏ - HOØA BÌNH
PhÇn I - S¸ng t¹o to¸n häc.

0(mod3)}. Khi ®ã víi mçi sè nguyªn d−¬ng r»ng tån t¹i v« h¹n sè nguyªn d−¬ng a mµ
n, tån t¹i sè nguyªn an sao cho P(an) chia hÕt Q(a) chia hÕt cho pp.
cho 3n.
Tß mß, t«i thö t×m mét lêi gi¶i kh¸c cho
Chøng minh. Ta chøng minh b»ng quy n¹p. bµi to¸n míi nµy. Vµ còng t×nh cê t«i ®−a ra
Víi n = 1, ta cã P(0) = d, P(-1) =- a + b – ®−îc mét lêi gi¶i míi, vµ tÊt nhiªn lµ còng
c + d ≡ - (a + c) + d ≡ 0(mod3), P(1) = a + víi “phong c¸ch” hoµn toµn míi: sö dông
b + c + d ≡ (a + c) + d(mod3). L−u ý r»ng kh¸i niÖm hÖ thÆng d− cña lý thuyÕt ®ång d−
trong 3 sè h¹ng liªn tiÕp cña mét cÊp sè thøc.
céng cã c«ng sai kh«ng chia hÕt cho 3, lu«n Bµi to¸n C còng chÝnh lµ néi dung cña bµi
tån t¹i mét sè chia hÕt cho 3. VËy víi n = 1, T8/336 trªn T¹p chÝ To¸n häc vµ Tuæi trÎ
bµi to¸n ®óng. th¸ng 06/2005 do t«i ®Ò xuÊt. XuÊt sø cña
Gi¶ sö tån t¹i ak ®Ó P(ak) chia hÕt cho 3k. bµi T8/336 lµ nh− vËy vµ cã lÏ, ®ã còng lµ
Ta cã P(ak + hP(ak)) = a(ak + hP(ak))3 + mét sù t×nh cê.
b(ak + hP(ak))2 + c(ak + hP(ak)) + d =
= P (ak ). ( 3a.ak2 .h + 3a.ak h 2 P (ak ) + h3 P 2 (ak ) +
Chøng minh Bµi to¸n C.
NhËn xÐt: Gi¸ trÞ t¹i pp ®iÓm nguyªn d−¬ng
+bh 2 P (ak ) + (2bak + c)h + 1) liªn tiÕp cña ®a thøc Q(x) lËp thµnh mét hÖ
thÆng d− ®Çy ®ñ (modpp).
Ta thÊy 2bak + c ≅ 0 (mod3) ⇒ tån t¹i ThËt vËy, trong pp sè nguyªn d−¬ng liªn
h ∈ {1; 2} sao cho (2bak + c)h + 1≡ 0(mod3). tiÕp, gi¶ sö cã u > v sao cho
Tõ ®ã chän ak+1 = ak + hP(ak) th× ta cã Q (u ) ≡ Q (u )(mod p p ) ⇔ ( p − 1)u p − u − 1
P(ak+1) chia hÕt cho 3k+1 (®pcm). ≡ ( p − 1)u p − u − 1(mod p p )
TÊt nhiªn lµ còng víi xu h−íng ®ã, t«i ⇔ ( p − 1) ( u p − v p ) − ( u − v ) ≡ 0(mod p p ) (*).
t×m c¸ch më réng bµi to¸n thªm n÷a, nh−ng Theo ®Þnh lý Fermat nhá, ta cã
qu¶ thùc lµ rÊt khã kh¨n. Sau mét vµi phÐp u ≡ u (mod p ) , v p ≡ v (mod p ) . Do ®ã tõ (*)
p

thö vµ dù ®o¸n t«i ®−a ra bµi to¸n sau mµ


theo t«i, ë mét khÝa c¹nh nµo ®ã, nã më réng ta cã ( p − 2) ( u − v ) ≡ 0(mod p ) . L¹i cã
cho Bµi to¸n HSG1. ( p; p − 2 ) = 1 , suy ra u ≡ v(mod p) .
Còng tõ (*) ta cã
Bµi to¸n B. Cho sè nguyªn tè lÎ p vµ ®a
thøc Q ( x) = ( p − 1) x p − x − 1 . Chøng minh ( )
( u − v ) ( p − 1) ( u p −1 + u p −2 v + ... + v p −1 ) − 1 ≡
r»ng víi mçi sè nguyªn d−¬ng n, tån t¹i v« ≡ 0(mod p ) . MÆt kh¸c u ≡ v(mod p ) ⇒
p

⇒ ( p − 1) ( u p −1 + u p − 2 v + ... + v p −1 ) − 1 ≡
h¹n sè nguyªn d−¬ng an mµ Q(an) chia hÕt
cho pn.
Víi lêi gi¶i còng gièng nh− c¸ch chøng ≡ ( p − 1). p − 1 ≅ 0(mod p p ) .
minh Bµi to¸n HSG1. Suy ra u ≡ v(mod p p ) .
§Æc biÖt tr−êng hîp n = p, ta cã bµi to¸n
riªng nh−ng d−êng nh− l¹i “khã” h¬n v× víi Chó ý lµ 0 < u – v < pp ⇒ u = v. NhËn
bµi to¸n míi nµy, chóng ta sÏ kh«ng dÔ dµng xÐt ®−îc chøng minh.
nghÜ ngay tíi ph−¬ng ph¸p quy n¹p ®Ó chøng HÖ qu¶ lµ trong pp sè nguyªn d−¬ng liªn
minh. tiÕp, tån t¹i duy nhÊt mét sè a ®Ó Q(a) chia
hÕt cho pp. Vµ do ®ã hiÓn nhiªn lµ trong tËp
Bµi to¸n C. Cho sè nguyªn tè lÎ p vµ ®a hîp v« h¹n c¸c sè nguyªn d−¬ng, tån t¹i v«
sè sè a mµ Q(a) chia hÕt cho pp. Bµi to¸n
thøc Q ( x) = ( p − 1) x p − x − 1 . Chøng minh
®−îc chøng minh.

TAÄP SAN TOAÙN HOÏC - 2007 13


TRÖÔØNG THPT CHUYEÂN HOAØNG VAÊN THUÏ - HOØA BÌNH
PhÇn I - S¸ng t¹o to¸n häc.

Trong lêi gi¶i Bµi to¸n B ta chØ ra ®−îc sù ⇒ x cã d¹ng 3k + 1 ⇒ P(x) = P(3k + 1) =
tån t¹i cña an nh−ng ®H kh«ng chØ ra ®−îc cã = 33(k3 + 52k2 22k +3).
bao nhiªu sè nh− vËy. C¸i thó vÞ ë c¸ch gi¶i * NÕu k = 3m + 2 ⇒
Bµi to¸n C kh«ng chØ lµ ë sù míi l¹ trong P(x) = 33(27m3 + 495m2 387m + 263)
c¸ch t− duy mµ cßn kh¾c phôc ®−îc ®iÓm kh«ng chia hÕt cho 34 víi mäi m.
h¹n chÕ cña ph−¬ng ph¸p tr−íc ®ã. Kh¸ bÊt
* NÕu k = 3m + 1 ⇒
ngê víi lêi gi¶i trªn, t«i chît nhí ®Õn bµi
P(x) = 34 (9m3 + 165m2 129m + 26)
to¸n HSG2 mµ m×nh ch−a gi¶i ®−îc. §em ¸p
kh«ng chia hÕt cho 35 víi mäi m.
dông ph−¬ng ph¸p míi nµy cho bµi to¸n ®ã
vµ t«i ®H thµnh c«ng! * NÕu k = 3m ⇒
Nh−ng t«i l¹i ®i tõ bµi to¸n …tæng qu¸t: P(x) = 34(9m3 + 156m2 + 22m + 1).
Ta thÊy ®a thøc Q(m) = (9m3 + 156m2 +
Bµi to¸n D. XÐt tËp hîp c¸c ®a thøc cã d¹ng 22m + 1) ∈T T vµ 1 ≤ x ≤ 32000 ⇔ 0 ≤ m ≤
T = { P(x) = ax3 + bx2 + cx + d / a ≠ 0, 31998 – 1. VËy P(x) chia hÕt cho 32000 ⇔ x =
b ≡ 0(mod3), c ≡ 0(mod3), a + c ≅ 9m + 1 vµ Q(m) chia hÕt cho 31996.
0(mod3)}. Khi ®ã gi¸ trÞ t¹i 3n ®iÓm nguyªn Theo hÖ qu¶ cña Bµi to¸n D suy ra: Trong
d−¬ng liªn tiÕp cña ®a thøc P(x) ∈ T lËp 9.31996 sè nguyªn liªn tiÕp 0, 1, 2, …,
thµnh mét hÖ thÆng d− ®Çy ®ñ (mod3n). 31998 − 1 tån t¹i ®óng 9 sè nguyªn a mµ Q(a)
chia hÕt cho 31996 ⇔ Trong ®o¹n [1; 32000]
Chøng minh. Trong 3n sè nguyªn d−¬ng tån t¹i ®óng 9 sè nguyªn a mµ P(a) chia hÕt
liªn tiÕp gi¶ sö cã u > v mµ Q(u) ≡ Q(v) cho 32000.
(mod3n) ⇔ au3 + bu2 + cu + d ≡ av3 + bv2 Ta còng dÔ dµng nhËn ra lµ trong ®o¹n
+ cv + d (mod3n) ⇔ a(u3 - v3)+ b(u2 - v2) +
[1;3n ] (n ≥ 1998) tån t¹i 3n – 1998 sè nguyªn a
+ c(u – v) ≡ 0(mod3n) (*).
mµ P(a) chia hÕt cho 32000. Bµi to¸n HSG2 ®H
Ta cã b≡0(mod3), u3≡u(mod3), v3≡ ®−îc gi¶i quyÕt trän vÑn!
v(mod3) nªn tõ (*) ⇒ a(u3 - v3)+ b(u2 - v2)
+ c(u – v) ≡ 0(mod3) III/ Lêi kÕt.
⇒ (a + c)(u - v) ≡ 0 (mod3) ⇒ u ≡ v(mod3), ChØ mét chót thay ®æi ®Ò bµi theo ý
do (a + c) ≅ 0(mod3). t−ëng cña m×nh c¸c b¹n cã thÓ t¹o ra ®−îc
Còng tõ (*) ta cã (u – v)[a(u2 + uv + v2) nh÷ng bµi to¸n míi còng kh¸ “hãc bóa” ®Êy
+ b(u + v) + c] ≡ 0(mod3n). chø! VÊn ®Ò ®Æt ra ë ®©y lµ trong tr−êng hîp
Mµ u ≡ v(mod3), c ≅ 0(mod3) ⇒ a(u2 + tæng qu¸t, ®a thøc P(x) bËc n th× kÕt qu¶ sÏ
ra sao? B¶n th©n t«i còng ch−a cã ®iÒu kiÖn
+ uv + v2) + b(u + v) + c ≅ 0(mod3) ⇒ u ≡
®Ó t×m hiÓu thªm, mong c¸c b¹n cïng quan
v (mod3n). VËy u = v ⇒ ®pcm. t©m coi nh− mét bµi tËp tr−íc khi kÕt thóc
bµi viÕt nµy.
HÖ qu¶ lµ: Trong 3n sè nguyªn d−¬ng liªn
tiÕp tån t¹i duy nhÊt mét sè a ®Ó Q(a) chia Nh− vËy ®Êy c¸c b¹n ¹, tõ mét sù t×nh cê
hÕt cho 3n. t«i ®H gi¶i ®−îc mét bµi to¸n khã vµ t×m ra
§©y chÝnh lµ sù tæng qu¸t cho Bµi to¸n ®−îc nhiÒu ®iÒu thó vÞ. Nh−ng ®Ó cã ®−îc sù
HSG2. Cô thÓ, lêi gi¶i cña bµi to¸n HSG2 “t×nh cê” ®ã lµ c¶ mét qu¸ tr×nh nç lùc
nh− sau: kh«ng ngõng vµ mét tr¸i tim ®am mª To¸n
häc mHnh liÖt. Cuèi cïng xin chóc c¸c b¹n
Lêi gi¶i Bµi to¸n HSG2. Ta cã P(x) = x3 thµnh c«ng vµ t×m ra ®−îc nhiÒu “c«ng
+153x2 -111x +38 ∉ T. Gi¶ sö P(x) chia tr×nh” cho riªng m×nh trong qu¸ tr×nh häc tËp
hÕt cho 32000 ⇒ P(x) ph¶i chia hÕt cho 3 vµ v−¬n lªn ë tÊt c¶ c¸c lÜnh vùc!

TAÄP SAN TOAÙN HOÏC - 2007 14


TRÖÔØNG THPT CHUYEÂN HOAØNG VAÊN THUÏ - HOØA BÌNH
PhÇn I - S¸ng t¹o to¸n häc.

SÖÛ DUÏNG TÍNH CHAÁT HAØM ÑÔN AÙNH


ÑEÅ GIAÛI BAØI TOAÙN PHÖÔNG TRÌNH HAØM
NguyÔn th¸I ngäc
Chuyªn to¸n k99-02
Sv. Líp §T8 – K48, Khoa §iÖn tö ViÔn th«ng - §H B¸ch Khoa Hµ Néi

HÖ qu¶. Cho f lµ mét ®¬n ¸nh , x¸c ®Þnh vµ


Trong c¸c k× thi Häc sinh giái, ta th−êng
liªn tôc trªn kho¶ng (a,b) vµ c¸c sè a', b' ∈
gÆp c¸c bµi to¸n vÒ gi¶i ph−¬ng tr×nh hµm.
(a,b) víi a' < b'. Khi ®ã:
§©y lµ d¹ng to¸n kh¸ quen thuéc víi c¸c
i) NÕu f(a') < f(b') th× f t¨ng ngÆt trªn [a,b]
b¹n. Trong cuèn "Ph−¬ng tr×nh hµm" cña GS
tøc lµ f(u) < f(v) nÕu a' < u < v < b'.
- TS.NguyÔn V¨n MËu, t¸c gi¶ ®H ®Ò cËp
t−¬ng ®èi s©u vÒ mét líp ph−¬ng tr×nh hµm. ii) NÕu f(a') > f(b') th× gi¶m ngÆt trªn [a',b']
Trong ph¹m vi bµi viÕt nµy, t«i xin ®−îc nªu nghÜa lµ f(u) > f(v) nÕu a' < u < v < b.
ra mét ph−¬ng ph¸p ®Ó gi¶i d¹ng to¸n nãi Ta sÏ chøng minh ®iÒu kh¼ng ®Þnh thø
trªn kh¸ hiÖu qu¶. §ã lµ ph−¬ng ph¸p sö nhÊt. V× f(a') < f(b') nªn nÕu a' < u < b' th×
dông tÝnh chÊt hµm ®¬n ¸nh. Tr−íc hÕt, t«i theo NhËn xÐt 1 ta cã: f(a') < f(u) < f(v).
xin nªu ®Þnh nghÜa vµ mét sè nhËn xÐt xoay V× f(a') < f(u) nªn nÕu a' < u < v ⇒ f(a')
quanh hµm ®¬n ¸nh: < f(u) < f(v) < f(b').
§Þnh nghÜa hµm ®¬n ¸nh. T−¬ng tù cho kh¼ng ®Þnh 2.
Hµm sè f : X→ Y NhËn xÐt 2.
x→ y = f(x) §iÒu kiÖn ¾t cã vµ ®ñ ®Ó mét hµm sè x¸c
®Þnh liªn, tôc trªn kho¶ng (a,b) ®¬n ¸nh lµ
®−îc gäi lµ mét hµm ®¬n ¸nh nÕu ∀x1, x2
hµm sè f(x) ®¬n ®iÖu ngÆt trªn kho¶ng ®ã.
thuéc X mµ x1 ≠ x2 suy ra f(x1)≠ f(x2) Chøng minh.
NhËn xÐt 1. a) NÕu f ®¬n ®iÖu ngÆt th× f ®¬n ¸nh:
Cho f lµ mét hµm sè x¸c ®Þnh, liªn tôc
Cho u ≠ v. Khi ®ã hoÆc u > v hoÆc u < v,
trong kho¶ng (a,b), khi ®ã, nÕu c¸c sè u, v
do vËy f(u) < f(v) hoÆc f(u) > f(v) nghÜa lµ
thuéc (a,b) sao cho u < v vµ f(u) < f(v) th×
víi bÊt kú w thuéc (u,v) lu«n cã: f(u) ≠ f(v).
f(u)<f(w)<f(v). b) NÕu f ®¬n ¸nh th× f ®¬n ®iÖu ngÆt: Víi a'
Chøng minh. §Ó chøng minh nhËn xÐt trªn < b' ∈ (a,b). Khi ®ã hoÆc f(a') < f(b') hoÆc
ta thõa nhËn ®Þnh lý vÒ gi¸ trÞ trung gian sau: f(a') > f(b') do vËy ta sÏ chøng minh:
Cho f(x) lµ hµm sè x¸c ®Þnh, liªn tôc HoÆc (i): NÕu f(a') <f(b') th× f t¨ng ngÆt.
trong ®o¹n [a,b], khi ®ã f(x) lÊy tÊt c¶ c¸c HoÆc (ii): NÕu f(a') >f(b') th× f gi¶m ngÆt.
gi¸ trÞ tõ f(a) ®Õn f(b). XÐt (i), cho u < v; u, v ∈ (a,b) ®Æt w =
Trë l¹i nhËn xÐt trªn, ta sÏ chøng minh min {a';u }; z=max{b';v} khi ®ã a', b', u, v
b»ng ph¶n chøng. ThËt vËy, nÕu f(u) < f(v) ®Òu thuéc ®o¹n [w, z] .
<f(w) (hoÆc f(w) < f(u) < f(v)) th× tõ gi¶ thiÕt Theo hÖ qu¶ cña NhËn xÐt 1,v× f ®¬n ¸nh
liªn tôc cña f vµ ®Þnh lÝ vÒ gi¸ trÞ trung gian nªn t¨ng ngÆt trªn [w,z]. V× u ≠ v, u, v ∈
cña mét hµm sè liªn tôc suy ra f(v) lµ gi¸ trÞ [w,z] nªn f(u) < f(v) vµ u, v lµ hai ®iÓm bÊt
trung gian cña f(u) vµ f(w). Do ®ã sÏ tån t¹i k× (u < v) trªn (a,b) nªn f t¨ng ngÆt trªn
v' thuéc [u;w] sao cho f(v') = f(u) vµ v' ≤ w (a,b).
< v do vËy v' ≠ v. §iÒu nµy m©u thuÉn víi Muèn chøng minh (ii) chØ cÇn thay f bëi
gi¶ thiÕt ®¬n ¸nh cña f. NhËn xÐt ®−îc chøng − f vµ lËp luËn t−¬ng tù .
minh .

TAÄP SAN TOAÙN HOÏC - 2007 15


TRÖÔØNG THPT CHUYEÂN HOAØNG VAÊN THUÏ - HOØA BÌNH
PhÇn I - S¸ng t¹o to¸n häc.

Nh− vËy lµ chóng ta ®H cã mét sè nhËn NÕu x0 = ± 1 tøc lµ f(1) = f(-1) = f(0) th×
xÐt vµ hÖ qu¶ kh¸ hay vÒ hµm ®¬n ¸nh. Sau ta cã f(2) = 2.f(1) = 0.
®©y xin ®−îc ®i vµo mét sè bµi to¸n cô thÓ: VËy 2 lµ nghiÖm cña f, tr¸i víi ®iÒu trªn.
Bµi to¸n 1. T×m tÊt c¶ c¸c hµm sè liªn tôc VËy x = 0 lµ nghiÖm duy nhÊt cña hµm
f : R → R tho¶ mLn ®iÒu kiÖn f(x).
f(x.f(y)) =y. f(x) , ∀x,y ∈ R. Cã f(x4+y) = x3.f(x) + f(y) = f(x4) + f(y)
Lêi gi¶i. Cho x = y = 0 ⇒ f(0) = 0. DÔ thÊy NÕu x ≥ 0 ⇒ f(x+y) = f(x) + f(y)
f(x) ≡ 0 lµ mét nghiÖm cña ph−¬ng tr×nh NÕu x < 0 ⇒ f(x+y) = f(-x-y) = -f(x-y)
hµm. = -(f(-x) + f(-y)) = f(x) + f(y).
XÐt f(x) ≠ 0. Cho x = y = 1 ⇒ f(f(1)) =f(1) . VËy f(x+y) = f(x) + f(y),∀x, y ∈ R.
Suy ra: f(x.f(f(1))) =f(1).f(x) =f(x.f(1))=f(x). Gi¶ sö ∃ x1 ≠ x2 mµ f(x1) = f(x2)
VËy f(1) =1. Gi¶ sö tån t¹i x1 ≠ x2 mµ f(x1) ⇒ f(x1+y) = f(x2+y),∀y∈ R.
= f(x2) . Ta cã f(x.f(x1)) = x1.f(x), ∀x∈R vµ ⇔ f(x1- x2) = 0. Hay f nhËn x1-x2 ≠ 0 lµm
(x.f(x2)) = x2. f(x), ∀x∈R. nghiÖm (V« lÝ).
⇒ x1. f(x) = x2. f(x) , ∀x∈R VËy f lµ ®¬n ¸nh .
⇒ x1 = x2 (v× f(x) ≠ 0 ), m©u thuÉn. Do f(f(y))=f(y)⇒ f(y) = y, ∀y∈R.
VËy f lµ ®¬n ¸nh vµ do f liªn tôc nªn theo Thö l¹i thÊy f(y) ≡ y lµ nghiÖm duy nhÊt
NhËn xÐt 2 suy ra f ®¬n ®iÖu ngÆt . cña ph−¬ng tr×nh hµm.
Cã f(1) >f(0) vËy f t¨ng ngÆt (HÖ qu¶ cña Bµi to¸n 3. T×m tÊt c¶ c¸c hµm f :N*→N*
NhËn xÐt 1) tháa mLn
Cã f(f(x.f(y))) =f(y.f(x)) = x.f(y), ∀x∈R f(m+f(n))= n+ f(m+ 2003),∀m,n∈N*
NÕu f(x.f(y)) > x.f(y) Lêi gi¶i. Gi¶ sö ∃ n1 ≠ n2 mµ f(n1) = f(n2)
⇔x.f(y) = f(f(x.f(y))) > f(x.f(y)) > x.f(y), v« lÝ ⇒ f(f(n1)+m) = n1 +f(m+2003), ∀m,∈N* vµ
NÕu f(x.f(y))< x.f(y) f(f(n2)+m)= n2 +f(m+2003), ∀m,∈N*, V« lÝ.
⇔ x.f(y) = f(f(x.f(y))) < f(x.f(y)) < x.f(y), v« lÝ VËy f lµ ®¬n ¸nh.
VËy f(x.f(y)) = x.f(y) Ta cã: f(f(1)+f(n)) = n + f(f(1)+2003) =
Thay x =1⇒ f(1) =x,∀x n+1 + f(2003+2003) = f(f(n+1)+2003).
Thö l¹i thÊy f(x) ≡ 0, f(1) ≡ x lµ hai Tõ ®ã f(f(1)+f(n)) = f(f(n+1)+ 2003).
nghiÖm cña ph−¬ng tr×nh hµm Do f lµ ®¬n ¸nh nªn
Bµi to¸n 2. T×m tÊt c¶ c¸c hµm f(x) x¸c ®Þnh f(1) + f(n) = f(n+1) + 2003.
trªn R cã h÷u han nghiÖm tho¶ mLn: B»ng quy n¹p ta suy ra: f(n) = an +b.
f(x4+y) = x3.f(x)+f(f(y)), ∀x,y∈ R. Thay vµo ®iÒu kiÖn cña bµi ta x¸c ®Þnh
(APMO- 2002) ®−îc: a=1, b=2003.
Lêi gi¶i. Cho x = 0 ⇒ f(f(y))=f(y), ∀y∈R VËy f(n)=n+2003, ∀n∈ N*.
⇒ f(x4+y) = x3.f(x) + f(y), ∀x, y ∈ R. Cuèi cïng xin nªu mét sè bµi to¸n mµ ta
cã thÓ sö dông tÝnh chÊt hµm ®¬n ¸nh ®Ó gi¶i
⇒ f(x4+y) = -x3.f(-x) + f(y), ∀x, y ∈ R
quyÕt . Chóc c¸c b¹n thµnh c«ng !
⇒ f(0) = 0 1. T×m tÊt c¶ c¸c hµm sè f :Q→Q tho¶
Cho y = 0 ⇒ f(x4) = x3.f(x), ∀x∈R mLn :f(f(x)+y)=x+ f(y),∀x,y∈Q.
NÕu ∃x0 ≠ 0 sao cho f(x0) = 0 2. T×m tÊt c¶ c¸c hµm sè f: R→R tho¶
⇒ f(x 04 ) = 0. NÕu x0≠ ±1 th× tån t¹i dHy: x1 mLn
= x0 , xn=x 4n−1 ,∀n = 2, 3, .... f(y-f(x))=f(x2002-y)-2001.y.f(x), ∀x,y∈R.
§©y lµ dHy v« sè sè h¹ng kh¸c nhau mµ (Chän häc sinh giái quèc gia 2001-2002).
f(x) nhËn lµm nghiÖm. Tr¸i gi¶ thiÕt.

TAÄP SAN TOAÙN HOÏC - 2007 16


TRÖÔØNG THPT CHUYEÂN HOAØNG VAÊN THUÏ - HOØA BÌNH
PhÇn I - S¸ng t¹o to¸n häc.

LÔØI GIAÛI CAÙC BAØI THI TOAÙN QUOÁC TEÁ 2003


Hµ H÷u Cao Tr×nh
Chuyªn to¸n K99 – 02
Líp K6 CNKHTN To¸n, §HKHTN - §HQG Hµ Néi

Lêi gi¶i. Gi¶ sö cÆp (a; b) tháa mHn bµi


T«i còng xin chia sÎ mét sè kinh nghiÖm
a2
trong gi¶i to¸n víi c¸c b¹n qua lêi gi¶i ®Ò thi to¸n. Do k = > 0 nªn 2ab2 -
to¸n quèc tÕ 2003. Hi väng r»ng qua ®©y c¸c 2ab − b + 1
2 3

b¹n sÏ rót ra cho m×nh nhiÒu ®iÒu bæ Ých. b


b3+1 > 0 vµ a ≥ .
Bµi sè 1. LÊy A lµ mét tËp con 101 phÇn tö 2
cña tËp S gåm c¸c sè tù nhiªn tõ 1 ®Õn MÆt kh¸c, ta cã a2 ≥ b2(2a - b) +1 > 0
1000000. Chøng minh r»ng: tån t¹i c¸c sè nªn hoÆc a > b hoÆc 2a = b (*)
t1, t2,..., t100 thuéc S sao cho c¸c tËp XÐt hai nghiÖm (a1, a2) cña ph−¬ng tr×nh
Ai = {x + t i / x ∈ A} , i = 1, 2,..., 100 ®«i a - 2kb2a + k(b3-1) = 0, víi gi¶ sö a1 ≥ a2
2

mét kh«ng giao nhau. .Theo ®Þnh lý Vi- et, ta cã a1+a2 = 2kb2 ⇒ a1
≥ kb2 > 0.
Lêi gi¶i. XÐt tËp D = {x − y / x, y ∈ A}, ta
H¬n n÷a, tõ a1a2 = k(b3-1), ta cã 0 ≤ a2 =
thÊy D cã nhiÒu nhÊt 101.100 +1 phÇn tö,
trong ®ã ch¾c ch¾n chøa phÇn tö 0. k (b 3 − 1) k (b 3 − 1)
≤ < b. KÕt hîp víi
NhËn xÐt: Ai, Aj cã giao kh¸c rçng khi vµ chØ a1 kb 2
khi ti - tj ∈ D (*), nªn ta chØ cÇn chän 100 b
phÇn tö kh«ng tháa mHn (*). Ta chän b»ng (*) ⇒ a2 = 0 hoÆc a2 = 2 .
2
quy n¹p: - NÕu a2 = 0 th× b - 1 = 0 ⇒ a1 = 2k, b =1.
3
ViÖc chän 1 phÇn tö lµ tÇm th−êng. Gi¶
sö chän ®−îc k phÇn tö, k ≤ 99 kh«ng vi b2 b2 b2 b
- NÕu a2 = th× k = vµ a1 = - .
ph¹m (*). Nh− vËy, ta vÉn cßn Ýt nhÊt 106 - 2 4 2 2
10101.k ≥ 1 sù lùa chän n÷a cho phÇn tö thø Nh− vËy, ta ®H t×m ®−îc (a;b) d−íi d¹ng
k+1. Tãm l¹i, ta cã thÓ chän ®−îc 100 sè t1, (2t; 1) hoÆc (t; 2t) hoÆc (8t4- t; 2t), t ∈ N*.
t2,..., t100 kh«ng vi ph¹m (*). Theo nguyªn lý Thö l¹i ®Òu thÊy tháa mHn.
quy n¹p, bµi to¸n ®−îc chøng minh. Chó ý: Cã thÓ suy ®−îc (*) b»ng c¸ch sau:
* Chó ý: Ph¸t biÓu tæng qu¸t sau vÉn ®óng XÐt hµm f(b) = 2ab2- b3+1, hµm nµy t¨ng
nhê phÐp chøng minh t−¬ng tù.  4a   4a 
“NÕu A lµ tËp con k ph©n tö cña tËp S gåm trªn 0;  , gi¶m trªn  ;+∞  vµ ta cã:
 3 3 
c¸c sè tù nhiªn tõ 1 ®Õn n vµ m lµ sè nguyªn f(a) = a + 1 > a , f(2a-1) = 4a - 4a+2 > a2,
3 2 2

d−¬ng tháa mLn: n > (m -1)( C k2 + 1), th× tån f(2a+1) = - 4a2- 4a < 0.
t¹i m sè t1, t2,..., tm thuéc S tháa mLn c¸c tËp a2
Ai x¸c ®Þnh nh− trªn ®«i mét kh«ng giao ⇒ NÕu b ≥ a vµ nguyªn d−¬ng th× b =
f (b)
nhau”.
Bµi sè 2. T×m tÊt c¶ c¸c cÆp sè nguyªn 4a
2a. ThËt vËy, nÕu a ≤ b ≤ th× f(b) ≥ f(a)
a2 3
d−¬ng (a; b) tháa mLn lµ mét a2 4a
2ab 2 − b 3 + 1 > a2 ⇒ < 1, v« lý. Cßn nÕu b >
sè nguyªn d−¬ng. f (b) 3
th× :

TAÄP SAN TOAÙN HOÏC - 2007 17


TRÖÔØNG THPT CHUYEÂN HOAØNG VAÊN THUÏ - HOØA BÌNH
PhÇn I - S¸ng t¹o to¸n häc.

i) NÕu b > 2a + 1 th× f(b) < f(2a+1) < 0, cã thÓ gi¶ sö mét trong hai gãc cßn l¹i t¹o
v« lý. bëi c¸c ®−êng chÐo chÝnh ≥ 600 vµ chøng
ii) NÕu b ≤ 2a-1 th× f(b) ≥ f(2a-1) > a2, v« minh t−¬ng tù, ta cã ngay ABCDEF cã tÊt c¶
lý. c¸c gãc b»ng nhau.
Bµi 3. Mçi cÆp c¹nh ®èi diÖn cña lôc gi¸c C¸ch 2. Sö dông vect¬ vµ nhËn xÐt ë c¸ch 1,
låi cã tÝnh chÊt sau: kho¶ng c¸ch c¸c b¹n hHy tù chøng minh (!).
3 Bµi 4. Cho ABCD lµ mét tø gi¸c néi tiÕp.
trung ®iÓm cña chóng gÊp lÇn tæng ®é Gäi P, Q, R lµ c¸c ch©n ®−êng vu«ng gãc h¹
2
tõ D xuèng BC, CA, AB. Chøng minh:
dµi cña chóng.
PQ=QR khi vµ chØ khi ph©n gi¸c cña
Chøng minh r»ng tÊt c¶ c¸c gãc cña lôc
∠ABC , ∠ADC vµ ®−êng chÐo AC ®ång quy.
gi¸c b»ng nhau.
Lêi gi¶i. Lêi gi¶i.
C¸ch 1. Ta sÏ chøng minh nhËn xÐt sau: NÕu C¸ch 1. Ta biÕt P, Q, R th¼ng hµng (®−êng
th¼ng Sims¬n). DÔ dµng cã ®−îc :
∠QPR ≥ 60 0 vµ L lµ trung ®iÓm cña QR th×
∆DCA ~ ∆DAR
3 ∆DAB ~ ∆DPQ
PL ≤ QR. DÊu b»ng x¶y ra khi vµ chØ
2 ∆DBC ~ ∆DRQ
khi PQR lµ tam gi¸c ®Òu. QR
Chøng minh nhËn xÐt. LÊy S sao cho QRS DA DR
DB.
⇒ = = BC = QR BA nªn PQ
lµ tam gi¸c ®Òu vµ P n»m trong phÇn giao
DC DP PQ PQ BC
cña nöa mÆt ph¼ng bê QR chøa S víi phÇn DB.
h×nh trßn ngo¹i tiÕp QRS.VËy P n»m trong BA
c¶ (L, LS) DA BA
= QR ⇔ = (1)
3 DC BC
⇒ PL < LS = QR (®pcm) Theo tÝnh chÊt cña ®−êng ph©n gi¸c, (1)
2 x¶y ra khi vµ chØ khi ph©n gi¸c
∠ABC , ∠ADC chia AC theo cïng mét tû sè,
tøc lµ chóng ®ång quy, (®pcm).

Trë l¹i bµi to¸n cña ta, gäi lôc gi¸c låi lµ
ABCDEF. XÐt mét ®−êng nèi trung ®iÓm N
cña DE víi trung ®iÓm M cña AB , AE ∩ BD
= P. Kh«ng mÊt tæng qu¸t gi¶ sö
∠APB ≥ 60 0 . V× tæng 3 gãc t¹o bëi c¸c
®−êng chÐo chÝnh liªn tiÕp b»ng 1800. Theo C¸ch 2. Gi¶ sö ph©n gi¸c ∠ABC , ∠ADC lÇn
nhËn xÐt trªn, ta cã : LA BA
l−ît c¾t AC ë L vµ M. Tõ = vµ
3 LC BC
MN = (AB+DE) ≥ PM+PN ≥ MN
2 MA DA
= ⇒ L≡M khi vµ chØ khi
DÊu b»ng buéc x¶y ra theo gi¶ MC DC
thiÕt ⇒ ABP lµ tam gi¸c ®Òu. §Õn lóc ®ã, ta

TAÄP SAN TOAÙN HOÏC - 2007 18


TRÖÔØNG THPT CHUYEÂN HOAØNG VAÊN THUÏ - HOØA BÌNH
PhÇn I - S¸ng t¹o to¸n häc.

BA DA 2
 n  2(n 2 − 1) n
BC DC
= ⇔ AB.CD = CB.AD (1)  ∑ xi − x j  ≤
  ∑ (xi − x j )2
 i , j =1  3
§Æt α = ∠ACB , γ = ∠CAB , ta cã: c¸c tø i , j =1

b) DÊu b»ng x¶y ra nÕu xi = k(2i-n-1) víi


gi¸c PDQC vµ APQR néi tiÕp nªn ∠PDQ
k nµo ®ã, nghÜa lµ x1, x2, ..., xn lµ mét cÊp sè
hoÆc b»ng α hoÆc b»ng 1800 - α . ∠QDR céng. MÆt kh¸c, gi¶ sö x1, x2, ..., xn lµ mét
hoÆc b»ng γ hoÆc b»ng 1800 - γ . Theo ®Ýnh cÊp sè céng víi c«ng sai d. Ta cã:
lý hµm sè sin ta cã: d x + xn
PQ = CDsin α , QR = ADsin γ xi = (2i − n − 1) + 1 . Gi¶m tÊt c¶
2 2
sin α DA x + xn
⇒ PQ = QR ⇔ = ®i mét l−îng 1 , ta thu ®−îc
sin γ DC 2
sin α BC n
MÆt kh¸c = nªn ta cã lu«n d
xi = (2i − n − 1) vµ ∑ xi = 0. Tõ ®ã, ta
sin γ BA 2 i =1
PQ = QR ⇔ AB.CD = CB. AD (2) cã ®¼ng thøc.
Tõ (1) vµ (2) suy ra ®pcm. Bµi 6. Cho p lµ mét sè nguyªn tè. Chøng
Bµi 5. Cho n lµ sè nguyªn d−¬ng vµ minh r»ng: tån t¹i sè nguyªn tè q tháa mLn
x1 ≤ x2 ≤ ... ≤ xn lµ c¸c sè thùc. víi mäi sè nguyªn n, sè np - p kh«ng chia hÕt
a) Chøng minh r»ng: cho q.
2
 n  2(n 2 − 1) n p p −1
 ∑ xi − x j  ≤
  ∑ (xi − x j )2 Lêi gi¶i. Ta cã:
p −1
= 1 + p + ... + p p −1 = -
 i , j =1  3 i , j =1

b) Chøng minh r»ng dÊu ®¼ng thøc x¶y ra ≡ p + 1(mod p 2 ) . Do ®ã ta cã thÓ lÊy Ýt nhÊt
khi vµ chØ khi x1,x2,...,xn lµ mét cÊp sè céng. p p −1
Lêi gi¶i. mét −íc nguyªn tè cña sao cho nã
a) Kh«ng mÊt tæng qu¸t, ta gi¶ sö p −1
n kh«ng ®ång d− víi 1(mod p ) . §Æt −íc nµy lµ
∑x
i =1
i =0 (do 2 vÕ cña bÊt ®¼ng thøc (B§T) q vµ ta chøng minh ®ã lµ sè cÇn t×m.
ThËt vËy, gi¶ sö tån t¹i sè tù nhiªn n sao
chØ phô thuéc (xi - xj)). Ta cã :
cho n p ≡ p (mod q ) .
− x j = 2∑ (xi − x j )
n

∑x
i , j =1
i
i< j
Ta cã n p ≡ p p ≡ 1(mod q) (theo ®Þnh
2

n nghÜa cña q). MÆt kh¸c, tõ ®Þnh lý nhá


= 2∑ xi (2i − n − 1) . Sö dông B§T Cauchy - Fermat n q −1 ≡ 1(mod q ) .Tõ p 2 / q − 1 , ta cã :
i =1
Schwarz, ta cã: ( p 2 , q − 1) = q . Tõ ®ã dÉn ®Õn
2 n p ≡ 1(mod q ) , nªn ta cã p ≡ 1(mod q ) . H¬n
 n  n n
 ∑ xi − x j  ≤ 4∑ (2i − n − 1) 2 ∑ xi2
  thÕ, 1+p+p2+...+pp -1≡ p (mod q), theo c¸ch
 i , j =1  i =1 i =1

n(n + 1)(n − 1) n lÊy q ta cã p≡0 (mod q), v« lý.


=4
3
∑ xi2 VËy ta cã ®pcm.
i =1 Cuèi cïng, rÊt mong c¸c b¹n sÏ lu«n say
MÆt kh¸c, ta cã: mª, t×m tßi ®Ó häc tèt bé m«n to¸n. C¸c b¹n
∑ (x − x j ) = n ∑ xi2 − ∑ xi
n n n n
hHy quyÕt t©m thËt cao vµ hHy tin r»ng: “ChØ
i , j =1
i
2

i =1 i =1
∑x
j =1
j +
biÕt r»ng cuèi cïng chóng t«i sÏ th¾ng” !
n n
+ n∑ x 2j = 2n∑ xi2 . Do vËy nªn:
j =1 i =1

TAÄP SAN TOAÙN HOÏC - 2007 19


TRÖÔØNG THPT CHUYEÂN HOAØNG VAÊN THUÏ - HOØA BÌNH
PhÇn I - S¸ng t¹o to¸n häc.

SOÁ PHÖÙC VÔÙI HÌNH HOÏC PHAÚNG


Vò H÷u Ph−¬ng
Chuyªn To¸n K00 – 03
Sv. Líp §T8 – K48, Khoa §iÖn tö ViÔn th«ng - §H B¸ch Khoa Hµ Néi

C¸c b¹n th©n mÕn, ®Ó gi¶i c¸c bµi to¸n a + ib ac + bd bc − ad


4. = 2 +i 2 , ë ®©y c2+d2
h×nh häc ph¼ng, chóng ta cã kh¸ nhiÒu c + id c + d 2
c +d 2

ph−¬ng ph¸p nh− sö dông vector, täa ®é, c¸c > 0.


phÐp biÕn h×nh.. nh−ng cã lÏ sè phøc lµ c«ng III. D¹ng h×nh häc, l−îng gi¸c cña sè
cô mµ nhiÒu b¹n cßn ch−a hoÆc Ýt sö dông v× phøc. Nh·n.
tÝnh míi l¹ cña nã. Bµi viÕt nµy xin ®−îc trao 1. D¹ng h×nh häc.
®æi víi c¸c b¹n mét sè kinh nghiÖm mµ t«i Trong mÆt ph¼ng täa ®é Oxy, nÕu ®iÓm M
cã vÒ sè phøc. cã täa ®é M(a;b) th× ng−êi ta biÓu diÔn nã
I. §Þnh nghÜa sè phøc. bëi sè phøc z = a +ib, gäi lµ nhHn cña ®iÓm
Tr−íc hÕt, c¸c b¹n hHy lµm quen víi ®¬n M.
vÞ ¶o mµ ng−êi ta kÝ hiÖu lµ i ®−îc x¸c ®Þnh
bëi ®¼ng thøc: i2 = -1 (®iÒu nµy cho phÐp
khai c¨n bÊt k× sè thùc nµo).
Víi ®¬n vÞ ¶o, ng−êi ta thiÕt lËp c¸c biÓu
thøc d¹ng: z = a +ib; a, b ∈ R (1).
1. §Þnh nghÜa.
Mçi biÓu thøc z = a +ib; a, b ∈ R ®−îc
gäi lµ mét sè phøc.
2. Mét sè kh¸i niÖm liªn quan.
Trong biÓu thøc (1), a ®−îc gäi lµ phÇn Tãm l¹i: ðiÓm M(a;b) cã nhHn lµ z = a +
thùc, b ®−îc gäi lµ phÇn ¶o cña sè phøc z. 
ib. Ng−êi ta còng nãi vector OM cã nhHn
§Ó gän h¬n, ng−êi ta kÝ hiÖu:
lµ z = a +ib.
a = im(z) = I(z); b = re(z) = R(z).
2. D¹ng l−îng gi¸c.
BiÓu thøc (1) ®−îc gäi lµ biÓu diÔn ®¹i sè
VÉn víi ®iÓm M trªn (kh¸c gèc täa ®é), ta
cña sè phøc vµ c¸c b¹n chó ý r»ng khii b = 0
xÐt gãc ®Þnh h−íng ϕ = (Ox, OM ) . §Æt r
th× z = a lµ mét sè thùc, khi a = 0, b ≠ 0 th×
ta gäi z = ib lµ sè thuÇn ¶o. a b
= a 2 + b 2 ta cã cos ϕ = , sin ϕ = , do ®ã
Sè phøc z = a - ib ®−îc gäi lµ sè phøc r r
liªn hîp cña sè phøc z. z = r ( cos ϕ + i sin ϕ ) (2)
TËp hîp c¸c sè phøc z ®−îc kÝ hiÖu lµ BiÓu diÔn (2) ®−îc gäi lµ biÓu diÔn l−îng
C = {a + ib / a, b ∈ R} . gi¸c cña sè phøc z.
II. PhÐp tÝnh trªn C. Ng−êi ta gäi r lµ modul cña z, kÝ hiÖu z
TÝnh chÊt giao ho¸n, kÕt hîp trong C §ång thêi, gäi ϕ lµ argument cña z, viÕt t¾t
gièng nh− trong R. lµ argz, tÊt nhiªn argz nhËn v« sè gi¸ trÞ :
C¸c phÐp céng, trõ, nh©n, chia nh÷ng sè
argz = ϕ + k2 π , k ∈ Z, vµ ta th−êng dïng ϕ
phøc víi biÓu diÔn ®¹i sè nh− sau:
1. (a+ib) + (c+id) = (a+c) + i(b+d) ∈ [0; 2 π ]. Víi hai sè phøc
2. (a+ib) - (c+id) = (a-c) + i(b-d) z1 = r1 ( cos ϕ1 + i sin ϕ1 )
3. (a+ib)(c+id) = (ac-bd) + i(ad+bc) z2 = r2 ( cos ϕ 2 + i sin ϕ2 ) ta cã:

TAÄP SAN TOAÙN HOÏC - 2007 20


TRÖÔØNG THPT CHUYEÂN HOAØNG VAÊN THUÏ - HOØA BÌNH
PhÇn I - S¸ng t¹o to¸n häc.

= arg(c.f.q) =
z2 z2 = r1r2 cos (ϕ1 + ϕ2 ) + i sin (ϕ1 + ϕ2 ) 
z1 r1 π
=  cos (ϕ1 − ϕ 2 ) + i sin (ϕ1 − ϕ2 )  = arg(1 + i )(2 + i )(3 + i ) = arg(10i ) = (®pcm)
z2 r2  2

§Ó cho tiÖn, b¾t ®Çu tõ ®©y ta kÝ hiÖu


®iÓm bëi ch÷ in hoa vµ nhHn cña nã lµ ch÷ in
th−êng. VÝ dô: ®iÓm Z cã nhHn lµ z = a + ib.
B©y giê, ta xÐt hai ®iÓm Z1, Z2 cã nhHn
t−¬ng øng lµ
z1 = r1 ( cos ϕ1 + i sin ϕ1 )
z2 = r2 ( cos ϕ 2 + i sin ϕ2 )
Lóc nµy th× tæng z3 = z1  + z2 sÏ
biÓu diÔn
   Bµi to¸n 2. Cho tø gi¸c låi ABCD. Dùng ra
vÞ trÝ cña ®iÓm Z3 mµ OZ 3 = OZ1 + OZ 2 phÝa ngoµi c¸c tam gi¸c c©n ®ång d¹ng
 ABP,BCD,CDR,DAS, mµ P,Q,R,S t−¬ng øng
HiÖu w = z1 - z2 sÏ lµ nhHn cña Z1 Z 2 . lµ c¸c ®Ønh c©n. Chøng minh: nÕu PQRS lµ
Gãc ®Þnh h−íng h×nh b×nh hµnh th× ABCD còng lµ h×nh b×nh
  hµnh.
( )
OZ1 , OZ 2 = arg z1 − arg z2 = arg 1
z
z2 Lêi gi¶i.
Víi 4 ®iÓm Z1, Z2, U1, U2, ta cã:
  U −U2
( )
arg Z1Z 2 , U1U 2 = arg 1
Z1 − Z 2
IV. TÝnh chÊt.
1) z + z = 2R(z)
2) z1 z 2 = z1 z 2 ; z1 + z 2 = z1 + z 2
3) z. z = z 2
  z − z1 z 2 − z1
4) Z1Z 2 ⊥ U1U 2 ⇔ 2 =
u 2 − u1 u 2 − u1
  p−a
V. Mét sè bµi to¸n. ( )
§Æt AB, AP = ϕ , thÕ th× ϕ = arg
p −b
hay

p − a AP
Bµi to¸n 1. Cho ba h×nh vu«ng ABCD, lµ = ( cos ϕ + i sin ϕ )
BEFC, EPQF nh− h×nh vÏ. Chøng minh p − b AB
π
r»ng: ∠ ACD + ∠ AFD + ∠ AQD = . AP 1 p−a 1
2 mµ = nªn = (1+tg ϕ ) suy
AB 2 cos ϕ p−b 2
Lêi gi¶i.
Dùng hÖ trôc täa ®é nh− h×nh vÏ vµ 1
nhËn AB lµm vect¬ ®¬n vÞ cña trôc hoµnh. ra p = (1 + tgϕ )( b − a ) + a .
2
Suy ra nhHn cña A, B, C, D, E, F, P, Q t−¬ng
øng lµ: a = 0; b =1; c =1 + i; d = i; e =2; f T−¬ng tù nh− vËy ta còng cã:
= 2 + i; p =3; q =3 + i.
1
Tõ ®ã: ∠ACD + ∠AFD + ∠AQD = q= (1 + tagϕ )( c − b ) + b ,
2
= (AB,AC) + (AE,AF) + (AP,AQ) =

= argc + argf + argq =

TAÄP SAN TOAÙN HOÏC - 2007 21


TRÖÔØNG THPT CHUYEÂN HOAØNG VAÊN THUÏ - HOØA BÌNH
PhÇn I - S¸ng t¹o to¸n häc.

1
r= (1 + tgϕ )( d − c ) + c
2
1
s = (1 + tgϕ )( a − d ) + d
2
Do PQRS lµ h×nh b×nh hµnh nªn s + q = p
+ r. Tõ ®©y dÔ dµng suy ra ®pcm.
NhËn xÐt. Bµi nµy khi ra cho chóng t«i,
thÇy gi¸o ®H cho ®iÒu kiÖn nhÑ h¬n lµ PQRS
lµ h×nh ch÷ nhËt ®Ó cã thÓ sö dông ®Þnh lý
“con nhÝm”. Nh−ng râ rµng víi sè phøc, ta
vÉn gi¶i quyÕt bµi to¸n mét c¸ch b×nh b '− a '
Do ®ã =
th−êng vµ còng hÕt søc tù nhiªn. TÊt nhiªn, c '− a '
sè phøc kh«ng ph¶i lóc nµo còng lµ h−íng
gi¶i quyÕt tèt cho c¸c bµi to¸n, bëi v× trong
tay c¸c b¹n cßn rÊt nhiÒu “vò khÝ” kh¸c, k ac bc 
nh−ng c¸c b¹n hHy cè g¾ng sö dông nã a − b − −b+a+ +b−a
th−êng xuyªn. §Ó kÕt thóc, xin giíi thiÖu víi 2 b a 
c¸c b¹n mét bµi thi häc sinh giái Quèc gia k ba bc 
mµ tù t«i ®H t×m ra lêi gi¶i b»ng sè phøc. a − c − −c+a+ +c−a
2 c a 
Bµi to¸n 3. XÐt c¸c tam gi¸c ABC kh«ng ®Òu kc(a + b )
cã c¸c ®−êng cao AD, BE, CF. LÊy A/, B/, C/ b −a
(k − 1) −
= . 2ab , Râ rµng,
sao/ cho AA/= k. AD; BB/= k.BE;
c −a ( kb(a + c )
CC =k.CF.(k ≠ 0). T×m tÊt c¶ c¸c gi¸ trÞ cña k − 1) −
k sao cho víi mäi tam gi¸c ABC kh«ng ®Òu 2ac
th× ∆ ABC ~ ∆ A/B/C/. ∆ABC ~ ∆A ' B ' C ' t−¬ng ®−¬ngvíi mét trong
b '− a ' b − a
(Thi HSGQG B¶ng A- 1995) hai ®iÒu kiÖn sau: = (1) hoÆc lµ
c '− a ' c − a
Lêi gi¶i. Dùng hÖ trôc täa ®é nh− h×nh vÏ, b '− a ' b−a
= (2). Trong ®ã (1)
ta cã AA ' = k . AD ⇒ a ' = k (d − a ) + a c '− a ' c−a
k bc  ⇔ c 2 a + c 2b = b 2 a + b 2 c ⇔
⇒ a ' = a +  b + c − a) −  , v× a (c − b)(c + b) + cb(c − b) = 0
2 a 
1 bc  ⇔ ab + bc + ca = 0 .Nh−ngvíi
d =  b + c + a −  . Dùng hÖ trôc täa ®é a = R ( cos ϕ1 + i sin ϕ1 ) ,
2 a 
nh− h×nh vÏ, ta cã: b = R ( cos ϕ2 + i sin ϕ 2 ) ,
AA ' = k . AD ⇒ a ' = k (d − a ) + a c = R ( cos ϕ3 + i sin ϕ3 ) th× ta cã:
k  bc   ab + bc + ca = 0 R = 0, v« lý.
⇒ a ' = a +  b + c − a ) −   v×
2  a 
1 bc 
VËy: ∆ABC ~ ∆A ' B ' C ' ⇔ (2)
d =  b + c + a −  .T−¬ng tù  kc ( a + b ) 
2 a  ⇔ b ( k − 1) − =
 2 ab 
k ca 
b ' = b +  c + a − b) −  ,  kb ( a + c )  2
2 b 
=  ( k − 1) −  ⇔k= .
 2ac  3
k ab 
c ' = c +  a + b − c) − 
2 c  2
VËy k = lµ gi¸ trÞ cÇn t×m./.
3

TAÄP SAN TOAÙN HOÏC - 2007 22


TRÖÔØNG THPT CHUYEÂN HOAØNG VAÊN THUÏ - HOØA BÌNH
PhÇn I - S¸ng t¹o to¸n häc.

Phöông trình haøm

vaø Söï Truø Maät


Bïi lª vò
Chuyªn to¸n K01-04
Sv. Líp 04TT, Khoa To¸n Tin, §HKHTN - §HQG TP Hå ChÝ Minh

Ph−¬ng tr×nh hµm lµ mét chuyªn ®Ò


Chøng minh. Do lim( n + 1 − n ) =
quan träng trong to¸n häc phæ th«ng, ®Æc n →∞

biÖt lµ trong c¸c k× thi häc sinh giái. Còng 1


= lim = 0 nªn ∀ ε > 0, tån t¹i
nh− nhiÒu vÊn ®Ò kh¸c trong To¸n häc, n →∞ n +1 + n
ph−¬ng tr×nh hµm còng mang vÎ ®Ñp riªng
cña nã. Bµi viÕt nµy xin ®−îc quan s¸t vÎ n0 sao cho n + 1 − n < ε , ∀ n > n0.
®Ñp ®ã d−íi gãc ®é lµ mèi liªn hÖ gi÷a Do lim( n0 − n ) = −∞
n →∞
ph−¬ng tr×nh hµm vµ sù trï mËt.
I / TËp hîp trï mËt. nªn ∀x ∈ R , ∀ ε > 0, ∃ n1 sao
1. §Þnh nghÜa. cho: n0 − n < ε , ∀ n > n1 . XÐt dHy (Un)
Cho hai tËp hîp A vµ B, A ⊂ B. Nãi A trï x¸c ®Þnh bëi: Un = n0 + n − n1 ∀ x ∈ N.
mËt trong B nÕu ∀x ∈ B, ∀ε > 0, ∃x ∈A sao
Ta cã Un+1 - Un= n0 + n + 1 − n + n0 < ε
cho x − x < ε .
vµ U0 = n0 − n1 < x − ε
2. Mét sè vÝ dô.
2.1. VÝ dô 1. Cho tËp hîp ⇒ ∃ m ∈ N* ®Ó U m −1 < x − ε < U m
k  ⇒ x + ε > Um-1 + 2ε > Um > x - ε
S1 =  n / k ∈ Z , n ∈ N  . Chøng minh rằng ⇒ U m − x < ε . Do Um ∈ S2 nªn S2 trï mËt
2 
tập S1 trï mËt trong R. trong R (®pcm).
1 2. 3. VÝ dô 3. Gi¶ sö tËp N ®−îc chia thµnh
Chøng minh. Do lim n = 0 nªn ∀ b > a, 2 tËp hîp con A vµ B, mçi tËp chøa v« h¹n
n →∞ 2
a 
∃ n0 sao cho: ∀ n > n0 ta cã b − a > n
1 phÇn tö. §Æt S3 =  / a, b ∈ N  . Chøng
2 b 
⇒ 2nb > 2na + 1, ∀ n > n0 minh rằng S3 trï mËt trong [0; +∞)
k Chøng minh. Do B cã v« h¹n phÇn tö nªn
⇒ ∃ k ∈ Z ®Ó b > n > a k
2 ∀ k ∈ N, ∀ ε > 0, ∃ b ∈ B sao cho b >
⇒ ∀ x ∈ R, ∀ ε >0, ∃ k ∈ Z, n ∈ N sao cho 2ε
k ⇒ b(x + ε ) > b(x - ε ) + k, ∀ x ∈ [0; + ∞ )
x +ε > n > x −ε ⇒ ∃ a ∈ A ®Ó b(x + ε ) > a > b(x - ε ) (do A
2
⇒ S1 trï mËt trong R (®pcm). cã v« h¹n phÇn tö )
2.2. VÝ dô 2. Cho tËp hîp a
⇒ x+ ε > > x- ε
S2 = { }
a − b / a, b ∈ N . Chøng minh rằng b
⇒ S3 trï mËt trong [0; + ∞ )
S2 trï mËt trong R. 3. §Þnh lý. Cho hai hµm sè
f ( x), g ( x) : X → X , trong ®ã f(x) liªn tôc,

TAÄP SAN TOAÙN HOÏC - 2007 23


TRÖÔØNG THPT CHUYEÂN HOAØNG VAÊN THUÏ - HOØA BÌNH
PhÇn I - S¸ng t¹o to¸n häc.

g(x) liªn tôc hoÆc ®¬n ®iÖu. Vµ A lµ tËp hîp Lêi gi¶i. §Æt
trï mËt trong X. Khi ®ã, nÕu f(x) = g(x) ∀ x e 2 f (0) − ef (1)
∈ A th× f(x) = g(x) ∀ x ∈ X. b= , a = f (0) − b ,
e2 −1
Chøng minh. Ta cã nhËn xÐt: Víi tËp A trï
ta cã f (0) = a + b , f (1) = ae + be−1 .
mËt trong tËp X th× ∀ x ∈ X, tån t¹i dHy (xn)
tháa mHn xn ∈ A, ∀ n ∈ N vµ lim xn = x. 1
n →+∞
Cho x = y = , ta cã
2
ThËt vËy, nÕu x ∈ A th× tÇm th−êng. NÕu
 1  2 − 
1 1
x ∉ A, theo ®Þnh nghÜa ta cã: f (0) + f (1) = f    e + e 2 
∀x ∈ X , ∀ε > 0, ∃x ∈A sao cho x − x < ε .  2  
1 1
1 −
MÆt kh¸c, dÔ cã tån t¹i dHy ( ε n) tháa mHn 0 ⇒ f( )= ae + be 2 2

< ε n < ε , ∀ n ∈ N. Víi mçi n ∈ N, ∃ xn ∈ 2


B»ng quy n¹p, dÔ dµng chøng minh ®−îc
A tháa x − x < ε n 1 1
1 − n

⇒ lim xn = x. NhËn xÐt ®−îc chøng minh. f( n )= ae + be , ∀ n ∈ N. Tõ ®ã, còng


2n 2

n →+∞ 2
Chó ý. Tõ ®©y ta cã thÓ dÔ dµng chøng minh b»ng quy n¹p, ta chøng minh ®−îc
®−îc nhËn xÐt m¹nh h¬n: Tån t¹i hai dHy  k 
k

k
f  n  = ae 2 + be 2 , ∀ n ∈ N.
( ) ( )
xn1 vµ xn2 tháa mHn 2 
i) xn1 , xn2 ∈ A, ∀n ∈ N Do S1 trï mËt trong R nªn ta cã
f ( x ) = ae x + be − x , ∀ x ∈ R. Thö l¹i thÊy
ii) xn1 < x < xn2 , ∀n ∈ N (*)
tháa mHn vµ ®ã lµ hµm cÇn t×m.
iii) lim xn1 = lim xn2 = x Bµi to¸n 2. T×m tÊt c¶ c¸c sè thùc d−¬ng k
n1 →+∞ n2 →+∞

Trë l¹i bµi to¸n, xÐt 2 tr−êng hîp: sao cho tån t¹i hµm f: R → R liªn tôc vµ tháa
- Tr−êng hîp1: g(x) liªn tôc. Theo nhËn xÐt mLn :
trªn, ∀ x ∈ X, tån t¹i dHy ( xn ) tháa mHn i) f(x.f(y)) = ykf(x), ∀ x, y ∈ R
( ) ( )( )
n −1
ii)f x 2 − y 2 = ∏ f x 2 − y 2 x− y ∀
n n i i
xn ∈ A, ∀n ∈ N vµ lim xn = x
n →+∞
i =1
⇒ g ( xn ) = f ( xn ) víi mäi n ∈ N x, y ∈ N vµ n lµ mét sè tù nhiªn cho tr−íc.
⇒ g(x) = f(x). Lêi gi¶i. Gi¶ sö tån t¹i sè k > 0 tháa mHn
- Tr−êng hîp 2: g(x) ®¬n ®iÖu. Kh«ng gi¶m ®iÒu kiÖn bµi to¸n. Tõ i) ta gi¶ sö f(x) = f(y)
tæng qu¸t, gi¶ sö g(x) t¨ng. Ta cã: tån t¹i hai ⇒ f(xf(y)) = f(xf(x))
( ) ( )
dHy xn1 vµ xn2 tháa mHn (*) ⇒ ykf(x) = xkf(x) ⇒ x = y
⇒ f ®¬n ¸nh .
⇒ f ( xn1 ) = g ( xn1 ) < g ( x) < g ( xn2 ) = f ( xn2 ) Cho x = 1, ta cã f(f(y)) = ykf(1).
⇒ g(x) = f(x) T¹i ®©y, cho y = 1 vµ y = xy ta ®−îc
VËy ®Þnh lý ®−îc chøng minh. Tõ ®©y, ta f(f(1)) = f(1) ⇒ f(1) = 1
cã thÓ thiÕt lËp ®−îc mét sè bµi to¸n . ⇒ f(f(xy)) = (xy)kf(1) = (xy)k
II. Mét sè Bµi to¸n. T¹i i) cho x = f(x), ta cã f(f(xy)) = ykf(f(x))
C¸c tËp S1, S2, S3 ®−îc ®Þnh nghÜa trong = (xy)k ⇒ f(xy) = f(x)f(y), ∀ x, y ∈ R.
c¸c vÝ dô ë môc trªn. VËy f nh©n tÝnh, kÕt hîp víi ii) dÔ dµng
Bµi to¸n 1. T×m tÊt c¶ c¸c hµm sè
f : R → R liªn tôc vµ tháa mLn : ( ) (
suy ra: f x − y = x − y ∀ x, y ∈ N. )
f(x + y) + f(x - y) = 2 f(x) chy. Do S2 trï mËt trong R nªn f(x) = x, ∀ x ∈ R.
Thay vµo i) ta cã k = 1 tháa mHn c¸c ®iÒu
Trong ®ã chy = ( e y + e− y )
1
2

TAÄP SAN TOAÙN HOÏC - 2007 24


TRÖÔØNG THPT CHUYEÂN HOAØNG VAÊN THUÏ - HOØA BÌNH
PhÇn I - S¸ng t¹o to¸n häc.

kiÖn bµi to¸n. VËy k = 1 lµ gi¸ trÞ duy nhÊt TiÕp theo, t«i thÊy r»ng “sù trï mËt” kh«ng
cÇn t×m. m¹nh b»ng “ sù phñ kÝn” vµ t«i ®H cè g¾ng tr¶ lêi
Bµi to¸n 3. KÝ hiÖu c©u hái: LiÖu “sù phñ kÝn” cã mèi liªn hÖ nµo
víi ph−¬ng tr×nh hµm?
 2p  Bµi to¸n 4. T×m tÊt c¶ c¸c hµm sè
S=  / p ∈ Z , k ∈ N  . T×m tÊt c¶ c¸c
 2k + 1  f : R+* → R+* tháa mLn :
hµm sè f: R → R liªn tôc vµ tháa mLn c¸c
f(x+y) = f(x2+y2) ∀ x, y > 0.
®iÒu kiÖn:
i) ∀ x ∈ S th× f(x) ∈ S, f2(0) ∈ S Lêi gi¶i. §Æt x + y = 2a , x2 + y2 = b
ii) f(xf(x) + f(y)) = y + f2(x), ) ∀ x, y ∈ S.
Lêi gi¶i. Cho x = 0 ⇒ f(f(y)) = f2(0) + y víi
⇒ f ( )
2a = f (b) ), ∀ 2a > b ≥ a > 0
Cè ®Þnh a > 0, ta cã: f (b) = const ,
mäi y ∈ S ⇒ f ®¬n ¸nh trong S.
Cho x = f(x) ∈ S, ta cã: [
∀b ∈ [a; 2a ) . KÝ hiÖu ∆ n = 2 n −1 a;2 n a ]
f ( f ( x). f ( f ( x)) + f ( y ) ) = [ f ( f ( x))] + y Do n −1
a = lim 2 a = +∞ nªn tËp
2 n
lim 2
n →+∞ n →+∞
⇒ f ( f ( x)( x + a) + f ( y ) ) = [( x + a )] + y ,
2
hîp c¸c ®o¹n ∆ n phñ kÝn (0; + ∞ ) (v× ∆ n
trong ®ã a = f2(0). T¹i ®©y, cho x = - a ∈ S,
ta cã f(f(y)) = y ⇒ a= 0 ⇒ f(0) = 0.
{ }
∩ ∆ n +1 = 2 n a )
Cho y = 0 ⇒ f(xf(x)) = f2(x) (1) ⇒ ∀ x > 0, tån t¹i n ∈ Z sao cho x ∈ ∆ n
T¹i ®©y, cho x = f(x), ta cã ⇒ f(x) = const. Thö l¹i thÊy ®óng.
f ( f ( x). f ( f ( x)) ) = [ f ( f ( x))] VËy f(x) = c, c ∈R lµ hµm cÇn t×m.
2

⇒ f(xf(x)) = x2 (2) Tõ ý t−ëng nµy, ta cã thÓ thiÕt lËp ®−îc


rÊt nhiÒu bµi to¸n d¹ng nh− sau:
Tõ (1) vµ (2) suy ra f2(x) = x2 , ∀ x ∈ S
Gi¶ sö tån t¹i y0 ∈ S ( y0 ≠ 0 ) sao cho f(y0) Cho hai hµm n biÕn g , h : X * → X ®−îc
liªn hÖ víi nhau bëi bÊt ®¼ng thøc kÐp trong
= - y0 th× t¹i ph−¬ng tr×nh hµm ban ®Çu, cho
X. T×m tÊt c¶ c¸c hµm f : X → X tháa mHn:
y = y0 ta cã:
f(xf(x) - y0) = y0 + f2(x)= x2+ y0 f ( g ()) = f (h()) .
⇒ [ f ( xf ( x) − y 0 )] = (x 2 + y 0 )
2 2 VÝ dô nh− bµi to¸n sau:
Bµi to¸n 5. Cho dLy hµm sè { f n ( x)}1 tháa

⇒ (xf(x) - y0)2 = (x2+y0)2


⇒ f(x) = - x, ∀ x ∈ S mLn f n ( x) : (1; +∞) → (1; +∞) sao cho
VËy f(x) = x, ∀ x ∈ S hoÆc f(x) = - x,  x + y  n   xn + yn 
∀ x∈ S (*) f n    = f n  ∀x, y > 1
NhËn xÐt.  2    2 
{
1/. Theo vÝ dô 3, xÐt tËp A= 2 p / p ∈ Z + ; } X¸c ®Þnh

∑f (x) .
{
B= 2k + 1 / k ∈ N }, khi ®ã tËp 2
n

 2p  (Bµi to¸n nµy xin “bá ngá”cho c¸c b¹n)


S /=  / p ∈ Z + , k ∈ N  trï mËt trong
 2k + 1  Trªn ®©y lµ mét sè kÕt qu¶ t«i khai th¸c
{0;+ ∞ ). Tõ ®©y, dÔ dµng suy ra tËp S trï mËt ®−îc vµ ch¾c ch¾n r»ng cßn rÊt nhiÒu nh÷ng
trong R (**) kÕt qu¶ ®Ñp n÷a, rÊt mong nh©n ®−îc sù trao
Tõ (*) vµ (**) suy ra f ( x) = x, ∀x ∈ R hoÆc ®æi cña c¸c b¹n. §Ó kÕt thóc bµi viÕt, xin
f ( x) = − x, ∀x ∈ R . Thö l¹i thÊy tháa mHn vµ phÐp ®−îc bµy tá mét ®iÒu rÊt t©m ®¾c mµ
®ã lµ 2 hµm cÇn t×m. t«i ®¨ häc ®−îc tõ ng−êi thÇy cña m×nh:
2/. Qua 3 bµi to¸n trªn ta thÊy mét sè kÕt “Häc to¸n còng nh− lµm bÊt k× mét viÖc g×,
qu¶ vÒ sù trï mËt ®H gióp mét phÇn kh«ng chóng ta hLy quyÕt t©m thËt cao, t«i tin r»ng
nhá trong viÖc ®Þnh h−íng gi¶i PTH. b¹n vµ t«i sÏ ®¹t ®−îc nh÷ng kÕt qu¶ t−¬ng
xøng víi sù cè g¾ng ®ã”./.

TAÄP SAN TOAÙN HOÏC - 2007 25


TRÖÔØNG THPT CHUYEÂN HOAØNG VAÊN THUÏ - HOØA BÌNH
PhÇn I - S¸ng t¹o to¸n häc.

Mäi tËp A ⊂ R , A ≠ ∅ bÞ chÆn trªn ®Òu cã


daõy soá vaø söï truø maät cËn trªn ®óng.
II. Mét sè tÝnh chÊt.
+
TREÂN R 1. TÝnh chÊt 1.
TËp Q c¸c sè h÷u tû lµ trï mËt trªn R.
2. TÝnh chÊt 2.
Hå Sü Tïng L©m NÕu tËp A trï mËt trªn R+ th× tËp
Chuyªn to¸n k01-04, ptnk 1 
B =  , r ∈ A còng trï mËt trªn R+.
§HQG TP Hå ChÝ Minh r 
Sv. Líp CNKHTN K3, Khoa To¸n Tin ViÖc chøng minh hai tÝnh chÊt trªn xem
§HKHTN - §HQG TP Hå ChÝ Minh nh− bµi tËp.
III. Mét sè bµi to¸n më réng.
I. §Þnh nghÜa. 1. Bµi to¸n 1.
ë ®©y t«i xin ®−îc nªu ra hai ®Þnh nghÜa Cho dLy sè {an} d−¬ng, t¨ng, kh«ng bÞ
cña sù trï mËt. chÆn. Chøng minh r»ng tËp hîp
XÐt c¸c tËp M, X, R sao M ⊂ X ⊂ R. m 
1. §Þnh nghÜa 1. A =  / m, n ∈ N +  trï mËt trªn R+.
X  an 
TËp M C gäi lµ trï mËt trªn tËp X nÕu vµ
Chøng minh. Ta sö dông bæ ®Ò sau:
chØ nÕu víi mäi p, q ∈ X, p > q, tån t¹i m ∈
Bæ ®Ò: Víi mäi sè α > 0 vµ dLy {an} cã
M sao cho p > m > q.
tÝnh chÊt lim an = ∞ th× ta còng cã lim α an
2. §Þnh nghÜa 2. n →∞ n →∞
X
TËp M C gäi lµ trï mËt trªn tËp X nÕu vµ = ∞ . XÐt p, q ∈ R tháa mHn p > q.
+

chØ nÕu víi mäi x ∈ X, tån t¹i dLy sao Suy ra lim (p - q)an = ∞ . Do ®ã, tån t¹i
n →∞
cho lim xn = x. sè n0 ∈ N sao cho (p - q) a n 0 > 2
+
n →∞
L−u ý r»ng hai ®Þnh nghÜa trªn lµ t−¬ng ⇒ p a n 0 > [p a n 0 -1] > q a n 0
®−¬ng. ViÖc chøng minh chóng t−¬ng ®−¬ng
m0
coi nh− bµi tËp. Chän m0 = [p a n 0 ] – 1, ta cã p > > q.
Mét sè ®Þnh nghÜa kh¸c: a n0
3. §Þnh nghÜa 3. VËy A trï mËt trªn R+.
Nãi r»ng tËp A ⊂ R bÞ chÆn trªn nÕu tån NhËn xÐt. ¸p dông Bµi to¸n 1, ta cã Q+ trï
t¹i x ∈ R sao cho x ≤ z víi mäi z ∈ A. PhÇn
tö z nh− thÕ ®−îc gäi lµ cËn trªn cña tËp A. m
mËt trªn R+,  n  trï mËt trªn R+.
Gi¶ sö tËp A bÞ chÆn trªn, z ®−îc gäi lµ 2 
cËn trªn ®óng cña A nÕu z lµ cËn trªn bÐ ¸p dông tÝnh chÊt 2, ta cã tËp A’
nhÊt cña A. CËn trªn ®óng cña A ®−îc ký a 
hiÖu lµ supA. =  n , m, n ∈ N +  còng trï mËt trªn R+.
4. §Þnh nghÜa 4. m 
Nãi r»ng tËp A ⊂ R bÞ chÆn d−íi nÕu tån 2. Bµi to¸n 2.
t¹i x ∈ R sao cho z ≤ x víi mäi z ∈ A. Cho {an } vµ {bn } lµ hai dLy sè d−¬ng,
PhÇn tö z nh− thÕ ®−îc gäi lµ cËn d−íi cña t¨ng ngÆt vµ kh«ng bÞ chÆn. §Æc biÖt, tËp
tËp A. hîp {a n +1 − a n } bÞ chÆn. Chøng minh r»ng
Gi¶ sö tËp A bÞ chÆn d−íi, z ®−îc gäi lµ
tËp hîp
cËn d−íi ®óng cña A nÕu z lµ cËn d−íi lín
nhÊt cña A. CËn d−íi ®óng cña A ®−îc ký a 
B =  m / m, n ∈ N +  trï mËt trªn R+.
hiÖu lµ infA.  bn 
5. Tiªn ®Ò vÒ cËn trªn.

TAÄP SAN TOAÙN HOÏC - 2007 26


TRÖÔØNG THPT CHUYEÂN HOAØNG VAÊN THUÏ - HOØA BÌNH
PhÇn I - S¸ng t¹o to¸n häc.

an
Chøng minh. XÐt p, q ∈ R+, p > q. §Æt
Do lim = k nªn chän ε > 0 sao cho
n →∞ n

M = Sup { an +1 − an } , K = Max {a1 , M } . p−q


.k > ε . Ta cã tån t¹i N sao cho:
V× lim bn = ∞ nªn tån t¹i m0 sao cho p+q
n →∞
an p−q
( p − q )bm0 > K . −k < .k − ε , ∀ n ≥ N.
n p+q
Ta cã tån t¹i n0 sao cho
p−q a p−q
pbn0 > an0 > qbn0 . ThËt vËy, do ⇔ .k + ε > n > .k - ε , ∀ n ≥ N.
p+q n p+q
K = Max {a1 , M } mµ lim an = ∞ nªn suy ra q p
n →∞ §Æt α = < =β.
tån t¹i sè tù nhiªn N tháa mHn 2q 2q
k +ε k −ε
aN > bn0 > aN −1 , h¬n n÷a ( p − q )bm0 > K nªn p+q p+q
pbm0 > aN . Chän n0 = N lµ xong. ¸p dông Bµi to¸n 1, tån t¹i v« sè bé (m,
n
an0 n) sao cho α < <β.
VËy p > > q ⇒ B trï mËt trªn R+. bm
bm0
Chó ý. Theo Bµi to¸n 1, chØ cã sù tån t¹i h÷u
NhËn xÐt. ¸p dông Bµi to¸n 2 ta cã hÖ qu¶ h¹n, nh−ng sù tån t¹i dÉn ®Õn sù tån t¹i v« h¹n v×
sau: nÕu ta chän α < α 1 < α 2 < ... < α n < β th×
Cho f: R+ → R+ tháa mHn:
i) f kh¶ vi trªn R+. gi÷a ( α i , α j ) tån t¹i mét sè, cho n dÇn tíi v«
ii) lim f ( x) = ∞ cïng th× ta cã v« sè sè. Do ®ã, cã thÓ chän
x →∞
n0
iii) f ' ( x) < M , ∀x ∈ R + \ (0;1) n0 > N vµ m0 sao cho α < < β . Mµ
b m0
Cho {bn } lµ dHy sè d−¬ng, t¨ng, kh«ng bÞ
p−q an p−q
chÆn. Ta cã: .k+ ε > 0 > . k - ε . Nh©n vÕ
p+q n0 p+q
 f ( m) 
B’ =  / m, n ∈ N +  trï mËt trªn R+. an
 bn  theo vÕ, ta cã: q < o < p .
b m0
 n 
Do ®ã tËp  m / m, n ∈ N +  trï mËt trªn R+. VËy C trï mËt trªn R+.
2  NhËn xÐt. ¸p dông Bµi to¸n 3, ta cã tËp
3. Bµi to¸n 3.  1 an
+
m ! sin  trï mËt trªn R . NÕu lim =∞
Cho dLy sè d−¬ng {an } t¨ng  n n →∞ n

an th× Bµi to¸n 3 kh«ng cßn ®óng n÷a. ThËt vËy,


tháa lim = k , k ∈ R+.
n →∞ n chän an = 2n , bm = 2m . Râ rµng lóc nµy C
Cho dLy sè d−¬ng {bn } t¨ng, kh«ng bÞ an
kh«ng trï mËt trªn R+. NÕu lim = 0 th×
chÆn. Chøng minh r»ng tËp hîp C = n →∞ n

 am Bµi to¸n 3 còng kh«ng cßn ®óng n÷a. ThËt vËy,


+
 / m, n ∈ N  trï mËt trªn R .
+ k +1
an = 2k , nÕu 22 ≤ n ≤ 22 − 1 vµ
k

b chän
 n 
bm = 2 .
m

Chøng minh. XÐt p, q ∈ R , p > q. +


Cuèi cïng xin chóc c¸c b¹n häc tËp vui vÎ vµ
cã kÕt qu¶ cao.

TAÄP SAN TOAÙN HOÏC - 2007 27


TRÖÔØNG THPT CHUYEÂN HOAØNG VAÊN THUÏ - HOØA BÌNH
PhÇn I - S¸ng t¹o to¸n häc.

MOÄT SOÁ BAØI TOAÙN SOÁ HOÏC veà


DAÕY TOÅNG CAÙC LUÕY THÖØA
TrÇn quèc hoµn
Chuyªn To¸n K02 – 05, THPT Chuyªn NguyÔn Tr`i – H¶i D−¬ng
Sv. Líp K50CA - §H C«ng NghÖ - §HQG Hµ Néi

Trong bµi b¸o nµy chóng ta ®Ò cËp ®Õn nhiÒu bµi to¸n tæng qu¸t còng nh− kÐo theo
tõ bµi to¸n nµy mµ c¸c thµnh viªn cña trang
mét sè bµi to¸n sè häc liªn quan ®Õn dHy web nµy ®H ®Ò ra. Sau ®©y t«i xin giíi thiÖu
tæng c¸c lòy thõa. Cho a1 , a2 ,.., am lµ c¸c 4 trong sè c¸c bµi to¸n ®ã. B¹n ®äc cã thÓ
sè nguyªn d−¬ng cè ®Þnh cho tr−íc. XÐt dHy t×m lêi gi¶i t−¬ng tù bµi to¸n trªn (tuy cã xö
lý mét sè kü thuËt khã h¬n).
sè sau un = a1 + a2 + ... + am trong ®ã n =
n n n

0, 1, 2, …. Bµi to¸n 1.1. Cho n > 1 lµ sè nguyªn d−¬ng.


Bµi to¸n 1. BiÕt r»ng tËp hîp c¸c −íc Chøng minh r»ng víi bÊt kú n sè nguyªn
nguyªn tè cña dLy un lµ h÷u h¹n. Chøng d−¬ng a1 , a2 ,..., an lu«n tån t¹i mét sè
minh r»ng a1 = a2 = ... = am . nguyªn d−¬ng k sao cho sè a1k + a2k + ... + ank
Lêi gi¶i. Tr−íc hÕt ®Æt d = ( a1 , a2 ,..., am ) cã mét −íc nguyªn tè kh«ng lµ −íc nguyªn
tè cña na1a2 ...an .
ta cã thÓ viÕt ai = dbi , trong ®ã
Bµi to¸n 1.2. Cho sè nguyªn d−¬ng {an }
( b1 , b2 ,..., bm ) = 1 . Khi ®ã dHy
víi m lµ sè nguyªn d−¬ng cho tr−íc vµ m sè
vn = b1n + b2n + ... + bmn còng cã h÷u h¹n c¸c nguyªn d−¬ng k1 , k2 ,..., km . DLy sè míi ®−îc
−íc nguyªn tè lµ p1 , p2 ,..., pk x¸c ®Þnh nh− sau:
Chän x nguyªn d−¬ng sao cho pix > m un = k1a1n + k2 a2n + ... + km amn . Chøng minh
víi mäi i = 1, 2, …, k. Khi ®ã víi mäi sè t r»ng dLy sè {un } cã h÷u h¹n −íc nguyªn tè
nguyªn d−¬ng lín h¬n x, ®Æt
k khi vµ chØ khi a1 = a2 = ... = am .
n(t ) = t ⋅ ∏ ϕ ( pix ) th× víi mäi i = 1, 2, …, k Bµi to¸n 1.3. Cho m nguyªn d−¬ng lín h¬n
i =1
ta cã bsn (t ) ≡ 0 ∨ 1(mod pix ) víi mäi s = 1, 2, 1 vµ P1 ( x), P2 ( x),..., Pm ( x) lµ c¸c ®a thøc hÖ
…, m. ( bsn (t ) ≡ 0(mod pix ) khi vµ chØ khi sè nguyªn kh«ng ©m kh«ng ®ång nhÊt víi 0.
C¸c sè nguyªn d−¬ng a1 , a2 ,..., am ®«i mét
bs chia hÕt cho pi ).
ph©n biÖt. X¸c ®Þnh hµm sè f : N → N sao
Chó ý lµ c¸c bi nguyªn tè cïng nhau vµ m
pi > m nªn dÔ thÊy un ( t ) sÏ kh«ng chia hÕt f (n) = ∑ Pi (n)ain víi mäi n ∈ N.
x
cho
i =1
cho pix víi mäi i = 0, 1, …, k. Do ®ã dÔ cã
k
Chøng minh r»ng tËp hîp c¸c −íc nguyªn tè
un ( t ) ≤ ∏ p x −1
, ∀t > x , ®iÒu nµy chØ xÈy ra cña hµm f lµ v« h¹n.
i =1
i
Bµi to¸n 1.4. Hµm f : N → Z ®−îc gäi lµ
khi b1 = b2 = ... = bm = 1 hay hµm gÇn ®a thøc nÕu f(m) – f(0) chia hÕt cho
a1 = a2 = ... = am . Bµi to¸n ®−îc chøng minh. m víi mäi m. Khi ®ã víi mäi f1 , f 2 ,..., f n lµ
c¸c hµm gÇn ®a thøc vµ c¸c sè nguyªn
XuÊt sø. Bµi to¸n nµy ®−îc ®Æt ra khi t¸c gi¶ d−¬ng ph©n biÖt a1 , a2 ,..., an th× tËp c¸c sè
®i t×m lêi gi¶i cho mét bµi to¸n rÊt thó vÞ
nh−ng hoµn toµn kh¸c trªn mathlink contest n

(sÏ cã dÞp giíi thiÖu víi b¹n ®äc trong nh÷ng nguyªn d−¬ng cã d¹ng ∑ f (k )a
i
k
i cã v« sè
dÞp kh¸c). Sau ®ã míi nã ®−îc chän lµm ®Ò i =1
thi gi¶i to¸n online th¸ng 10 n¨m 2006 trªn −íc nguyªn tè.
trang web www.diendantoanhoc.net, cã rÊt

TAÄP SAN TOAÙN HOÏC - 2007 28


TRÖÔØNG THPT CHUYEÂN HOAØNG VAÊN THUÏ - HOØA BÌNH
PhÇn I - S¸ng t¹o to¸n häc.

Bµi to¸n 2. BiÕt r»ng víi mäi n ®ñ lín th× un Tr−êng hîp 2: p1 = 2 suy ra
lµ sè chÝnh ph−¬ng. Chøng minh r»ng m lµ  p1   2  p 2 −1

sè chÝnh ph−¬ng.  p  =  p  = ( −1) 8


Lêi gi¶i. Tr−íc hÕt ta chøng minh bæ ®Ò sau:    
§Æt
Bæ ®Ò: Cho a lµ sè nguyªn d−¬ng sao cho
víi mäi p nguyªn tè ®ñ lín ta cã a lµ sè k
 p   p1  k  pi   p 
chÝnh ph−¬ng theo modp. Khi ®ã ta cã kÕt s = ∏  =  ∏    =
luËn a lµ sè chÝnh ph−¬ng. i = 2  pi   p  i = 2  p   pi 
Chøng minh. Ph¶n chøng, gi¶ sö a kh«ng lµ p 2 −1
k
( p −1)( pi −1)
sè chÝnh ph−¬ng. Khi ®ã kh«ng gi¶m tæng = ( −1) 8
+∑ 4 (1)
i =2
qu¸t ta gi¶ sö a kh«ng cã −íc nguyªn tè
chÝnh ph−¬ng nµo kh¸c 1 suy ra Râ rµng nÕu k = 1 hay a = p1 = 2 ta cã
a = p1 p2 ... pk víi p1 < p2 < ... < pk lµ c¸c sè ngay m©u thuÉn (Ch¼ng h¹n, chän p nguyªn
nguyªn tè ph©n biÖt. tè ®ñ lín cã d¹ng 8n + 3). Ta chØ cÇn xÐt khi
Theo bµi ra tån t¹i N 0 sao cho víi k > 1. B©y giê lËp luËn t−¬ng tù tr−êng hîp
1 sÏ tån t¹i sè nguyªn tè p sao cho p > N 0
a
mäi sè nguyªn tè p > N 0 th×   = 1 hay  p
 p vµ p ≡ 1(mod 8) ,   = −1 vµ
k
 pi   p2 
∏   = 1 . XÐt c¸c tr−êng hîp sau:
i =1  p 
 p
 p  = 1, ∀i = 3,..., k . Tõ ®ã ta còng dÔ thÊy
Tr−êng hîp 1: p1 > 2 , ®Æt  i
k
 p k  pp  ( p −1)( pi −1) m©u thuÉn nh− trªn. Bæ ®Ò ®−îc chøng minh.
s = ∏   = ∏    i  = ( −1)∑ 4 Trë l¹i Bµi to¸n 2, gi¶ sö tån t¹i N sao
i =1  pi  i =1  pi   p  cho mäi n > N th× an lµ sè chÝnh ph−¬ng.
NhËn xÐt r»ng: Tån t¹i b sè nguyªn XÐt tÊt c¶ c¸c sè nguyªn tè p > N + 1 vµ
b p > Max {ai } th× u p −1 ≡ m(mod p ) vËy m lµ
d−¬ng sao cho (b; p1 ) = 1 vµ   = −1 (®iÒu
 p1  sè chÝnh ph−¬ng modp víi mäi p nguyªn tè
nµy lµ hiÓn nhiªn v× trong mét hÖ thÆng d− ®ñ lín nªn suy ra lµ sè chÝnh ph−¬ng (theo
p −1 bæ ®Ò trªn). Bµi to¸n ®−îc chøng minh.
thu gän mod p1 th× cã ®óng sè chÝnh Xung quanh dHy sè nµy t«i xin ®−îc ®Ò
2 xuÊt 3 bµi to¸n sau, rÊt mong sù quan t©m
ph−¬ng mod p1 ). Khi ®ã ta chØ viÖc chän sè ®Õn lêi gi¶i cña chóng.
t nguyªn d−¬ng sao cho Bµi to¸n 3. Tån t¹i hay kh«ng dLy sè nguyªn
a1 , a2 ,... sao cho dLy sè cã v« h¹n sè h¹ng
 t ≡ 1(mod 8)
 kh¸c kh«ng vµ ®ång thßi dLy sè
t ≡ b(mod p1 ) ; i = 2,...k un = a1n + a2n + ... + ann cã h÷u h¹n −íc nguyªn
 t ≡ 1(mod p ) tè. (Chó ý r»ng nÕu dLy cã h÷u h¹n sè kh¸c
 i
kh«ng th× nã nh− hÖ qu¶ cña bµi to¸n 1 ta cã
Khi ®ã tån t¹i sè nguyªn tè p > N 0 sao tÊt c¶ c¸c sè h¹ng kh¸c kh«ng cña dLy
cho p ≡ t (mod 8 p1... pk ) (Theo nguyªn t¾c an ®Òu b»ng nhau).
 p Bµi to¸n 4. BiÕt r»ng dLy un cã h÷u h¹n −íc
Dirichlet). Tõ ®©y suy ra   = −1 vµ sè chÝnh ph−¬ng. Chøng minh r»ng
 p1  a1 = a2 = ... = am
 p Bµi to¸n 5. Cho A lµ mét cÊp sè céng d¹ng
 p  = 1, ∀i = 2,..., k nªn ta sÏ cã s = -1 ax + b víi a, b nguyªn, a > 0. (a, b) = 1, x
 i BiÕt r»ng dLy un chøa h÷u h¹n −íc nguyªn
nh−ng víi chó ý r»ng p -1 chia hÕt cho 8 nªn
tè trong A. Chøng minh r»ng
s = 1 theo c«ng thøc trªn. §iÒu nµy m©u
thuÉn víi kÕt qu¶ võa cã. a1 = a2 = ... = am ./.

TAÄP SAN TOAÙN HOÏC - 2007 29


TRÖÔØNG THPT CHUYEÂN HOAØNG VAÊN THUÏ - HOØA BÌNH
PhÇn I - S¸ng t¹o to¸n häc.

CAÂN BAÈNG HEÄ SOÁ TRONG BAÁT ÑAÚNG THÖÙC COÂ-SI


NguyÔn L©m TuyÒn
Chuyªn To¸n K99 –02
Sv. Líp §iÒu khiÓn Tù §éng 1 - K47, §H B¸ch Khoa Hµ Néi

Sö dông bÊt ®¼ng thøc (B§T) ®H biÕt mµ Ph−¬ng ph¸p suy luËn:
®Æc biÖt lµ B§T C«-si lµ ph−¬ng ph¸p Sù chªnh lÖch vÒ sè mò cña c¸c biÓu
th−êng ®−îc ¸p dông ®Ó gi¶i c¸c bµi to¸n vÒ thøc x3 + y 3 vµ P( x; y ) = x + y gîi cho ta
B§T nãi chung. Nh÷ng bµi to¸n cùc trÞ, nhÊt sö dông B§T C«-si ®Ó h¹ bËc cña x3 + y 3 .
lµ tr−êng hîp cã thªm c¸c ®iÒu kiÖn phô
th−êng g©y khã kh¨n cho ng−êi gi¶i trong Nh−ng ta cÇn ¸p dông cho bao nhiªu sè vµ lµ
viÖc −íc l−îng hÖ sè vµ xÐt ®iÒu kiÖn ®Ó dÊu nh÷ng sè nµo? C¨n cø vµo bËc cña c¸c biÕn
®¶ng thøc xÈy ra. Bµi viÕt nµy tr×nh bµy mét sè x vµ y trong c¸c biÓu thøc trªn, ta thÊy cÇn
ph−¬ng ph¸p ®¸nh gi¸ th«ng qua B§T C«-si ph¶i ¸p dông B§T C«-si lÇn l−ît cho
®Ó tõ ®ã, chuyÓn bµi to¸n cùc trÞ vÒ viÖc gi¶i x3 vµ y 3 cïng víi 5 h»ng sè d−¬ng t−¬ng øng
mét ph−¬ng tr×nh (PT) hoÆc hÖ ph−¬ng tr×nh kh¸c ®Ó lµm xuÊt hiÖn x vµ y . MÆt kh¸c
(HPT) mµ viÖc gi¶i quyÕt lµ dÔ dµng hoÆc cã
do x, y d−¬ng vµ vai trß cña chóng nh− nhau
®−êng lèi râ rµng h¬n, ®ã lµ ph−¬ng ph¸p
c©n b»ng hÖ sè. Còng tõ ph−¬ng ph¸p nµy, nªn ta dù ®o¸n P ( x; y ) ®¹t Max khi x = y .
víi mét chót s¸ng t¹o, chóng ta cã thÓ tæng 1
Tõ (1) suy ra x = y = 3 vµ ta ®i ®Õn lêi gi¶i
qu¸t vµ t¹o ra ®−îc nh÷ng bµi to¸n míi. 2
Tr−íc hÕt xin nªu l¹i mµ kh«ng chøng nh− sau.
minh hai B§T quen thuéc sau: Lêi gi¶i. ¸p dông B§T C«-si cho 6 sè
1
i) B§T C«-si tæng qu¸t: d−¬ng: 1 sè x3 vµ 5 sè , ta cã:
2
a1 + a2 + ... + an ≥ n n a1a2 ...an 5
5
ii) B§T C«-si suy réng: 1 1 −
x + 5. ≥ 6 6 x3 .   = 6.2 6 x
3

α1a1 + α 2 a2 + ... + α n an ≥ 2 2


1 1
(
≥ (α1 + α 2 + ... + an ) a1 a2 ...an
α1 α 2 αn
) a1 + a2 +...+ an DÊu “=” xÈy ra ⇔ x = 3
2
Trong hai B§T trªn th× a1 , a2 ,..., an kh«ng T−¬ng tù nh− vËy:
©m, α1 , α 2 ,..., α n d−¬ng vµ dÊu ®¼ng thøc xÈy 1 3 1

5 5

y + 5. ≥ 6 y .   = 6.2 6 y
3 6
ra khi vµ chØ khi a1 = a2 = ... = an . 2 2
Chóng ta b¾t ®Çu tï bµi to¸n sau: 1
DÊu “=” xÈy ra ⇔ y = 3
2
VÝ dô 1. Cho c¸c sè thùc d−¬ng x, y tháa
Céng theo vÕ c¸c B§T trªn ta ®−îc:
mLn ®iÒu kiÖn x3 + y 3 = 1 (1). T×m gi¸ trÞ lín
( )
5

nhÊt (Max) cña biÓu thøc ( x 3 + y 3 ) + 5 ≥ 6.2 6
x + y (2)
P( x; y ) = x + y 1
DÊu “=” xÈy ra ⇔ x = y = 3
.
2

TAÄP SAN TOAÙN HOÏC - 2007 30


TRÖÔØNG THPT CHUYEÂN HOAØNG VAÊN THUÏ - HOØA BÌNH
PhÇn I - S¸ng t¹o to¸n häc.

Tõ (1) vµ (2) suy ra: B»ng c¸ch lµm ng−îc l¹i c¸c b−íc trªn ta
P ( x; y ) = x + y ≤ 2 sÏ thu ®−îc Max { P ( x; y )} = (1 + 2 2 )
6 5 5
6 5

1
DÊu b»ng xÈy ra ⇔ x = y = 3
, tháa
2 NhËn xÐt. Tõ c¸ch ph©n tÝch trªn ta thÊy cã
mHn ®iÒu kiÖn (1). thÓ thay ®æi d÷ kiÖn cña bµi to¸n sao cho
VËy Max { P( x; y )} = 6 25 . HPT sau khi c©n b»ng hÖ sè cã thÓ gi¶i ®−îc.
Ch¼ng h¹n nh− c¸c bµi to¸n d−íi ®©y:
VÝ dô 2. Cho c¸c sè thùc d−¬ng x, y tháa Bµi to¸n 1. Cho c¸c sè nguyªn d−¬ng
mLn ®iÒu kiÖn x3 + y 3 ≤ 1 (3). T×m gi¸ trÞ lín m, p, q sao cho m ≥ Max { p, q} . HLy t×m
nhÊt (Max) cña biÓu thøc
GTLN cña biÓu thøc P ( x; y ) = ax p + y q
P( x; y ) = x + 2 y
trong hai tr−êng hîp sau, biÕt r»ng a lµ
Ph−¬ng ph¸p suy luËn: h»ng sè d−¬ng vµ x, y lµ c¸c biÕn sè kh«ng
ë vÝ dô 1, chóng ta ®H nhanh chãng dù ©m tháa mLn ®iÒu kiÖn x m + y m ≤ 1 :
®o¸n ®−îc Max P ( x; y ) ®¹t ®−îc khi x = y ,
m+q
tõ ®ã tÝnh ®−îc x, y . Nh−ng trong bµi to¸n i) p =
2
nµy, vai trß cña x vµ y lµ kh«ng b×nh ®¼ng.
2m + q
Tuy nhiªn ta hHy gi¶ sö P ( x; y ) ®¹t Max khi ii) p =
3
x =α Bµi to¸n 2. Cho c¸c sè thùc d−¬ng a, b, c, d
 nµo ®ã vµ dù ®o¸n α , β ë ®iÒu kiÖn
y = β vµ c¸c sè nguyªn m, n tháa mLn ®iÒu kiÖn
biªn cña (3), tøc lµ α 3 + β 3 = 1 (4). Ta viÕt: m > n > 0 . T×m gi¸ trÞ lín nhÊt cña biÓu
5 thøc P ( x; y; z ) = ax n + by n + cz n trong ®ã
x + 5.α ≥ 6 x . (α
3 3 6 3
)
3 5
= 6.α 2
x x, y, z lµ c¸c biÕn sè kh«ng ©m tháa mLn
5 ®iÒu kiÖn x m + y m + z m ≤ d .
y + 5.β ≥ 6 y . ( β
3 3 6 3
)
3 5
= 6.β 2
y
Suy ra VÝ dô 3. T×m gi¸ trÞ nhá nhÊt cña biÓu thøc
P ( x; y; z ) = a ( x 2 + y 2 ) + z 2 . Trong ®ã a lµ sè
5 5

(x 3
+ y 3 ) + 5. (α 3 + β 3 ) ≥ 6.α 2
x + 6.β 2
y
thùc d−¬ng vµ x, y, z lµ c¸c biÕn sè tháa mLn
®iÒu kiÖn xy + yz + zx = 1 (6)
§Ó xuÊt hiÖn P ( x; y ) ë vÕ ph¶i, ta cÇn
Ph−¬ng ph¸p suy luËn:
chän α , β sao cã tû lÖ:
Do vai trß cña x vµ y lµ nh− nhau nªn ta
5 5
6.α 2 x : 6.β 2 y = 1. x : 2. y dù ®o¸n P ( x; y; z ) ®¹t Min khi
5 x = y = α z (α > 0) (7). ¸p dông B§T C«-si
 α 2 1 α 1 cho hai sè d−¬ng ta cã
⇔   = ⇔ = 5 (5)
β  2 β 4 x 2 + y 2 ≥ 2 xy ≥ 2 xy
x 2 + (α z ) ≥ 2 x α z ≥ 2α xz ⇔
2

VËy tõ (4) vµ(5) ta thu ®−îc HPT:


 1
α =
1 ⇔ x 2 + α z 2 ≥ 2 xz
 α 1  α
 =5 
3
1+ 25 2
 β 4 ⇔  y 2 + (α z ) ≥ 2 y α z ≥ 2α yz ⇔
2
5
α 3 + β 3 = 1 β = 4
  1

3
1+ 25 2 ⇔ y 2 + α z 2 ≥ 2 yz
α
Tõ c¸c B§T trªn suy ra:

TAÄP SAN TOAÙN HOÏC - 2007 31


TRÖÔØNG THPT CHUYEÂN HOAØNG VAÊN THUÏ - HOØA BÌNH
PhÇn I - S¸ng t¹o to¸n häc.

 1 2 Vµ ta cÇn t×m Min cña biÓu thøc


1 +  ( x + y ) + 2α z ≥ 2 ( xy + yz + zx )
2 2
P ( x; y; z ) = x + 2 y + 3 z
 α
x =αz
Gi¶ sö P ( x; y; z ) ®¹t Min khi 
VÕ ph¶i cña B§T trªn lµ h»ng sè, v× vËy y = β z
ta cÇn t×m α ®Ó cã tû lÖ:
¸p dông B§T C«-si suy réng ta cã:
 1 12 xyz ≥ 2 x + 8 y + 21z ≥
1 +  : 2α = a :1
 α y
x
1 + 1 + 8a ≥ 2α   + 8β   + 21z ≥
⇔ 2aα 2 − α − 1 = 0 ⇒ α = , α  β 
4a 1

1 − 1 + 8a   x 2α  y 8 β 21  2α +8 β + 21
α= < 0 lo¹i. ≥ ( 2α + 8β + 21)      z 
 
4a α   β  
Cïng víi (6) vµ (7) ta cã HPT:
⇒ x8 β + 21 y 2α + 21 z 2α +8 β ≥ A (α , β ) (10)
 xy + yz + zx = 1 (α 2 + 2α ) z 2 = 1
 ⇔ Trong ®ã biÓu thøc A (α , β ) chØ phô
 x = y =αz  x = y = α z
thuéc vµo α , β .
Gi¶i HPT nµy víi α nh− trªn ta ®−îc:
Còng theo B§T C«-si suy réng ta cã:
 16a 2
 z = ± P ( x, y, z ) = x + 2y + 3z
( )
2
 8a + 1 8a + 1 + 1 y
⇔ x
= α   + 2β   + 3z ≥
x = y = ± 4a α  β 

 (
8a + 1 8a + 1 + 1 )   x α  y  2 β 3  α + 2 β +3
1

B»ng c¸ch lµm ng−îc l¹i ta tÝnh ®−îc ≥ (α + 2 β + 3 )      z 


 
xy + yz + zx 4 α   β  
Min { P ( x; y; z )} = =
α
1
1 + 1 + 8a = B (α , β ) ( xα y 2 β z 3 )α + 2 β +3 (11)
NhËn xÐt. B»ng c¸ch lµm t−¬ng tù nh− trªn
chóng ta cã thÓ gi¶i trän vÑn ®−îc bµi to¸n Trong ®ã biÓu thøc B (α , β ) chØ phô
tæng qu¸t h¬n sau: thuéc vµo α , β .
Bµi to¸n 3. Cho c¸c h»ng thùc d−¬ng a, b, c §èi chiÕu (10) vµ (11) ta thÊy cÇn chän
vµ c¸c biÕn sè x, y, z tháa mLn ®iÒu kiÖn α , β sao cho cã tû lÖ:
xy + yz + zx ≥ 1 . T×m gi¸ trÞ nhá nhÊt cña
α : 2β : 3 = ( 8β + 21) : ( 2α + 21) : ( 8β + 2α )
biÓu thøc P ( x; y; z ) = ax 2 + by 2 + cz 2 .
 8β + 21 α
 8β + 2α = 3
VÝ dô 4. XÐt c¸c sè thùc d−¬ng a, b, c tháa 
⇔
mLn ®iÒu kiÖn 21ab + 2bc + 8ca ≤ 12 . HLy
 2α + 21 = 2 β
t×m gi¸ trÞ nhá nhÊt cña biÓu  8β + 2α 3
1 2 3
thøc P (a; b; c) = + + . 2α 2 + 8αβ = 24 β + 63
a b c ⇔
16 β + 4αβ = 6α + 63
2
(§Ò thi chän §TVN dù thi IMO 2001)
Ph−¬ng ph¸p suy luËn: 2α 2 − 63
1 1 1 Tõ PT thø nhÊt ⇒ β = . Thay
§Æt a = , b = , c = . §iÒu kiÖn cña 8(3 − α )
x y z
vµo PT thø hai ta cã:
bµi to¸n të thµnh 2 x + 8 y + 21z ≤ 12 xyz (9).

TAÄP SAN TOAÙN HOÏC - 2007 32


TRÖÔØNG THPT CHUYEÂN HOAØNG VAÊN THUÏ - HOØA BÌNH
PhÇn I - S¸ng t¹o to¸n häc.

2
 2α 2 − 63  5 5 2 2
2α 2 − 63 ⇔ x = 3 x1 = 3, y = y1 = , z = z1 =
16 
 8 ( 3 − α ) 
+ 4 α = 6α + 63
  8 ( 3 − α ) 1 4
4
3
4 3 3

⇔ 4α + 78α − 306α − 567 = 0


3 2 ⇔ a = ,b = ,c =
3 5 2
⇔ ( 2α − 9 ) ( 2α 2 + 48α + 63) = 0 15
VËy Min P ( a, b, c ) = .
9 15 2
⇔α = ( do α > 0 ) ⇒ β = .
2 8 NhËn xÐt. Së dÜ ta ®Æt c¸c biÕn míi x1 , y1 , z1
Khi P ( x, y, z ) ®¹t Min th× tÊt c¶ c¸c B§T lµ v× ta ®H x¸c ®Þnh ®−îc bé sè (x,y,z) ®Ó
trªn ®Òu trë thµnh ®¼ng thøc, nghÜa lµ P ( x, y, z ) ®¹t Min. MÆt kh¸c viÖc xÐt dÊu
  b»ng sÏ trë nªn dÔ dµng h¬n bÕu c¸c biÕn
2 x + 8 y + 21z = 12 x=3 tham gia khi xÈy ra dÊu ®¼ng thøc lµ b»ng
  nhau vµ ®Òu b»ng 1.
 9  5
 x = α z = z ⇔ y = Mét ®iÒu thó vÞ vµ ®¸ng chó ý ë ®©y lµ
 2  4 c¸c B§T (12), (13) t−¬ng ®èi ®¬n gi¶n,
 18  2 nh−ng qua phÐp ®æi biÕn ®H trë thµnh B§T
 y = βz = 5 z  z = 3 kh¸c phøc t¹p h¬n rÊt nhiÒu. Chóng ta hHy

Tíi ®©y, ®iÓm mÊu chèt cña bµi to¸n ®H thö vËn dông ®iÒu nµy ®Ó t¹o ra nh÷ng bµi
®−îc gi¶i quyÕt vµ ta ®i ®Õn mét lêi gi¶i to¸n míi rÊt thó vÞ, xuÊt ph¸t tõ bæ ®Ò sau:
t−¬ng ®èi ng¾n gän cho bµi to¸n nh− sau: Bæ ®Ò: Cho c¸c sè thùc α , β , γ , λ ≥ 0 vµ
5 2 x, y, z , t > 0 . Khi ®ã ta cã:
Lêi gi¶i. §Æt x = 3 x1 , y = y1 , z = z1 khi i) NÕu
4 3
®ã ®iÒu kiÖn (9) trë thµnh α x + β y + γ z + λt ≤ (α + β + γ + λ ) xyzt
5 2 5 2 th×
2.3 x1 + 8. y1 + 21. z1 ≤ 12.3 x1. y1. z1
4 3 4 3 ( β + γ + λ ) x + (γ + λ + α ) y + ( λ + α + β ) z +
⇔ 3 x1 + 5 y1 + 7 z1 ≤ 15 x1 y1 z1 .
+ ( α + β + γ ) t ≥ 3 (α + β + γ + λ ) (14)
P ( x, y, z ) = P ( x1 , y1 , z1 ) = ii) NÕu
5
= 3 x1 + 2. y1 + 3 z1
2 ( β + γ + λ ) x + (γ + λ + α ) y + ( λ + α + β ) z +
+ ( α + β + γ ) t ≥ 3 (α + β + γ + λ )
4 3
th×
1
= ( 6 x1 + 5 y1 + 4 z1 ) α x + β y + γ z + λt ≥ (α + β + γ + λ ) xyzt (15)
2
¸p dông B§T C«-si tæng qu¸t cho 15 sè Chøng minh. Tr−êng hîp α = β = γ = λ = 0
d−¬ng ta cã: th× bæ ®Ò hiÓn nhiªn ®óng. Ta xÐt khi
15 x1 y1 z1 ≥ 3 x1 + 5 y1 + 7 z1 ≥ 1515 x y z3 5 7
1 1 1 (12) α 2 + β 2 + γ 2 + λ2 > 0.
1 i) ¸p dông B§T C«-si suy réng ta cã:
P ( x, y , z ) = ( 6 x1 + 5 y1 + 4 z1 ) ≥
2 (α + β + γ + λ ) xyzt ≥ α x + β y + γ z + λt ≥
1 1
≥ .15.15 x16 y15 z14 (13) ≥ (α + β + γ + λ ) ( x y z t α β
)
γ λ α + β +γ + λ
2
β +γ + λ γ + λ +α λ +α + β α + β + γ
Tõ (12) suy ra x16 y15 z14 ≥ 1 , do ®ã tõ (13) ⇒x y z t ≥1
15 Nh− vËy: ( β + γ + λ ) x + ( γ + λ + α ) y +
ta ®−îc P ( x, y, z ) ≥
2 + ( λ + α + β ) z + (α + β + γ ) t ≥
§¼ng thøc xÈy ra ⇔ x1 = y1 = z1 = 1
≥ 3 (α + β + γ + λ ) ×

TAÄP SAN TOAÙN HOÏC - 2007 33


TRÖÔØNG THPT CHUYEÂN HOAØNG VAÊN THUÏ - HOØA BÌNH
PhÇn I - S¸ng t¹o to¸n häc.

1 1 1 1
× ( x β +γ + λ y γ + λ +α z λ +α + β t α + β +γ )α + β +γ + λ ≥ - Thay t = 1, λ = 1, α = , β = , γ = vµo
2 3 6
≥ 3 (α + β + γ + λ ) 1 2 4
(14) vµ ®Æt x = ,y = ,z = ta cã bµi
§¼ng thøc xÈy ra ⇔ x = y = z = t = 1 . 2a 3b 3c
ii) ¸p dông B§T C«-si suy réng ta cã: to¸n sau:
Bµi to¸n 5. Cho c¸c sè thùc d−¬ng a, b, c
3 (α + β + γ + λ ) ≥ ( β + γ + λ ) x +
tháa mLn ®iÒu kiÖn 8a 2 ( b + c ) + 27abc ≤ 16 .
+ ( γ + λ + α ) y + ( λ + α + β ) z + (α + β + γ ) t ≥
5 10 22
≥ 3 (α + β + γ + λ ) × Chøng minh r»ng: + + ≥ 6 . §¼ng
4a 9b 9c
1 thøc x¶y ra khi nµo?
×( x β +γ + λ
y γ + λ +α
z λ +α + β α + β + γ
t ) α + β +γ + λ

⇒ 1 ≥ x β +γ + λ y γ + λ +α z λ +α + β t α + β +γ - V× khi xÈy ra ®¼ng thøc ë hai Bµi to¸n 4 vµ


1 2 4
1
5 ®Òu cã a = , b = , c = nªn khi kÕt hîp
⇔ ( xα y β z γ t λ )α + β +γ + λ ≥ xyzt 2 3 3
Nh− vËy: hai bµi to¸n trªn ta cã:
α x + β y + γ z + λt ≥ Bµi to¸n 6. Cho c¸c sè thùc d−¬ng a, b, c
1
tháa mLn ®iÒu kiÖn 72ab + 9bc + 24ca +
≥ (α + β + γ + λ ) ( x α y β z γ t λ ) α + β + γ + λ ≥ 18abc ≤ 56 vµ 8a 2 ( b + c ) + 27abc ≤ 16 .
≥ (α + β + γ + λ ) xyzt Chøng minh r»ng:
17 19 166
+ + ≥ 21 .
§¼ng thøc x¶y ra ⇔ x = y = z = t = 1 . 4a 9b 9c
§¼ng thøc x¶y ra khi nµo?
Bæ ®Ò ®−îc chøng minh.

Sö dông bæ ®Ò trªn b»ng c¸ch thay vµo 1


- Thay t = , λ = 1, α = 1, β = 2, γ = 3 vµo
nh÷ng gi¸ trÞ ®Æc biÖt vµ b»ng nh÷ng c¸ch x
ph¸t biÓu kh¸c nhau, ta sÏ cã nh÷ng kÕt qu¶ 1 2 4
(14) vµ ®Æt x = ,y = ,z = ta cã bµi
kh¸c nhau: 2a 3b 3c
- Víi t = 1, λ = 0, α = 3, β = 5, γ = 7 , thay x, to¸n sau:
5 2 Bµi to¸n 7. Cho c¸c sè thùc a, b, c d−¬ng
y, z, t lÇn l−ît bëi 3x, y , z vµo (14), sau
4 3 3 10 16
tháa mLn ®iÒu kiÖn + + + 12a ≤ 21 ,
1 1 1 a 3b 3c
®ã ®Æt a = , b = , c ta ®−îc Bµi to¸n vÝ
x y z 1 4 4 28
chøng minh r»ng + + + 2a ≥ .
dô 4. 2a 3b c 9abc
§¼ng thøc x¶y ra khi nµo?
- Thay t = 1, λ = 1, α = 1, β = 2, γ = 3 vµo (14) B»ng c¸ch thay ®æi d÷ kiÖn bµi to¸n theo
1 2 4 h−íng trªn chóng ta sÏ cã ®−îc rÊt nhiÒu bµi
vµ ®Æt x = ,y = ,z = ta cã bµi to¸n: to¸n míi. C¸c b¹n hHy thö tiÕp tôc suy nghÜ
2a 3b 3c theo h−íng trªn vµ theo h−íng tæng qu¸t cho
Bµi to¸n 4. Cho c¸c sè thùc d−¬ng a, b, c tr−êng hîp nhiÒu biÕn h¬n n÷a. §Ó kÕt thóc
tháa mLn ®iÒu kiÖn 72ab + 9bc + 24ca + bµi viÕt nµy, ®Ò nghÞ c¸c b¹n gi¶i mét sè bµi
+ 18abc ≤ 56. Chøng minh r»ng: tËp sau vµ hHy cè g¾ng më réng chóng theo
3 10 16 c¸ch cña m×nh. §ã lµ mét viÖc lµm thùc sù
+ + ≥ 15 . §¼ng thøc x¶y ra khi nµo?
a b c cÇn thiÕt khi häc to¸n . Chóc c¸c b¹n thµnh
c«ng!
(Xem tiÕp trang 80)

TAÄP SAN TOAÙN HOÏC - 2007 34


TRÖÔØNG THPT CHUYEÂN HOAØNG VAÊN THUÏ - HOØA BÌNH
PhÇn I - S¸ng t¹o to¸n häc.

PHÖÔNG PHAÙP SÖÛ DUÏNG ÑÒNH NGHÓA

ÑEÅ TÍNH GIÔÙI HAÏN


Lª b¶o kh¸nh
Líp 12A To¸n, K01- 04 - THPT Chuyªn NguyÔn HuÖ, Hµ T©y
Sv. Khoa Kinh tÕ §èi ngo¹i, §H Ngo¹i Th−¬ng Hµ Néi

Lêi Ban biªn tËp. C¸c b¹n th©n mÕn! Nh− - NÕu lim xn = a , xn > b víi mäi n > N th× a
n →∞
c¸c b¹n ®L biÕt, cã rÊt nhiÒu ph−¬ng ph¸p
®Ó tÝnh giíi h¹n cña mét dLy sè. Mçi ph−¬ng ≥ b.
ph¸p ®Òu cã nh÷ng ®iÓm m¹nh ®Æc tr−ng - NÕu lim xn = a , lim yn = b vµ xn ≥ yn víi
n →∞ n →∞
cho riªng m×nh. Tuy kh«ng ph¶i lµ mét mäi n > N, th× a ≥ b
ph−¬ng ph¸p míi l¹, nh−ng sö dông ®Þnh - NÕu lim xn = a th× lim xn = a . §iÒu
nghÜa ®Ó tÝnh giíi h¹n vÉn lµ mét ph−¬ng n →∞ n →∞

ph¸p kinh ®iÓn, nã mang mét s¾c th¸i vµ vÎ ng−îc l¹i kh«ng ®óng.
®Ñp riªng. Vµ còng ®óng nh− t¸c gi¶ bµi b¸o - lim xn = 0 ⇔ lim xn = 0 .
n →∞ n →∞
nµy nhËn xÐt, ®©y lµ mét ph−¬ng ph¸p rÊt
s©u s¾c vÒ mÆt to¸n häc. §Ó vËn dông thµnh Trong c¸c bµi to¸n vÒ t×m giíi h¹n, ta cã
th¹o ph−¬ng ph¸p nµy th× chóng ta cÇn ph¶i thÓ dïng mét sè dÊu hiÖu/ph−¬ng ph¸p nh−
cã mét c¸i nh×n s©u s¾c vÒ b¶n chÊt còng nguyªn lý ®¬n ®iÖu, nguyªn lý kÑp, ... Tuy
nh− ý nghÜa cña lý thuyÕt giíi h¹n. Xin giíi nhiªn, trong mét sè bµi to¸n khã cÇn ph¶i
thiÖu cïng b¹n ®äc: vËn dông trùc tiÕp ®Þnh nghÜa ®Ó chøng minh
nh−ng ®iÒu nµy ®ßi hái chóng ta ph¶i hiÓu
I. §Þnh nghÜa. biÕt mét c¸c t−¬ng ®èi s©u vÒ giíi h¹n. T«i
Tr−íc hÕt, chóng ta hHy cïng nh¾c l¹i vÒ xin b¾t ®Çu c¸c vÝ dô tõ c¬ b¶n ®Õn phøc t¹p.
®Þnh nghÜa giíi h¹n cña mét dHy sè:
Cho dLy sè thùc ( xn ) , a ∈ R . Ta nãi III. Mét sè vÝ dô minh häa.
1. Bµi to¸n1.
lim xn = a nÕu ∀ ε > 0, ∃ N sao cho ∀ n >
n →∞ Cho dHy sè thùc ( xn ) kh«ng ©m tháa mHn
N th× x n − a < ε . xn
lim = 0 . Chøng minh r»ng
n →∞ n
II. Vµi tÝnh chÊt thËt c¬ b¶n. Max { xi }
lim i =1,n = 0.
- Giíi h¹n cña mét dLy nÕu tån t¹i th× duy n →∞ n
nhÊt.
Lêi gi¶i . Tõ gi¶ thiÕt, suy ra víi mäi ε > 0,
- lim xn = a , a ∈ (p; q) th× tån t¹i N sao cho
n →∞ x
tån t¹i sè tù nhiªn m sao cho n < ε ,
∀ n > N, xn ∈ (p; q). n
∀n≥m

TAÄP SAN TOAÙN HOÏC - 2007 35


TRÖÔØNG THPT CHUYEÂN HOAØNG VAÊN THUÏ - HOØA BÌNH
PhÇn I - S¸ng t¹o to¸n häc.

Max { xi } ∞

⇒ ∃ n0 ≥ m tháa i =1, n

1. ∑c1
n = +∞ .
n0
2. lim yn = +∞ .
xk x n →∞
NÕu (m - 1) ≥ k ≥ 1 th× ta cã < k <ε
n k
3. lim xn = 0 .
xn xk n →∞
NÕu n ≥ k ≥ m th× ta cã: < <ε Chøng minh.
n k
Max { xi }
1 ⇒ 2. Theo bÊt ®¼ng thøc Bernoulli, ta cã :
VËy ta lu«n cã i =1,n <ε n n
n
yn = ∏ (1 + ci ) ≥ 1+ ∑ ci
Max { xi } 1 1
⇒ lim i =1,n = 0 (®pcm) ⇒ lim yn = +∞ .
n →∞ n n →∞

2. Bµi to¸n 2. n

Cho dLy sè thùc ( an ) kh«ng ©m tháa 2 ⇒ 3. Do xn yn = ∏ (1 − ci2 ) <1 nªn dÔ cã


1
n
a
mLn lim n = 0 vµ n ≥ ∑ ai . Chøng minh lim xn = 0 .
n →∞
n →∞ n
1 ∞
n

∑a i
3 ⇒ 1. Gi¶ sö ∑c 1
n < +∞ , ta cã ngay tån t¹i

r»ng lim i =1
=0. 1 m
n →∞ n2 n0 ∈ N * sao cho
i = n0 +1 2
∑c i <
, víi mäi m > n0.

Lêi gi¶i. Theo Bµi to¸n 1 ta cã : Khi ®ã, víi mäi m > n0, ta cã:
Max { xi } m  m  xn
lim i =1,n = 0 . MÆt kh¸c, dÔ thÊy xm = xn0 . ∏ (1 − ci ) ≥ xn0 . 1 − ∑ ci  > 0
n →∞ n i = n0 +1  i = n0 +1  2
n n
(Còng theo bÊt ®¼ng thøc Bernoulli)
Max { xi }
i =1, n
Max { xi }
i =1, n
∑ ai ∑a i
≥ . i =1
≥ i =1
≥0 ⇒ lim xn > 0 , tr¸i gi¶ thiÕt ⇒ ®pcm.
n n n n2 n →∞
Theo nguyªn lý kÑp, suy ra ®pcm.
4. Bµi to¸n 4.
3. Bµi to¸n 3.
Cho 2 dLy ( xn ) , ( yn ) tháa mLn
§©y lµ mét bæ ®Ò cã nhiÒu øng dông. lim xn = 0 ,
n →∞
n
Víi dLy sè thùc ( xn ) . §Æt S n = ∑ xi vµ kÝ

1 ∑y
1
n < +∞ . Chøng minh r»ng:

hiÖu lim S n = ∑ xi . XÐt dLy sè
n →∞
( cn ) tháa  n 
lim  ∑ xi yn +1−i  = 0
1 n →∞
 1 
mLn ®iÒu kiÖn 0 < cn < 1 , víi mäi n = 0, 1, 2,
n n
Lêi gi¶i. Tõ gi¶ thiÕt suy ra tån t¹i c¸c sè K,
… §Æt xn = ∏ (1 − ci ) , yn = ∏ (1 + ci ) .
1 1 L > 0 sao cho x n < K, víi mäi n ≥ 1 vµ
Khi ®ã 3 kh¼ng ®Þnh sau lµ t−¬ng ®−¬ng: ∞

∑y
1
n < L . Ngoµi ra ta cßn cã:

TAÄP SAN TOAÙN HOÏC - 2007 36


TRÖÔØNG THPT CHUYEÂN HOAØNG VAÊN THUÏ - HOØA BÌNH
PhÇn I - S¸ng t¹o to¸n häc.

Víi mäi ε > 0, tån t¹i sè tù nhiªn m sao 6. Bµi to¸n 6.


n
ε Cho dLy sè thùc kh«ng ©m ( an ) tháa mLn
cho ∑y
i=m K+L
i, ∀ n > m.
<

Víi mäi ε > 0, tån t¹i sè tù nhiªn n0 sao ∑a n < +∞ . Víi mçi x > 0, kÝ hiÖu N(x) lµ
ε
1

cho xn < , ∀ n > n 0. sè sè an > x . Chøng minh r»ng lim xN ( x) .


K+L x →0

Chän N = Max {m; n0 } th× víi mäi n > N,


ta cã:
Lêi gi¶i. Do ∑a 1
n < +∞ nªn ∀ ε > 0, ∃ n0
n N n ∞
ε
∑ xi y n+1−i = ∑ xi y n+1−i + ∑ xi y n+1−i ≤ sao cho ∑a
n0 +1
n <
2
i =1 i =1 i = N +1

ε ε ε ε
≤ K + L = ε ⇒ ®pcm ⇒ Víi x < th× N ( x) ≤ n0 + < 2n0
K+L K+L 2n0 2x
5. Bµi to¸n 5. ∞
ε
Cho dHy sè thùc d−¬ng ( an ) tháa mHn : Chó ý r»ng tõ ∑a
n0 +1
n <
2
suy ra cã kh«ng

1) ∑a < +∞ ε
n qu¸ sè ai > x, i ≥ n0+1.
1 2x
2) an +1 < an (1 + an ) víi mäi n. ⇒ xN(x) < ε ⇒ ®pcm.
Chøng minh r»ng lim nan = 0
n →∞ Trªn ®©y lµ mét sè vÝ dô g¾n trùc tiÕp víi
®Þnh nghÜa vµ më réng ®Þnh nghÜa giíi h¹n
Lêi gi¶i. Gi¶ sö lim nan ≠ 0 . Chän ε bÊt k× dHy. Cã rÊt nhiÒu bµi to¸n vµ ®Ò thi cÇn sö
n →∞
dông ®Þnh nghÜa ®Ó trùc tiÕp t×m giíi h¹n
sao
(Nh− c©u 3 - ®Ò thi HSGQG 2000 - 2001
cho 1 > ε > 0. Tõ gi¶ thiÕt ph¶n chøng, ta
B¶ng A) mong c¸c b¹n chó ý vµ t×m hiÓu
thÊy cã v« sè n ®Ó nan > ε . Chän m > 1 sao thªm!
cho mam > ε ⇒ am −1 (1 + am −1 ) > am >
ε ε ε ε
> > (1 + ) ⇒ am-1 > ... ************************************
m m +1 m +1 m +1
TiÕp tôc qu¸ tr×nh nµy, ta ®−îc
ε
a m
>
m−   m
m+ 
2
2
LÊy tæng l¹i, ta ®−îc:
ε ε ε
am + ... + a m
> +....+ >
m 4
m−   m
m+ 2
2
Qu¸ tr×nh nµy cã thÓ tiÕp diÔn, suy ra ta
cã thÓ chän ®−îc v« sè tæng rêi cã ®é lín
ε ∞
v−ît ⇒ ∑ a n = +∞ , M©u thuÉn ⇒ ®pcm.
4 1

TAÄP SAN TOAÙN HOÏC - 2007 37


TRÖÔØNG THPT CHUYEÂN HOAØNG VAÊN THUÏ - HOØA BÌNH
PhÇn I - S¸ng t¹o to¸n häc.

DÔ thÊy r»ng diÓm Lemoine cña mét tam


gi¸c th× n»m trong tam gi¸c ®ã. §Þnh lý sau

Ñieåm LEMOINE cho ta mét tiªu chuÈn ®Ó nhËn biÕt ®iÓm


Lemoine.
§Þnh lÝ 1:

Trong Tam Giaùc


Cho ®iÓm L n»m trong ∆ABC . Gäi H, J,
K lÇn l−ît lµ h×nh chiÕu cña L trªn c¸c c¹nh
BC, CA, AB t−¬ng øng. Khi ®ã L lµ ®iÓm
Lemoine cña ∆ABC nÕu vµ chØ nÕu L lµ
Lª v¨n ®Ýnh träng t©m cña tam gi¸c HJK.
Líp 3clc – k51 to¸n, ®hsp hµ néi i

Chóng ta h¼n ai còng biÕt ®Õn bµi to¸n


næi tiÕng sau:
Bµi to¸n 1:T×m ®iÓm M trong mÆt ph¼ng
∆ABC sao cho tæng MA+MB+MC ®¹t gi¸
trÞ nhá nhÊt.
§©y lµ mét bµi to¸n khã ®−îc ®Æt ra kh¸ Chøng minh ®¼ng thøc nµy dùa trªn 2
l©u tr−íc khi Toricelli - ng−êi ®Çu tiªn t×m ra ®¼ng thøc vÐct¬ quen thuéc sau mµ viÖc chi
lêi gi¶i. ThÕ nh−ng khi n©ng c¸c ®¹i l−îng tiÕt ho¸ kh«ng cã g× lµ khã kh¨n, c¸c b¹n
trong Bµi to¸n 1 lªn bËc hai th× vÊn ®Ò l¹i hÕt hHy thiÕt lËp
coi nh− bµi tËp.
  
søc ®¬n gi¶n . §ã lµ néi dung bµi to¸n mµ S (∆LBC ) LA + S (∆LCA) LB + S (∆LAB ) LC = 0 ,
chóng ta sau nµy ®Òu biÕt r»ng, träng t©m G vµ a  b  a   , trong ®ã
LH + LJ + LK = 0
cña ∆ABC lµ lêi gi¶i duy nhÊt cña nã. LH LJ LK
Bµi to¸n 1a: S (∆XYZ ) chØ diÖn tÝch cña ∆XYZ .
T×m ®iÓm M n»m trªn mÆt ph¼ng tam Mét ®Æc tr−ng ®−a ®Õn c¸ch dùng ®iÓm
gi¸c ABC sao cho ®¹i l−îng Lemoine ®−îc chØ ra trong ®Þnh lÝ sau:
MA + MB + MC ®¹t gi¸ trÞ nhá nhÊt.
2 2 2 §Þnh lÝ 2:
Mét bµi to¸n kh¸c còng ®−îc ®Æt ra mét C¸c ®−êng ®èi trung cña ∆ABC t¹i ®iÓm
c¸ch rÊt tù nhiªn tõ Bµi to¸n 1. Lemoine cña nã.
Cho ®iÓm M n»m trong tam gi¸c ABC, Chøng minh. Gäi AM, AD, AS lÇn l−ît lµ
gäi H, J, K lÇn l−ît lµ h×nh chiÕu cña M trªn trung tuyÕn , ph©n gi¸c, vµ ®−êng ®èi trung
c¸c c¹nh BC, CA, AB t−¬ng øng. X¸c ®Þnh vÞ (lµ ®−êng ®èi xøng víi trung tuyÕn qua ph©n
trÝ cña ®iÓm M sao cho tæng S = MH + MJ gi¸c) xuÊt ph¸t tõ ®Ønh A cña tam gi¸c ABC.
+ MK ®¹t gi¸ trÞ nhá nhÊt.
ViÖc ph¸t triÓn Bµi to¸n 1 theo h−íng
n©ng c¸c ®¹i l−îng cña tæng S lªn bËc hai sÏ
dÉn ta tíi kh¸i niÖm sau, trong ®ã a, b, c lµ
ký hiÖu ®é dµi ba c¹nh BC, CA vµ AB cña
tam gi¸c ABC.
§iÓm Lemoine trong tam gi¸c:
§iÓm L thuéc mÆt ph¼ng chøa tam gi¸c
ABC ®−îc gäi lµ ®iÓm Lemoine cña tam gi¸c
   
®ã nÕu a 2 .LA + b2 .LB + c 2 .LC = 0 .

TAÄP SAN TOAÙN HOÏC - 2007 38


TRÖÔØNG THPT CHUYEÂN HOAØNG VAÊN THUÏ - HOØA BÌNH
PhÇn I - S¸ng t¹o to¸n häc.

Trªn c¸c tia AB, AC lÇn l−ît lÊy c¸c ®iÓm MH MJ MK


C1, B1 sao cho AB =AB1, AC=AC1. = = ⇔
a b c
Ký hiÖu S1 = AS ∩ B1C1 vµ gäi B2 vµ C2 S (∆MBC ) S (∆MCA) S (∆MAB )
⇔ = =
lÇn l−ît lµ giao ®iÓm cña ®−êng th¼ng BC a 2
b 2
c2
  
 
víi c¸c ®−êng th¼ng qua B1, C1 vµ song song ⇔ a 2 .MA + b2 .MB + c 2 .MC = 0
víi AS. ⇔ M lµ ®iÓm Lemoine cña ∆ABC
Tõ tÝnh ®èi xøng, dÔ thÊy S1 lµ trung ®iÓm
B1C1, do ®ã theo ®Þnh lÝ Talet, S lµ trung Lêi kÕt. §iÓm Lemoine cña ∆ABC cã nhiÒu
  
®iÓm B2C2: B2 S + C2 S = 0 (1) tÝnh chÊt thó vÞ kh¸c. Ch¼ng h¹n, nã còng lµ
Còng theo ®Þnh lÝ Talet ta cã: ®iÓm lµm cùc tiÓu biÓu thøc
BS AB AB a CS CA CA b a MA + b MB + c MC , hay c¸c h×nh
2 2 2 2 2 2
= = = = = =
C2 S AC1 AC b B2 S B1 A BA c chiÕu cña nã lªn c¸c c¹nh cña ∆ABC t¹o
   thµnh mét tam gi¸c néi tiÕp ∆ABC cã tæng
Tõ ®ã vµ (1) suy ra b 2 .BS + c 2 .CS = 0 (2). b×nh ph−¬ng c¸c c¹nh nhá nhÊt mµ trong
VËy, nÕu gäi L lµ ®iÓm Lemoine cña tam khu«n khæ bµi viÕt kh«ng thÓ tr×nh bµy cÆn
   
gi¸c ABC th× a 2 .LA + b2 .LB + c 2 .LC = 0 kÏ ®−îc.
   
( ) (
⇔ a 2 LS + SA . + b 2 . LS + SB + ) §Ó kÕt thóc xin nh¾c l¹i c¸c hÖ thøc x¸c
  
(
c 2 LS + SC = 0)  
®Þnh träng t©m, t©m ®−êng trßn néi tiÕp vµ
®iÓm Lemoine cña mét tam gi¸c:
⇔ ( a 2 + b 2 + c 2 ) LS + a 2 SA +    
   GA + GB + GC = 0
   
+b 2 SB + c 2 SC = 0
   a.IA  2  IC 2=
+ b.IB + c. 0 
⇔ ( a 2 + b 2 + c 2 ) LS + a 2 SA = 0 a .LA + b .LB + c .LC = 0
2

⇒ L n»m trªn ®−êng ®èi trung SA. T−¬ng tù Nh÷ng hÖ thøc nµy gîi ý cho ta nghiªn
nh− vËy, ta còng cã L n»m trªn hai d−êng cøu c¸c ®iÓm M tho¶ mHn:
   
®èi trung cßn l¹i cña ∆ABC ⇒ ®pcm. a k .MA + bk .MB + c k .MC = 0
Nh÷ng tÝnh chÊt cña ®iÓm M nh− thÕ
B©y giê ta trë l¹i xÐt bµi to¸n ®H nªu ë ®ang chê b¹n kh¸m ph¸. Chóc c¸c b¹n thµnh
trªn . c«ng!

Bµi to¸n 1b.


T×m ®iÓm M n»m trong ∆ABC sao cho
tæng c¸c b×nh ph−¬ng c¸c kho¶ng c¸ch tõ M
®Õn c¸c c¹nh cña tam gi¸c lµ nhá nhÊt. T«i nghe th× t«i sÏ quªn
Lêi gi¶i. VÉn ký hiÖu H, J, K nh− ë h×nh vÏ T«i nghÜ th× t«i sÏ nhí
thø nhÊt. Ta cã: T«i häc th× t«i sÏ hiÓu
4 S ∆2ABC = ( a.MH + b.MJ + c.MK ) ≤
2

≤ ( a 2 + b 2 + c 2 )( MH 2 + MJ 2 + MK 2 )
4 S ∆2ABC
⇒ MH 2 + MJ 2 + MK 2 ≥
a2 + b2 + c2
§¼ng thøc x¶y ra khi vµ chØ khi

TAÄP SAN TOAÙN HOÏC - 2007 39


TRÖÔØNG THPT CHUYEÂN HOAØNG VAÊN THUÏ - HOØA BÌNH
PhÇn I - S¸ng t¹o to¸n häc.

B ' R A ' P C 'Q


ta cã . . =1
B ' P A'Q C ' R
Suy ra A’, B’, C’ th¼ng hµng.
Sau khi chøng minh bµi to¸n trªn, t«i ®H
ÑÖÔØNG thö më réng nã cho elipse (Thay ®−êng trßn
bëi elipse). T«i ®H cè g¾ng x©y dùng kh¸i
TROØN niÖm vÒ ph−¬ng tÝch cña mét ®iÓm víi elipse
nh−ng ®H thÊt b¹i. Mäi chuyÖn chØ s¸ng tá

VAØ
khi t«i häc vÒ phÐp chiÕu song song. ThËt
VAØ vËy, nÕu ta ®em chiÕu mÆt ph¼ng chøa
®−êng trßn ngo¹i tiÕp lôc gi¸c lªn mét mÆt
elipse ph¼ng kh«ng song song víi nã th× thay cho
®−êng trßn, ta cã ®−êng elipse.
L−u nh− hoµ Ta cã ®Þnh lý 2. Mét lôc gi¸c néi tiÕp trong
Chuyªn To¸n K02 – 05 mét elip th× 3 giao ®iÓm cña 3 cÆp c¹nh ®èi
Sv. Líp §iÒu KhiÓn Tù ®éng, KSTN - K50 sÏ th¼ng hµng.
§¹i häc B¸ch Khoa Hµ Néi Qu¸ sung s−íng v× ph¸t hiÖn cña m×nh,
t«i liÒn t×m vµ më réng mét lo¹t ®Ýnh lý kh¸c
Ñ−êng trßn lu«n lµ ®Ò tµi hÊp dÉn ®èi cã “dÝnh d¸ng” ®Õn ®−êng trßn.
§Þnh lý 3. ( §Þnh lý Briasong).
víi mçi häc sinh khi häc vÒ h×nh häc. NÕu
C¸c ®−êng chÐo nèi c¸c ®Ønh ®èi diÖn
biÕt ®µo s©u suy nghÜ, b¹n sÏ kh¸m ph¸ ra
cña mét lôc gi¸c ngo¹i tiÕp ®−êng trßn th×
nh÷ng ®iÒu thó vÞ mµ nÕu kh«ng ph¶i b¹n
ph¸t hiÖn ra chóng th× còng ch¼ng ai nãi cho ®ång quy.
b¹n c¶. §Þnh lý 4. ( ®Þnh lý Newton).
Ta xÐt ®Þnh lý 1. §iÒu kiÖn cÇn vµ ®ñ ®Ó tø gi¸c ngo¹i tiÕp
Bµi to¸n Pascal. NÕu 1 lôc gi¸c néi tiÕp ®−êng trßn lµ trung ®iÓm 2 ®−êng chÐo víi
trong mét ®−êng trßn th× 3 giao ®iÓm cña 3 t©m cña ®−êng trßn néi tiÕp th¼ng hµng.
cÆp c¹nh ®èi diÖn th¼ng hµng. Dùa vµo phÐp chiÕu song song, ta cã thÓ
Chøng minh.(B¹n ®äc tù vÏ h×nh). thay côm tõ “®−êng trßn” b»ng “elipse”.
NÕu nh− ta thay phÐp chiÕu song song bëi
Gi¶ sö ABCDEF lµ lôc gi¸c néi tiÕp. C¸c
phÐp chiÕu xuyªn t©m th× côm tõ “®−êng
cÆp c¹nh ®èi AB, DE, BC, EF, CD, FA c¾t trßn” ®−îc thay bëi “conic” vµ ta thu ®−îc
nhau theo thø tù t¹i A’, B’, C’. Gäi P, Q, R ®Þnh lý:
t−¬ng øng lµ giao ®iÓm cña c¸c cÆp ®−êng §Þnh lý 5. Mét lôc gi¸c néi tiÕp trong mét
th¼ng AB vµ EF, AB vµ CD, CD vµ EF. ®−êng conic th× 3 giao ®iÓm cña 3 cÆp c¹nh
Theo ®Þnh lý Mª-nª-la-uyt trong tam gi¸c ®èi diÖn sÏ th¼ng hµng.
PQR víi c¸c c¸t tuyÕn BCB’, DEA’, CFA ta §ã còng lµ néi dung cña ®Þnh lý Pascal
cã c¸c hÖ thøc: mµ «ng tr×nh bµy trong c«ng tr×nh “Nghiªn
CQ B ' R BP DQ ER A ' P cøu vÒ Conic” (1640).
. . = . . =
CR B ' P BQ DR EP A ' Q C©u chuyÖn vÒ ®−êng trßn vµ ®−êng elip
C ' Q FR AP vÉn cßn nhiÒu hÊp dÉn dµnh cho b¹n ®äc
= . . =1 kh¸m ph¸. Sau ®©y lµ mét sè vÊn ®Ò t«i ®−a
C ' R FP AQ
ra ®Ó cïng nghiªn cøu:
Nh©n c¸c vÕ cña 3 ®¼ng thøc trªn víi 1. §Þnh lý Briasong sÏ biÕn ®æi nh− thÕ
nhau vµ chó ý ®Õn ph−¬ng tÝch c¸c ®iÓm P, nµo qua phÐp chiÕu xuyªn t©m?
Q, R víi ®−êng trßn: 2. HLy t×m ®iÒu kiÖn cÇn vµ ®ñ cho tø
AP BP AQ BQ CR DR gi¸c néi tiÕp ®−îc elipse.
. = . = . =1
FQ EQ CQ DQ ER FR

TAÄP SAN TOAÙN HOÏC - 2007 40


TRÖÔØNG THPT CHUYEÂN HOAØNG VAÊN THUÏ - HOØA BÌNH
PhÇn I - S¸ng t¹o to¸n häc.

Moät soá phöông phaùp xaùc ñònh


Giôùi haïn cuûa daõy soá
NguyÔn L©m TuyÒn
Chuyªn To¸n K99 – 02
Sv. Líp §iÒu khiÓn Tù ®éng 1 - K47, §H B¸ch Khoa Hµ Néi

DHy sè lµ mét chñ ®Ò quan träng trong f(x n ) – f( β ) = (x n - β ). f’(c n ) ⇒ |x n +1 - β |


tr−¬ng tr×nh to¸n phæ th«ng, ®Æc biÖt lµ c¸c = | x n - β |.|f’(c n )| ≤
k× thi Häc sinh giái to¸n. Bµi viÕt nµy xin
≤ | x n - β |.c ≤ ... ≤ | x0 - β |.cn+1
®−îc giíi thiÖu vÒ giíi h¹n cña dHy sè, mét
vÊn ®Ò mµ khi t×m hiÓu ®Õn, ng−êi lµm to¸n VËy 0 ≤ |xn- β | ≤ |x0- β |. cn
lu«n t×m thÊy nh÷ng ®iÒu míi l¹ vµ thó vÞ. ⇒ lim x n = β , do lim c n = 0 .
n → +∞ n → +∞
Còng nh− c¸c lÜnh vùc kh¸c cña to¸n häc,
giíi h¹n dHy sè rÊt ®a d¹ng vÒ thÓ lo¹i vµ NhËn xÐt ®−îc chøng minh.
phong phó vÒ ph−¬ng ph¸p. Ngoµi mét sè CÇn l−u ý r»ng hµm sè g(x) = f(x) - x cã
c¸ch th«ng th−êng nh− sö dông ®Þnh nghÜa, g’(x) = f’(x) - 1 < 0 nªn sù tån t¹i cña β lµ
®Þnh nghÜa tÝch ph©n, ®Þnh nghÜa ®¹o hµm, duy nhÊt.
hay chøng minh mét dHy ®¬n ®iÖu vµ bÞ chÆn I.1. VÝ dô 1. DLy sè (xn) ®−îc x¸c ®Þnh nh−
sau ®ã gi¶i ph−¬ng tr×nh truy nã ®Ó t×m giíi sau:
h¹n v.v … Chóng ta còng cÇn chó ý tíi mét 1
sè ph−¬ng ph¸p kh¸c t−¬ng ®èi hiÖu qu¶ cho x0 = a, xn+1 = .ln(x 4n +2006) - 2007,
2006
d¹ng to¸n nµy. Sau mçi ph−¬ng ph¸p ë bµi víi mäi n = 0, 1, 2 …
b¸o nµy ®Òu cã nªu vÝ dô ¸p dông. Chøng minh r»ng dLy sè (xn) cã giíi h¹n
I. Ph−¬ng ph¸p sö dông ®Þnh lÝ Lagrange. h÷u h¹n khi n → + ∞ .
Tr−íc hÕt xin nh¾c l¹i Lêi gi¶i. XÐt hµm sè
§Þnh lÝ Lagrange: Gi¶ sö hµm sè f(x) x¸c
1
®Þnh vµ cã ®¹o hµm trªn ®o¹n [a;b], khi ®ã f(x) = . ln(x4 + 2006) - 2007.
tån t¹i mét ®iÓm c ∈ (a; b) sao cho 2006
f’(c) . (a - b) = f(a) - f(b). NhËn thÊy f(x) x¸c ®Þnh, liªn tôc trªn R
Tõ ®ã ta cã hÖ qu¶ sau: vµ dHy (xn) ®−îc viÕt l¹i nh− sau:
HÖ qu¶. Gi¶ sö hµm sè f(x) cã ®¹o hµm trªn x0 = a, xn+1 = f(xn) víi mäi n = 0, 1, 2 …
miÒn x¸c ®Þnh D, tho¶ mLn ®iÒu kiÖn /f’(x)/ 4x3
Ta cã: f’(x) =
≤ c < 1 víi c lµ h»ng sè vµ ph−¬ng tr×nh f(x) 2006( x 4 + 2006)
= x cã nghiÖm duy nhÊt β thuéc D, khi ®ã Theo bÊt ®¼ng thøc C«si th×
dLy sè (x n ) (n = 0, 1, 2, …) x¸c ®Þnh bëi x0 x4 x4 x4
x4 + 2006 > + + + 27 ≥ 4 x 3 .
thuéc D vµ x n +1 = f(x n ) cã giíi h¹n lµ β khi 3 3 3
n dÇn tíi v« h¹n. 1
Tõ ®ã suy ra f ' ( x) < .
Chøng minh. Gi¶ sö f ' ( x) ≤ c < 1. 2006
Gäi β lµ nghiÖm cña ph−¬ng tr×nh f(x) =
Theo ®Þnh lÝ Lagrange, víi mçi n, tån t¹i c n
x, β tån t¹i vµ duy nhÊt v× hµm sè
n»m gi÷a x n vµ β sao cho

TAÄP SAN TOAÙN HOÏC - 2007 41


TRÖÔØNG THPT CHUYEÂN HOAØNG VAÊN THUÏ - HOØA BÌNH
PhÇn I - S¸ng t¹o to¸n häc.

g ( x) = f ( x) − x liªn tôc, g’(x) = f’(x) -1 < 0 g( c )<0 ⇒ ph−¬ng tr×nh f(x) =x cã duy
1 nhÊt nghiÖm trªn [ 0; c ]. Gäi nghiÖm ®ã lµ
vµ cã g(0) = . ln 2006 − 2007 < 0 ,
2003 α , khi ®ã theo nhËn xÐt trªn ta cã
lim g ( x) = +∞ . Do ®ã theo hÖ qu¶ trªn ta lim x n = α (®pcm)
x → +∞ n → +∞
cã lim x n = β , (®pcm).
n → +∞ I.3. Bµi tËp.
I.2. VÝ dô 2. Cho sè thùc c > 2. DLy sè (xn), Bµi tËp 1. DLy sè (xn) ®−îc x¸c ®Þnh bëi x0
n = 0, 1, 2, 3,… ®−îc x¸c ®Þnh theo c¸ch sau 2 x n2 π
∈ R vµ xn+1 = arctg − (1 + ) , víi mäi
: x0 = c , xn+1 = c − c + x n (n = 0, 1, 3 3 9
2,…) nÕu c¸c biÓu thøc d−íi dÊu c¨n lµ n = 1, 2, 3, …
kh«ng ©m. Chøng minh r»ng: dLy (xn) ®−îc Chøng minh r»ng dLy nµy cã giíi h¹n
x¸c ®Þnh víi mäi gi¸ trÞ n vµ tån t¹i lim x n . h÷u h¹n khi n → +∞ . T×m giíi h¹n ®ã.
n → +∞ Bµi tËp 2. DLy sè (yn) cã y0 tuú ý, tho¶
(VietNam 1999 - 2000)
1 + y n2
Lêi gi¶i. mLn:yn+1= − ln y n + y n2 + 1 , ∀n.
* Ta chøng minh (xn) x¸c ®Þnh víi mäi n 3
b»ng qui n¹p. Tõ gi¶ thiÕt Chøng minh r»ng: tån t¹i lim y n
n → +∞
c > 2 ⇒ c − c + xo = c − c + c > c − 2 c > 0 Bµi tËp 3. Cho dLy sè (zn) tho¶ mLn:
⇒ x1 ®−îc x¸c ®Þnh. 16zn+1 = 6coszn + 3cos2zn + 2cos3zn –
Gi¶ sö x k (k ≥ 1) ®H ®−îc x¸c ®Þnh. Khi 3(8 π -1), víi mäi n ∈ N. T×m lim z n .
n → +∞

®ã do 0 < x k < c < c 1


Bµi tËp 4. Cho sè thùc a ∉ ( ;0) vµ sè thùc b
⇒ c − c + x k > c − 2c > 0 2
tuú ý. DLy sè (xn) víi n = 0, 1, 2, … ®−îc x¸c
⇒ x k +1 ®−îc x¸c ®Þnh ⇒ ®pcm. ®Þnh theo c¸ch sau: x0 = b, xn+1 =
* Ta cã 0 ≤ x n ≤ c , ∀ n. a
.ln( x n2 + a2) - a2, víi mäi n = 0, 1, 2,...
3
XÐt hµm sè f(x) = c − c + x trªn Chøng minh r»ng dLy sè (xn) cã giíi h¹n h÷u
[0; c ]. h¹n khi n → +∞ .
−1 π
Ta cã : f’(x) = Bµi tËp 5. Gi¶ sö 0 < α < . XÐt dLy sè
4 c − c + x. c + x 2
(un) ®−îc x¸c ®Þnh bëi:
Chó ý c > 2 ⇒
u1 >0 vµ un+1= un sin2 α +
3 3 1
⇒ c(c − 2) + c .( c − ) + >0 2002. cos α 2
4 4 64 víi n = 1, 2, 3...Chøng minh
u ntg α
2

1 1
⇒ (c − ) 2 > c + c ⇒ c − c + c > r»ng dLy sè (un) cã giíi h¹n khi n → +∞ vµ
8 8
VËy tÝnh giíi h¹n ®ã.

4. c − c + x . c + x > 4. c − c + c . c > II. Ph−¬ng ph¸p sö dông c«ng thøc tæng


qu¸t.
1 1 II.1. VÝ dô 2. Cho dLy sè kh«ng ©m (un)
> . 2 = 2 ⇒ f '( x) <
8 2. tho¶ mLn ®iÒu kiÖn :
XÐt hµm sè g(x) = f(x) – x trªn [ 0; c ]. 1 1 1
4un + 2 ≤ ( + + ... + ).un +1 + ... +
Ta cã g’(x) = f’(x) –1 < 0 vµ g(0)>0, n +1 n + 2 n+n

TAÄP SAN TOAÙN HOÏC - 2007 42


TRÖÔØNG THPT CHUYEÂN HOAØNG VAÊN THUÏ - HOØA BÌNH
PhÇn I - S¸ng t¹o to¸n häc.

1 1 1
+( + + ... + ).un Bµi tËp 2. Cho dLy sè kh«ng ©m (yn) cã tÝnh
n + n n + n +1 n + 3n
víi mäi n = 1, 2, 3... chÊt: yn+2 ≤ uyn + vyn ( ∀ n), trong ®ã u, v lµ
Chøng minh r»ng dLy sè (un) héi tô vµ c¸c sè thùc d−¬ng vµ u + v <1. T×m lim y n .
n → +∞
t×m giíi h¹n ®ã.
Lêi gi¶i. Víi mäi n = 1, 2, 3, ... ta cã: III. Ph−¬ng ph¸p sö dông ®Þnh lÝ Stolz -
1 1 1 Cesaro.
+ + ... + < 1 vµ
n +1 n + 2 n+n §Þnh lÝ Stolz. Cho c¸c dHy sè (xn) vµ (yn).
1 1 1 1 3 Gi¶ sö r»ng (yn) lµ dHy d−¬ng
+ + ... + < (2n + 1). ≤ x − xn
n + n n + n +1 n + 3n n+n 2 t¨ng, lim yn = +∞ vµ lim n +1 = L , khi
n +1 − y n
3 n →+∞ n → +∞ y
Nh− vËy ta cã: 4u n+ 2 ≤ u n +1 + .u n , víi
2 x
mäi n = 1, 2, 3 .... ®ã ta còng cã lim n = L .
n → +∞ y
XÐt dHy (vn) nh− sau: v1 = u1, v2 = u2 vµ n
Chøng minh. Tõ ®Þnh nghÜa giíi h¹n vµ gi¶
3
4vn+2 = vn+1+ vn , ∀ n = 1, 2, 3.... x − xn
2 thiÕt lim n +1 = L , suy ra víi mäi
n +1 − y n
n → +∞ y
3
Ph−¬ng tr×nh ®Æc tr−ng 4 X 2 − X − = 0 ε > 0 bÊt kú tån t¹i sè tù nhiªn N0 sao cho
2 víi mäi k ≥ No , lu«n cã :
3 1
⇒ X1 = , X 2 = − x k +1 − x k ε
4 2 −L <
3 1 y k +1 − y k 2
⇒ v n = C1 .( ) n + C 2 .(− ) n.
4 2 ε
⇔ −( yk − yk −1 ). < xk − xk −1 − L.( yk − yk −1 ) <
⇒ lim v n = 0 (*) 2
n → +∞
TiÕp theo, ta chøng minh b»ng quy n¹p ε
< ( yk − yk −1 ).
r»ng: v n ≥ u n ≥ 0 víi mäi n = 1, 2, 3, … 2
(**) Cho k = No, N0 + 1,...., n - 1 råi céng theo
ThËt vËy, víi n = 1, 2 th× (**) ®óng theo tõng vÕ c¸c B§T ®ã ta ®−îc:
c¸ch ®Þnh nghÜa dHy (vn). ε
−( yn − yN0 ). < xn − xN0 − L.( yn − y N0 ) <
Gi¶ sö (**) ®H ®óng víi mäi n ≤ k+1. 2
Khi ®ã ta cã: ε ε x ε
3 3 < ( yn − yN0 ). ⇒ − < n − L − zn < (1)
4vk+2 = vk+1 + vk ≥ uk+1 + .u k ≥ 4.u k + 2 2 2 yn 2
2 2
x N − Ly N 0
⇒ v k + 2 ≥ u k + 2 ≥ 0 . NhËn xÐt (**) ®−îc Trong ®ã zn = 0 .
chøng minh. Cuèi cïng, sö dông ®Þnh lÝ kÑp yn
trong lý thuyÕt giíi h¹n dHy sè vµ (*), (**) ë V× lim y n = + ∞ , lim z n = 0 ⇒ tån t¹i sè
n → +∞ n → +∞
trªn ta ®−îc lim u n = 0.
n → +∞ tù nhiªn M 0 sao cho víi mäi n > M 0 th× :
II.2. NhËn xÐt. Yªu cÇu cña ph−¬ng ph¸p lµ
ε ε
ph¶i n¾m v÷ng c¸ch gi¶i cña ph−¬ng tr×nh − < zn <
(2)
sai ph©n vµ kÜ n¨ng biÕn ®æi truy håi. 2 2
Tõ (1) vµ (2) suy ra víi mäi
II.3.Bµi tËp. n > Max { N 0 ; M 0 } ta lu«n cã:
Bµi tËp 1. Gi¶ sö ph−¬ng tr×nh ax2 +bx +c
=0, (a ≠ 0), cã hai nghiÖm thùc ph©n biÖt. ε xn ε x
− <−L< ⇔ n −L <ε
XÐt dLy sè (xn) ®−îc x¸c ®Þnh bëi sè xo cho 2 yn 2 yn
tr−íc vµ ®iÒu kiÖn xn(axn-1 + b) + c = 0 víi n x
= 1, 2, 3... HLy t×m lim x n theo x0. ⇔ lim n = L (®pcm).
n → +∞ n → +∞ y
n

TAÄP SAN TOAÙN HOÏC - 2007 43


TRÖÔØNG THPT CHUYEÂN HOAØNG VAÊN THUÏ - HOØA BÌNH
PhÇn I - S¸ng t¹o to¸n häc.

Tr−êng hîp ®Æc biÖt, víi xn = v1 + v2 ( ai −1) 1− a


 k 
+...+vn vµ yn = n, ta cã ®Þnh lÝ : XÐt hµm sè f(x) = 1 + ∑ x a −1  −1.
 i =1 
§Þnh lÝ Cesaro vÒ trung b×nh cña mét d·y Ta cã f(0) = 0 vµ
sè: −a
Cho dLy (vn) vµ ®Æt wn =  k aai −−1a   k ( ai −1)

f '( x) = (1 − a )  ∑ x  .  1 + ∑ x a −1  .
v1 + v 2 + ... + v n  i =1   i =1 
. Khi ®ã nÕu (vn) héi tô ®Õn
n Do c¸c ai < a < 0 ⇒ f’(0) = 1- a.
L khi n t¨ng lªn v« h¹n th× dLy (wn) còng héi Sö dông ®Þnh nghÜa ®¹o hµm, ta thÊy
tô ®Õn L khi n t¨ng lªn v« h¹n. f ( x ) − f ( 0)
f’(0) = lim = 1- a (*)
x →0 x
III.1.VÝ dô 3. Cho k sè thùc ©m ph©n biÖt
a1, a2, ..., ak (k ≥ 1). DLy sè (Tn) ®−îc x¸c f ( xn )
Hay nãi riªng lim = 1− a
®Þnh bëi Ti > 0 ( i = 1, 2, ..., k) cho tr−íc n → +∞ xn
tho¶ mLn ®iÒu kiÖn : (do f(0) = 0, lim x n = 0 )
k n → +∞
Tn+1 = Tn + ∑ Tnai , víi mäi n ≥ k.
i =1
⇒ lim v n = 1 − a .
n → +∞

T×m tÊt c¶ c¸c sè thùc α sao cho dLy sè


* Cuèi cïng, ¸p dông ®Þnh lý Cesaro víi
Tnα
(Un) x¸c ®Þnh bëi Un = , n = 0, 1, 2, … vn = =T 1n−+a1 - T 1n−a ta ®−îc
n
cã giíi h¹n h÷u h¹n kh¸c kh«ng khi n dÇn Tn1+−1a − T11−a
lim wn = lim = lim v n = 1 − a .
tíi v« h¹n vµ t×m giíi h¹n ®ã. n → +∞ n → +∞ n n → +∞
Lêi gi¶i. Tn1− a
* Tr−íc hÕt ta chøng minh lim Tn = + ∞ . ⇒ lim = 1− a .
n → +∞ n → +∞ n
Gi¶ sö ng−îc l¹i, dHy (Tn) bÞ chÆn. Khi ®ã, VËy α = 1- a lµ mét gi¸ trÞ cÇn t×m.
do (Tn) lµ mét dHy d−¬ng, t¨ng nªn tån t¹i MÆt kh¸c v× lim Tn = + ∞ nªn :
giíi h¹n lim Tn = β h÷u h¹n, tõ n → +∞
n → +∞
k
+ ∞, α > 1− a
®ã β = β + ∑ β ai , v« lÝ. lim Tnα −(1−a ) = 
i =1 n → +∞
 0, α < 1− a
* TiÕp theo ®Æt a = Max{a i } vµ xÐt dHy T 1− a
1≤i ≤ k Cuèi cïng, v× Un = n .Tnα −(1−a ) vµ
vn = Tn1+−1a − Tn1− a .Ta sÏ chøng minh 1− a
n
T
lim v n = 1 − a . lim n = 1 − a nªn
n → +∞ n → +∞ n
ThËt vËy, ta cã:
+ ∞, α > 1− a
k
lim U n = 
vn = ( Tn + ∑T
i =1
n
ai
)1-a - T 1n−a = n → +∞
 0, α < 1− a
k
= T 1n−a .[(1 + ∑ Tnai −1 )1− a − 1] .
i =1 * KÕt luËn. §Æt a = Max{a i }.
a −1 1≤i ≤ k
§Æt xn = T víi chó ý lim x n = 0 (v× a
n
n → +∞ - NÕu a = 1 th× kh«ng tån t¹i sè α tho¶ mHn
< 0) th× ta cã: ®Ò bµi.
( ai −1) 1− a - NÕu a ≠ 1 th× gi¸ trÞ duy nhÊt cÇn t×m lµ
 
1 + ∑ xn −1 α = 1 − a . Vµ lóc ®ã lim U n = 1 − a .
a −1
 n → +∞
vn =  
xn

TAÄP SAN TOAÙN HOÏC - 2007 44


TRÖÔØNG THPT CHUYEÂN HOAØNG VAÊN THUÏ - HOØA BÌNH
PhÇn I - S¸ng t¹o to¸n häc.

NhËn xÐt. nv n3 + 1
v1 = a, v n +1 = víi n = 0, 1, 2,
i) §©y lµ bµi to¸n tæng qu¸t cña bµi to¸n v n2 .3 n 3 + n 2
sè 3 trong ®Ò thi chän §éi tuyÓn ViÖt Nam
dù thi IMO n¨m 1996. C¸c b¹n còng cã thÕ … T×m lim n C 2nn .v n theo c¸c tr−êng hîp
n → +∞
thÊy mét sè tr−êng hîp ®Æc biÖt cña bµi to¸n cña a, trong ®ã C nk sè tæ hîp chËp k cña n
nµy trªn t¹p chÝ To¸n häc vµ Tuæi trÎ hoÆc
trong c¸c ®Ò thi Olympic. Nh−ng trong c¸c phÇn tö.
tr−êng hîp ®Æc biÖt ®ã, bµi to¸n cßn cã lêi
gi¶i s¬ cÊp h¬n th«ng qua c¸c bÊt ®¼ng thøc Bµi tËp 4. Cho 2 dLy sè xn vµ un (n = 0, 1, 2,
®¸nh gi¸ vµ vËn dông nguyªn lý kÑp. …) ®−îc x¸c ®Þnh theo c¸ch sau :

ii) §©y lµ d¹ng to¸n khã vµ ®iÓn h×nh 
trong lý thuyÕt giíi h¹n cña dHy sè. Trong  u1 > 0
lêi gi¶i trªn, chóng ta thÊy sù hiÖn diÖn cña  1 2007 −i
nhiÒu ph−¬ng ph¸p t×m/chøng minh giíi h¹n. u n +1 = u n + k + ∑ u n , ∀ n ≥ 1
Trong ®ã, c¸c b¹n l−u ý ph−¬ng ph¸p sö  u n i =1
dông ®Þnh nghÜa ®¹o hµm (*). §©y lµ mét  2007

ph−¬ng ph¸p “m¹nh” vµ t−¬ng ®èi tinh tÕ.  2006 .u 2006

VÒ b¶n chÊt, tuy chØ lµ mét tr−êng hîp riªng xn = n


 n +1
cña quy t¾c L’hospital, nh−ng rÊt hiÖu qu¶
T×m tÊt c¶ c¸c sè thùc d−¬ng k sao cho
0 ∞
trong viÖc xÐt c¸c giíi h¹n d¹ng , cÊp dLy xn dÇn tíi 2007 khi n dÇn tíi v« h¹n.
0 ∞ (Lêi gi¶i mêi c¸c b¹n xem trong chuyªn môc
1. Sö dông ®Þnh nghÜa ®¹o hµm, trong lêi
gi¶i trªn chóng ta ®H chøng minh ®−îc “Nh÷ng bµi to¸n hay vµ c¸c bµi to¸n tù
lim v n = 1 − a mét c¸ch nhanh chãng mµ s¸ng t¹o” )
n → +∞
kh«ng kÐm phÇn tù nhiªn. C¸c b¹n hHy t×m Bµi tËp 5. T×m giíi h¹n sau, trong ®ã k lµ sè
hiÓu thªm vÒ kü thuËt còng nh− øng dông
cña ph−¬ng ph¸p nµy. 1k + 2 k + ... + n k
tù nhiªn cho tr−íc: lim .
n → +∞ n k +1
III.2. Bµi tËp.
Bµi tËp 1. Cho dLy sè (Tn) (n = 0, 1, 2, …)
®−îc x¸c ®inh bëi T1 > 0 cho tr−íc, T n+1 = Trªn ®©y lµ mét sè ph−¬ng ph¸p gi¶i
1 c¸c bµi to¸n vÒ giíi h¹n cña dHy sè. Hi väng
Tn + , víi mäi n = 0, 1, 2, … r»ng c¸c b¹n sÏ t×m tßi ra nhiÒu ph−¬ng ph¸p
Tn
kh¸c vµ kh«ng chØ trong lÜnh vùc nµy mµ cßn
HLy t×m tÊt c¶ c¸c sè thùc α sao cho dLy
trong tÊt c¶ c¸c lÜnh vùc kh¸c cña to¸n häc.

sè (Un) x¸c ®Þnh bëi Un = n (∀n ≥ 1) cã
n Cã thÓ nãi “ To¸n häc nh− mét viªn kim
giíi h¹n h÷u h¹n khi n dÇn tíi v« h¹n. c−¬ng, cµng mµi giòa cµng s¸ng ®Ñp”. Vµ
(§Ò dù tuyÓn ViÖt Nam 1996) chóng ta hHy cïng lµm cho to¸n häc ngµy
Bµi tËp 2. Cho dLy sè a n x¸c ®Þnh nh− sau: cµng s¸ng h¬n, ®Ñp h¬n c¸c b¹n nhÐ !
1
a1 = 2, a n+1 = a n + 2 , n = 1, 2, …
an
a3
Chøng minh r»ng lim n = 3 .
n → +∞ n
(§Ò thi Olympic 30 - n¨m 2001 )

Bµi tËp 3. Cho a > -1. LËp dLy sè ( v n ), n =


0, 1, 2, … nh− sau:

TAÄP SAN TOAÙN HOÏC - 2007 45


TRÖÔØNG THPT CHUYEÂN HOAØNG VAÊN THUÏ - HOØA BÌNH
PhÇn I - S¸ng t¹o to¸n häc.

MOÄT LÔÙP caùc baøi toaùn Baát Ñaúng Thöùc


NguyÔn Minh phóc
Chuyªn to¸n K05 – 08
Líp 11T, THPT Chuyªn Hoµng V¨n Thô

C ¸c b¹n th©n mÕn! Khi häc m«n To¸n, còng ®H suy nghÜ nh− vËy vµ tù ®Æt cho m×nh
bµi to¸n sau:
kh«ng Ýt b¹n kÓ c¶ häc sinh chuyªn To¸n,
®Òu c¶m thÊy m«n To¸n cã nhiÒu ®iÒu thËt Chøng minh r»ng víi mäi a, b, c ∈ R+* vµ m,
khã hiÓu. Nh−ng thùc chÊt mçi vÊn ®Ò ®Òu n ∈ N * , m > n ta lu«n cã:
cã mét ch×a khãa riªng cña nã, viÖc t×m ra
ch×a khãa phô thuéc vµo møc ®é lÉn c«ng am bm cm
+ + ≥ a m−n + bm−n + c m−n ( I )
søc mµ b¹n bá ra. §«i khi chØ cÇn bá ra chót bn c n an
Ýt thêi gian ®µo s©u suy nghÜ, ta cã thÓ khai §Ó chøng minh bµi to¸n trªn, ta ¸p dông
th¸c ®−îc nh÷ng hÖ thèng kiÕn thøc mét trùc tiÕp B§T Cauchy cho m sè d−¬ng:
c¸ch h÷u Ých. am
Trong bµi viÕt nµy t«i xin giíi thiÖu mét (m − n) n +n b m − n ≥ na m − n ,
c¸ch ¸p dông hiÖu qu¶ B§T Cauchy vèn ®H b
rÊt quen thuéc ®èi víi c¸c b¹n häc sinh bm
(m − n) n + nc m − n ≥ nb m − n ,
THCS lÉn cÊp THPT. c
cm
Nh− c¸c b¹n ®H biÕt, trong ch−¬ng tr×nh (m − n) n + na m − n ≥ nc m − n
THCS, ta th−êng gÆp nh÷ng bµi to¸n B§T cã a
d¹ng sau:
Céng vÕ c¸c B§T trªn råi rót gän ta cã
a 3 b3 c3 (I). Râ rµng (1) chØ lµ tr−êng hîp riªng cña
+ + ≥ a 2 + b 2 + c 2 (1)
b c a (I) øng víi m = 3, n = 2. Nh−ng chØ víi
Hay a 5 + b5 + c5 ≥ a 4 b + b 4 c + c 4 a (2) B§T(I) qu¸ tr×nh t×m kiÕm cña chóng ta vÉn
(a, b, c ∈ R+* ) ch−a kÕt thóc. Víi trùc gi¸c, ta c¶m thÊy cßn
cã mét biÓu thøc n»m gi÷a hai vÕ cña (I).
Khi dïng B§T Cauchy ®Ó gi¶i to¸n, mçi VÉn lµ biÓu thøc cã cïng bËc lµ (m - n),
bµi ta ®Òu cã nh÷ng c¸ch ghÐp c¸c hÖ sè nh−ng bËc cña biÕn l¹i gi¶m ®i. Ta xÐt bµi
kh¸c nhau. Nh− ë B§T(1), ta cã thÓ ghÐp to¸n:
c¸c hÖ sè nh− sau: Víi a, b, c ∈ R+* . Chøng minh:
3 3 3 3
a a b b a 5 b5 c5 a 4 b 4 c 4
+ + b 2 ≥ 3a 2 ; + + a 2 ≥ 3b 2 ; + + ≥ + + (3)
b b c c b2 c2 a 2 b c a
c 3
c 3
¸p dông trùc tiÕp B§T Cauchy cho 5 sè
+ + a 2 ≥ 3c 2 . d−¬ng ta cã:
a a
Céng c¸c B§T trªn vÕ theo vÕ ta cã ngay a5 a4 b5 b4
4 2 + b3 ≥ 5 ; 4 2 + c3 ≥ 5 ;
®iÒu ph¶i chøng minh. Sau khi gi¶i nhiÒu bµi b b c c
to¸n d¹ng trªn, h¼n c¸c b¹n còng nhËn ra c5 c4
r»ng hai vÕ cña c¸c B§T lu«n ®ång bËc vµ 4 2 +a ≥5 ;
3

bËc cña biÕn ë vÕ lín h¬n còng lín h¬n. T«i a a

TAÄP SAN TOAÙN HOÏC - 2007 46


TRÖÔØNG THPT CHUYEÂN HOAØNG VAÊN THUÏ - HOØA BÌNH
PhÇn I - S¸ng t¹o to¸n häc.

Céng vÕ víi vÕ c¸c B§T trªn ta ®−îc: Céng tÊt c¶ l¹i ta ®−îc ngay (III). Nh−
a 5 b5 c5 vËy tæng kÕt l¹i ta cã chuçi B§T sau:
4( 2 + 2 + 2 ) + a 3 + b3 + c3 ≥ a, b, c ∈ R+* ; m, n, k , r ∈ N * ; m ≥ n ≥ k ; m − n ≥ r :
b c a
a4 b4 c 4 A≥ B≥C ≥ D
≥ 5( + + ) Trong ®ã
b c a
Mµ theo (I) víi m = 4, n = 1 th× a m−k bm−k c m−k
B = n−k + n −k + n −k ,
a4 b4 c4 b c a
+ + ≥ a 3 + b3 + c3 nªn ta cã C = a + b + c m −n ,
m−n m−n
b c a
ngay (3). D = a m−n− r br + bm−n −r c r + c m−n −r a r
B»ng c¸ch chøng minh hoµn toµn t−¬ng
tù vÝ dô trªn ta cã c¸ch gi¶i bµi to¸n sau: VËy lµ sau mét sè nhËn xÐt vÒ c¸c bµi
to¸n quen thuéc vµ ¸p dông B§T Cauchy
Víi a, b, c ∈ R+* , m, n, k ∈ N * , m ≥ n ≥ ®óng c¸ch, chóng ta ®H cã thÓ gi¶i mét lo¹t
k th× ta cã: c¸c bµi to¸n phæ biÕn mµ kh«ng gÆp qu¸
nhiÒu khã kh¨n. ChiÕc ch×a khãa ë ®©y chÝnh
a m bm c m a m −k bm −k c m −k lµ sù kÕt hîp hÖ sè ®óng c¸ch trong B§T
+ + ≥ + + ≥
bn c n a n b n − k c n − k a n− k Cauchy. Khi dïng ch×a khãa nµy b¹n cã thÓ
≥ a m − n + b m − n + c m − n (II) më thªm nhiÒu c¸nh cöa kh¸c mµ chóng ta
Hay M ≥ N ≥ P. ch−a thÓ “tham quan” hÕt trong bµi viÕt nµy.

Lêi gi¶i. ¸p dông B§T Cauchy cho m sè Cuèi cïng, xin chóc c¸c b¹n häc giái vµ
d−¬ng ta cã: thµnh c«ng trong cuéc sèng!
am a m−k
( m − k ) n + kb m−n ≥ m n−k ;
b b
b m
bm−k
( m − k ) n + kc m−n ≥ m n−k ;
c c
c m
a m−k
( m − k ) n + ka m−n
≥ m
a a n−k
Céng theo tõng vÕ c¸c B§T trªn ta cã:
( m − k ) M + kP ≥ mN
Nh−ng theo (I) th× N ≥ P ⇔ kN ≥ kP , tõ
®ã ta cã ®−îc (II).

TiÕp theo ta kÐo dµi chuçi B§T ra víi bµi


to¸n sau:
Víi mäi a, b, c ∈ℜ*+ ; m, n ∈ N * ta cã:
a m+n + bm+n + c m+n ≥ a m bn + bm c n + c m a n
(III)
Chøng minh thËt dÔ dµng, chØ cÇn ¸p
dông B§T cho (m + n) sè d−¬ng:
ma m + n + nb m + n ≥ ( m + n ) a mb n ;
mb m + n + nc m + n ≥ ( m + n ) b m c n ;
mc m + n + na m + n ≥ ( m + n ) c m a n

TAÄP SAN TOAÙN HOÏC - 2007 47


TRÖÔØNG THPT CHUYEÂN HOAØNG VAÊN THUÏ - HOØA BÌNH
PhÇn I - S¸ng t¹o to¸n häc.

Moät soá khaùi nieäm veà


GOÙC ÑÒNH HÖÔÙNG
TrÇn quang thä
Chuyªn to¸n k01 - 04
Sv. Líp Kinh tÕ qu¶n lý c«ng, §H Kinh TÕ Quèc D©n Hµ Néi

Gãc ®Þnh h−íng lµmét kh¸i niÖm rÊt m¬ II. C¸ch ®o gãc ®Þnh h−íng gi÷a hai
hå, phøc t¹p nh−ng ®ã còng lµ mét c«ng cô vect¬.
m¹nh trong h×nh häc ph¼ng . T«i xin giíi 1. §Þnh nghÜa.
thiÖu mét sè vÊn ®Ò liªn quan ®Õn kh¸i niÖm Sè ®o theo ra®ian/®é cña gãc ®Þnh h−íng
nµy. gi÷a hai vect¬ chÝnh lµ sè ®o theo ra®ian/®é
cña cung ®Þnh h−íng mµ nã sinh ra.
I. Gãc ®Þnh h−íng gi÷a hai vect¬. Hai gãc ®Þnh h−íng gi÷a hai vect¬ ®−îc
1. §Þnh nghÜa.    gäi lµ b»ng nhau nÕu sè ®o cña chóng b»ng
Cho hai vect¬ AB , CD kh¸c 0 . Mçi mét nhau.
sù quay xung quanh ®iÓm A, theo mét h−íng
 2. C¸c c«ng thøc c¬ b¶n .
x¸c ®Þnh cña AB sao cho vect¬ míi nhËn
 Nhê ®Þnh nghÜa gãc ®Þnh h−íng gi÷a hai
®−îc cïng h−íng víi CD , ®−îc gäi lµ gãc vect¬ vµ sè ®o gãc cña nã ta, cã c«ng thøc:
   
®Þnh h−íng gi÷a haivect¬ AB , CD . ( )
(1.1) AB, CD ≡ α (mod 2π )
Trong ®ã α lµ sè ®o theo ra®ian cña mét
 
(
trong c¸c gãc AB, CD . )
   
(
(1.2) AB ↑↑ CD ⇔ AB, CD ≡ 0(mod 2π ) )
   
( ) (
(1.3) AB, CD ≡ − CD, AB (mod 2π ))
   
( ) (
(1.4) AB, CD ≡ π + − AB, CD (mod 2π ) )
 
2. Mét sè quy −íc.
Trªn mét mÆt ph¼ng, h−íng ng−îc víi
( )
≡ π + AB, −CD (mod 2π )
   
h−íng quay cña kim ®ång hå ®Þnh h−íng gäi ( ) (
(1.5) AB, CD ≡ − AB, −CD (mod 2π ) )
lµ h−íng d−¬ng. H−íng trïng víi h−íng 3. HÖ thøc Sacl¬.
quay cña kim ®ång hå ®Þnh h−íng gäi lµ      
h−íng ©m. ( ) ( ) (
AB, CD ≡ AB, EF + EF , CD (mod 2π ) )
HÖ qu¶.
     
Nh− vËy, gãc gi÷a hai vect¬ ®−îc gäi lµ
d−¬ng/©m nÕu sù quay sinh ra nã lµ sù quay ( ) ( ) (
AB, CD ≡ EF , CD − EF , AB + (mod 2π ) )
theo h−íng d−¬ng/©m.

TAÄP SAN TOAÙN HOÏC - 2007 48


TRÖÔØNG THPT CHUYEÂN HOAØNG VAÊN THUÏ - HOØA BÌNH
PhÇn I - S¸ng t¹o to¸n häc.

III. Cung ®Þnh h−íng.


1. §Þnh nghÜa. h−íng víi vect¬ CD, ®−îc gäi lµ mét gãc
Khi vect¬ AB quay quanh ®iÓm A theo ®Þnh h−íng gi÷a hai ®−êng th¼ng AB vµ CD.
mét h−íng x¸c ®Þnh ®Ó sinh ra gãc
 
( )
AB, CD th× ®iÓm B thùc hiÖn mét chuyÓn
®éng, quü ®¹o chuyÓn ®éng ®ã ®−îc gäi lµ
mét cung ®Þnh h−íng.
Cung ®Þnh h−íng ®−îc gäi lµ d−¬ng/©m
nÕu gãc ®Þnh h−íng sinh ra nã lµ gãc
d−¬ng/©m.

2. C¸ch ®o cung ®Þnh h−íng.


NÕu cung ®Þnh h−íng lµ d−¬ng th× tØ sè
gi÷a ®é dµi cña nã vµ ®é dµi cña vect¬ sinh Gãc ®Þnh h−íng gi÷a hai ®−êng th¼ng
ra nã ®−îc coi lµ sè ®o cña cung ®Þnh h−íng ®−îc lµ gãc d−¬ng/©m nÕu gãc gi÷a hai
vect¬ sinh ra nã d−¬ng/©m.
®ã tÝnh theo ra®ian.
Hai cung ®Þnh h−íng ®−îc gäi lµ b»ng
nhau nÕu sè ®o cña chóng b»ng nhau. 2. Sè ®o gãc ®Þnh h−íng gi÷a hai ®−êng
th¼ng.
3. C¸c c«ng thøc c¬ b¶n. a. §Þnh nghÜa.
Theo ®Þnh nghÜa cung ®Þnh h−íng vµ sè Sè ®o tÝnh theo ra®ian/®é cña gãc ®Þnh
®o cña nã, ta cã ngay c¸c c«ng thøc c¬ b¶n h−íng gi÷a hai ®−êng th¼ng chÝnh lµ sè ®o
sau, ®óng víi c¸c cung ®Þnh h−íng trong gãc tÝnh theo ra®ian/®é cña gãc ®Þnh h−íng
cïng mét ®−êng trßn. gi÷a hai vect¬ sinh ra nã.
Ký hiÖu s® BB ' ≡ α (mod 2π ) (2.1) lµ sè Hai gãc ®Þnh h−íng gi÷a hai ®−êng th¼ng
®o cung BB ' , trong ®ã α lµ sè ®o theo ®−îc gäi lµ b»ng nhau nÕu sè ®o cña chóng
ra®ian cña mét trong c¸c cung ®Þnh h−íng b»ng nhau.
BB ' .
b. C¸c c«ng thøc c¬ b¶n.
(2.2) B trïng B′ ⇔ s® BB ' ≡ 0(mod 2π )
Tõ ®ÞnhnghÜa vµ sè ®o gãc ®Þnh h−íng
(2.3) s® BB ' - s® BB ' (mod 2π ) gi÷a hai ®−êng th¼ng ta cã:
(3.1) ( AB, CD ) ≡ α (mod 2π )
4. HÖ thøc Sacl¬.
Theo ®Þnh nghÜa cung ®Þnh h−íng vµ sè Trong ®ã α lµ sè ®o theo ra®ian/®é cña
®o cña nã ta cã hÖ thøc sau: mét trong c¸c gãc ®Þnh h−íng (AB,CD).
 
s® AB ≡ s® AC + s® CB (mod 2‫)ח‬ ( )
(3.2) ( AB, CD ) ≡ AB, CD (mod 2π )
 
HÖ qu¶. Cho ba ®iÓm A, B, C cïng thuéc
mét ®−êng trßn ta th× ta cã hÖ thøc sau: ( )
HoÆc ( AB, CD ) ≡ − AB, CD (mod 2π )
 
s® AB ≡ s® CB - s® CA (mod 2‫)ח‬ ( )
HoÆc ( AB, CD ) ≡ AB, −CD (mod 2π )
 
IV. Gãc ®Þnh h−íng gi÷a hai ®−êng th¼ng. ( )
(3.3) ( AB, CD ) ≡ AB, CD (mod π )
1. §Þnh nghÜa. (3.4) AB song song hoÆc trïng víi CD
Cho hai ®−êng th¼ng AB, CD vµ O lµ mét
⇔ (AB,CD) ≡ 0 ( mod Π)
®iÓm bÊt k× trªn ®−êng th¼ng AB. Mçi sù
quay quanh ®iÓm O theo mét h−íng x¸c ®Þnh (3.5) ( AB, CD ) ≡ − ( CD, AB ) (mod π )

cña vect¬ AB sao cho sau khi quay, vect¬
míi nhËn ®−îc cïng h−íng hoÆc ng−îc c. HÖ thøc sacl¬.

TAÄP SAN TOAÙN HOÏC - 2007 49


TRÖÔØNG THPT CHUYEÂN HOAØNG VAÊN THUÏ - HOØA BÌNH
PhÇn I - S¸ng t¹o to¸n häc.

(AB,CD) ≡ (AB,EF) + (EF,CD) (mod Π) Cho ABC vµ A'B'C', c¸c ®iÒu kiÖn sau
t−¬ng ®−¬ng:
Chøng minh. Theo (3.3), ta cã: a. ∆ABC vµ ∆A'B'C' ®ång d¹ng, cïng
 
( )
( AB, CD ) ≡ AB, CD (mod π ) h−íng.
AB A ' B '
Còng theo (3.3) ta cã : b. =
 
( )
AC A ' C '
( AB, EF ) ≡ AB, EF (mod π )    
  ( ) ( )
AB, AC ≡ A ' B ', A ' C ' (mod 2π )
( )
( EF , CD ) ≡ EF , CD (mod π )  
( )
 
( )
c. BA, BC ≡ B ' A ', B ' C ' (mod 2π )
⇒ ( AB, EF ) + ( EF , CD ) ≡    
    ( ) ( )
CA, CB ≡ C ' A ', C ' B ' (mod 2π )
( ) (
≡ AB, EF + EF , CD (mod π ) )    
⇒ ( AB, EF ) + ( EF , CD ) ≡
( ) ( )
d. BA, BC ≡ B ' A ', B ' C ' (mod 2π )
   
(
 
) (
 
≡ AB, EF + EF , CD (mod 2π )) ( ) ( )
CA, CB ≡ C ' A ', C ' B ' (mod 2π )
  e. (BC,B'C') ≡ (CA,C'A')
( )
≡ AB, CD (mod 2π ) ≡ (AB,A'B') ( mod 2Π)
 
(
Mµ ( AB, CD ) ≡ AB, CD (mod π ) ) 3. §Þnhlý 3.
⇒ ( AB, EF ) + ( EF , CD ) ≡ a.
a // a '
( AB, CD ) (mod π )  ⇒ ( a, b ) ≡ ( a ', b ' ) (mod 2π )
HÖqu¶. (AB,CD) ≡ (EF,CD) - (EF,AB) (mod  b // b '
Π) b.
a ⊥ a '
 ⇒ ( a, b ) ≡ ( a ', b ' ) (mod π )
VI. C¸c ®Þnh lý. b ⊥ b '

1. §Þnh lý 1. 4. §Þnh lý 4.
Cho ∆ABC vµ ∆A'B'C' khi ®ã c¸c ®iÒu Cho bèn ®iÓm A, B, C, D n»m trªn ®−êng
kiÖn sau t−¬ng ®−¬ng : trßn (O). Khi ®ã
   
a. ∆ABC vµ ∆A'B'C' b»ng nhau vµ cïng
h−íng ( ) ( )
2 AB, CD ≡ 2 AB, DC ≡2 ( AB, CD )
b. AB = A'B' ≡ s®AB + s®CD ≡ s®AD+ s® BC (mod 2Π)
AC = A'C'
   
( ) ( )
AB, AC ≡ A ' B ', A ' C ' (mod 2π ) 5. §Þnh lý 5.
Cho ®−êng trßn (O) vµ c¸c ®iÓm A, B, M
c. BC = B'C' n»m trªn (O) . Ta cã:
   
( ) ( )
BA, BC ≡ B ' A ', B ' C ' (mod 2π )
 
(
 
) (
a. 2 MA, MB ≡ OA, OB (mod 2π ) )
   
( ) ( )
CA, CB ≡ C ' A ', C ' B ' (mod 2π ) b. 2(MA,MB) ≡ (OA,OB) (mod Π)
d. BC = B'C'
6. §Þnh lý 6.
(BA,BC) ≡ (B'A',B'C') (mod Π)
Qua phÐp quay t©m O, gãc quay α c¸c
(CA,CB) ≡ (C'A',C'B') (mod Π) ®iÓm A, B biÕn thµnh c¸c ®iÓm A', B'. Khi
®ã:
 
2. §Þnh lý 2. ( )
AB, A ' B ' ≡ α (mod 2π )

TAÄP SAN TOAÙN HOÏC - 2007 50


TRÖÔØNG THPT CHUYEÂN HOAØNG VAÊN THUÏ - HOØA BÌNH
PhÇn I - S¸ng t¹o to¸n häc.

( AB, A ' B ') ≡ α (mod π ) 2. Bµi tËp 2. C¸c chÊt ®iÓm A, B lÇn l−ît
chuyÓn ®éng ®ång thêi trªn c¸c ®−êng trßn
7. §Þnh lý 7.
Qua phÐp biÕn h×nh f, c¸c ®iÓm A, B, C, t©m O1 vµ O2 víi vËn tèc gãc nh− nhau (
D lÇn l−ît biÕn thµnh A',B',C',D'. Khi ®ã: theo cïng mét h−íng ). Chøng minh r»ng
a. NÕu f lµ mét trong c¸c phÐp biÕn ®Ønh C cña tam gi¸c ®Òu ABC còng chuyÓn
h×nh sau ®©y: TÞnh tiÕn, ®èi xøng t©m, quay, ®éng ®ång thêi trªn mét ®−êng trßn nµo ®ã.
vÞ tù, th×
    3. Bµi tËp 3. Cho tam gi¸c ABC, hai ®−êng
( ) ( )
AB, CD ≡ A ' B ', C ' D ' (mod 2π ) th¼ng ®èi xøng víi AB vµ BC qua c¹nh AC
b. NÕu f lµ phÐp ®èi xøng trôc th×: c¾t nhau t¹i K. Chøng minh r»ng ®−êng
   
( ) ( )
AB, CD ≡ C ' D ', A ' B ' (mod 2π )
th¼ng BK ®i qua t©m ®−êng trßn néi tiÕp tam
gi¸c ABC.
( AB, CD ) ≡ ( C ' D ', A ' B ') (mod π )
8. §Þnh lý 8.
Cho tam gi¸c ABC, c¸c ®iÒu kiÖn sau lµ
t−¬ng ®−¬ng:
a. ABC c©n t¹i A
   
( ) (
b. BA, BC ≡ CA, CB (mod 2π ) ) “Nh÷ng ph¸t minh cña t«i ®−îc b¾t nguån
tõ ®©u −? Håi t«i cßn ®i häc, c¸c b¹n cña
c. (BA,BC) ≡ (CB,CA) (mod Π) t«i lu«n hiÓu rÊt nhanh cßn t«i th× kh«ng
®−îc nh− vËy. V× vËy khi häc vÊn ®Ò nµo
VI. Mét sè bµi tËp ¸p dông. th× t«i còng ph¶i häc tËp nhiÒu h¬n vµ suy
nghÜ l©u h¬n c¸c b¹n, thËm chÝ nhiÒu vÊn
Bµi sè 1. Cho tam gi¸c ABC ®Òu, M thuéc ®Ò vÒ sau nµy t«i míi hiÓu. Bëi lý do ®ã cho
®−êng trßn ngo¹i tiÕp tam gi¸c ABC. MA, nªn c¸c vÊn ®Ò t«i th−êng hiÓu s©u h¬n so
MB, MC c¾t c¸c ®−êng th¼ng BC, CA, AB víi c¸c b¹n. ChØ cã vËy th«i”
t¹i A1, B1, C1 . Chøng minh r»ng tån t¹i duy - Albert Einstein –
nhÊt mét ®−êng th¼ng tiÕp xóc víi c¸c
®−êng trßn ®−êng kÝnh MA1, MB1, MC1.

Lêi gi¶i. C¸c b¹n tù vÏ h×nh.


Ta cã : ( MC1, C0C1 ) ≡ (MC,BA).
Mµ (MC,BA) ≡ (MC,MB) + ( MB,BC) +
+ (BC,BA)(modΠ)
≡ (MB,BC)(modΠ)
≡ (MB,BC)(modΠ)
≡ (MC0,C0A0)(modΠ)
⇒ C0A0 lµ tiÕp tuyÕn cña ®−êng trßn
(MC1C0)
Vµ C0A0 lµ tiÕp tuyÕn cña ®−êng trßn
(MA1A0)
Vµ C0B0 lµ tiÕp tuyÕn cña ®−êng trßn
(MB1B0) ⇒ ®pcm.

TAÄP SAN TOAÙN HOÏC - 2007 51


TRÖÔØNG THPT CHUYEÂN HOAØNG VAÊN THUÏ - HOØA BÌNH
PhÇn I - S¸ng t¹o to¸n häc.

Tieâu Chuaån Hoäi Tuï > 0, do lim xn = x nªn tån t¹i sè tù nhiªn N
n →∞
sao cho:

TOÅNG QUAÙT xi − x <


2k
ε
víi mäi i,j ≥ N.

M = Sup { xi − x , i ∈ N } . Ta cã:
Bïi lª vò n N

Chuyªn to¸n K01-04 ∑ ( xi − x)cin ≤


i =1
∑ (x
i =1
i − x)cin +
Sv. Líp 04TT, Khoa To¸n Tin, §HKHTN, n N
§HQG TP Hå ChÝ Minh ∑ ( xi − x)cin
i = N +1
≤ ∑ xi − x cin
i =1

NguyÔn th¸I ngäc


n
ε
Chuyªn to¸n k99-02
+ ∑x
i = N +1
i − x cin ≤ NM . cin +
2k
.
Sv. Líp §T8 – K48 n
ε
Khoa §iÖn tö ViÔn th«ng ⇒ ∑c
i = N +1
in
2
, víi mäi n ≥ N.
< NM . cin +
§H B¸ch Khoa Hµ Néi
Do lim cin = 0 nªn lu«n tån t¹i N0 > N
n →∞

ε
sao cho: cin < , víi mäi n ≥ N0
Chóng ta ®H tiÕp xóc víi rÊt nhiÒu 2 NM
Nh− vËy, víi mäi ε > 0, tån t¹i N0 sao
ph−¬ng ph¸p, tiªu chuÈn t×m giíi h¹n dHy sè. n
Bµi viÕt nµy t«i xin ®−îc giíi thiÖu mét tiªu cho víi mäi n ≥ N0 ta cã ∑ (x i − x)cin <
chuÈn héi tô tæng qu¸t. i =1

ε.
Tiªu chuÈn héi tô tæng qu¸t: n

Gi¶ sö víi mçi sè thùc n thuéc tËp hîp Suy ra lim


n→∞ ∑ (x
i =1
i − x )cin = 0
c¸c sè tù nhiªn N, c1n ,..., cnn lµ c¸c sè thùc
n
sao cho:
∑ xc
n

(i) lim cin = 0 víi mäi i.


n →∞
⇒ lim
n→∞ ∑x c
i =1
i in
= lim
n→∞
i =1
in = xc (®pcm)

n
(ii) cn = ∑ cin → c Mét c¸ch chñ ®éng, ta khai th¸c tiªu
i =1 chuÈn héi tô tæng qu¸t.
n
(iii) kn = ∑c i =1
in lµ dLy bÞ chÆn 1
XÐt dHy cin = , n = 1, 2, …, ta cã lim cin
n→∞
n
(nÕu cij kh«ng ©m th× kh«ng cÇn ®iÒu kiÖn n

nµy) =0,víi mäi i,vµ cn = ∑ cin = 1 . Ta thu ®−îc:


i =1
(iv) lim xn = x
n→∞
n KÕt qu¶ 1:
Khi ®ã ta cã: lim ∑ xi cin = xc . “ Cho dLy sè thùc {an} héi tô vÒ a. Víi
n →∞
i =1
n

∑a
n
Chøng minh. Do kn = ∑ cin lµ dHy bÞ chÆn, mäi n, ta ®Æt bn = n −1. i .
Khi ®ã dLy {bn}
i =1 i =1
gi¶ sö {kn } bÞ chÆn bëi k. Ta cã: víi mäi ε còng héi tô vÒ a”

TAÄP SAN TOAÙN HOÏC - 2007 52


TRÖÔØNG THPT CHUYEÂN HOAØNG VAÊN THUÏ - HOØA BÌNH
PhÇn I - S¸ng t¹o to¸n häc.

2i KÕt qu¶ 5:
XÐt dHy cin = , ta cã lim cin = 0 víi mäi Cho c¸cdLy sè thùc {an}, {bn}, {vn}, {un}
2n+1 n →∞
n
1 tháa mLn c¸c ®iÒu kiÖn sau:
i vµ cn = ∑ cin = 1 − . Do ®ã lim cn = 1 . (i) lim an = a
i =1 2n n →∞ n→∞

Ta thu ®−îc: (ii) lim vn= ∞ , vn ≠ 0 víi mäi n.


n→∞
n n
KÕt qu¶ 2: (iii) lim vn−1 ∑ ui = 1 , ∑ u i v n−1 bÞ chÆn.
n →∞
“ Cho dLy sè thùc {an} héi tô vÒ a. Víi i =1 i =1
n n
(iv) bn= v n−1 ∑ u i ai
mäi n, ta ®Æt bn = n −1. ∑2 a
i =1
i
i .
Khi ®ã dLy
i =1

{bn} còng héi tô vÒ a” Khi ®ã, ta cã lim bn=a


n→∞

XÐt dHy cin = 2i , ta cã lim cin = 0 víi Chøng minh. §Æt cin = vn−1ui . Do (ii) nªn
n(n + 1) n →∞
lim cin = 0 . Còng theo gi¶ thiÕt, ta cã:
n n →∞
mäi i vµ cn = ∑ cin = 1 . Ta thu ®−îc: n n
i =1 cn = ∑ cin = vn−1 ∑ ui , do ®ã lim cn = 1 . H¬n
n →∞
i =1 i =1

KÕt qu¶ 3: n n

“ Cho dLy sè thùc {an} héi tô vÒ a. Víi n÷a kn = ∑ cin = ∑ ui vn−1 lµ dHy bÞ chÆn.
i =1 i =1

2 n
VËy, ®iÒu kiÖn cña tiªu chuÈn héi tô tæng
mäi n, ta ®Æt bn = n −1 .∑ ai . Khi qu¸t ®H tháa mHn vµ ta sÏ cã lim bn =
n(n + 1) i =1 n →∞
®ã dLy {bn} còng héi tô vÒ a” n

2 n −i lim ∑ ai cin = a .
XÐt dHy cin = . n →∞
( n − i )! i =1
B©y giê chóng ta sÏ xem xÐt tiªu chuÈn
DÔ thÊy lim cin = 0 . L¹i cã: nãi trªn tæng qu¸t nh− thÕ nµo nhÐ!
n →∞
n −i
n
2 2n −1 2n − 2 Tiªu chu©n Stolz: Tiªu chuÈn nµy th−êng

i =1 ( n − i !
=
) ( ) (
n − 1 !
+
n − 2 ) !
+ ... + 1 → e 2 .
®−îc ph¸t biÓu d−íi hai d¹ng t−¬ng ®−¬ng
nh− sau: (c¸c b¹n hHy tù chøng minh sù
VËy ta thu ®−îc kÕt qu¶: t−¬ng ®−¬ng nµy xem nh− bµi tËp).
D¹ng 1:
KÕt qu¶ 4: Gi¶ sö {xn} vµ {yn} lµ hai dLy sè thùc sao
cho:
“ Cho dLy sè thùc { xn } trong ®ã
(i) {yn} lµ dLy t¨ng vµ lim yn = ∞
n n −i n→∞
2
lim xn = x . Khi ®ã yn = ∑ xi còng xn − xn−1
i =1 ( n − i ) ! =L
x →∞
(ii) lim
n→∞ y − y
n n−1
héi tô vµ héi tô vÒ e2 x ”
x
Khi ®ã lim n = L .
n→∞ y
Sau khi dïng ph−¬ng ph¸p ®Þnh nghÜa vµ n
thu ®−îc 4 kÕt qu¶ trªn, t«i ®H rót ra ®−îc D¹ng 2:
mét kÕt qu¶ më réng. Gi¶ sö {U n } vµ {Vn } lµ hai dHy sè thùc
sao cho:

TAÄP SAN TOAÙN HOÏC - 2007 53


TRÖÔØNG THPT CHUYEÂN HOAØNG VAÊN THUÏ - HOØA BÌNH
PhÇn I - S¸ng t¹o to¸n häc.

n s¸ng t¹o ra ®−îc rÊt nhiÒu bµi to¸n. Sau ®©y


(i) Vn > 0 víi mäi n vµ lim ∑ Vi = +∞ mêi c¸c b¹n lµm mét sè bµi to¸n t−¬ng tù coi
n →∞
i =1
nh− bµi tËp tr−íc khi kÕt thóc bµi viÕt nµy:
Un
(ii) lim =t .
Vn n→∞ Bµi tËp 1:
U +U + ... +Un Cho dHy sè { xn } cã giíi h¹n lim xn = x vµ
Khi ®ã, lim 1 2 =t. n →∞
n→∞ V +V + ... +V
1
c¸c sè thùc α ≠ ; a, b > 0 . §Æt :
1 2 n

2
Chøng minh:
(i + ai + b )
2 α
n
U
§Æt xn = n , cin =
Vi
, i = 1, 2, …, n. yn = ∑ xi . T×m giíi h¹n dHy
Vn V1 + ... +Vn i =1 n 2α + 1
n sè { yn } khi n → ∞
Ta cã lim xn = t vµ cn = ∑ cin = 1 ,
n →∞
i =1

lim cn = 0 Bµi tËp 2:


n →∞
DHy sè { xn } cã giíi h¹n lim xn = x .
Theo tiªu chuÈn héi tô tæng qu¸t, ta cã: n →∞
n T×m giíi h¹n cña c¸c tæng sau:
lim ∑ xi cin = xc hay lµ: n
n + i2
n →∞
i =1 a) ∑ 3 3 xi khi n → ∞ .
U1 + U2 + .. + U n i =1 n + i
lim =1 n
i i −1
n→∞ V1 + V2 + .. + Vn b) ∑ i xi khi n → ∞ .
i =1 n

Tiªu chuÈn trung b×nh Cesaro.


Cho c¸c dLy sè thùc {an}, {bn}, {vn} tháa Bµi tËp 3:
mLn c¸c ®iÒu kiÖn: Cho u0 , u1 , α ∈ R . Vµ dHy {un } tho¶ mHn:
(i) lim an = a , lim bn = b , bn > 0 víi mäi n. 1 n n
uiα
n →∞
bna1 + .. + ba
n →∞
un +1 = ∑ i
n + 1 i =0
u . T×m lim
n →∞

i =0 n + 1
.
(ii) vn = 1 n
víi mäi n.
n
Khi ®ã lim vn = ab.
n→∞
Chøng minh.
b
§Æt cin = n+1−i , suy ra lim cin = 0 .
n n →∞
n

n ∑b i
H¬n n÷a, cn = ∑ cin = nªn theo kÕti =1

n i =1

qu¶ 1, ta cã lim cn = b . Theo tiªu chuÈn héi


n →∞
tô tæng qu¸t, ta cã
n
lim vn = lim ∑ ai cin = ab .
n →∞ n →∞
i =1

Chóng ta thÊy ®Êy, chØ cÇn linh ho¹t


trong c¸ch chän dHy {cin } lµ chóng ta cã thÓ

TAÄP SAN TOAÙN HOÏC - 2007 54


TRÖÔØNG THPT CHUYEÂN HOAØNG VAÊN THUÏ - HOØA BÌNH
PhÇn I - S¸ng t¹o to¸n häc.

ng Dng ð
nh lý Stolz
TRONG Tìm Gii H n c a Dãy S
Ng« NhÊt S¬n
Chuyªn To¸n K01-04
Sv. K49 Khoa §iÖn tö viÔn th«ng, §H B¸ch Khoa Hµ Néi

ðÞnh lý Stolz tá ra rÊt cã hiÖu qu¶ trong an +1 − an =


an
=
1
.
viÖc t×m giíi h¹n cña dHy sè. Bµi viÕt nµy xin an (an + 1) + an 1
giíi thiÖu mét sè vÝ dô thÓ hiÖn øng dông 1+ +1
an
trùc tiÕp cña ®Þnh lý nµy ®èi víi viÖc t×m giíi
h¹n cña dHy sè. XÐt dHy: bn = n, ta thÊy dHy (bn) t¨ng vµ
a −a 1 1
lim n +1 n = lim = .
n +1 − bn
I. §Þnh lý Stolz. n →∞ b n →∞ 1 2
Cho hai dLy sè thùc (an) vµ (bn) tháa 1+ +1
an
mLn:
i) b1 < b2 < .... < bn Theo ®Þnh lý Stolz, suy ra ®pcm.
ii) lim bn = ∞
n → +∞ 2. Bµi to¸n 2. Cho dLy sè d−¬ng (xn) tháa
a n +1 − a n 1
iii) lim =k mLn xn+1 = xn + , ∀n ≥ 1
n → +∞ bn +1 − bn xn
an xn
Khi ®ã ta còng cã lim =k Chøng minh r»ng lim =1
n → +∞ n → +∞
bn 2n
II. Mét sè bµi to¸n. Lêi gi¶i. Tõ gi¶ thiÕt, ta cã:
1
x n2+1 = x n2 + 2 + 2, ∀n ≥ 1
1. Bµi to¸n 1. Cho dLy sè thùc d−¬ng (an) xn
tháa mLn : §Æt an +1 = xn2+1 , b n = 2n . Khi ®ã (an),
a 2n +1 = a1 + a 2 + .... + a n , ∀n ≥ 1
(bn) t¨ng vµ bn → +∞ vµ xn → +∞ ..
a 1 Ta l¹i cã :
Chøng minh r»ng: lim n = .
bn 2 a −a x2 − x2
Lêi gi¶i. DÔ thÊy dHy (an) t¨ng vµ lim n +1 n = lim n +1 n =
n +1 − bn
n →+∞ b n →+∞ 2
a n2+1 = a n2 + a n , ∀n ≥ 1 , nªn lim an = +∞
n →+∞ 1
2+ 2
MÆt kh¸c: xn
= lim =1
an +1 − an = an (an + 1) − an n →+∞ 2
⇒ ( an +1 − an ) ( )
an ( an + 1) + an =
Theo ®Þnh lý Stolz suy ra:
a x2
lim n = 1 ⇒ lim n = 1 .
= an (an + 1) − an2 n → +∞ b n → +∞ 2 n
n

TAÄP SAN TOAÙN HOÏC - 2007 55


TRÖÔØNG THPT CHUYEÂN HOAØNG VAÊN THUÏ - HOØA BÌNH
PhÇn I - S¸ng t¹o to¸n häc.

3. Bµi to¸n 3. Cho dLy (xn) x¸c ®Þnh nh− Bµi to¸n ®−îc gi¶i quyÕt. Sau ®©y lµ mét sè
sau: ®Ò bµi ®Ó c¸c b¹n tù luyÖn:
i) 0 <x0<1 Bµi tËp 1. Cho dHy (Un) x¸c ®Þnh nh− sau:
−1
ii) xn+1 = xn(1-xn)
Chøng minh r»ng: U0 > 0, Un+1 = Un - e U n2
, ∀n ∈ N
a) lim nxn = 1. Chøng minh r»ng lim U n2 ln n = 1 .
n → +∞ n →∞

n(1 − nx n ) Bµi tËp 2. Cho dHy (Un) x¸c ®Þnh nh− sau:
b) lim =1 U0 = 3, Un+1 = U n2 − 2, ∀n ∈ N
n → +∞ ln n
Lêi gi¶i. n −1

a) Ta cã: xn+1 = xn(1-xn) vµ 0 < x0 <1 nªn TÝnh lim


n → +∞
2n
∏U
i =1
i

xn ≠ 0, ⇒ 1 − 1 = 1 , ∀n ≥ 0
x n +1 x n 1 − x n Cuèi cïng, rÊt mong qua bµi viÕt nµy c¸c b¹n
1 sÏ cã nhiÒu kinh nghiÖm khi gi¶i c¸c bµi to¸n vÒ
§Æt an = n, bn =
xn giíi h¹n.
DÔ dµng chøng minh ®−îc (xn) gi¶m vµ
→ 0 nªn bn → +∞ . VËy, theo ®Þnh lý Stolz,
a
ta cã lim nxn = lim n = 1.
n → +∞ n → +∞ b
n
b) Ta cã :
n n “NÕu nh− t«i ®` ph¸t hiÖn nh÷ng ch©n lý
 1  1 míi mÎ nµo ®ã trong Khoa häc th× t«i cã
lim 1 +  = e ⇒ lim ln1 +  = 1
n → +∞
 n n → +∞
 n thÓ kh¼ng ®Þnh r»ng tÊt c¶ c¸c ch©n lý ®ã
Theo kÕt qu¶ phÇn a) ta cã: ®Òu, hoÆc lµ nh÷ng hÖ qu¶ trùc tiÕp cña
xn n¨m hay s¸u bµi to¸n chñ yÕu mµ t«i ®`
gi¶i ®−îc, hoÆc tïy thuéc vµo c¸c bµi to¸n
nxn 1 − xn
lim = 1 ⇒ lim =1 ®è vµ t«i xem chóng nh− nh÷ng cuéc chiÕn
n →+∞ 1 − x n →+∞  1 
n
ln 1 +  ®Êu trong ®ã niÒm vui th¾ng lîi thuéc vÒ
 n t«i”
1 1
− −1 - Decartes -
x xn
⇒ lim n +1 =1
n →+∞ ln( n + 1) − ln n

1 1 
− (n + 1) −  − n 
x  xn  =1
⇒ lim n +1
n →+∞ ln(n + 1) − ln n
1
§Æt an = − n vµ bn = ln n , ta cã ngaydHy
xn
(bn) t¨ng vµ bn → +∞
a a − an
⇒ lim n = lim n +1 =1
n +1 − bn
n → +∞ b n → +∞ b
n

1 − nx n n(1 − nx n )
⇒ lim = 1 ⇒ lim =1
n → +∞ x n ln n n → +∞ ln n

TAÄP SAN TOAÙN HOÏC - 2007 56


TRÖÔØNG THPT CHUYEÂN HOAØNG VAÊN THUÏ - HOØA BÌNH
PhÇn I - S¸ng t¹o to¸n häc.

ÖÙNG DUÏNG CUÛA MOÄT BAØI TOAÙN TOÅNG QUAÙT


NguyÔn hµ thuËt
Chuyªn to¸n k01-04
Sv. Líp Tin 6 – K49, Khoa CNTT, §H B¸ch Khoa Hµ Néi

Ña thøc lµ mét m¶ng to¸n hay vµ khã f ( P ).ϕ (Q) + f (Q).ϕ ( P ) + ϕ ( P ).ϕ (Q) = ϕ ( H )
trong ch−¬ng tr×nh to¸n trung häc phæ th«ng. (2). B©y giê ta so s¸nh sè h¹ng cã bËc cao
ChÝnh v× thÕ nã th−êng ®−îc khai th¸c trong nhÊt ë hai vÕ cña (2). Ta thÊy:
c¸c ®Ò thi Häc sinh giái trong, ngoµi n−íc vµ deg { f ( P).ϕ (Q )} = kp + lq
mét sè khu vùc. §Ó lµm t¨ng thªm vÎ ®Ñp
cña m¶ng to¸n ®a thøc, trong ph¹m vi bµi deg { f (Q).ϕ ( P )} = kq + lp
viÕt nµy t«i xin tr×nh bµy vÒ mét bµi to¸n ®a deg {ϕ ( P).ϕ (Q)} = lp + lq
thøc tæng qu¸t, cã nhiÒu øng dông.
I. Bµi to¸n gèc. deg {ϕ ( H )} = lh = lp + lq
Cho P ( x), Q( x), H ( x) lµ c¸c ®a thøc tháa BËc cña ®a thøc vÕ ph¶i lµ l ( p + q ) , cßn
mLn ®iÒu kiÖn:
cña vÕ tr¸i lµ Max {kp + lq; kq + lp; l ( p + q)} .
i) P ( x), Q( x), H ( x) cã c¸c hÖ sè thùc.
Do ®ã, nÕu p ≠ q th× hiÓn nhiªn ta cã bËc
ii) Min {deg P ( x);deg Q( x)} ≥ 1 , trong ®ã ký
cña vÕ tr¸i lín h¬n vÕ ph¶i.
hiÖu deg chØ bËc cña ®a thøc. NÕu p = q th× xÐt hÖ sè cao nhÊt hai vÕ
iii) Max {deg P( x);deg Q( x)} = deg( P + Q) cña (2): DÔ thÊy hÖ sè bËc cao nhÊt cña vÕ
Khi ®ã, víi mäi sè nguyªn d−¬ng k, tån ph¶i lµ c(ab)l trong khi cña vÕ tr¸i lµ
t¹i kh«ng qu¸ mét ®a thøc hÖ sè thùc f ( x)
c(ab)l = c(ab)l (a k −l + b k −l ) . L−u ý lµ tõ ®iÒu
cã bËc k, cã hÖ sè bËc cao nhÊt b»ng 1 vµ
kiÖn iii) cña gi¶ thiÕt ta cã a ≠ −b , nªn tõ
tháa mLn ®ång nhÊt thøc
trªn ta cã m©u thuÉn. DÉn ®Õn
f ( P ( x)). f (Q( x)) = f ( H ( x)) (1). ϕ ( x) ≡ 0 (®pcm).
Chøng minh. Gi¶ sö f ( x) lµ cã bËc k tháa II. Mét sè bµi to¸n hÖ qu¶.
mHn ®iÒu kiÖn ®Ò bµi vµ ϕ ( x) lµ ®a thøc cã Bµi 1. T×m tÊt c¶ c¸c ®a thøc f ( x) tháa
bËc bÐ h¬n k. Ta sÏ chøng minh r»ng: NÕu
mLn f 2 ( x) = f ( x 2 ), ∀x ∈ R (3)
®a thøc ( f + ϕ ) tháa mHn (1) th× ®a thøc
Lêi gi¶i. NÕu deg f ( x) < 1 . Ta cã
ϕ ( x) ph¶i ®ång nhÊt víi ®a thøc kh«ng.
ThËt vËy, gäi a, b, c lÇn l−ît lµ hÖ sè bËc  f ( x) = 0, ∀x ∈ R
 f ( x) = 1, ∀x ∈ R
cao nhÊt cña P ( x), Q( x) vµ ϕ ( x) . 
§Æt deg H ( x) = h , deg P ( x) = p ≥ 1 vµ NÕu deg f ( x) = n ≥ 1 . Ta thÊy f ( x) = x n
deg Q( x) = q ≥ 1 . Khi ®ã tõ (1) ⇒ lµ ®a thøc tháa mHn (3). Ta chøng minh
deg H ( x) = deg P ( x) + deg Q( x) , hay lµ f ( x) tån t¹i duy nhÊt víi mçi n ∈ N * . ThËt
h = p + q . §Æt deg ϕ ( x) = l th× 0 ≤ l ≤ k − 1 . vËy, gi¶ sö tån t¹i ®a thøc ϕ ( x) tháa mHn
V× gi¶ thiÕt f ( x) vµ ( f + ϕ )( x) ®Òu tháa ®iÒu kiÖn 0 < deg ϕ ( x) = k < n vµ ( f + ϕ )
mHn (1) nªn ta cã c¸c ®ång nhÊt: tháa mHn (3). Thay vµo (3) ta cã:
f ( P ( x)). f (Q( x)) = f ( H ( x)) vµ ϕ 2 ( x) + f 2 ( x) + 2ϕ ( x) f ( x) = ϕ 2 ( x) + f 2 ( x)
( f ( P) + ϕ ( P) ) . ( f (Q) + ϕ ( P) ) = f (H ) + ϕ (H ) ⇒ ϕ 2 ( x) + 2ϕ ( x) f ( x) = ϕ 2 ( x) ( ∀x ∈ R )
Tõ ®©y suy ra So s¸nh bËc hai vÕ ta thÊy bËc cña vÕ tr¸i
lµ n + k lín h¬n bËc cña vÕ ph¶i lµ 2k

TAÄP SAN TOAÙN HOÏC - 2007 57


TRÖÔØNG THPT CHUYEÂN HOAØNG VAÊN THUÏ - HOØA BÌNH
PhÇn I - S¸ng t¹o to¸n häc.

⇒ ®pcm. VËy tÊt c¶ c¸c ®a thøc cÇn t×m lµ Víi an = 1 , gi¶ sö x0 lµ nghiÖm thùc
 f ( x) = 0, ∀x ∈ R (kh¸c 0) cña f ( x) . Tõ (4) suy ra dHy sè ®¬n
 f ( x) = 1, ∀x ∈ R ®iÖu v« h¹n sau ®©y ®Òu lµ nghiÖm cña
 f ( x) : xk +1 = 2 xk3 + xk víi mäi k = 0, 1, 2, …
 f ( x) = x n , ∀x ∈ R
§ã lµ ®iÒu v« lý, thµnh thö f ( x) v« nghiÖm
Chó ý. Ngoµi c¸ch gi¶i trªn, ta cã thÓ gi¶i
quyÕt bµi to¸n theo h−íng kh¸c nh− sau: thùc. Chøng tá f ( x) cã bËc ch½n (v× ®a thøc
XÐt khi deg f ( x) = n ≥ 1 . Gi¶ sö bËc lÎ lu«n lu«n cã nghiÖm thùc).
B©y giê ta t×m tÊt c¶ c¸c ®a thøc f ( x) cã
f ( x) = a0 x n + a1 x n −1 + ... + an −1 x + an , (a0 ≠ 0)
bËc ch½n 2n tháa mHn (4).
Gäi k lµ sè nguyªn d−¬ng nhá nhÊt tháa NhËn thÊy r»ng ®a thøc
mHn ak ≠ 0
f ( x) = ( x 2 + 1) , ∀x ∈ R tháa mHn (4). Ta sÏ
n

⇒ f ( x) = a0 x n + ak x n − k + ... + an −1 x + an . Thay
chøng minh víi mçi n th× ®a thøc f ( x) nh−
vµo (3) suy ra ngay a0 = 1 . MÆt kh¸c, so
trªn lµ duy nhÊt. Gi¶ sö tån t¹i ®a thøc F(x)
s¸nh hÖ sè cña x 2 n − k cña hai vÕ ta cã
cã bËc 2n nh−ng kh¸c f ( x) = ( x 2 + 1) , tháa
n

2a0 ak = 0 ⇒ ak = 0 .
mHn ®iÒu kiÖn (1).
VËy an = an −1 = ... = a1 = 0 Ph©n tÝch F ( x) = f ( x) + g ( x) trong ®ã
⇒ f ( x) = a0 x n = x n . 0 < deg { g ( x)} = k < 2n . Thay vµo (4) ta cã:
Bµi 2. T×m tÊt c¶ c¸c ®a thøc f ( x) tháa
F ( x).F (2 x 2 ) = F (2 x 3 + x), ∀x ∈ R
mLn f ( x). f (2 x 2 ) = f (2 x3 + x), ∀x ∈ R (4)
(§Ò Dù tuyÓn Quèc tÕ 1979) ⇔ ( f ( x) + g ( x) ) . ( f (2 x 2 ) + g (2 x 2 ) ) =
Lêi gi¶i. NÕu deg f ( x) < 1 . Ta cã = f (2 x3 + x) + g (2 x3 + x), ∀x ∈ R
 f ( x) = 0, ∀x ∈ R ⇔ g ( x). f (2 x 2 ) + g (2 x 2 ). f ( x) +
 f ( x) = 1, ∀x ∈ R + g ( x).g (2 x 2 ) = g (2 x 2 + x), ∀x ∈ R .

NÕu deg f ( x) = n ≥ 1 . Tr−íc hÕt ta chøng So s¸nh bËc ë hai vÕ cña ®ång nhÊt thøc
minh f ( x) kh«ng cã nghiÖm thùc. ThËt vËy, trªn ta thÊy bËc cña vÕ tr¸i lµ 2n + k trong
gi¶ sö khi bËc cña vÕ ph¶i lµ 3k < 2n + k. M©u
thuÉn nhËn ®−îc chøng tá kh«ng tån t¹i ®a
f ( x) = a0 x n + a1 x n −1 + ... + an −1 x + an , (a0 ≠ 0) thøc thøc F(x) cã bËc 2n nh−ng kh¸c
Thay f ( x) vµo (4) vµ so s¸nh hÖ sè cao
f ( x) = ( x 2 + 1) ®ång thêi tháa mHn (1).
n

nhÊt, hÖ sè tù do cña hai vÕ ®¼ng thøc võa


nhËn ®−îc, ta cã: VËy tÊt c¶ c¸c ®a thøc cÇn t×m lµ:
 a0 = 1 
f ( x) = 0, ∀x ∈ R
 an2 = an  
 2 n ⇒   an = 0 .  f ( x) = 1, ∀x ∈ R
a0 .2 = a0 .2
n
 a = 1 
 n  f ( x) = ( x 2 + 1) , ∀x ∈ R
n

NÕu an = 0 , viÕt 
§Ó hiÓu s©u thªm vÒ bµi to¸n tæng qu¸t
f ( x) = x k .g ( x), g ( x) ≠ ∀x ∈ R trªn, mêi c¸c b¹n lµm mét sè bµi tËp sau:
Thay vµo (4) ta cã Bµi tËp. T×m tÊt c¶ c¸c ®a thøc cã hÖ sè thùc
x k . ( 2 x 2 ) .g ( x).g ( x 2 ) = (2 x 3 + x)k g (2 x3 + x) f ( x) tháa mLn:
k

a) f 2 ( x − 2) = f ( x 2 − 2), ∀x ∈ R
, víi mäi x ∈ R ⇒
(§Ò thi Häc sinh giái Acmenia 1996)
( 2x )
2 k
.g ( x).g ( x 2 ) = (2 x 2 + 1) k g (2 x3 + x) , b) f ( x). f ( x − 1) = f ( x 2 ), ∀x ∈ R
víi mäi x ∈ R . (§Ò thi Häc sinh giái Ai x¬ len 1997)
Cho x = 0 ⇒ g(0) = 0, v« lý.

TAÄP SAN TOAÙN HOÏC - 2007 58


TRÖÔØNG THPT CHUYEÂN HOAØNG VAÊN THUÏ - HOØA BÌNH
PhÇn I - S¸ng t¹o to¸n häc.

Tp Dt
0 → 0 →
Q 90 ( MM 1 ) + Q 90 ( MM 2 ) +
→ → → → →
+ Q 90 ( MM 3 ) = A1 A2 + A2 A3 + A3 A1 = 0
0

saùng taïo Tõ ®ã, theo tÝnh chÊt 2 ta cã ngay ®iÒu


ph¶i chøng minh.
B©y giê ta sÏ më réng bµi to¸n.

§Æng Phïng H−ng I. Më réng cho tam gi¸c.


Chuyªn To¸n K02 – 05
Sv. §H Kinh tÕ Quèc d©n Hµ Néi NhËn thÊy 2 ®iÒu r»ng:
→ → → →
A = 0 ⇔ k A = 0 , ∀k ≠ 0
→ → → →
Kh«ng ngõng ph¸t triÓn vµ s¸ng t¹o sau Q α ( A) = 0 ⇔ A = 0 , ∀α
mçi bµi to¸n lµ mét ®iÒu cÇn thiÕt víi mçi
häc sinh yªu to¸n. NiÒm vui sÏ t¨ng lªn khi 1. Bµi to¸n. Cho tam gi¸c A1A2A3 vµ M lµ
c¸c b¹n t×m ®−îc nh÷ng kÕt qu¶ míi mÎ tõ ®iÓm thuéc miÒn trong tam gi¸c. M1, M2, M3
bµi to¸n võa gi¶i xong. D−íi ®©y lµ mét vÝ
 → → 
dô mµ t«i ®H hoµn thµnh. Xin giíi thiÖu cïng lµ c¸c ®iÓm tháa mLn  MM 1 , A2 A3 
b¹n ®äc:  
 → →   → → 
Bµi to¸n gèc (Bµi to¸n con nhÝm) . =  MM 2 , A3 A1  =  MM 3 , A1 A2  = α
Cho tam gi¸c A1A2A3. Gäi M lµ ®iÓm bÊt    
k× n»m trong tam gi¸c A1A2A3. M1M2 M3 lµ MM 1 MM 2 MM 3
c¸c ®iÓm tho¶ mLn MM, MM2, MM3 lÇn l−ît vµ = = = k .Chøng minh
A2 A3 A3 A1 A1 A2
vu«ng gãc vµ b»ng A3A2, A2A1, A1A3. §ång → → → →
thêi c¸c tia MM1, MM2, MM3 lÇn l−ît c¾t r»ng: MM 1 + MM 2 + MM 3 = 0
c¸c ®−êng A3A2, A3A1, A1A2 . Chøng minh
r»ng: Lêi gi¶i. XÐt phÐp quay vect¬:
→ → → →
→ → →
MM 1 + MM 2 + MM 3 = 0 Q 90 ( MM 1 + MM 2 + MM 3 ) =
0

=
Lêi gi¶i. Tr−íc hÕt ta nh¾c l¹i 2 tÝnh chÊt c¬ → → →
b¶n cña phÐp quay vect¬ :
0 0 0
Q 90 ( MM 1 ) + Q 90 ( MM 2 ) + Q 90 ( MM 3 ) =
→ → →
- NÕu a + b = c th× 1 → → → →
= ( A1 A2 + A2 A3 + A3 A1 ) = 0
→ → → k
Qα  c  = Qα  a  + Qα  c 
     
2. HÖ qu¶.
→ → → →
- Qα  A = 0 ⇔ A = 0 Tõ bµi to¸n trªn, ta cã thÓ suy ra c¸c bµi to¸n
  sau:
C¶ hai tÝnh chÊt trªn ®óng víi mäi gãc
quay α . 2.1. HÖ qu¶ 1. Víi M lµ träng t©m tam gi¸c
Trë l¹i bµi to¸n. XÐt phÐp quay th×:
→ → → → → → →
Q 90 ( MM 1 + MM 2 + MM 3 ) =
0

MM 1 + MM 2 + MM 3 = 0

TAÄP SAN TOAÙN HOÏC - 2007 59


TRÖÔØNG THPT CHUYEÂN HOAØNG VAÊN THUÏ - HOØA BÌNH
PhÇn I - S¸ng t¹o to¸n häc.

→ → → Chøng minh. T−¬ng tù phÇn trªn nªn xin


2.2. HÖ qu¶ 2. Víi i1 , i2 , i3 lµ c¸c vect¬ ®¬n dµnh cho b¹n ®äc coi nh− bµi tËp kÕt thóc
→ → → bµi b¸o nµy.
vÞ lÇn l−ît cïng h−íng víi MA1 , MA2 , MA3 .
Khi ®ã: B»ng niÒm say mª, ham thÝch häc tËp vµ
→ → s¸ng t¹o, t«i xin chóc c¸c b¹n sÏ t×m ®−îc
i1 sin ∠M 2 MM 3 + i2 sin M 3 MM 1 + nh÷ng kÕt qu¶ ®Ñp tõ nh÷ng bµi to¸n ®Çu.
→ →
+ i3 sin M 1MM 2 = 0

Chøng minh. Tõ A1, A2, A3 lÇn l−ît vÏ c¸c


®−êng th¼ng vu«ng gãc víi MA1, MA2, MA3
vµ chóng c¾t nhau t¹i B1, B2, B3. Theo bµi
to¸n “con nhÝm”, ta cã : Kh«ng cã chiÕn th¾ng nµo vinh quang h¬n
→ → → →
i1 B2 B3 + i2 B3 B1 + i3 B1 B2 = 0 lµ chiÕn th¾ng chÝnh b¶n th©n m×nh.
→ → → →
⇔ i1 sin B1 + i2 sin B2 + i3 sin B3 = 0 -V.I.LeNin-
Tõ ®ã suy ra ®pcm.

2.3. HÖ qu¶ 3.
→ → → →
S A2 MA3 A1 M + S A1MA3 A2 M + S A1MA2 A3 M = 0
HÖ thøc nµy ®−îc chøng minh tõ HÖ qu¶
2.1. Tõ ®©y ta suy ra:
→ → → →
OA sin 2 A + OB sin 2 B + OC sin 2C = 0
víi tam gi¸c ABC nhän vµ O lµ t©m ®−êng
trßn ngo¹i tiÕp tam gi¸c.
→ → → →
a IA + b IB + c IC = 0 víi I lµ t©m
®−êng trßn néi tiÕp tam gi¸c.

II. Më réng bµi to¸n cho ®a gi¸c.

Cho ®a gi¸c A1A2...An. M lµ ®iÓm n»m


trong ®a gi¸c. Mi lµ c¸c ®iÓm tháa mLn:
   
( ) (
MM 1 , A2 A3 = MM 2 , A3 A4 = .... = )
 
( )
= MM n , A1 A2 = α vµ
MM 1 MM 2 MM n
= = ... = =k
A2 A3 A3 A4 A1 A2
Chøng minh r»ng:
→ → → →
MM 1 + MM 2 + ..... + MM n = 0

TAÄP SAN TOAÙN HOÏC - 2007 60


TRÖÔØNG THPT CHUYEÂN HOAØNG VAÊN THUÏ - HOØA BÌNH
PhÇn I - S¸ng t¹o to¸n häc.

Vaän duïng ñònh lyù Saùch Giaùo Khoa


Linh Hoaït
TrÞnh Anh TuÊn
Chuyªn To¸n, K2001 – 2004
Sv. Líp ¤ T« - K49, Khoa C¬ KhÝ - §H B¸ch Khoa Hµ Néi

C¸c b¹n th©n mÕn ! Ký hiÖu S(XYZ) lµ diÖn tÝch cña tam gi¸c
ViÖc ®äc vµ thuéc ®Þnh lý s¸ch gi¸o khoa XYZ, ta cã:
qu¶ thËt lµ dÔ dµng. Song ®Ó øng dông nã 1
mét c¸ch tèi −u th× qu¶ thËt lµ khã kh¨n. Sau . AH . AG. sin HAG
S ( AHG ) 2 AH AG
®©y t«i cïng b¹n t×m hiÓu mét vµi vÝ dô ®Ó = = .
thÊy ®iÒu ®ã. S ( ABI ) 1 AB AI
. AB. AI . sin BAI
Ai còng biÕt träng t©m cña tam gi¸c lµ 2
giao ®iÓm cña 3 ®−êng trung tuyÕn, lµ ®iÓm T−¬ng tù nh− vËy:
chia trung tuyÕn theo tØ sè 1 : 3 … Nh−ng, S ( AGK ) AK AG
= .
®Ó ¸p dông vµo bµi to¸n sau ®©y, ph¶i thËt S ( AIC ) AC AI
“khÐo lÐo”:
S ( AHK ) AK AH
= .
1. Bµi to¸n 1. Cho ∆ABC vµ G lµ träng t©m S ( ABC ) AC AB
cña tam gi¸c ®ã. Mét ®−êng th¼ng bÊt kú ®i ¸p dông c¸c ®¼ng thøc võa thiÕt lËp ë trªn,
qua G c¾t AB, AC lÇn l−ît t¹i H vµ K . S ( AHK ) S ( AHG ) S ( AKG )
ta cã: = +
AC AB S ( ABC ) 2 S ( ABC ) 2 S ( ABC )
Chøng minh r»ng + =3.
AK AH AK AH AH AG AK AG
⇔ 2 . = . + .
AC AB AB AI AC AI
Lêi gi¶i. AC AB AI
⇔ + = 2. = 3.
AK AH AG

B©y giê ta xÐt bµi to¸n khã h¬n sau:

2. Bµi to¸n 2. Cho tø diÖn SABC víi G lµ


träng t©m cña tø diÖn ®ã. MÆt ph¼ng (P) qua
G c¾t AB, AC lÇn l−ît t¹i H, K. Chøng minh
4 V ( ASHK ) 1
r»ng: ≤ ≤
9 V ( ABCD ) 2
ë ®©y, ký hiÖu V chØ thÓ tÝch cña tø diÖn.

Lêi gi¶i.

TAÄP SAN TOAÙN HOÏC - 2007 61


TRÖÔØNG THPT CHUYEÂN HOAØNG VAÊN THUÏ - HOØA BÌNH
PhÇn I - S¸ng t¹o to¸n häc.

AC AB Gi¶ sö SG ∩ MP ( ABC ) = G ' . T−¬ng tù


Tõ Bµi to¸n 1 trªn, ta cã + = 3. nh− Bµi to¸n 1, th«ng qua mét kÕt qu¶ quen
AK AH
biÕt vÒ tû lÖ thÓ tÝch cña hai h×nh chãp tam
AC 1 AB 1
§Æt = , (0 < x ≤ 1) ⇒ = 3− . gi¸c h×nh thµnh trªn cïng mét gãc tam diÖn,
AK x AH x ta thiÕt lËp ®−îc c¸c tû lÖ thøc sau:
AB 1 V ( SA ' B ' G ) SA '.SB '.SG
§Ó ý lµ ≥ 1 nªn x ≤ . Sö dông c«ng =
AH 2 V ( SABG ') SA.SB '.SG '
thøc tÝnh thÓ tÝch cho c¸c tø diÖn SAKH vµ V ( SA ' C ' G ) SA '.SC '.SG
SABC vµ víi chó ý: Hai tø diÖn nµy cã cïng =
®−êng cao h¹ tõ ®Ønh S t−¬ng øng, ta ®−îc: V ( SACG ') SA.SC '.SG '
1 V ( SC ' B ' G ) SC '.SB '.SG
. AH . AK . sin A =
V ( SAKH ) S ( AKH ) 2 V ( SCBG ') SC.SB '.SG '
= = =
V ( SABC ) S ( ABC ) 1 V ( SA ' B ' C ') V ( SA ' B ' G ) V ( SA ' C ' G )
. AB. AC.SinA = + +
2 V ( SABC ) 3V ( SABG ') 3V ( SACG ')
AH AK x x2 V ( SC ' B ' G )
= . = .x = +
AB AC 3 x − 1 3x − 1 3V ( SCBG ')
2
x SA SB SC
§Æt f ( x) = vµ xÐt sù biÕn thiªn ⇒ + + = 4.
3x − 1 S ' A' S ' B ' S 'C '
1  Bµi to¸n ®−îc chøng minh.
cña hµm sè nµy trªn miÒn  ;1 ta dÔ dµng
2 
4. Bµi to¸n 3.
4 1
cã ®−îc ≤ f ( x) ≤ , tõ ®ã cã ®−îc kÕt Cho h×nh hép ABCD.A’B’C’D’. Chøng
9 2 minh r»ng ®−êng th¼ng AC’ ®i qua träng
qu¶ cña bµi to¸n. t©m cña tam gi¸c A’BD vµ ®iÓm nµy chia
®o¹n th¼ng AC’ theo tØ sè1: 2.
3. Më réng bµi 1.
Cho tø diÖn SABC víi träng t©m G. Mét Lêi gi¶i.
mÆt mÆt ph¼ng (P) bÊt kú ®i qua träng G c¾t
c¸c c¹nh SA, SB, SC lÇn l−ît t¹i A’, B’, C’. (Xem h×nh vÏ trang sau)
SA SB SC
Chøng minh r»ng: + + =4
S ' A' S ' B ' S 'C '

Lêi gi¶i.

TAÄP SAN TOAÙN HOÏC - 2007 62


TRÖÔØNG THPT CHUYEÂN HOAØNG VAÊN THUÏ - HOØA BÌNH
PhÇn I - S¸ng t¹o to¸n häc.

nhau qua mÆt ph¼ng (A1B1C1). VËy, ¸p dông


bµi to¸n trªn ta cã ngay ®iÒu ph¶i chøng
minh.

Sau ®©y lµ hai bµi tËp dµnh cho b¹n ®äc

Bµi tËp 1.
Gi¶ sö G lµ träng t©m cña tø diÖn ABCD
vµ M lµ mét ®iÓm bÊt kú thuéc mÆt ph¼ng
(BCD). Qua M dùng ®−êng th¼ng song song
víi BG c¾t mÆt ph¼ng (ACD) t¹i B1. T−¬ng
tù x¸c ®Þnh c¸c ®iÓm C1, D1. Chøng minh
Ký hiÖu O = BD ∩ AC , I = AC '∩ A ' O . Ta
r»ng ®−êng th¼ng MG ®i qua träng t©m cña
IO AO 1 tam gi¸c B1C1D1.
cã = = . Mµ O lµ trung ®iÓm cña
IA ' A ' C ' 2
BD nªn suy ra I lµ träng t©m cña tam gi¸c Bµi tËp 2.
A’BD. Cho tø diÖn SABCcã c¸c c¹nh SA = a, SB
= b, SC = c. Mét mÆt ph¼ng bÊt kú (P) ®i
Tõ Bµi to¸n 3 lµ mét bµi to¸n ®¬n gi¶n cã qua träng t©m G cña tø diÖn c¾t SA, SB, SC
trong s¸ch gi¸o khoa, nÕu linh ho¹t mét chót t¹i D, E, F t−¬ng øng. T×m gi¸ trÞ nhá nhÊt
ta sÏ chøng minh ®−îc bµi to¸n khã trªn t¹p cña biÓu thøc:
chÝ To¸n häc vµ Tuæi trÎ sau: 1 1 1
2
+ 2+
5. Bµi to¸n 4. SD SE SF 2
Cho tø diÖn ABCD vµ mét ®iÓm M bÊt kú
thuéc mÆt ph¶ng (BCD). Qua M kÎ c¸c Tõ c¸c vÝ dô trªn ta thÊy nÕu biÕt vËn
®−êng th¼ng MA1 song song AB c¾t mÆt dông linh ho¹t, chÝnh x¸c vµ thªm mét chót
s¸ng t¹o th× t«i tin ch¾c r»ng c¸c b¹n cã thÓ
ph¼ng (ACD) t¹i B1. C¸c ®iÓm C1, A1 ®−îc
gi¶i ®−îc rÊtnhiÒu bµi to¸n khã kh¸c nhau.
x¸c ®Þnh t−¬ng tù. Chøng minh r»ng ®−êng
Chóc c¸c b¹n ngµy cµng häc tèt m«n
th¼ng AM ®i qua träng t©m cña tam gi¸c to¸n!
A1B1C1.

Lêi gi¶i.
Kh«ng cã viÖc g× khã
ChØ sî lßng kh«ng bÒn
§µo nói vµ lÊp biÓn
QuyÕt chÝ ¾t lµm nªn

- Hå ChÝ Minh -

DÔ thÊy M, A, A1, B1, C1 cïng thuéc mét


h×nh hép. Trong ®ã M vµ A ë hai phÝa ®èi

TAÄP SAN TOAÙN HOÏC - 2007 63


TRÖÔØNG THPT CHUYEÂN HOAØNG VAÊN THUÏ - HOØA BÌNH
PhÇn I - S¸ng t¹o to¸n häc.

MÔÛ ROÄNG KHAÙI NIEÄM

TAÂM TÆ CÖÏ CHO TÖÙ DIEÄN


Hoµng an giang
Chuyªn to¸n k01-04
Sv. Líp §iÖn tö 4 – K49, Khoa §iÖn tö ViÔn th«ng - §H B¸ch Khoa Hµ Néi

******************************

T rong h×nh häc ph¼ng, ®Æc biÖt lµ trong Ta cã thÓ kh¸i qu¸t 2 tÝnh chÊt trªn nh− sau :
tam gi¸c, kh¸i niÖm t©m tØ cù vµ mét sè tÝnh Cho tam gi¸c ABC vµ M lµ mét ®iÓm bÊt
chÊt cña nã lµ mét c«ng cô ®Ó gi¶i to¸n rÊt kú n»m trong tam gi¸c. §Æt Sa = SMBC , Sb =
hay, nhÊt lµ nh÷ng bµi to¸n liªn quan tíi viÖc SMCA , Sc = SMAB. Khi ®ã :
tÝnh ®é dµi kho¶ng c¸ch gi÷a mét sè ®iÓm S a .MA + S b .MB + S c .MC = O
®Æc biÖt trong tam gi¸c.
Bµi viÕt nµy sÏ më réng vÊn ®Ò trªn cho
tr−êng hîp tø diÖn. II. Më réng vÊn ®Ò cho tø diÖn.

I. Kh¸i niÖm vµ tÝnh chÊt cña t©m tØ cù 1. §Þnh lý 2.


trong tam gi¸c.
Trong kh«ng gian cho bèn ®iÓm A, B, C,
1. §Þnh lý 1. D kh«ng ®ång ph¼ng vµ c¸c sè thùc x, y, z, t
Cho 3 ®IÓm A, B, C kh«ng th¼ng hµng vµ tho¶ mLn x + y + z + t ≠ 0 . Khi ®ã tån t¹i duy
3 sè thùc x, y, z tho¶ mLn x + y + z ≠ 0 . Khi nhÊt mét ®iÓm M sao cho
®ã tån t¹i duy nhÊt ®iÓm M sao cho : x.MA + y.MB + z.MC + t.MD = O (*).
x.MA + y.MB + z.MC = O
§iÓm M nh− vËy ®−îc gäi lµ t©m tØ cù Chøng minh. Ta cã :
cña 3 ®IÓm A, B, C øng víi bé sè (x, y, z). x.MA + y.MB + z.MC + t.MD = O
  
⇔ x.MA + y.( MA + AB ) +
2. Mét sè tÝnh chÊt.     
+ z.( MA + AC ) + t.( MA + AD ) = O
 
a. Cho tam gi¸c ABC vµ G lµ träng t©m ⇔ ( x + y + z + t ).MA + y.MA +
  
tam gi¸c . Khi ®ã GA + GB + GC = O . + z. AC + t. AD = O
b. Cho tam gi¸c ABC vµ I lµ t©m ®−êng
y. AB + z. AC + t. AD
trßn néi tiÕp tam gi¸c. §Æt AB = c, BC = a, ⇔ AM =
x+ y+ z+t
CA = b. Khi ®ã ta cã a.IA + b.IB + c.IC = O .
⇒ Lu«n tån t¹i M tho¶ mHn (*)

TAÄP SAN TOAÙN HOÏC - 2007 64


TRÖÔØNG THPT CHUYEÂN HOAØNG VAÊN THUÏ - HOØA BÌNH
PhÇn I - S¸ng t¹o to¸n häc.

B©y giê ta chøng minh M lµ duy nhÊt. c. Cho tø diÖn ABCD vµ I lµ t©m mÆt cÇu néi
ThËt vËy, gi¶ sö tån t¹i ®iÓm M’ kh¸c M tiÕp tø diÖn. Khi ®ã :
tho¶ mHn (*), khi ®ã: S a .IA + S b .IB + S c .IC + S d .ID = O
O = x.MA + y.MB + z.MC + t.MD trong ®ã S a , Sb , S c , S d lÇn l−ît lµ diÖn tÝch
   
( )
= − x.M ' A + y.M ' B + z.M ' C + t.M ' D c¸c tam gi¸c BCD, CDA, DAB, ABC.
= ( x + y + z + t ) MM '
III. øng dông ®Ó tÝnh kho¶ng c¸ch gi÷a
V× x + y + z + t ≠ 0 nªn MM ' = 0 c¸c ®iÓm ®Æc biÖt trong tø diÖn.
⇒ M’ ≡ M. VËy tån t¹i duy nhÊt M tho¶ mHn
(*). §iÓm M tho¶ mHn ®Þnh lý 2 ®−îc gäi lµ Ta ký hiÖu G, H, O, I lÇn l−ît lµ träng
t©m tØ cù cña tø ®iÓm A, B, C, D theo bé sè t©m, trùc t©m (nÕu cã), t©m mÆt cÇu ngo¹i
(x, y, z, t). tiÕp vµ néi tiÕp tø diÖn ABCD. §Æt AB = a,
BC = b, CD = c, DA = d, AC = e, BD = f.
2. TÝnh chÊt. • TÝnh OG: Ta cã
T−¬ng tù nh− trong tam gi¸c ta chøng     
minh ®−îc tÝnh mét sè tÝnh chÊt sau: 4OG = OA + OB + OC + OD
⇒ 16OG 2 = OA2 + OB 2 + OC 2 + OD 2 +
 
+2 ∑ OA.OB =
A , B ,C , D

= OA2 + OB 2 + OC 2 + OD 2 +
+2 ∑ (R
a ,b ,c , d , e , f
2
− a2 )

⇒ OG 2 =
= R 2 − ( a 2 + b2 + c2 + d 2 + e2 + f 2 ) .
1
16
B»ng c¸ch t−¬ng tù, ta tÝnh ®−îc c¸c kÕt qu¶
sau:

∑ S a Sb a 2
OI 2 = R 2 − a ,b ,c ,d ,e , f 2
 
 ∑ Sa 
 a ,b , c , d , e , f 
a. Cho tø diÖn ABCD vµ 1 ®iÓm M bÊt kú 1
n»m trong tø diÖn. HG 2 = R 2 − . ∑ a 2
§Æt Va = VMBCD ; Vb = VMCAD ; Vc = VMDAB ; 16 a ,b ,c , d ,e , f
Vd = VMABC. Khi ®ã : 1
OH 2 = 4 R 2 − . ∑ a 2
Va .MA + Vb .MB + Vc .MC + Vd .MD = O 4 a ,b , c , d , e , f
§©y chØ lµ mét øng dông cña t©m tØ cù cho
b. Cho tø diÖn ABCD vµ G lµ träng t©m tø tø diÖn. RÊt mong c¸c b¹n më réng vµ kh¸i
diÖn. Khi ®ã : qu¸t vÊn ®Ò nµy thªm n÷a!
GA + GB + GC + GD = O

HÖ qu¶: Víi M lµ ®iÓm bÊt kú ta lu«n cã:


4 MG = MA + MB + MC + MD

TAÄP SAN TOAÙN HOÏC - 2007 65


TRÖÔØNG THPT CHUYEÂN HOAØNG VAÊN THUÏ - HOØA BÌNH
PhÇn I - S¸ng t¹o to¸n häc.

Phöông phaùp logic meänh ñeà


Ph¹m phóc l©n
Chuyªn To¸n K01 – 04
Sv. Líp T§H3 – K49, Khoa §iÖn, §H B¸ch Khoa Hµ Néi
====================================

L «gic suy luËn lµ d¹ng to¸n ch−a ®−îc MÖnh ®Ò “A vµ B” lµ héi cña 2 mÖnh ®Ò
khai th¸c nhiÒu trong c¸c ®Ò thi häc sinh A vµ mÖnh ®Ò B. Ký hiÖu: A^B.
giái, cã lÏ v× trong nã mang c¶ nguån gèc, MÖnh ®Ò “A hoÆc B” lµ tuyÓn cña 2
nÒn t¶ng cña sù ph¸t triÓn to¸n häc nªn viÖc mÖnh ®Ò A vµ mÖnh ®Ò B. Ký hiÖu: AvB.
ph¸t triÓn nã chØ ®−îc thÓ hiÖn qua nh÷ng MÖnh ®Ò “A kÐo theo B” lµ phÐp kÐo theo
vÊn ®Ò kh¸c. Ph−¬ng ph¸p logic mÖnh ®Ò cã cña 2 mÖnh ®Ò A vµ mÖnh ®Ò B.
thÓ gióp ta dÔ dµng mét sè bµi to¸n phøc t¹p
Ký hiÖu: A→ B.
®ßi hái t− duy nh−ng nã còng ®ßi hái b¹n
b, Mét sè c«ng thøc th−êng sö dông:
ph¶i cã nh÷ng hiÓu biÕt nhÊt ®Þnh vÒ vÊn ®Ò
nµy. Tr−íc hÕt ta cÇn quy ®Þnh:
A : lµ phñ ®Þnh cña mÖnh ®Ò A.
I. Mét sè vÊn ®Ò vÒ ph−¬ng ph¸p logic A=1: BiÓu diÔn cho mÖnh ®Ò A cã gi¸ trÞ
mÖnh ®Ò. ch©n lÝ ®óng.
A = 0 : BiÓu diÔn cho mÖnh ®Ò A cã gi¸
1. Nguyªn t¾c gi¶i to¸n. trÞ ch©n lÝ sai.
Ph−¬ng ph¸p l«gic mÖnh ®Ò sö dông c¸c A = B : BiÓu diÔn cho 2 mÖnh ®Ò A vµ B
phÐp to¸n logic nh−: tuyÓn, héi, kÐo theo, ... cã gi¸ trÞ ch©n lÝ nh− nhau.
®Ó suy ra kÕt qu¶. C¸c c«ng thøc:
A → B = A∨ B
2. Quy tr×nh gi¶i to¸n. A ∧ (B ∨ C) = A ∧ B ∨ A ∧ C
Gåm 3 b−íc: A ∨ B ∧ C = ( A ∨ B) ∧ ( A ∨ C )
a, B−íc 1: Chän c¸c biÕn mÖnh ®Ò thÝch hîp,
biÓu diÔn c¸c mÖnh ®ª cña bµi to¸n qua c¸c A∨ B = A∧ B
biÕn vµ diÔn ®¹t thªm mét sè quan hÖ. A∧ B = A∨ B
b, B−íc 2: Dïng c¸c phÐp suy luËn logic ®Ó
suy ra c¸c gi¸ trÞ cña biÕn. A ⇔ B = A∧ B∨ A∧ B
c, B−íc 3: KÕt luËn c¸c mÖnh ®Ò. A ∧ ( A ∨ B) = A
A ∨ ( A ∧ B) = A
3. Nh¾c l¹i c¸c phÐp to¸n vµ mét sè c«ng
thøc hay sö dông. A∨ B = B ∨ A
a, C¸c phÐp to¸n tuyÓn, héi, kÐo theo. A∧ B = B ∧ A
Cho 2 mÖnh ®Ò A vµ mÖnh ®Ò B. Khi ®ã A ∧ ( B ∧ C ) = ( A ∧ B) ∧ C
mÖnh ®Ò: A ∨ ( B ∨ C ) = ( A ∨ B) ∨ C

TAÄP SAN TOAÙN HOÏC - 2007 66


TRÖÔØNG THPT CHUYEÂN HOAØNG VAÊN THUÏ - HOØA BÌNH
PhÇn I - S¸ng t¹o to¸n häc.

A∨ A = A B×nh kh«ng lµm ®æ mùc.


A∧ A = A Vinh lµm ®æ mùc.
Bµi to¸n trªn cã thÓ gi¶i ®−îc b»ng phÐp
A∨ A =1 suy luËn trùc tiÕp, song ph−¬ng ph¸p nµy
A∧ A = 0 gÆp nhiÒu khã kh¨n khi gi¶i c¸c bµi to¸n cã
sè l−îng mÖnh ®Ò lín nh− bµi to¸n sau.
A= A
II. Mét sè bµi to¸n. Bµi to¸n 2. Bèn b¹n Hïng, Nhµn, Nam,
Bµi to¸n 1. Ba b¹n An, B×nh, Vinh ngåi lµm T©m quyÕt ®Þnh ®i du lÞch ë 4 ®Þa ®iÓm kh¸c
bµi xung quanh mét chiÕc bµn. C¸c b¹n ®L nhau: H¹ Long, §å S¬n, SÇm S¬n, Héi An.
lµm ®æ mùc ra bµn. Khi thÇy gi¸o hái 3 b¹n Hä cïng tho¶ thuËn:
tr¶ lêi nh− sau:
An nãi: Vinh kh«ng lµm ®æ mùc, ®ã lµ do 1, NÕu Hïng kh«ng ®i H¹ Long th× Nam
B×nh. kh«ng ®i §å S¬n.
B×nh nãi: Vinh lµm ®æ mùc, An kh«ng 2, NÕu Nhµn kh«ng ®i H¹ Long vµ Héi An
lµm. th× Hïng ®i H¹ Long.
Vinh nãi: B×nh kh«ng lµm ®æ mùc. 3, NÕu Nam kh«ng ®i Héi An th× Nhµn ®i
Trong 3 b¹n cã 2 b¹n nãi sai cßn 1 b¹n SÇm S¬n.
nãi ®óng. HLy t×m ra ng−êi lµm ®æ mùc. 4, NÕu T©m kh«ng ®i H¹ Long th× Nhµn ®i
Lêi gi¶i. Ký hiÖu: a lµ mÖnh ®Ò “An lµm ®æ H¹ Long.
mùc”, b lµ mÖnh ®Ò “ B×nh lµm ®æ mùc”, c 5, NÕu T©m kh«ng ®i H¹ Long th× Nhµn ®i
lµ mÖnh ®Ò “Vinh lµm ®æ mùc”. Ta cã: H¹ Long.
An nãi: A = c∧b Hái 4 b¹n sÏ ®i du lÞch ë ®©u?
B×nh nãi: B = a ∧ c
Vinh nãi: C = b Lêi gi¶i. Chän c¸c biÕn mÖnh ®Ò nh− sau:
V× 2 trong 3 mÖnh ®Ò trªn lµ ®óng nªn: Hïng ®i H¹ Long: al
T = A∨ B = c ∧b∨ c ∧ a =1 Nhµn ®i H¹ Long: bl
Nhµn ®i Héi An: bh
H = A∨ C = c ∧b ∨ b = c ∨ b =1
Nhµn ®i SÇm S¬n: bs
K = B ∨C = c ∧ a ∨b =1 Nam ®i Héi An: ch
T = K = H =1 Nam ®i §å S¬n: cd
T©m ®i H¹ Long: dl
T ∧ K ∧ H =1 T©m ®i §å S¬n: dd
⇔ (c ∧ b ∧ c ∨ c ∧ b ∧ b ∨ c ∧ c ∧ a ∨ c N¨m tho¶ thuËn cña c¸c b¹n lµ:
∧ a ∧ b ) ∧ (c ∧ a ∨ b ) = 1 M = al → cd ≡ al ∨ cd

⇔ ( c ∧ b ∨ c ∧ a ∧ b ) ∧ ( c ∧ a ∨ b) = 1 ( )
N = bl ∧ bh → al ≡ bl ∨ bh ∨ al
⇔ (c ∧ b ∧ c ∧ a ∨ c ∧ b ∧ b ∨ c ∧ a ∧ b ∧ c H = ch → bs ≡ ch ∨ bs
∧a ∨ c ∧ a ∧ b ∧ b = 1 P = d l → bl ≡ dl ∨ bl
⇔ c ∧ a ∧b =1 Q = d d → bl ≡ d d ∨ bl
c = 1  V× h−íng ®i ®−îc c¶ 4 ng−êi nhÊt trÝ nªn:
  M ∧ N ∧ P ∧ H ∧Q =1
⇔ a = 1
 
b = 1 Ta cã:
VËy: An kh«ng lµm ®æ mùc.

TAÄP SAN TOAÙN HOÏC - 2007 67


TRÖÔØNG THPT CHUYEÂN HOAØNG VAÊN THUÏ - HOØA BÌNH
PhÇn I - S¸ng t¹o to¸n häc.

(
M ∧ N = al ∨ cd ∧ ( bl ∨ bh ∨ al ) = ) Líp 12 Tin cã c¸c b¹n: Hoµ, TuÊn Anh,
Hïng, Minh, Trung.
= ( al ∧ bl ) ∨ ( al ∧ bh ) ∨ ( al ∧ al ) ∨ Hai líp ®−a ra c¸c yªu cÇu nh− sau:

( ) ( ) ( )
1, Líp 12 To¸n kh«ng chÊp nhËn TuÊn Anh
∨ c d ∧ bl ∨ cd ∧ bh ∨ cd ∧ bl vµ Minh cïng ®−îc vµo ®éi tuyÓn.
= ( al ∧ bh ) ∨ al ∨ cd ∧ bl ∨ ( ) 2, Líp 12 Tin yªu cÇu Vò vµ ThiÖn kh«ng
cïng ®−îc tuyÓn.
(
∨ c d ∧ bh ∨ cd ∧ al ) ( ) 3, Líp 12 To¸n yªu cÇu: Trung kh«ng ®−îc
tuyÓn nÕu TuÊn Anh vµ Hoµ cïng ®−îc
(
= al ∨ c d ∧ bl ∨ cd ∧ bh ) ( ) tuyÓn.
4, Líp 12 Tin yªu cÇu: NÕu TuÊn Anh ®−îc
M ∧N ∧H =
tuyÓn th× L©n vµ ThiÖn ph¶i ngåi trªn kh¸n
(
=  a l ∨ c d ∧ bl ∨ c d ∧ bh  ∧
  ) ( ) ®µi.
5, NÕu Trung kh«ng ®−îc tuyÓn th× T©m
∧ ( c h ∧ bs ) còng ph¶i ngåi nhµ.
( ) (
= ( al ∧ch ) ∨( al ∧bs ) ∨ cd ∧bl ∧ch ∨ cd ∧bl ∧bs ) 6, S¬n sÏ tõ giL s©n cá nÕu Trung ®−îc ®¸.
7, NÕu L©n ®−îc ®¸ th× Hïng sÏ nghØ.
= ( a ∧c ) ∨( a ∧b ) ∨( c ∧b ∧c )
l h l s d l h
Sau 15 phót suy nghÜ thÇy Trùc ®L chän
( al ∧ ch ) ∨ ( al ∧ bs ) ∨  ®−îc 6 b¹n vµo ®éi tuyÓn mµ kh«ng lµm c¸c
M ∧N ∧H ∧P= 
 cd ∧ bl ∧ ch dl ∧ bl 
 (  ) b¹n bÊt hoµ.
Hái thÇy Trùc ®L chän ai ?
= ( al ∧ ch ∧ d l ) ∨ ( al ∧ ch ∧ bl ) ∨ §¸p sè: L©n, Vò, T©m, Hoµ, Minh,
Trung.
( al ∧ bs ∧ dl ) ∨ ( al ∧ ch ∧ bl ) ∨
( al ∧ bs ∧ dl ) ∨ ( al ∧ bs ∧ bl ) ∨ Bµi tËp 2. N¨m b¹n Vò, L©n, ThÞnh, S¬n,
(c ∧ b ∧ c ∧ d ) ∨ (c
d l h l d ∧ bl ∧ ch ∧ bl ) TiÕn ®¸ bãng trong nhµ ®a chøc n¨ng mét
trong 5 b¹n ®L ®¸ vì cöa kÝnh. Khi thÇy Phó
= (c ∧ b ∧ c )
d l h
hái c¸c b¹n ®L tr¶ lêi nh− sau.
Vò: “ChØ cã thÓ lµ TiÕn hoÆc S¬n lµm
V× M ∧ N ∧ H ∧ P = 1 vì!”
Nªn cd ∧ bl ∧ ch = 1 TiÕn cLi l¹i: “Em vµ ThÞnh kh«ng lµm vì
kÝnh”.
cd = 1
  S¬n: “C¶ hai b¹n trªn ®Òu nãi sai!”
⇒ bl = 1  L©n: “Kh«ng! mét trong hai b¹n trªn ®L
 c = 1 nãi ®óng!”.
 k  ThÞnh: “L©n ¹, B¹n nãi kh«ng ®óng!”.
VËy Nhµn ®i H¹ Long, Nam ®i Héi An Mét c¸ch t×nh cê, thÇy Phó ®L biÕt ®−îc 3
Theo mÖnh ®Ò 5: T©m kh«ng ®i §å S¬n trong 5 c©u nãi lµ ®óng. Sau 15 phót thÇy
nªn T©m ®i SÇm S¬n, Hïng ®i §å S¬n. Phó ®L t×m ra S¬n ®L ®¸ vì kÝnh. Hái thÇy
®L suy luËn nh− thÕ nµo?
III. Bµi tËp tù luyÖn:

Bµi tËp 1. Thµnh lËp ®éi bãng tr−êng THPT


Chuyªn Hoµng V¨n Thô cã 10 b¹n tham gia
dù tuyÓn. Cô thÓ lµ:
Líp 12 To¸n cã c¸c b¹n: Vò, S¬n, L©n,
T©m, ThiÖn.

TAÄP SAN TOAÙN HOÏC - 2007 68


TRÖÔØNG THPT CHUYEÂN HOAØNG VAÊN THUÏ - HOØA BÌNH
PhÇn I - S¸ng t¹o to¸n häc.

PHEÙP CHIEÁU
VAØ ÖÙNG DUÏNG CUÛA PHEÙP CHIEÁU
NguyÔn L©m TuyÒn X = mÆt ph¼ng, Y = mÆt ph¼ng ®H lo¹i trõ
Chuyªn To¸n K99 – 02 trong ®Þnh nghÜa trªn).
Sv. Líp §iÒu khiÓn Tù §éng 1 - K47
§H B¸ch Khoa Hµ Néi 2. §Þnh nghÜa 2. H×nh chiÕu cña mét
 vÐct¬.
H×nh chiÕu cña mét vÐct¬ AB theo

Trong ch−¬ng tr×nh to¸n phæ th«ng,
ph−¬ng X lªn gi¸ chiÕu Y lµ mét vÐct¬ A ' B ' ,
trong ®ã A’, B’ lÇn l−ît lµ h×nh chiÕu cña
chóng ta ®H ®−îc lµm quen víi mét trong c¸c ®iÓm A vµ B theo ph−¬ng X lªn gi¸ chiÕu
nh÷ng kü thuËt chøng minh trong h×nh häc, Y.
®ã lµ phÐp chiÕu. Tuy nhiªn ®©y lµ ph−¬ng Minh häa cho mét vµi phÐp chiÕu vÐc t¬:
ph¸p mµ c¸c b¹n th−êng Ýt sö dông. Bµi viÕt
nµy xin ®−îc tr×nh bµy vÒ phÐp chiÕu vµ øng
dông cña kü thuËt nµy trong viÖc chøng
minh c¸c ®¼ng thøc vÐc t¬.

I. §Þnh nghÜa.

Cho hai ®Þnh d¹ng h×nh häc X, Y lµ c¸c


®−êng th¼ng hoÆc mÆt ph¼ng. Gi÷a X vµ Y cã
c¸c quan hÖ: Song song (//), trïng ( ≡ ) hoÆc
c¾t nhau ( ∩ ). Ch¼ng h¹n khi X = ®−êng
th¼ng, Y = ®−êng th¼ng vµ nÕu hai ®−êng
th¼ng nµy c¾t nhau t¹i I th× ta viÕt X ∩ Y = I. Ph−¬ng chiÕu lµ ®−êng th¼ng, gi¸ chiÕu lµ
Nh− vËy ký hiÖu hoµn toµn nh− quy −íc cña mÆt ph¼ng
c¸c quan hÖ h×nh häc ®H biÕt.

1. §Þnh nghÜa 1. H×nh chiÕu cña mét ®iÓm.


Cho hai ®Þnh d¹ng h×nh häc X vµ Y kh«ng
cïng lµ mÆt ph¼ng, cã quan hÖ c¾t nhau vµ
mét ®iÓm A. Qua A dùng ®Þnh d¹ng h×nh häc
Z cã cïng tÝnh chÊt vµ song song víi X. Khi
®ã ®iÓm A’ = Z ∩ Y ®−îc gäi lµ h×nh chiÕu
cña ®iÓm A theo ph−¬ng X lªn gi¸ chiÕu Y.
Ch¼ng h¹n, khi X = ®−êng th¼ng, Y =
®−êng th¼ng ta cã kh¸i niÖm vÒ h×nh chiÕu
cña mét ®iÓm theo ph−¬ng ®−êng th¼ng lªn
mét ®−êng th¼ng. Kh«ng cã kh¸i niÖm vÒ
h×nh chiÕu cña mét ®iÓm lªn mét mÆt ph¼ng Ph−¬ng chiÕu lµ mÆt ph¼ng, gi¸ chiÕu lµ
theo ph−¬ng mÆt ph¼ng (øng víi tr−êng hîp ®−êng th¼ng

TAÄP SAN TOAÙN HOÏC - 2007 69


TRÖÔØNG THPT CHUYEÂN HOAØNG VAÊN THUÏ - HOØA BÌNH
PhÇn I - S¸ng t¹o to¸n häc.

II. TÝnh chÊt vµ ®Þnh lý. theo quy −íc th«ng th−êng.
 Ch¼ng h¹n h×nh
chiÕu a ' cña vÐc t¬ a theo ph−¬ng ®−êng
1. TÝnh chÊt 1. th¼ng l lªn mÆt ph¼ng (α ) ®−îc viÕt lµ:
H×nh chiÕu cña mét vÐc t¬ tæng b»ng tæng  
c¸c h×nh chiÕu cña c¸c vÐc t¬ thµnh phÇn. ()
a ' = Ch(lα ) a .
Kh«ng mÊy khã kh¨n ®Ó nhËn ra tÝnh
®óng ®¾n cña tÝnh chÊt nµy. C¸c b¹n cã thÓ
V. Mét sè vÝ dô ¸p dông.
chøng minh trong tr−êng hîp tæng cña hai
vÐc t¬, tõ ®ã suy ra ®−îc cho tr−êng hîp n ≥
1. VÝ dô 1.
2 vÐc t¬.
Cho tam gi¸c ABC vµ I lµ t©m ®−êng trßn
néi tiÕp tam gi¸c ®ã. §Æt a = BC, b = CA, c
2. TÝnh chÊt 2.
= AB. Chøng minh r»ng:
H×nh chiÕu cña mét vÐct¬ b»ng vÐct¬ -    
kh«ng khi vµ chØ khi vÐc t¬ chiÕu ®ã cã a.IA + b.IB + c.IC = 0
ph−¬ng trïng víi ph−¬ng chiÕu.
L−u ý tr−êng hîp vÐc t¬ chiÕu b»ng vÐct¬
- kh«ng, kh¼ng ®Þnh vÉn hoµn toµn ®óng.

3. §Þnh lý chiÕu.
Mét vÐc t¬ cã h×nh chiÕu theo Ýt nhÊt hai
ph−¬ng kh¸c nhau (Hai ph−¬ng cã tån t¹i
quan hÖ c¾t) ®Òu b»ng vÐc t¬ - kh«ng khi vµ
chØ khi ®ã lµ vÐc t¬ - kh«ng.
§Þnh lý nµy suy trùc tiÕp tõ tÝnh chÊt cña    
Lêi gi¶i. §Æt T = a.IA + b.IB + c.IC vµ ký
vÐc t¬ - kh«ng vµ tÝnh chÊt 2 ë trªn phÐp 
chiÕu. hiÖu A ' = AI ∩ BC . Ta cã: ChBC ( )
AI
T =
  
III. C¸c b−íc chøng minh mét ®¼ng thøc = ChBC

(
AI
)
a.IA + ChBC

(
AI


)
b.IB + ChBC( )AI
c.IC
vÐc t¬. = 0 + b. A ' B + c. A ' C
B−íc 1. ChuyÓn ®¼ng thøc vÐc t¬ cÇn   c  
chøng minh thµnh d¹ng ®¼ng thøc cã vÕ ph¶i = b.  A ' B + . A ' C 
lµ vÐc t¬ - kh«ng.  b 
 
B−íc 2. Ký hiÖu vÐc t¬ bªn vÕ tr¸i lµ T = 0 (Theo tÝnh chÊt ®−êng ph©n gi¸c).
 
vµ chän hai ph−¬ng chiÕu kh¸c nhau, sau ®ã T−¬ng tù ChCA ( )
BI

 
T = 0 , do ®ã theo ®Þnh
chøng minh h×nh chiÕu cña vÐc t¬ T theo
c¸c ph−¬ng ®ã (Lªn gi¸ chiÕu t−¬ng øng) lý chiÕu ta cã T = 0 , ®pcm.
®Òu b»ng vÐc t¬ - kh«ng.
B−íc 3. Dùa vµo ®Þnh lý chiÕu më môc 3, 2. VÝ dô 2.
kÕt luËn ®−îc ®¼ng thøc vÐc t¬ cÇn chøng Cho tø diÖn ABCD vµ ®iÓm M bÊt kú n»m
minh. trong tø diÖn ®ã. Ký hiÖu VA , VB , VC , VD
lÇn l−ît lµ thÓ tÝch cña c¸c h×nh tø diÖn nhá
IV. Ký hiÖu. MBCD, MCDA, MDAB vµ MABC. Chøng

Khi chiÕu mét vÐc t¬ a theo ph−¬ng minh r»ng:
     
chiÕu X lªn gi¸ chiÕu Y, ta ®−îc vÐc t¬ a ' . VA .MA + VB .MB + VC .MA + VD .MD = 0
 
()
§Ó cho gän, ta ký hiÖu a ' = ChYX a . Trong
®ã c¸c ®Þnh d¹ng h×nh häc X, Y ®−îc ký hiÖu

TAÄP SAN TOAÙN HOÏC - 2007 70


TRÖÔØNG THPT CHUYEÂN HOAØNG VAÊN THUÏ - HOØA BÌNH
PhÇn I - S¸ng t¹o to¸n häc.

Lêi gi¶i.   


a) §Æt T = HG − 2.GO , ta cÇn chøng
 
minh T = 0 . ThËt vËy, Gi¶ sö
AH ⊥ BC = H ' , AG ∩ BC = M . KÎ
GG ' ⊥ BC nh− trªn h×nh vÏ.
Ta cã OM ⊥ BC , do ®ã:
  
AH
ChBC ( )
T = ChBC

AH
( )
HG - ChBC

AH
( )
2.GO =
= H ' G ' − 2.G ' M .

Lêi gi¶i.    


§Æt T = VA .MA + VB .MB + VC .MA + VD .MD .
Ký hiÖu A ' = AM ∩ mp ( BCD ) .
 
Ta cã: ChAM ( )
( BCD )
T = ChAM (
( BCD )
)
VA .MA +
 
+ ChAM (
( BCD )


)
VA .MA + ChAM (
( BCD )


)
VA .MA =
= VA .MA + (VB + VC + VD ) MA '
  G ' M GM 1
= VA .MA + (V − VA ) MA ' (V lµ thÓ tÝch cña tø Nh−ng dÔ thÊy = = nªn ta
H ' G ' GA 2
diÖn ABCD).    
 
= VA . A ' A + V .MA '
cã ngay ChBC
AH
( )
T = H ' G ' − 2.G ' M = 0 .
 
 V   
= V .  A A ' A + MA ' 
T−¬ng tù nh− vËy: ChCA
BH
( )
T = 0 , tõ ®ã cã
V  ®pcm.
   
= 0 (v× A =
V MA '
lµ tû lÖ thÓ tÝch cña hai tø b) §Æt T = HG − GO , ta cÇn chøng minh
 
V AA ' T = 0 . Ký hiÖu nh− sau: AH ∩ ( BCD ) = H ' ,
diÖn chung ®¸y MBCD vµ ABCD).
  AG ∩ ( BCD ) = GA , GG ' ⊥ ( BCD ) = G ' ,
T−¬ng tù: ChBM ( )
( CDA )
T = 0 , tõ ®ã theo
OO ' ⊥ ( BCD ) = O ' .
®Þnh lý chiÕu ta cã ®pcm.

3. VÝ dô 3.
a) Cho tam gi¸c ABC. Ký hiÖu H, G, O
lÇn lµ trùc t©m, träng t©m vµ t©m ®−êng trßn
ngo¹i tiÕp cña tam gi¸c. Chøng minh r»ng:
 
HG = 2GO
b) Gi¶ sö ABCD cã c¸c ®−êng cao ®ång
quy t¹i mét ®iÓm H (ABCD lµ mét tø diÖn
trùc t©m). Gäi G, O lÇn l−ît lµ träng t©m vµ
t©m mÆt cÇu ngo¹i tiÕp cña tø diÖn. Chøng
minh r»ng G lµ trung ®iÓm cña ®o¹n th¼ng
 
HO, nghÜa lµ HG = GO .

TAÄP SAN TOAÙN HOÏC - 2007 71


TRÖÔØNG THPT CHUYEÂN HOAØNG VAÊN THUÏ - HOØA BÌNH
PhÇn I - S¸ng t¹o to¸n häc.

III. Bµi tËp.


DÔ thÊy H’ lµ trùc t©m, G A lµ träng t©m
vµ O’ lµ t©m ®−êng trßn ngo¹i tiÕp cña tam Bµi 1. Cho tam gi¸c ®Òu ABC t©m O. Gi¶ sö
gi¸c BCD. Thùc hiÖn phÐp chiÕu vu«ng gãc M lµ mét ®iÓm bÊt kú n»m trong tam gi¸c vµ
 D, E, F lÇn l−ît lµ h×nh chiÕu cña M lªn c¸c
vÐc t¬ T lªm mÆt ph¼ng (BCD) ta cã:
   c¹nh BC, CA vµ AB t−¬ng øng. Chøng minh
Ch(AH
BCD )( )
T = H 'G ' − G 'O '    3 
r»ng MD + ME + MF = MO .
2
HLy më réng bµi to¸n cho tr−êng hîp tø
diÖn ®Òu.

Bµi 2. VÏ qua träng t©m G cña tam gi¸c


ABC mét ®−êng th¼ng d. Gäi H, I, K t−¬ng
øng lµ h×nh chiÕu cña A, B, C trªn d. Chøng
minh r»ng trong ba ®o¹n th¼ng AH, BI vµ
CK, cã mét ®o¹n b»ng tæng hai ®o¹n th¼ng
cßn l¹i.

Bµi 3. Cho tam gi¸c ABC vµ mét ®iÓm M bÊt


kú n»m trong tam gi¸c. C¸c ®−êng th¼ng
AM, BM, CM lÇn l−ît c¾t c¸c c¹nh BC, CA
Nh−ng theo tÝnh
chÊt cña träng t©m tø vµ AB t¹i A’, B’, C’. Chøng minh r»ng M lµ
diÖn th× AG = 3.GGA vµ theo kÕt qu¶ cña träng t©m cña tam gi¸c ABC khi vµ chØ khi
 
phÇn a) H ' GA = 2GAO ' , nªn ta cã M lµ träng t©m cña tam gi¸c A’B’C’.
   
H ' G ' = G ' O ' ⇒ Ch(AH ( )
BCD ) T = 0 . Bµi 4. Cho tø diÖn ABCD víi I lµ t©m mÆt
 
Còng nh− vËy, Ch(BH ( )
CDA) T = 0 , nªn theo
cÇu néi tiÕp. Chøng minh r»ng:
    
S A .IA + S B .IB + SC .IA + S D .ID = 0
®Þnh lý chiÕu, ta cã ®pcm.
Trong ®ã S A , S B , SC , S D lµ diÖn tÝch
Trªn ®©y lµ mét sè kh¸i niÖm vµ øng c¸c mÆt ®èi diÖn víi c¸c ®Ønh A, B, C, D cña
dông cña phÐp chiÕu (song song). Nh− c¸c tø diÖn.
b¹n ®H thÊy, phÐp chiÕu t−¬ng ®èi hiÖu qu¶
trong viÖc chøng minh c¸c ®¼ng thøc vÐc t¬. Bµi 5. Gi¶ sö r»ng ®L cã c¸c ®¼ng thøc vÐc
§iÒu quan träng trong phÐp chøng minh nµy t¬ ë c¸c bµi to¸n trªn. HLy thiÕt lËp c¸c
lµ chóng ta cÇn ph¶i lùa chän ph−¬ng chiÕu ®¼ng thøc ®¹i sè míi dùa trªn b×nh ph−¬ng
vµ gi¸ chiÕu sao cho hîp lý th× lêi gi¶i míi cña c¸c ®¼ng thøc vÐc t¬ ®ã.
sö dông phÐp t−¬ng tù vµ ng¾n gän ®−îc.
øng dông phÐp chiÕu ®Ó chøng minh ®¼ng Bµi 6. Gi¶ sö M lµ mét ®iÓm n»m trong tø
thøc vÐc t¬ chØ lµ mét khÝa c¹nh nhá cña diÖn ABCD. C¸c ®−êng th¼ng AM, BM, CM,
phÐp chiÕu, thùc tÕ cßn nhiÒu øng dông quan DM theo thø tù c¾t c¸c mÆt ph¼ng (BCD),
träng kh¸c cña kü thuËt nµy mµ néi dung (CDA), (DAB), (ABC) t¹i A’, B’, C’, D’. MÆt
n»m ngoµi khu«n khæ cña bµi b¸o. C¸c b¹n ph¼ng (α ) qua M, song song víi mÆt ph¼ng
hHy chó ý t×m hiÓu thªm vµ ®Ó kÕt thóc, mêi (BCD) lÇn l−ît c¾t A’B’, A’C’, A’D’ t¹i XYZ.
c¸c b¹n gi¶i c¸c bµi tËp sau. L−u ý lµ mçi Chøng minh r»ng M lµ träng t©m cña tam
mét c¸ch chiÕu kh¸c nhau sÏ cho ta mét lêi gi¸c X
gi¶i kh¸c nhau.

TAÄP SAN TOAÙN HOÏC - 2007 72


TRÖÔØNG THPT CHUYEÂN HOAØNG VAÊN THUÏ - HOØA BÌNH
PhÇn I - S¸ng t¹o to¸n häc.

MOÄT SOÁ BAØI TOAÙN BAÁT ÑAÚNG THÖÙC CHOÏN LOÏC


(**) ⇔ ( x − 1) 2 (2 x + 1) ≥ 0, ∀x > 0 ,
L−u nh− hoµ lu«n ®óng. Thµnh thö
Chuyªn To¸n K02-05 1 2a + 3 3
Sv. Líp §iÒu KhiÓn Tù ®éng, KSTN - K50
∑ 2a 2 − 6a + 9 ≤ ∑ 25 = 5 .
§¹i häc B¸ch Khoa Hµ Néi DÊu b»ng xÈy ra ⇔ a = b = c.

Bµi to¸n t−¬ng tù:


§Ò thi USA – 2003
BÊt ®¼ng thøc (B§T) lµ mét d¹ng to¸n Cho x, y, z > 0. Chøng minh r»ng
kh«ng ®ßi hái nhiÒu kiÕn thøc nh−ng l¹i hay (2 x + y + z )2
xuÊt hiÖn trong c¸c kú thi Häc sinh giái
∑ 2 x2 + ( y + z )2 ≤ 8
To¸n. V× thÕ nã ®−îc kh¸ nhiÒu b¹n chän ®Ó
viÕt trong tËp san nµy. T«i còng xin ®−îc Bµi sè 2. (§Ò thi chän §T tr−êng PTNK TP
®ãng gãp mét bµi viÕt vÒ d¹ng to¸n nµy, qua Hå ChÝ Minh 2004).
®ã muèn giíi thiÖu tíi c¸c b¹n mét sè d¹ng Cho x, y, z tháa mLn hÖ ®iÒu kiÖn
to¸n vÒ B§T th−êng xuÊt hiÖn trong c¸c kú
 x+ y+ z = 4
thi to¸n nh÷ng n¨m gÇn ®©y.  2
x + y + z = 6
2 2

Bµi sè 1. (Japan 1997). Cho a, b, c > 0. T×m gi¸ trÞ lín nhÊt (GTLN) vµ gi¸ trÞ
Chøng minh r»ng: nhá nhÊt (GTNN) cña biÓu thøc
(b + c − a ) 2 (c + a − b ) 2 S = x6 + y 6 + z 6 .
S= + +
(b + c) 2 + a 2 (c + a ) 2 + b 2
(a + b − c) 2 3 Lêi gi¶i. Ta cã xy + yz + zx =
+ ≥ (*) 1
( a + b) 2 + c 2 5 = ( x + y + z ) 2 − ( x 2 + y 2 + z 2 )  = 5
2
Lêi gi¶i. Kh«ng mÊt tÝnh tæng qu¸t ta gi¶ sö §Æt xyz = a.
a + b + c = 3. Khi ®ã ta cã:
(2a − 3) 2  x + y+ = 4
S =∑ 2 vµ 
§iÒu kiÖn cho hÖ  xy + yz + zx = 5 cã
a + (a − 3)2
 xyz = a
(2a − 3) 2 3 
(*) ⇔ ∑ 2 ≥ nghiÖm thùc t−¬ng ®−¬ng víi ®iÒu kiÖn cho
a + (a − 3) 2
5
ph−¬ng tr×nh bËc ba sau cã nghiÖm (cã thÓ lµ
1 3
⇔∑ 2 ≤ nghiÖm béi) t 3 − 4t 2 + 5t − a = 0 .
2a − 6a + 9 5
50
1 2x + 3 DÔ thÊy ®iÒu kiÖn nµy lµ ≤a≤2
NhËn xÐt: ≤ , ∀x > 0 (**) 27
2x − 6x + 9
2
25
ThËt vËy, L¹i cã x 6 + y 6 + z 6 =
= ( x 2 + y 2 + z 2 )3 − 3( x 2 + y 2 )( y 2 + z 2 )( z 2 + x 2 )

TAÄP SAN TOAÙN HOÏC - 2007 73


TRÖÔØNG THPT CHUYEÂN HOAØNG VAÊN THUÏ - HOØA BÌNH
PhÇn I - S¸ng t¹o to¸n häc.

= ( x 2 + y 2 + z 2 )3 − 3( x 2 y 2 + y 2 z 2 + z 2 x 2 )( x 2 + y 2 + z 2 ) + Bµi sè 4. Cho 0 < p ≤ q . Chøng minh r»ng


+3x 2 y 2 z 2 = 3a 2 + 144a − 234 = f (a ) . ap aq
 50 
∑ b p + c p ∑ bq + c q , ∀a, b, c > 0 (*)

Do f '(a ) = 6a + 144 > 0, ∀a ∈  ; 2 
 27 
Nªn Lêi gi¶i.
 50   aq ap 
10438
= f   ≤ f (a ) ≤ f ( 2 ) = 66 (*) ⇔ ∑  q − ≥0
b +c b +c
q p p
243  27  
VËy GTLN cña biÓu thøc S lµ 66 khi
(x,y,z) = (1,1,2). GTNN cña S lµ
10438
khi a q (b p + c p ) − a p (b q + c q )
243 ⇔∑ ≥0
5 5 2
(b q + c q )(b p + c p )
(x,y,z) = ( , , ).
3 3 3
a pb p (a q − p − b q − p ) + a p c p (a q − p − c q − p )
⇔∑ ≥0
Bµi to¸n t−¬ng tù. (b q + c q )(b p + c p )
ViÖt Nam – B – 2004.
Cho x, y, z > 0 tháa mLn ®iÒu kiÖn x + y ⇔ ∑ {a p b p (a q − p − b q − p ) ×
+ z = 4 vµ xyz = 2. T×m GTLN vµ GTNN
cña P = x 4 + y 4 + z 4 .  1 1 
× p − p q 
≥0
 (b + c )(b + c ) (a + c )(a + c )  
p q q p q

Bµi sè 3.
a) Cho tam gi¸c ABC tï. HLy t×m sè thùc (**)
k bÐ nhÊt sao cho DÔ thÊy (**) lu«n ®óng do
S = sin 3 A + sin 3 B + sin 3 C < k lµ ®óng a b (a q − p − bq − p ) ×
p p

víi mäi tam gi¸c tï ABC.


 1 1 
b) §Ò bµi nh− trªn nh−ng víi ®iÒu × p − p q 
≥0
 (b + c )(b + c ) (a + c )(a + c ) 
p q q p q
kiÖn ABC lµ tam gi¸c bÊt kú.
víi mäi a, b, c > 0 .
Lêi gi¶i. Cïng víi c¸c biÓu thøc ho¸n vÞ kh¸c, ta
π π cã ®pcm.
a) Cho A→ ,B → , C → 0 thÊy
2 2
S →2. §Ó cã Bµi sè 5.
S = sin A + sin B + sin C < k víi mäi tam
3 3 3 Cho a, b, c > 0. Chøng minh r»ng
2 2 2
gi¸c ABC th× k ≥ 2 .  a   b   c  3
Kh«ng mÊt tÝnh tæng qu¸t, ta gi¶ sö   +  +  ≥
 a+b b+c c+a  4
π
A> >B≥C. Khi ®ã
2 a b c
Lêi gi¶i. §Æt x = ,y = ,z = ta cã
sin 2 B + sin 2 C < sin 2 A b c a
vµ sin A > sin B ≥ sin C . VËy nªn x, y, z > 0 vµ xyz = 1 . Ta cÇn chøng minh
S = sin 3 A + sin 3 B + sin 3 C < 1 3
< sin 3 A + sin A ( sin 2 B + sin 2 C ) < 2sin 3 A < 2 ∑ ( x + 1)2 ≥ 4
VËy k = 2 lµ tÊt c¶ c¸c gi¸ trÞ cÇn t×m.
b) Lêi gi¶i xin ®−îc dµnh cho b¹n ®äc
xem nh− mét bµi tËp.

TAÄP SAN TOAÙN HOÏC - 2007 74


TRÖÔØNG THPT CHUYEÂN HOAØNG VAÊN THUÏ - HOØA BÌNH
PhÇn I - S¸ng t¹o to¸n häc.

NhËn xÐt. Víi x > 0 th×


x
+
1

3 SÖÛ DUÏNG ÑAÚNG THÖÙC
x + 1 ( x + 1) 2
4
(*)
ThËt vËy (*) ⇔ ( x − 1) ≥ 0 lu«n ®óng.
2 ÑEÅ CHÖÙNG MINH BAÁT
Gi¶ sö x ≥ y ≥ z . Ta cã xy ≥ 1 vµ z ≤ 1
1 1 2
ÑAÚNG THÖÙC
L¹i cã 2 + 2 − =
x + 1 y + 1 xy + 1
Vò viÖt dòng
( x − y ) 2 ( xy − 1)
= ≥0 Líp 12t, Chuyªn to¸n k04 – 07
(1 + x 2 )(1 + y 2 )(1 + xy ) THPT Chuyªn Hoµng V¨n thô
VËy nªn
1 1 1 1  1
∑ ( x + 1)2 ≥ 2  x 2 + 1 + y 2 + 1  + ( z + 1)2 ≥
 
BÊt ®¼ng thøc lµ d¹ng to¸n cã nhiÒu c¸ch
1 1 z 1 3 kh¸c nhau ®Ó chøng minh. Chóng ta th−êng
≥ + ≥ + ≥
1 + xy ( z + 1) 2
z + 1 ( z + 1) 2
4 thÊy c¸c bÊt ®¼ng thøc cã ®iÒu kiÖn ®i kÌm
VËy ta cã ®pcm nh− a + b + c = 1, abc = 1, … Vµ trong
nhiÒu tr−êng hîp, chóng ta ph¶i tù t¹o ra
Bµi to¸n t−¬ng tù. nh÷ng ®¼ng thøc ®Ó chøng minh bÊt ®¼ng
§Ò chän §TVN n¨m 2005. thøc. Sau ®©y lµ mét sè vÝ dô cô thÓ.
Cho a, b, c > 0. Chøng minh r»ng
3
 a   b   c  3
3 3 VÝ dô 1.
  +  +  ≥ Cho a, b, c lµ 3 sè ®«i mét kh¸c nhau.
 a+b b+c c+a  8 Chøng minh r»ng:
Korean 2000.
a 3 − b3 b3 − c3 c3 − a3 9
Cho a, b, c > 0. Chøng minh r»ng + + ≥
( a − b) (b − c ) ( c − a ) 4
3 3 3
3 3 3
 a 2  b 2  c 2 3 2
1≤   +  +  ≤ Lêi gi¶i. §©y lµ bÊt ®¼ng thøc kh¸ quen
 a+b b+c c+a  2 thuéc,viÖc chøng minh ®i tõ ®¼ng thøc sau:
Bµi sè 6. (Dùa theo mét bµi IMO)  a + b  b + c   c + a 
− 1 − 1 −  1 − =
Cho ( an ) lµ dLy c¸c sè nguyªn d−¬ng  a − b  b − c   c − a 
ph©n biÖt. Hái ( an ) ph¶i tháa mLn ®iÒu kiÖn  a + b  b + c  c + a 
= 1 + 1 + 1 + 

 ak   a − b  b − c  c − a 
g× ®Ó chuçi ∑  k
2 

cã gi¸ trÞ h÷u h¹n?
a+b b+c b+c c+a
k =1 ⇔ + +
Cã thÓ so s¸nh chuçi trªn víi mét chuçi a −b b−c b−c c−a
®iÒu hßa dÉn ®Õn c©u tr¶ lêi lµ kh«ng tån t¹i c+a a+b
+ = −1
dHy sè nµo nh− vËy. ViÖc chøng minh t−êng c −a a −b
minh xin ®−îc dµnh cho b¹n ®äc coi nh− bµi a+b b+c c+a
tËp tr−íc khi kÕt thóc bµi viÕt nµy. §Æt x = ,y= ,z =
Trªn ®©y lµ mét sè bµi to¸n vÒ B§T ®H a −b b−c c−a
tõng xuÊt hiÖn trong mét sè kú thi To¸n gÇn th× xy + yz + zx = −1 . MÆt kh¸c, ta lu«n cã
®©y. Chóc c¸c b¹n t×m thÊy ®−îc nhiÒu ®iÒu (x + y + z) ≥0
2

bæ Ých.
⇔ x 2 + y 2 + z 2 ≥ −2 ( xy + yz + zx )

TAÄP SAN TOAÙN HOÏC - 2007 75


TRÖÔØNG THPT CHUYEÂN HOAØNG VAÊN THUÏ - HOØA BÌNH
PhÇn I - S¸ng t¹o to¸n häc.

 4 y + z + x   4z + x + y   ≥
2 2

+  +  
 a+b b+c c+a
2 2 2
 3   3  
⇒  +  +  ≥2
 a −b   b−c   c −a    4 x + y + z   4 y + z + x   4 z + x + y 
2

≥ a   + b  + c 
 a+b b+c
2 2

⇔  +1+   +   3   3   3 
 a−b  b−c  ⇒ ®pcm.
c+a
2

+1 +   +1 ≥ 5 VÝ dô 3.
c−a Chøng minh r»ng trong tam gi¸c ABC ta
a 2 + b2 b2 + c 2 c 2 + a 2 5 cã:
⇔ + + ≥ (1)
( a − b ) (b − c ) (c − a ) 2
2 2 2
16
a 4 + b 4 + c 4 ≥ ma mb mc ( ma + mb + mc )
L¹i cã 9
Lêi gi¶i. Víi mäi a, b, c ta lu«n cã ®¼ng
 a+b b+c c+a
2 2 2

  −1 +   −1 +   − 1 ≥ −1 thøc sau:
 a −b   b−c  c−a  2 2
 2b 2 + 2c 2 − a 2   2c 2 + 2a 2 − b 2 

ab
+
bc
+
ca 1
≥ − (2)   +  +
( a − b) (b − c ) (c − a )
2 2 2
4  3   3 
2
Céng tõng vÕ (1) vµ (2) ta cã ®pcm.  2a 2 + 2b 2 − c 2 
+  = a +b +c
4 4 4

 3 
VÝ dô 2.
Chøng minh r»ng víi mäi sè thùc a, b,c, 2b 2 + 2c 2 − a 2
L¹i cã ma2 = ,
x, y, z th× 4
a ( y + z) + b ( z + x) + c ( x + y) + mb2 =
2 a 2 + 2c 2 − b 2
,
4
+ (a 2
+ b2 + c2 ) ( y + z ) + ( z + x ) + ( x + y )  ≥

2 2 2
 2a 2 + 2b 2 − c 2
mc2 = nªn
4

4
3
( a + b + c )( x + y + z )
9
( ma + mb4 + mc4 ) = a 4 + b4 + c 4
16 4

Lêi gi¶i. BÊt ®¼ng thøc ®H cho t−¬ng ®−¬ng Nh−ng hiÓn nhiªn lµ
víi: ma4 + mb4 + mc4 ≥ ma mb mc ( ma + mb + mc )
(a 2
+ b 2 + c 2 ) ( y + z ) + ( z + x ) + ( x + y )  ≥

2 2 2

Tõ ®ã cã ®pcm.

VÝ dô 4.
 4 x + y + z   4 y + x + z   4z + x + y  Gäi r lµ b¸n kÝnh ®−êng trßn néi tiÕp tam
≥ a  + b  + c 
 3   3   3  gi¸c ABC. Chøng minh t»ng:
ChØ cÇn nhËn thÊy r»ng: 1 1 1 1
2
+ 2+ 2≤ 2
 4x + y + z   4 y + z + x   4z + x + y 
2 2 2 a b c 4r
   +  +  = Lêi gi¶i. Trong tam gi¸c ABC ta cã ®¼ng
 3   3   3  1 1
= ( x + y ) + ( y + z ) + ( z + x ) th× bµi to¸n sÏ
2 2 2 thøc sau: = +
4r 2
( b + c − a )( c + a − b )
trë nªn ®¬n gi¶n. ThËt vËy ¸p dông bÊt ®¼ng 1 1
thøc Bunhiacopski ta cã: + +
 4 x + y + z 
2 ( c + a − b )( a + b − c ) ( a + b − c )( b + c − a )
(a 2
+ b 2 + c 2 )   +
 3 

TAÄP SAN TOAÙN HOÏC - 2007 76


TRÖÔØNG THPT CHUYEÂN HOAØNG VAÊN THUÏ - HOØA BÌNH
PhÇn I - S¸ng t¹o to¸n häc.

Ta cã 6)
1 1 1 a−b b−c c−a a −b b−c c−a
= ≥ 2 ⋅ ⋅ = + +
( b + c − a )( c + a − b ) c2 − ( a − b )
2
c c a b c a b

1 1 1 7) Trong tø gi¸c ABCD ta cã:


= ≥
( c + a − b )( a + b − c ) a − (b − c)
2 2
a2 B+D
S = ( p−a)( p−b)( p−c)( p−d) −abcd cos2
1 1 1 2
= ≥
( a + b − c )( b + c − a ) b2 − ( c − a )
2
b2 trong ®ã p lµ n÷a chu vi cßn a, b, c, d lµ ®é
dµi c¸c c¹nh cña tø gi¸c.
Céng tõng vÕ bÊt ®¼ng thøc trªn ⇒ ®pcm.
8) Cho tam diÖn O.ABC vu«ng t¹i O, khi ®ã:
Sau ®©y lµ mét sè ®¼ng thøc (b¹n ®äc tù
chøng minh), hy väng qua ®ã c¸c b¹n cã thÓ
2
S ABC = SOAB
2
+ SOAC
2
+ SOBC
2
.
s¸ng t¹o ra nhiÒu bÊt ®¼ng thøc hay vµ ®Ñp
h¬n. 9) §Æt S = a1 + a2 + ... + an ta lu«n cã:
a−b b−c c−a
 2 S − na1   2 S − na2 
2 2
1) ⋅ ⋅ =
a + b + 2c b + c + 2a c + a + 2b   +  + ...
 n   n 
a−b b−c c−a
= + +  2 S − nan 
2
a + b + 2c b + c + 2a c + a + 2b ... +   = a1 + a2 + ... + an , ∀n ∈ N
2 2 2 *

 n 
a2 b2
2) + + 10)
( a − b )( a − c ) ( b − a )( b − c )
y + z + 2x x + y + 2z x + y + 2z z + x + 2 y
c2 ⋅ + ⋅ +
+ =1 z−y y−x y−x x−z
( c − a )( c − b ) z + x + 2 y y + z + 2x
+ ⋅ = −1
x−z z−y
(1 + a )(1 + b) (1 + b)(1 + c)
3) + +
(c − a )(c − b) (a − b)(a − c) 11)
(1 + c)(1 + a ) c a a b b c
+ = −1 ⋅ + ⋅ + ⋅ = −1
(b − c)(b − a ) a −b b−c b−c c − a c − a a −b

4) ( z + y − x ) + ( x + z − y )2 + ( x + y − z ) =
2 2

Chó ý lµ trong c¸c ®¼ng thøc nãi trªn th×


 5x − z − y   5 y − x − z   5z − x − y 
2 2 2

= ®iÒu kiÖn lµ mÉu sè ë c¸c ph©n thøc lµ kh¸c


 +  +  kh«ng.
 3   3   3 

 x + y + z −t   y + z +t − x 
2 2

5)   +  +
 2   2 
 z+t + x− y  t + x+ y− z 
2 2

+  +  =
 2   2 
= x2 + y2 + z 2 + t 2

TAÄP SAN TOAÙN HOÏC - 2007 77


TRÖÔØNG THPT CHUYEÂN HOAØNG VAÊN THUÏ - HOØA BÌNH
PhÇn I - S¸ng t¹o to¸n häc.

l−îng cùc tiÓu” trong lý thuyÕt vËt lý. §Ó


Tieáp caän Toaùn c©n b»ng n¨ng l−îng c¬ thÓ víi m«i tr−êng
khi trêi rÐt th× chó chim cÇn ph¶i trao ®æi
nhiÖt (cô thÓ lµ cung cÊp n¨ng l−îng) nh»m

baèng Vaät Lyù chèng l¹i t¸c ®éng tõ phÝa m«i tr−êng.
Nh−ng ®Ó møc “tæn thÊt” n¨ng l−îng lµ Ýt
nhÊt, chó chim ph¶i “xï l«ng” ®Ó h¹n chÕ
dÉn nhiÖt. Viªn bi chuyÓn ®éng xuèng d−íi
NguyÔn L©m TuyÒn ®Ó cã ®−îc møc thÕ n¨ng cùc tiÓu. Giät
Chuyªn To¸n K99 – 02 Anilin cã d¹ng h×nh cÇu ®Ó cã ®−îc møc
Sv. Líp §iÒu khiÓn Tù ®éng 1 K47 n¨ng l−îng tù do (lµ thÕ n¨ng phô tiÒm tµng
§H B¸ch Khoa Hµ Néi cña c¸c ph©n tö líp bÒ mÆt) ®¹t cùc tiÓu. V×
víi mét khèi chÊt cã thÓ tÝch nhÊt ®Þnh th×
h×nh cÇu lµ h×nh cã diÖn tÝch bÒ mÆt nhá
Ng−êi ta th−êng nãi “VËn dông To¸n vµo nhÊt!
VËt lý” nh−ng Ýt khi nãi vËn dông theo chiÒu
h−íng ng−îc l¹i, cã ch¨ng chØ lµ tõ vÊn ®Ò Tr−êng hîp ®Æc biÖt vµ ®iÓn h×nh cña
VËt lý cô thÓ mµ ®Æt ra nhiÖm vô nghiªn cøu nguyªn lý vÒ “møc n¨ng l−îng cùc tiÓu” lµ
míi cho To¸n häc. Bµi b¸o nµy xin ®−îc giíi nguyªn lý “thÕ n¨ng cùc tiÓu” trong tr−êng
thiÖu víi b¹n ®äc mét h−íng vËn dông lùc thÕ. Tr−êng lùc thÕ lµ tr−êng mµ ë ®ã
ng−îc l¹i, ®ã lµ c¸ch tiÕp cËn to¸n häc b»ng c«ng dÞch chuyÓn mét vËt kh«ng phô thuéc
vËt lý. vµo d¹ng ®−êng dÞch chuyÓn mµ chØ phô
thuéc vµo ®iÓm ®Çu vµ ®iÓm cuèi cña dÞch
Trong to¸n häc Phæ th«ng, ai còng biÕt chuyÓn, vÝ dô tr−êng träng lùc cña tr¸i ®Êt lµ
tíi bµi to¸n næi tiÕng cña Toricelli sau: “Cho mét tr−êng thÕ. Néi dung cña nguyªn lý nµy
tam gi¸c nhän ABC, t×m trong tam gi¸c mét lµ: “Mät vËt/hÖ vËt trong tù nhiªn ®Òu cã
®iÓm M sao cho tæng MA + MB + MC ®¹t xu h−íng tù dÞch chuyÓn vÒ tr¹ng th¸i cã
gi¸ trÞ bÐ nhÊt”. Ta còng biÕt r»ng ®iÓm M møc thÕ n¨ng cùc tiÓu”. V× t¹i ®ã ®éng
tháa mHn bµi to¸n lµ ®iÓm n»m trong tam n¨ng kh«ng thÓ t¨ng ®−îc n÷a vµ ban ®Çu
gi¸c vµ nh×n ba c¹nh AB, BC, CA cña tam nÕu vËt ®øng yªn th× sÏ ®øng yªn mHi mHi.
gi¸c ®ã d−íi mét gãc 1200. §iÓm M ®ã cßn
®−îc gäi lµ ®iÓm Toricelli. B©y giê, chóng ta sÏ gi¶i bµi to¸n
Toricelli cã ¸p dông nguyªn lý thÕ n¨ng cùc
Bµi to¸n trªn cã mét lêi gi¶i rÊt ®Ñp b»ng tiÓu.
phÐp quay. Néi dung lêi gi¶i còng nh− phÇn
më réng cña bµi to¸n nµy còng ®H ®−îc ®Ò Chän mÆt ®Êt ph¼ng lµ ph−¬ng lµm mèc
cËp ®Õn trong nhiÒu tµi liÖu, ®Æc biÖt lµ t¹p tÝnh thÕ n¨ng. H−íng d−¬ng lµ h−íng lªn
chÝ THTT, ë ®©y t¸c gi¶ bµi b¸o xin kh«ng trªn (phÝa ra xa mÆt ®Êt). Trong kh«ng gian
nªu l¹i. ta dùng tam gi¸c nhän ABC cã ph−¬ng
(ABC) song song víi ph−¬ng mÆt ®Êt. LÊy
Chóng ta hHy ®Ó ý mét sè hiÖn t−îng: T¹i nh÷ng ®o¹n d©y treo kh«ng khèi l−îng,
sao mét chó chim l¹i xï l«ng mçi khi trêi kh«ng giHn, cã chiÒu dµi l1 = l2 = l3 = l vµ
rÐt? Mét viªn bi t¹i sao ®Æt trªn ®Ønh dèc l¹i nh÷ng qu¶ nÆng cã khèi l−îng ®¬n vÞ. Nèi
tù ®éng l¨n xuèng d−íi? Hay t¹i sao mét c¸c ®o¹n d©y l¹i mét ®iÓm chung M, ®Çu cßn
giät Anilin khi nhá vµo trong n−íc l¹i cã l¹i nèi víi c¸c qu¶ nÆng vµ v¾t chóng qua
h×nh d¹ng mét khèi cÇu? … c¸c ®Ønh cña tam gi¸c ABC nh− trong h×nh
Cã thÓ gi¶i thÝch mét c¸ch ®Þnh tÝnh nh− vÏ 1.
sau th«ng qua nguyªn lý vÒ “møc n¨ng Gi¶ thiÕt c¸c sîi d©y dÞch chuyÓn kh«ng
ma s¸t t¹i c¸c ®Ønh vµ gi¶ thiÕt m«i tr−êng

TAÄP SAN TOAÙN HOÏC - 2007 78


TRÖÔØNG THPT CHUYEÂN HOAØNG VAÊN THUÏ - HOØA BÌNH
PhÇn I - S¸ng t¹o to¸n häc.
     
thùc nghiÖm lµ lý t−ëng. Râ rµng hÖ sÏ tù Ta cã P1 + P2 + P3 = 0 vµ v× P1 = P2 =
®éng chuyÓn dÞch theo møc thÕ n¨ng gi¶m 
dÇn vµ sÏ dõng l¹i t¹i tr¹ng th¸i mµ hÖ ®H ®¹t P3 = mg ⇒ ∆A1 B1C1 ®Òu ⇒ ∠AMB =
tíi møc thÕ n¨ng cùc tiÓu, vÞ trÝ c©n b»ng
bÒn. ∠BMC = ∠CMA = 1200 (*). M chÝnh lµ
®iÓm Toricelri cÇn t×m. V× ∆ABC nhän nªn
vÞ trÝ cña M nh− trªn tån t¹i vµ duy nhÊt.
VÊn ®Ò sÏ kh¸c ®i nÕu ∆ABC cã mét
gãc lín h¬n 1200, gi¶ sö lµ gãc A. Khi ®ã sÏ
kh«ng tån t¹i ®iÓm M tháa mHn (*) vµ hiÓn
nhiªn lµ, bµi to¸n ch−a ®−îc gi¶i quyÕt.
Nh−ng chóng ta chó ý hai sù kiÖn sau:

Sù kiÖn 1: Khi ®iÓm M trong tam gi¸c vµ


nh×n hai c¹nh AB, AC c¸c gãc b»ng nhau
( ∠AMB = ∠AMC ) th× ®iÓm M cã xu h−íng
chuyÓn ®éng vÒ phÝa ®iÓm A.

Ký hiÖu c¸c kho¶ng c¸ch, chiÒu dµi còng


nh− c¸c lùc t¸c dông nh− trªn h×nh vÏ . Theo
®ã th× thÕ n¨ng t¹i vÞ trÝ c©n b»ng lµ:
Wt = (ha + hb + hc ).mg ®¹t Min
⇔ (ha + hb + hc ) ®¹t Min
⇔ ( xa + xb + xc ) ®¹t Min (v× xa + ha = Kh«ng mÊy khã kh¨n ®Ó nh×n thÊy tÝnh

xb + hb = xc + hc = h = const) ®óng ®¾n cña sù kiÖn nµy v× hîp lùc P khi
®ã cã h−íng vÒ phÝa ®iÓm A.
⇔ ( MA + MB + MC ) ®¹t Min (v×
MA + xa = MB + xb = MC + xc = l = const) Sù kiÖn 2: Khi ®iÓm M trong tam gi¸c vµ
§iÓm M ë vÞ trÝ c©n b»ng (®øng yªn), do nh×n hai c¹nh AB, AC c¸c gãc kh«ng b»ng

®ã häp lùc P t¸c dông lªn nót M ph¶i b»ng nhau (ch¼ng h¹n ∠AMB > ∠AMC ) th× ®iÓm
 M cã xu h−íng dÞch chuyÓn vÒ phÝa c¹nh cã
0 . C¸c lùc t¸c dông lªn nót M cã modul (®é
gãc nh×n nhá. Cô thÓ, khi ∠AMB > ∠AMC
lín) chÝnh b»ng träng lùc c¸c qu¶ nÆng (m.g) 
vµ cã chiÒu h−íng tõ M ra c¸c ®Ønh t−¬ng th× hîp lùc P cã h−íng vÒ phÝa c¹nh AC.
øng (H.2).

Tõ 2 sù kiÖn trªn, ta ®i ®Õn kÕt luËn: ThÕ


n¨ng cña hÖ tù do gi¶m dÇn (®iÒu nµy t−¬ng

TAÄP SAN TOAÙN HOÏC - 2007 79


TRÖÔØNG THPT CHUYEÂN HOAØNG VAÊN THUÏ - HOØA BÌNH
PhÇn I - S¸ng t¹o to¸n häc.

øng víi tæng MA + MB + MC nhá dÇn) øng C©n b»ng hÖ sè … (tiÕp trang 34 )
víi vÞ trÝ cña ®iÓm M chuyÓn ®éng theo
®−êng zic z¾c vµ cã xu h−íng tiÕn vÒ phÝa
Bµi tËp
®iÓm A. L−u ý r»ng ®iÓm M ≡ A còng vÉn
tháa mHn ®iÒu kiÖn cña ®iÓm M trong sù
Bµi tËp 1. T×m gi¸ trÞ nhá nhÊt cña biÓu thøc
kiÖn 1. VËy ®iÓm A chÝnh lµ ®iÓm cÇn t×m.
Bµi to¸n ®H ®−îc gi¶i quyÕt. x1000 y1000 z1000 t1000
+ 99 + 99 + 99
Tõ c¸ch tiÕp cËn vÊn ®Ò nh− trªn, t«i xin y 99 z t x
®Ò xuÊt mét sè bµi to¸n sau: trong ®ã x, y, z, t lµ c¸c sè thùc d−¬ng tháa
mHn x 2 + y 2 + z 2 + t 2 = 1
VÊn ®Ò 1. Trong tam gi¸c ABC, t×m tËp hîp
Bµi tËp 2. Gäi x lµ sè lín nhÊt trong ba sè
tÊt c¶ c¸c ®iÓm M nh×n 2 c¹nh AB, AC cña
d−¬ng x, y, z. T×m gi¸ trÞ nhá nhÊt cña biÓu
tam gi¸c ®ã c¸c gãc b»ng nhau.
x y z
thøc + 1 + + 3 1 + .
VÊn ®Ò 2. Cho tam gi¸c ABC, t×m trong tam y z x
gi¸c mét ®iÓm M sao cho tæng xMA +y MB Bµi tËp 3. Cho c¸c sè thùc d−¬ng a, b, c, d
+ zMC ®¹t gi¸ trÞ bÐ nhÊt, trong ®ã x, y, z lµ tháa mHn ®iÒu kiÖn:
c¸c sè thùc d−¬ng cho tr−íc.  12 6 4 15 6 32 
Max  + + ; + +  ≤ 32
VÊn ®Ò 3. Cho ®a gi¸c låi A1A2 … An. T×m  ab bc ca bc cd db 
®iÓm trªn h×nh ®a gi¸c ®ã ®iÓm M sao cho T×m gi¸ trÞ nhá nhÊt cña biÓu thøc
13 22 7
tæng MA1 + MA2 + … + MAn ®¹t gi¸ trÞ bÐ P ( a , b, c , d ) = 5a + b + c + a .
nhÊt. Chøng minh r»ng cã duy nhÊt mét 3 3 4
®iÓm M tháa mHn ®iÒu kiÖn ®ã. (§S: MinP ( a, b, c, d ) = 18 ®¹t ®−îc khi
1 3 3
VÊn ®Ò 2 còng kh«ng ph¶i lµ míi l¹, nã a = , b = ,c = ,d = 2)
còng ®H ®−îc ®Ò cËp vµ gi¶i quyÕt trong mét 2 2 4
sè tr−êng hîp trªn t¹p chÝ THTT (n¨m 1994) Bµi tËp 4. Cho c¸c sè thùc d−¬ng ai , bi ( i =
cña thÇy NguyÔn Minh Hµ. Mêi c¸c b¹n xem 1, 2, 3 ) vµ x1 , x2 , x3 , x4 tháa mHn c¸c ®iÒu
xÐt c¸c bµi to¸n trªn theo c¸ch tiÕp cËn b»ng
vËt lý vµ b»ng thuÇn tóy to¸n häc. kiÖn a1 x1 + a2 x2 + a3 x3 ≤ ( a1 + a2 + a3 ) x1 x2 x3
vµ b1 x2 + b2 x3 + b3 x4 ≤ ( b1 + b2 + b3 ) x2 x3 x4 .
Nh− vËy ®Êy c¸c b¹n ¹, mèi liªn hÖ gi÷a
Chøng minh r»ng khi ®ã ta cã bÊt ®¼ng
To¸n häc vµ c¸c m«n häc kh¸c còng thËt thó
vÞ! thøc: ( a2 + a3 ) x1 + ( a1 + a3 + b2 + b3 ) x2 +
+ ( a1 + a2 + b1 + b3 ) x3 + ( b1 + b4 ) x4 ≥
≥ 2 ( a1 + a2 + a3 + b1 + b2 + b3 )
Vµ ®¼ng thøc xÈy ra khi vµ chØ khi:
x1 = x2 = x3 = x4 = 1 ./.

Kh«ng ai t¾m hai lÇn trªn mét dßng s«ng Keep it simple: As simple as possible, but
no simpler
TriÕt häc - Albert Einstein -

TAÄP SAN TOAÙN HOÏC - 2007 80


TRÖÔØNG THPT CHUYEÂN HOAØNG VAÊN THUÏ - HOØA BÌNH
PhÇn I - S¸ng t¹o to¸n häc.

Baát ñaúng thöùc schur


Cho a, b, c lµ c¸c sè thùc d−¬ng tháa
mLn abc = 1. Chøng minh r»ng:

vaø öùng duïng 



1 
b 
1 
 a −1 +   b −1+   c −1 +  ≤ 1
c 
1
1
a
Lêi gi¶i. §Æt x = a, y = 1, z = = ac . Khi ®ã
b
Tr−¬ng quèc h−ng x y z
a = , b = , c = . Ta cã B§T ®H cho
Chuyªn to¸n k04 – 07 y z x
Líp 12T, THPT Chuyªn Hoµng V¨n Thô t−¬ng ®−¬ng víi
( x − y + z ) ( y − z + x) ( z − x + y )
⋅ ⋅ ≤1
y z x
1. BÊt ®¼ng thøc Schur. §©y chÝnh lµ B§T b) khi r = 1 cña B§T
Cho c¸c sè thùc x, y, z ≥ 0; r > 0 ta cã Schur. VËy ta cã ®pcm. DÊu ®¼ng thøc xÈy
bÊt ®¼ng thøc (B§T) sau ra khi a = b = c = 1.
x r ( x − y )( x − z ) + y r ( y − z )( y − x) +
+ z r ( z − x)( z − y ) ≥ 0 . 3. VÝ dô 2.
Cho a, b, c lµ c¸c sè thùc d−¬ng tháa
DÊu ®¼ng thøc xÈy ra khi x = y = z = 1
mLn a + b + c = 1. Chøng minh r»ng:
hoÆc hai trong ba sè x, y, z b»ng nhau vµ sè
cßn l¹i b»ng 0. 5 ( a 2 + b 2 + c 2 ) ≤ 6 ( a 3 + b3 + c 3 ) + 1(*)
Chøng minh. Kh«ng mÊt tÝnh tæng qu¸t gi¶ Lêi gi¶i. B§T Schur cã hai vÕ lµ ®ång bËc.
sö x ≥ y ≥ z . Khi ®ã NÕu muèn ¸p dông th× ta cÇn ph¶i chuyÓn
x r ( x − y )( x − z ) ≥ y r ( x − y )( y − z ) , B§T(*) cho hai vÕ cã cïng bËc. Cô thÓ ta
lµm nh− sau:
l¹i cã z r ( z − x)( z − y ) ≥ 0 nªn ta cã ®pcm.
(*) ⇔ 5 ( a 2 + b 2 + c 2 ) ( a + b + c ) ≤
§Æc biÖt, víi r = 1 ta cã mét B§T cã kh¸
nhiÒu øng dông: ≤ 6 ( a 3 + b 3 + c 3 ) + ( a + b + c)3 (V× a + b +
“Víi x, y, z lµ c¸c sè kh«ng ©m ta lu«n
c = 1). Khai triÓn B§T trªn ta cã:
cã:
2(a 3 + b3 + c3 + 3abc) ≥
x ( x − y )( x − z ) + y ( y − z )( y − x ) +
≥ 2(a 2b + ab 2 + b 2 c + bc 2 + c 2 a + ca 2 )
+ z ( z − x )( z − y ) ≥ 0 ”.
⇔ 2(a 3 + b3 + c3 + 3abc) ≥
Cô thÓ lµ khi khai triÓn B§T trªn ta viÕt ≥ ab(a + b) + bc(b + c) + ca (c + a )
®−îc d−íi ba d¹ng sau:
§©y lµ B§T a) trong tr−êng hîp r = 1
a) x3 + y 3 + z 3 + 3 xyz ≥
cña B§T Schur, do vËy ta cã ®pcm.
≥ xy ( x + y ) + yz ( y + z ) + zx ( z + x )
b) xyz ≥ ( x + y − z )( y + z − x )( z + x − y ) VÝ dô 3. (IMO – 1984).
Cho x, y, z lµ c¸c sè thùc kh«ng ©m tháa
c) 4( x + y + z )( xy + yz + zx) ≤
mLn x + y + z = 1. Chøng minh r»ng
≤ ( x + y + z ) + 9 xyz
3
7
0 ≤ xy + yz + zx − 2 xyz ≤ (*)
XÐt mét sè vÝ dô cô thÓ sau: 27
Lêi gi¶i. Chóng ta chuyÓn B§T nµy vÒ d¹ng
2. VÝ dô 1. (IMO – 2000). ®ång bËc. Dùa vµo gi¶ thiÕt x + y + z = 1 ta
cã: (*) ⇔

TAÄP SAN TOAÙN HOÏC - 2007 81


TRÖÔØNG THPT CHUYEÂN HOAØNG VAÊN THUÏ - HOØA BÌNH
PhÇn I - S¸ng t¹o to¸n häc.

0 ≤ ( x + y + z )( xy + yz + zx) − 2 xyz ≤ ≥ 4(ab + bc + ca) − ( a + b + c ) =


2

7 = 2(ab + bc + ca) − (a 2 + b 2 + c 2 )
≤ ( x + y + z )3
27 VËy (2) ®−îc chøng minh hay (*) ®−îc
⇔ 0 ≤ x 2 y + xy 2 + y 2 z + yz 2 + z 2 x + zx 2 + xyz ≤ chøng minh. Sau ®©y lµ mét sè bµi tËp nhá
7 cho b¹n ®äc:
≤ ( x + y + z )3
27
Bµi tËp 1. (Chän §T Mü n¨m 2003).
Râ rµng B§T bªn tr¸i lu«n ®óng do x, y, z
Cho a, b, c lµ c¸c sè thùc thuéc kho¶ng
kh«ng ©m.
Ta cã B§T ë vÕ ph¶i t−¬ng ®−¬ng víi  π
 0,  . Chøng minh r»ng
( x + y + z )( xy + yz + zx) − 2 xyz ≤  2
7 sin asin(a −b)sin(a − c) sin bsin(b − c)sin(b − a)
≤ ( x + y + z ) (1) + +
3

27 sin(b + c) sin(c + a)
MÆt kh¸c theo B§T c) ë trªn ta cã sin c sin(c − a)sin(c − b)
( x + y + z )( xy + yz + zx) ≤ + ≥0
sin(a + b)
1
≤ ( x + y + z ) + 9 xyz  (2)
3

4 
Bµi tËp 2. (Chän §T Mü n¨m 2002).
Theo B§T AM – GM ta cã:
Cho c¸c sè thùc a, b, c d−¬ng. Chøng
1
xyz ≤ ( x + y + z ) (3)
3
minh r»ng
27 a+b+c 3
Tõ (2) vµ (3) suy ra B§T(1) ®−îc chøng − abc ≤
minh. 3

VÝ dô 4. (APMO - 2004) .
≤ max {( a− b , ) (
2
b− c , ) (
2
c− a )}
2

Cho a, b, c lµ c¸c sè thùc d−¬ng. Chøng


minh r»ng:
( a 2 + 2 )( b2 + 2 )( c2 + 2 ) ≥ 9 ( ab + bc + ca ) (*)
“Mét cuéc sèng c©n b»ng lµ g× −? H`y häc
Lêi gi¶i. Ta cã
mét thø g× ®ã vµ nghÜ mét thø g× ®ã vµ vÏ
(*) ⇔ ( abc ) + 2 ( a 2b 2 + b 2 c 2 + c 2 a 2 ) +
2
vµ s¬n vµ nh¶y vµ ch¬i hµng ngµy”
+4 ( a 2 + b 2 + c 2 ) + 8 ≥ 9( ab + bc + ca )(1)
- Robert Fulghum -
Do x + y ≥ 2 xy ®óng víi mäi x, y nªn
2 2

ta cã a 2 + b 2 + c 2 ≥ ab + bc + ca .
Còng vËy, do x 2 y 2 + 1 ≥ 2 xy ®óng víi
mäi x, y nªn ta cã
2 ( a 2b 2 + b 2 c 2 + c 2 a 2 ) + 6 ≥ 4(ab + bc + ca )
Khi ®ã B§T(1) ®−îc chøng minh khi
B§T sau ®−îc chøng minh:
( abc ) + 2 ≥ 2 ( ab + bc + ca ) − ( a 2 + b2 + c2 ) (2)
2

Dïng B§T AM - GM hai lÇn vµ dïng


B§T c) ë trªn ta cã:
abc
( abc ) + 2 ≥ 3 3 ( abc ) ≥ 9 ≥
2 2

a +b+c

TAÄP SAN TOAÙN HOÏC - 2007 82


TRÖÔØNG THPT CHUYEÂN HOAØNG VAÊN THUÏ - HOØA BÌNH
PhÇn I - S¸ng t¹o to¸n häc.

moät soá baøi taäp veà Toaùn Rôøi Raïc


Bïi m¹nh qu©n
Chuyªn to¸n k04 – 07
Líp 12T, THPT Chuyªn Hoµng V¨n Thô

Rêi r¹c lµ mét d¹ng to¸n kh«ng cã mét i) S n +1 = S n nÕu 2 ng−êi di


ph−¬ng ph¸p gi¶i cô thÓ, ®Ó gi¶i ®−îc bµi chuyÓn cã mét ng−êi ë « ®en,
to¸n d¹ng nµy cÇn dùa trªn nh÷ng suy luËn mét ng−êi ë « tr¾ng.
logic vµ chÝnh x¸c. §ã lµ ®iÒu rÊt cã ý nghÜa ii) S n +1 = S n − 2 nÕu 2 ng−êi di
trong viÖc rÌn luyÖn trÝ th«ng minh, sù logic
vµ tÝnh linh ho¹t, gióp n©ng cao hiÖu qu¶ gi¶i chuyÓn cïng ë « tr¾ng.
quyÕt c«ng viÖc trong nhiÒu lÜnh vùc: Tõ häc iii) S n +1 = S n − 2 nÕu 2 ng−êi di
tËp ®Õn Khoa häc vµ ®êi sèng. chuyÓn cïng « ®en.
Sau ®©y lµ mét sè bµi to¸n mµ t«i muèn Tõ ®ã suy ra Sn+1 ≡ Sn (mod 2) ∀n ∈ N
göi ®Õn c¸c b¹n. ⇒ Sn ≡ S0 ≡ 1 (mod 2) ∀n ∈ N . Yªu cÇu
Bµi sè 1. Mét dLy cã 2007 phßng, ban ®Çu cña bµi to¸n lµ tån t¹i n ®Ó S n = 0 nªn
trong mçi phßng cã mét ng−êi. Sau mçi ngµy kh«ng thÓ thùc hiÖn ®−îc.
cã 2 ng−êi nµo ®ã chuyÓn sang phßng kÒ víi b) Gi¶ sö mçi ng−êi khi ë phßng nµo th×
phßng m×nh ®ang ë nh−ng theo 2 chiÒu ghi sè phßng ®ã vµo mét chiÕc b¶ng cña
ng−îc nhau. Hái: riªng m×nh. Gäi Tn lµ tæng c¸c sè ghi trªn
a) LiÖu cã mét ngµy nµo ®ã kh«ng cã b¶ng cña tÊt c¶ c¸c thµnh viªnsau b−íc
ng−êi nµo ë trong phßng ch½n hay chuyÓn thø n. Ta cã: Tn = T0 =1 + 2 + … +
kh«ng?
2007 = 2015028 = 1004.2007.
b) LiÖu cã mét ngµy nµo ®ã cã 1004
⇒ Kh«ng thÓ tån t¹i mét ngµy nµo ®ã cã
ng−êi ë phßng 2007 ®−îc kh«ng?
1004 ng−êi ë phßng 2007, v× nÕu nh− vËy th×
c)
c¸c phßng cßn l¹i sÏ kh«ng cã ng−êi v× tæng
Lêi gi¶i.
kh«ng ®æi Tn ®H ®ñ, vµ ®©y lµ ®iÒu v« lý.
a) T« ®en c¸c « ch½n nh− h×nh vÏ:

…………………………… Bµi sè 2. Cho a = 1, b = 2 thùc hiÖn trß


………………… ch¬i nh− sau: Tõ 2 sè a, b ®−îc phÐp viÕt a
+ b + ab.
Khi ®ã cã 1004 « tr¾ng vµ 1003 « ®en. a) Hái cã thÓ viÕt ®−îc 2001
Gäi Sn lµ tæng sè ng−êi trong c¸c phßng kh«ng?
ch½n (tøc lµ trong c¸c « ®en) sau ngµy thø n. b) Hái cã thÓ viÕt ®−îc 11111
Sau mçi ngµy cã 2 ng−êi chuyÓn sang kh«ng?
phßng kÒ víi nã. Nªn hä sÏ chuyÓn tõ « mµu
®en sang « mµu tr¾ng. Cã c¸c kh¶ n¨ng sau Lêi gi¶i. Tõ c¸c sè viÕt ®−îc cã thÓ lËp ra
xÈy ra: dHy sau:

TAÄP SAN TOAÙN HOÏC - 2007 83


TRÖÔØNG THPT CHUYEÂN HOAØNG VAÊN THUÏ - HOØA BÌNH
PhÇn I - S¸ng t¹o to¸n häc.

 u1 = 1, u2 = 2 Gi¶ sö cã ®−îc tr¹ng th¸i nh− trªn yªu


 (i, j < n, i ≠ j ) cÇu cña ®Ò bµi th× tÝch c¸c sè ë ®Ønh ®−îc t«
un = ui + u j + ui u j mµu lµ -1. §iÒu nµy m©u thuÉn.V× vËy
XÐt dHy ( vn ) : vn = un + 1 kh«ng tån t¹i tr¹ng th¸i A2 mang dÊu (-):
 v1 = 2, v2 = 3(*) c¸c ®Ønh kh¸c mang dÊu (+).
⇒
vn = un + 1 = ( ui + 1) ( u j + 1) = vi v j (**) b) Khi k = 4.
Tõ c«ng thøc (*) vµ (**) ⇒ vn chØ cã thÓ T« mµu c¸c ®Ønh lÎ A1 , A3 ,..., A9 , A11 . Mçi
ph©n tÝch ®−îc thµnh: lÇn ®æi dÊu còng cã ch½n ®Ønh ®−îc t« mµu
vn = 2α 3β (α , β ∈ N + ) bÞ ®æi dÊu. Do ®ã tÝch c¸c sè ë c¶ ®Ønh ®−îc
t« mµu lu«n b»ng (-1) kh«ng ®æi.
Tõ ®ã ⇒ vn = 2α 3β (α , β ∈ N + ) Gi¶ sö cã tr¹ng th¸i tháa mHn ®Ò bµi th×
Mµ 2001 + 1 = 2.7.143, 11111 + 1 = tÝch c¸c sè ë ®Ønh t« mµu b»ng 1. §iÒu nµy
8.3.463, v× thÕ kh«ng thÓ viÕt ®−îc c¸c sè lµ m©u thuÉn. Do vËy kh«ng tån t¹i tr¹ng
2001 vµ 11111. th¸i nh− ®H yªu cÇu.

Bµi sè 3. Chia h×nh trßn thµnh 12 phÇn b»ng Qua mét sè vÝ dô ë trªn, t«i hy väng c¸c
nhau nh− h×nh vÏ. Ban ®Çu A1 ®−îc ®¸nh b¹n cã thÓ thÊy phÇn nµo sù thó vÞ cña To¸n
rêi r¹c. Chóc c¸c b¹n häc tèt vµ cã ®−îc
dÊu (-): C¸c ®iÓm tõ A2 ®Õn A12 ®−îc ®¸nh
nhiÒu thµnh c«ng!
dÊu (+) … Mçi lÇn cho phÐp lÊy ra k ®Ønh
liªn tiÕp vµ ®æi dÊu ®ång thêi c¸c ®Ønh
®ã.Hái cã thÓ tån t¹i tr¹ng th¸i A2 dÊu (-),
c¸c ®Ønh cßn l¹i mang dÊu (+) ®−îc kh«ng?
a) Cho k = 3.
b) Cho k = 4.

Lêi gi¶i. §ång nhÊt (+) = 1 vµ (-) = -1

Lêi Ban biªn tËp. Nh÷ng bµi to¸n rêi r¹c


mµ b¹n Bïi M¹nh Qu©n giíi thiÖu lµ nh÷ng
bµi to¸n rÊt hay vµ thó vÞ ®óng nh− c¸i vèn
cã cña To¸n rêi r¹c. Mong c¸c b¹n chó ý t×m
tßi thªm nhiÒu bµi to¸n hay kh¸c. §iÒu ®ã sÏ
gióp cho viÖc häc – lµm to¸n trë nªn phong
phó vµ ý nghÜa.

a) Khi k = 3.
T« mµu A2 , A3 , A5 , A6 , A8 , A9 , A11 , A12 . Mçi
lÇn ®æi dÊu th× cã ch½n ®Ønh t« mµu ®æi
dÊu.Nªn tÝch c¸c sè ë c¸c ®Ønh ®−îc t« mµu
lu«n kh«ng ®æi vµ b»ng 1.

TAÄP SAN TOAÙN HOÏC - 2007 84


TRÖÔØNG THPT CHUYEÂN HOAØNG VAÊN THUÏ - HOØA BÌNH
PhÇn I - S¸ng t¹o to¸n häc.

SÖÛ DUÏNG HAØNG ÑIEÅM ÑIEÀU HOØA


ÑEÅ GIAÛI BAØI TOAÙN CÖÏC TRÒ
TrÇn thÞ linh ph−¬ng
Chuyªn to¸n k06 – 09.
Líp 10T, THPT Chuyªn Hoµng V¨n Thô

Lêi Ban biªn tËp. Trong qu¸ tr×nh chuÈn bÞ AB AB


ra m¾t tê TËp san, chóng t«i ®L nhËn ®−îc ⇔ 1+ = −1 −
CA DA
rÊt nhiÒu ý kiÕn ®ãng gãp vµ bµi viÕt göi vÒ.
Trong ®ã cã c¶ nh÷ng b¹n míi häc líp 10. AB AB
⇔2= +
Chóng t«i xin tr©n träng c¶m ¬n mäi sù AC AD
®ãng gãp quý b¸u ®ã cña c¸c b¹n. Hy väng 2 1 1
r»ng nh÷ng b¹n häc sinh Phæ th«ng sÏ tiÕp ⇔ = +
AB AD AC
tôc c«ng viÖc mµ c¸c thÕ hÖ ®i tr−íc cßn
ch−a hoµn thµnh … Sau ®©y lµ bµi viÕt cña
t¸c gi¶ nhá tuæi nhÊt trong TËp san nµy, víi 2. TÝnh chÊt 2.
néi dung vÒ hµng ®iÓm ®iÒu hßa. Gäi SC vµ SD lÇn l−ît lµ ph©n gi¸c trong vµ
ngoµi cña tam gi¸c ABS. Khi ®ã (A, B, C, D)
lµ mét hµng ®iÓm ®iÒu hßa (2)
I. §Þnh nghÜa. Chøng minh.
Trªn mét trôc cho 4 ®iÓm A, B, C, D. Ta
nãi A, B, C, D lËp thµnh 1 hµng ®iÓm ®iÒu
hßa nÕu (A, B, C, D) = -1.

II. Mét sè tÝnh chÊt.


1. TÝnh chÊt 1( HÖ thøc §Ò C¸c).
A, B, C, D lµ hµng ®iÓm ®iÒu hßa th×
2 1 1
= + (1)
AB AC AD
V× SC lµ ph©n gi¸c trong cña tam gi¸c
Chøng minh. CA SA CA SA
SAB nªn: = ⇒ =−
CB DB CB SB CB SB
Tõ (A,B,C,D) = -1 ta cã: =− V× SD lµ ph©n gi¸c ngoµi cña tam gi¸c
CA DA
SAB nªn:
CA + AB DA + AB
Hay : =− DA SA DA SA CA DA
CA DA = ⇒ = ⇒ =−
DB SB DB SB CB DB
⇒ ( A, B, C , D ) = −1 (®pcm).

TAÄP SAN TOAÙN HOÏC - 2007 85


TRÖÔØNG THPT CHUYEÂN HOAØNG VAÊN THUÏ - HOØA BÌNH
III. Mét sè bµi to¸n. N. T×m vÞ trÝ cña I ®Ó gi¸ trÞ cña biÓu thøc
1
Bµi to¸n 1. Cho gãc xOy vµ mét ®iÓm M cè ®¹t gi¸ trÞ lín nhÊt.
CM ⋅ BN
®Þnh trªn ®−êng ph©n gi¸c Ot cña gãc ®ã.
Mét ®−êng th»ng ∆ bÊt kú quay quanh M Lêi gi¶i.
c¾t xOy t¹i P vµ Q. T×m vÞ trÝ cña ®−êng
1
th¼ng ∆ ®Ó biÓu thøc ®¹t gi¸ trÞ
OP ⋅ OQ
lín nhÊt.

Lêi gi¶i.

VÏ ®−êng cao AH, gi¶ sö AH ∩ ∆ = P ,


AH ∩ BM = K , CK ∩ AB = R ,
CP ∩ AB = S .
V× ®−êng th¼ng ∆ song song víi BC nªn
theo ®Þnh lý Ta - lÐt ta cã:
Qua M kÎ ®−êng th¼ng vu«ng gãc víi Ot PQ IP PK IP IP
c¾t Ox vµ Oy lÇn l−ît t¹i A vµ B. LÊy I trªn = ; = =
trôc Ox sao cho OI = OQ. QH HC HK BH HC
Ta cã MA lµ ph©n gi¸c trong cña tam gi¸c PQ KP
Do ®ã: =− ⇒ ( K , P , Q , H ) = −1
IMP vµ MD ⊥ MA . Do ®ã MO lµ ph©n gi¸c QH KH
ngoµi cña tam gi¸c IMP. Khi ®ã c¸c tia CK, CP, CQ, CH lËp thµnh
Theo tÝnh chÊt 2 ta cã I, A, P, O lµ hµng mét chïm ®iÒu hßa. L¹i cã AB c¾t chïm
®iÓm ®iÒu hßa. Theo hÖ thøc §Ò C¸c ta cã:
®iÒu hßa t¹i R, S, N, B ⇒ (R, S, B, N) = -1
1 1 2
+ = = const. (Mét tÝnh chÊt quen thuéc cña chïm ®iÒu
OP OI OA hßa).
1 1 2
Mµ OI = OQ nªn + = =
OP OQ OA Nh− vËy, theo hÖ thøc §Ò - C¸c th×:
const Theo bÊt ®¼ng thøc Cauchy th× 1 1 2 1 1 2
+ = ⇒ + = = const
1 1 1 BN BR BS BN CM BS
®¹t gi¸ trÞ lín nhÊt khi = Do vËy theo bÊt ®¼ng thøc Cauchy ta cã
OP ⋅ OQ OP OQ
1
hay khi P ≡ A, Q ≡ B . Tõ ®ã ta tÝnh ®−îc gi¸ ®¹t gi¸ trÞ lín nhÊt khi BN = CM,
CM ⋅ BN
1
trÞ cô thÓ cña . hay lµ khi I ≡ P .
OP ⋅ OQ
Bµi to¸n 2. ************************************
Cho tam gi¸c ABC c©n t¹i A vµ mét
®−êng th¼ng ∆ song song víi BC. §iÓm I di
®éng trªn ∆ . BI c¾t AC t¹i M, CI c¾t AB t¹i
PhÇn iI

LÞch sö vµ øng dông To¸n häc


PhÇn II - lÞch sö vµ øng dông to¸n häc

Më ®Çu

Khi xH héi cµng ph¸t triÓn th× sù ph©n hãa gi÷a c¸c ngµnh nghÒ cµng
gi¶m mÆc dï tÝnh chuyªn m«n hãa vµ sù ph©n c«ng lao ®éng trong xH héi
ngµy cµng trë nªn s©u s¾c. Cïng víi xu h−íng ®ã, mèi liªn hÖ gi÷a nh÷ng
ngµnh khoa häc c¬ b¶n lµ To¸n häc – VËt lý häc – Hãa häc còng ngµy cµng
gÇn nhau vµ mËt thiÕt h¬n. Cã nh÷ng thêi ®iÓm chóng ta sÏ kh«ng thÓ hoÆc
kh«ng nhÊt thiÕt ph¶i ph©n biÖt gi÷a To¸n häc vµ VËt lý, To¸n häc vµ Hãa
häc … Bëi ®¬n gi¶n, nhiÖm vô cña ta lµ gi¶i quyÕt mét vÊn ®Ò thùc tÕ - cô
thÓ, cÇn ph¶i vËn dông kiÕn thøc lý luËn tæng hîp.

Nhµ to¸n häc Polya ®H nãi: “Gi¶i ®−îc bµi to¸n lµ mét thµnh c«ng, nh−ng
sÏ th«ng minh h¬n nÕu chóng ta nghÜ ra c¸ch thø hai!”. Vµ sÏ cßn tuyÖt vêi
h¬n thÕ n÷a khi nh÷ng sù th«ng minh b¾t nguån tõ to¸n häc ®ã ®−îc øng
dông vµo trong thùc tÕ, vµo khoa häc kü thuËt ®Ó phôc vô cho cuéc sèng
con ng−êi.

§H tõng häc To¸n, ®H tõng yªu To¸n vµ ®H tõng gÆp nhiÒu khã kh¨n khi
®Þnh h−íng nghµnh nghÒ khi cßn häc Phæ th«ng. Chóng t«i còng ®H tõng
®Æt ra nh÷ng c©u hái m¬ hå r»ng häc to¸n ®Ó lµm g×? Chøng minh bÊt ®¼ng
thøc ®Ó lµm g×? … Cã lÏ ®H cã kh«ng Ýt b¹n còng ®H tõng th¾c m¾c nh− thÕ.
ChÝnh v× vËy BBT ®H cã ý t−ëng vµ quyÕt ®Þnh thªm chuyªn môc “øng
dông to¸n häc”. Chuyªn môc tuy chØ mang tÝnh giíi thiÖu nh−ng chóng t«i
hy väng sÏ phÇn nµo gióp c¸c b¹n b−íc ®Çu h×nh dung ®−îc vÞ trÝ còng nh−
môc ®Ých cña to¸n häc trong Khoa häc kü thuËt còng nh− trong ®êi sèng.
Xin giíi thiÖu cïng b¹n ®äc.

TAÄP SAN TOAÙN HOÏC - 2007 88


TRÖÔØNG THPT CHUYEÂN HOAØNG VAÊN THUÏ - HOØA BÌNH
PhÇn II - lÞch sö vµ øng dông to¸n häc

söï phaùt trieån cuûa soá hoïc


Phïng ngäc th¾ng
Chuyªn to¸n K01 – 04
Sv. Líp K45B To¸n, §¹i Häc Vinh – NghÖ An

C¸c b¹n th©n mÕn, sè häc lµ mét trong §Ó biÓu thÞ nh÷ng ch÷ sè t−¬ng ®èi lín,
nh÷ng bé m«n ®¨ xuÊt hiÖn tõ rÊt l©u trong c¸ch ®ã kh«ng dïng ®−îc. V× vËy, ®H xuÊt
lÜnh vùc to¸n häc. Qua bao giai ®o¹n th¨ng hiÖn nh÷ng ký hiÖu cho sè 10 (theo hÖ ®Õm
trÇm, qua bao thÕ hÖ c¸c nhµ to¸n häc x©y thËp ph©n), vµ ë mét sè d©n téc, cho sè
dùng vµ ph¸t triÓn, sè häc ®H dÇn kh¼ng ®Þnh 5(t−¬ng øng víi hÖ ghi c¬ sè n¨m) theo sè
vÞ trÝ cña m×nh trong bÇu trêi To¸n häc. Bµi ngãn tay trªn mét bµn tay. VÒ sau, nh÷ng ký
viÕt nµy xin ®−îc giíi thiÖu víi c¸c b¹n ®«i hiÖu cho c¸c sè lín ®−îc h×nh thµnh. Nh÷ng
nÐt vÒ sù ph¸t triÓn sè häc b»ng nh÷ng g× mµ ký hiÖu nµy ë nh÷ng d©n téc kh¸c nhau cã
t«i ®H biÕt vµ s−u tÇm ®−îc. h×nh thøc kh¸c nhau vµ thay ®æi theo thêi
gian. C¶ nh÷ng hÖ ghi sè tøc lµ nh÷ng c¸ch
I. Sù ph¸t triÓn vÒ sè. kÕt hîp c¸c ch÷ sè ®Ó biÓu thÞ c¸c sè lín
còng kh¸c nhau. Tuy vËy, trong phÇn lín
Khi ®Õm c¸c vËt riªng biÖt, ®¬n vÞ lµ sè nh÷ng hÖ ghi sè, c¬ së thËp ph©n cã gi¸ trÞ
nhá nhÊt, nghÜa lµ kh«ng cÇn chia nã ra c¬ b¶n, do hÖ ®Õm thËp ph©n lµ hÖ ®Õm −u
thµnh nhiÒu thµnh phÇn vµ th−êng lµ còng tiªn.
kh«ng thÓ lµm ®−îc (VÝ dô trong khi ®Õm
c¸c viªn ®¸, nÕu thªm nöa viªn th× ta ®−îc 3 III. Nh÷ng hÖ ghi sè cña mét sè d©n téc.
viªn chø kh«ng ph¶i lµ 2,5 viªn, còng nh−
kh«ng thÓ bÇu ®−îc 2,5 ng−êi vµo ®oµn chñ 1. HÖ ghi sè cæ Hy - L¹p.
tÞch!). Tuy nhiªn, trong nh÷ng phÐp ®o c¸c Thêi cæ Hy - L¹p cã c¸ch ghi sè ®−îc phæ
®¹i l−îng mét c¸ch s¬ sµi, viÖc chia ®¬n vÞ biÕn gäi lµ c¸ch ghi sè Attich (Attich lµ mét
thµnh nhiÒu thµnh phÇn còng cÇn thiÕt, thÝ xø ë Hy L¹p, cã thñ ®« lµ Athen). Nh÷ng sè
dô nh− ®o chiÒu dµi b»ng b−íc ch©n ... V× 1, 2, 3, 4 ®−îc biÓu thÞ b»ng nh÷ng g¹ch: | ||
thÕ, ngay tõ håi cæ x−a ®H h×nh thµnh kh¸i ||| ||||. Sè 5 ®−îc ghi b¨ng dÊu . (Lèi viÕt cæ
niÖm ph©n sè. VÒ sau, ng−êi ta thÊy cÇn ph¶i cña ch÷ “pi”, ch÷ ®Çu cña tõ “pentª” nghÜa
më réng h¬n kh¸i niÖm sè. Do ®ã xuÊt hiÖn lµ n¨m). Nh÷ng sè 6, 7, 8, 9 ®−îc ký hiÖu
nh÷ng sè v« tû, sè ©m vµ sè phøc. b»ng , , , . Sè 10 ®−îc ký hiÖu
Sè “kh«ng” tham gia vµo hµng ngò c¸c b»ng ∆ (Ch÷ ®Çu cña tõ “®ªca”, nghÜa lµ
sè kh¸ muén. Tho¹t ®Çu, tõ “kh«ng” cã m−êi.). Nh÷ng sè 100, 1000 vµ 10 000 ®−îc
nghÜa lµ kh«ng cã sè. Thùc vËy, ch¼ng h¹n ký hiÖu b»ng H, X, M lµ nh÷ng ch÷ c¸i ®Çu
nÕu lÊy 3 bít ®i 3 th× coi nh− kh«ng cßn l¹i cña nh÷ng tõ chØ sè t−¬ng øng. Nh÷ng sè 50,
g× c¶. ViÖc coi c¸i “kh«ng cã g×” ®ã lµ mét 500, 5000 ®−îc ký hiÖu b»ng c¸ch phèi hîp
c¬ së khi xÐt ®Õn nh÷ng sè ©m. c¸c dÊu chØ 5 vµ 10, 5 vµ 100, 5 vµ 1000, cô
thÓ lµ: A H M . Cßn nh÷ng sè kh¸c, trong
II. Ch÷ sè. ph¹m vi chôc ngh×n ®Çu tiªn ®−îc ghi nh−
sau: HHA = 256, XXX HHHH = 7800.
Ch÷ sè lµ c¸ch viÕt dïng ®Ó chØ sè. Thêi
cæ, c¸c sè ®−îc biÓu thÞ b»ng nh÷ng nÐt Vµo thÕ kû thø III tr−íc c«ng nguyªn,
th¼ng (c¸c c¸i g¹ch): Mét g¹ch biÓu thÞ cho c¸ch ghi sè Attich ®−îc thay thÕ b»ng mét
sè mét, hai g¹ch biÓu thÞ cho sè hai v.v ... hÖ gäi lµ hÖ ghi sè I«ni (mét xø ë TiÓu ¸).
C¸ch ghi ®ã b¾t nguån tõ nh÷ng nh¸t kh¾c. Thêi cæ ®¹i, nh÷ng d©n téc Do Th¸i, ¶ - rËp
HiÖn nay, c¸ch ghi ®ã cßn ®−îc gi÷ trong vµ nhiÒu d©n téc kh¸c ë CËn §«ng còng ®Òu
nh÷ng “ch÷ sè La ML” ®Ó biÓu thÞ c¸c sè 1, dïng c¸ch ghi sè b»ng c¸c b¶ng ch÷ c¸i nh−
2, 3, ...

TAÄP SAN TOAÙN HOÏC - 2007 88


TRÖÔØNG THPT CHUYEÂN HOAØNG VAÊN THUÏ - HOØA BÌNH
PhÇn II - lÞch sö vµ øng dông to¸n häc

vËy. Kh«ng râ c¸ch ®ã xuÊt hiÖn tr−íc nhÊt MH, m−êi hai ch÷ sè ®Çu tiªn ®−îc kÝ hiÖu
ë d©n téc nµo. nh− sau:
I, II, III, IV, V, VI, VII, VIII, IX, X, XI, XII.
2. C¸ch ghi sè Xlav¬.
C¸c d©n téc Xlav¬ ph−¬ng Nam vµ Víi c¸ch kÝ hiÖu ®ã, c¸c phÐp tÝnh sè häc
ph−¬ng §«ng dïng c¸ch ghi sè b»ng c¸c ch÷ trªn nh÷ng sè ®ã gÆp rÊt nhiÒu khã kh¨n.
c¸i. Mét sè lÊy gi¸ trÞ sè theo thø tù trong Tuy vËy, c¸ch ghi sè La MH vÉn chiÕm −u
b¶ng ch÷ c¸i Xlav¬. Mét sè kh¸c (Trong sè thÕ ë n−íc Italia cho tíi thÓ kû 13, cßn ë c¸c
®ã cã d©n téc Nga) chØ lÊy nh÷ng ch÷ nµo cã n−íc T©y ©u th× ®Õn thÕ kû 16.
c¶ trong b¶ng ch÷ c¸i Hy L¹p vµ chØ nh÷ng
ch÷ ®ã lµm ch÷ sè. Trªn c¸c ch÷ biÓu thÞ sè 4. C¸ch ghi sè theo vÞ trÝ cña ng−êi Ên §é.
ng−êi ta ®Æt mét ®Çu ®Æc biÖt. Nh− vËy, gi¸ C¸c miÒn kh¸c nhau ë Ên §é cã nh÷ng
trÞ cña c¸c ch÷ t¨ng theo tr×nh tù s¾p xÕp c¸c hÖ ghi sè kh¸c nhau. Mét trong nh÷ng hÖ ®ã
ch÷ trong b¶ng ch÷ c¸i Hy L¹p (Tr×nh tù ®−îc phæ biÕn trªn toµn thÕ giíi vµ ngµy nay
trong b¶ng ch÷ c¸i Xlav¬ h¬i kh¸c mét ®ang ®−îc sö dông réng rHi. Theo hÖ Êy, c¸c
chót). ch÷ sè cã d¹ng cña nh÷ng tÝnh tõ chØ sè
T¹i n−íc Nga, c¸ch ghi sè Xlav¬ cßn t−¬ng øng trong ng«n ng÷ cæ Ên §é (Theo
®−îc gi÷ ®Õn cuèi thÕ kØ 17. D−íi triÒu Pi«t b¶ng ch÷ c¸i §ªvanagari)
®¹i ®Õ, c¸ch ghi sè A rËp ®−îc phæ biÕn h¬n, §Çu tiªn ng−êi ta biÓu thÞ nh÷ng sè 1, 2,
c¸ch ghi nµy ngµy nay ng−êi Nga vÉn dïng. 3, 4, ... ; 9, 10, 20, 30, ... 90, 100, 1000 b»ng
C¸ch ®¸nh sè Xlav¬ chØ cßn trong c¸c s¸ch nh÷ng dÊu phÈy. Dùa vµo ®ã ng−êi ta ký
thÇn häc. hiÖu nh÷ng sè kh¸c. VÒ sau, ng−êi ta ®−a
thªm dÊu ®Æc biÖt (ChÊm ®Ëm, vßng trßn) ®Ó
3. Ch÷ sè La M`. chØ nh÷ng hµng trèng. Nh÷ng dÊu chØ nh÷ng
Ng−êi cæ La MH dïng c¸ch ghi sè gäi lµ sè lín h¬n 9, kh«ng ®−îc dïng ®Õn n÷a vµ
“c¸ch ghi sè La ML”, c¸ch nµy cßn dïng c¸ch ghi sè §ªvanagari chuyÓn thµnh hÖ ghi
®Õn ngµy nay. Chóng ta hiÖn dïng c¸ch ®ã sè thËp ph©n theo vÞ trÝ. Cho ®Õn nay ng−êi
®Ó ghi l¹i mét sè nh÷ng niªn hiÖu, ®¸nh ta vÉn ch−a biÕt b−íc chuyÓn tiÕp ®ã ®−îc
trang s¸ch... D¹ng míi nhÊt cña nh÷ng ch÷ thùc hiÖn nh− thÕ nµo vµ vµo thêi nµo. Vµo
sè La mH nh− sau : I = 1, V = 5, L = 50, C = kho¶ng gi÷a thÓ kû thø VIII, hÖ ghi sè theo
100, D = 500, M = 1000.
Tr−íc kia h×nh d¹ng cña chóng cã h¬i vÞ trÝ ®−îc sö dông réng rHi ë Ên §é. Trong
kh¸c. Ch¼ng h¹n, nh− sè 1000 ®−îc biÓu thÞ kho¶ng thêi gian nµy, hÖ ®ã còng ®H th©m
b»ng dÊu (/) vµ sè 500 b»ng dÊu /). nhËp vµo c¸c n−íc kh¸c(§«ng D−¬ng, Trung
VÒ nguån gçc cña ch÷ sè La MH, kh«ng quèc, T©y T¹ng, c¸c vïng thuéc c¸c n−íc
cã ý kiÕn nµo ®¸ng tin cËy c¶. Cã thÓ lµ ch÷ Céng hßa Trung ¸ cña Liªn X«, ...) Trong
sè V tho¹t ®Çu tiªn ®−îc dïng ®Ó biÓu thÞ viÖc truyÒn b¸ c¸ch ghi sè Ên §é vµo ®Êt ¶
bµn tay, cßn ch÷ sè X th× do hai ch÷ sè n¨m rËp, s¸ch chØ dÉn do M«hamet ë xø
hîp l¹i. Còng vËy, dÊu chØ 1000 cã thÓ ®−îc Kh«rªzmi (Ngµy nay lµ miÒn Kh«rªzmi
t¹o thµnh b»ng c¸ch ghÐp hai dÊu chØ sè thuéc n−íc Céng hßa Uzbªkistan) cã mét vai
500(hoÆc ng−îc l¹i) trß quyÕt ®Þnh. S¸ch nµy ë T©y ¢u ®H ®−îc
Trong c¸ch ghi sè La MH ng−êi ta thÊy râ dÞch sang tiÕng Latin vµo thÕ kû thø XII.
vÕt tÝch cña hÖ ®Õm c¬ sè n¨m. Cßn trong §Õn thÕ kû thø XIII, c¸ch ghi sè Ên §é qua
ng«n ng÷ cña ng−êi La MH (ng«n ng÷ La tay ng−êi ¶- rËp, nªn gäi nã lµ c¸ch ghi sè
Tinh) th× kh«ng thÊy vÕt tÝch nµo cña hÖ ®Õm “¶ rËp”. C¸ch gäi tªn sai vÒ mÆt lÞch sö ®ã
c¬ sè n¨m. Nh− thÕ nghÜa lµ ng−êi La MH ®H cho ®Õn hiÖn nay vÉn tån t¹i.
m−în nh÷ng ch÷ sè nµy cña d©n téc kh¸c
(rÊt cã thÓ lµ ng−êi xø Et¬ruri) H×nh d¸ng cña nh÷ng ch÷ sè Ên §é ®H
TÊt c¶ c¸c sè nguyªn (cho tíi 5000), ®Òu tr¶i qua nhiÒu biÕn ®æi. D¹ng nh÷ng ch÷ sè
®−îc viÕt nhê c¸ch ghÐp l¹i nh÷ng ch÷ sè ngµy nay chóng ta viÕt ®−îc h×nh thµnh vµo
trªn. Theo c¸ch nµy, nÕu mét ch÷ sè lín thÕ kû thø XVI.
®øng tr−íc mét ch÷ sè bÐ th× chóng céng víi
nhau vµ ng−îc l¹i. Theo c¸ch viÕt ch÷ sè La Finish! Chóc c¸c b¹n lu«n say mª t×m
kiÕm vÎ ®Ñp to¸n häc.

TAÄP SAN TOAÙN HOÏC - 2007 89


TRÖÔØNG THPT CHUYEÂN HOAØNG VAÊN THUÏ - HOØA BÌNH
PhÇn II - lÞch sö vµ øng dông to¸n häc

Toán hc và Töï Ñoäng Hoùa


NguyÔn L©m TuyÒn
Chuyªn To¸n K99 – 02
Sv. Líp §iÒu khiÓn Tù §éng 1 - K47, §H B¸ch Khoa Hµ Néi

C«ng nghÖ th«ng tin vµ Tù ®éng hãa lµ U(t) [N]: Lùc Ðp lªn bé gi¶m chÊn.
hai lÜnh vùc c«ng nghÖ ®ang ph¸t triÓn t−¬ng Nguyªn nh©n chÝnh g©y ra ®é dÞch chuyÓn
®èi m¹nh mÏ trong nh÷ng n¨m gÇn ®©y. T«i (®é lón) y(t) cña nã.
xin ®−îc giíi thiÖu mét sè kh¸i niÖm c¬ b¶n Fc [N]: Lùc c¶n (lùc ®µn håi) cña lß xo
vÒ Kü thuËt §iÒu khiÓn tù ®éng. Qua ®ã cã ®é cøng c.
phÇn nµo sÏ gióp c¸c b¹n hiÓu ®−îc vai trß Fd [N]: Lùc c¶n cña bé gi¶m tèc cã hÖ sè
cña To¸n häc trong Tù ®éng hãa vµ øng c¶n d. T−¬ng tù nh− lß xo nh−ng “mÒm dÎo”
dông cña lÜnh vùc nµy trong thùc tÕ. h¬n. §−îc chÕ t¹o b»ng khÝ nÐn hoÆc b»ng
dÇu.
Trong hÇu hÕt c¸c ph−¬ng tiÖn giao th«ng Fm [N]: Hîp lùc t¸c dông lªn bé gi¶m
vµ mét sè thiÕt bÞ m¸y mãc kh¸c, chóng ta chÊn.
biÕt ®Õn bé gi¶m chÊn hay cßn gäi lµ bé
gi¶m xãc. Nã cã t¸c dông lµm “mÒm” hãa Chän hÖ trôc täa ®é Oy cã chiÒu h−íng
c¸c xung ®éng m¹nh kh«ng mong muèn ®Ó xuèng d−íi. Gèc täa ®é O lµ vÞ trÝ cña hÖ
®¶m b¶o an toµn, tr¸nh g©y mÖt nhäc cho thèng cÇn gi¶m chÊn (®−îc c« vÒ vËt cã khèi
con ng−êi vµ b¶o vÖ m¸y mãc, ®¶m b¶o cho l−îng m, g¾n chÆt víi bé gi¶m chÊn) ë vÞ trÝ
mét hÖ thèng/d©y chuyÒn s¶n xuÊt ho¹t ®éng c©n b»ng, nghÜa lµ khi ch−a cã lùc Ðp U(t)
mét c¸ch b×nh th−êng, tr¬n tru. t¸c ®éng lªn bé gi¶m chÊn. XÈy ra, ch¼ng
h¹n nh− khi « - t« ®i trªn ®o¹n ®−êng b»ng
Chóng ta xÐt mét bé gi¶m chÊn ®¬n gi¶n ph¼ng, kh«ng cã “æ gµ”.
nh− trong h×nh vÏ:
U(t)
Theo ®Þnh luËt II cña Newton, ta cã:
Fm d2y
Fm = 2
m dt
O
y(t) Trong ®ã hîp lùc Fm = U(t) – Fc – Fd , víi
dy (t )
Fc = c.y(t), Fd = d. . Hai ®¼ng thøc sau
c d dt
Fc Fd
cïng cã ®−îc nhê b¶n chÊt cña thiÕt bÞ, ®H
y ®−îc kh¶o s¸t trong lý thuyÕt vËt lý.

S¾p xÕp l¹i c¸c ®¼ng thøc trªn, ta thu


H×nh 1. Bé gi¶m chÊn ®¬n gi¶n. d 2 y (t ) dy (t )
®−îc: m. 2
+ d. + c. y (t ) = U (t ) (*)
dt dt
Hµm sè “quan träng” nhÊt ë ®©y lµ y(t), v×
Trong ®ã c¸c ký hiÖu ®−îc hiÓu nh− sau: y(t) chÝnh lµ ®é dÞch chuyÓn (hay ly ®é) cña

TAÄP SAN TOAÙN HOÏC - 2007 90


TRÖÔØNG THPT CHUYEÂN HOAØNG VAÊN THUÏ - HOØA BÌNH
PhÇn II - lÞch sö vµ øng dông to¸n häc

bé gi¶m chÊn. §©y lµ mét hµm theo biÕn To¸n häc nh− ®H ®Ò cËp trªn, coi nh− ®H cã
thêi gian t, víi gèc thêi gian lµ thêi ®iÓm ta s½n.
kh¶o s¸t, th«ng th−êng lµ khi b¾t ®Çu xuÊt Quay trë l¹i vÊn ®Ò, yqd (t ) lµ nghiÖm
hiÖn “chÊn ®éng” lµm n¶y sinh U(t) ≠ 0 tæng qu¸t thuÇn nhÊt cña (*) nghÜa lµ khi (*)
(U(t) ta ®ang xÐt lµ ®¹i l−îng v« h−íng). kh«ng cã vÕ ph¶i. Nã ®Æc tr−ng cho qu¸
HiÓn nhiªn ®iÒu ta mong muèn lµ dao ®éng tr×nh qu¸ ®é, ®ã lµ qu¸ tr×nh phøc t¹p nhÊt,
y(t) t¾t dÇn theo thêi gian, nghÜa lµ g©y ra dao ®éng cña bé gi¶m chÊn.
lim y (t ) = 0 . Vµ tÊt nhiªn, y (t ) → 0 cµng Qu¸ tr×nh x¸c lËp lµ mét qu¸ tr×nh æn
t →∞
nhanh cµng tèt. Lóc ®ã hÇu nh− kh«ng cã ®inh. VÊn ®Ò chØ cßn xÐt qu¸ tr×nh qu¸ ®é
¶nh h−ëng xÊu tõ “æ gµ” hay c¸c t¸c ®éng yqd (t ) . NghiÖm nµy cã d¹ng:
®ét ngét kh«ng mong muèn kh¸c. yqd (t ) = C1.e p1t + C2 .e p2t
§Ó kh¶o s¸t tiªu chÝ trªn, ta sÏ kh¶o s¸t
y(t) th«ng qua ph−¬ng tr×nh vi ph©n (PTVP) Trong ®ã p1 , p2 lµ nghiÖm cña PT ®Æc
(*). Trong To¸n häc, (*) cßn ®−îc biÓu tr−ng (**): m. p 2 + d . p + c = 0
diÔn/ký hiÖu ë hai d¹ng kh¸c: NÕu ∆ = d 2 − 4mc ≥ 0 th× PT(**) cã 2
m. y "(t ) + d . y '(t ) + c. y (t ) = U (t ) nghiÖm thùc. NÕu ∆ < 0 th× (**) cã 2 nghiÖm
ii i
hoÆc lµ: m. y (t )+ d . y (t )+ c. y (t ) = U (t ) d
phøc liªn hîp p1,2 = −α ± i β ( α = ,
2m
Ta chuyÓn (*) vÒ d¹ng ®¹i sè th«ng qua ∆
β= , i lµ ®¬n vÞ ¶o: i 2 = −1 ).
phÐp biÕn ®æi Laplace. §ã lµ d¹ng ph−¬ng 2m
tr×nh ®Æc tr−ng mµ nhiÒu b¹n ®H quen biÕt
khi gi¶i c¸c bµi to¸n sai ph©n liªn quan ®Õn HÖ thøc ¥ - Le: eiϕ = cos ϕ + i sin ϕ do
dHy sè. ChØ kh¸c lµ ë ®©y, “®èi t−îng” ta nhµ To¸n häc ¥ - Le ph¸t minh ra, vµ øng
kh¶o s¸t lµ qu¸ tr×nh liªn tôc chø kh«ng ph¶i
lµ qu¸ tr×nh rêi r¹c nh− ë dHy sè. Cô thÓ, víi ϕ = π th× ta cã eiπ = −1 chÝnh lµ ®¼ng
ph−¬ng tr×nh ®Æc tr−ng cña PTVP (*) lµ: thøc ®−îc coi lµ “®Ñp nhÊt mäi thêi ®¹i” v×
m. p 2 + d . p + c = 0 (**) trong ®ã cã mÆt c¶ 3 sè siªu viÖt e, i, π !

Nhê cã ®¼ng thøc trªn mµ ta viÕt l¹i ®−îc


Trong To¸n häc, ng−êi ta còng chØ ra
nghiÖm tæng qu¸t cña (*):
r»ng nghiÖm cña (*) sÏ cã d¹ng:
y (t ) = y0 (t ) + yqd (t ) yqd (t ) = C1.e p1t + C2 .e p2t
Trong ®ã y0 (t ) lµ nghiÖm riªng cña (*) cã = C1.e( −α +iβ )t + C2 .e( −α −iβ )t
vÕ ph¶i, ®Æc tr−ng cho qu¸ tr×nh x¸c lËp, = 2C.e−α t .cos β t
nghÜa lµ khi thêi gian kh¶o s¸t lµ ®ñ lín. (V× yqd (0) = 0 ⇒ C1 = - C2 = C)
NghiÖm riªng lµ mét nghiÖm bÊt kú tháa
Râ rµng nÕu cã α > 0 th× sÏ cã
mHn (*) vµ tháa mHn ®Çy ®ñ c¸c yªu cÇu vÒ
s¬ kiÖn. ë ®©y ®Ó ®¬n gi¶n ta coi nh− thêi lim e −α t = 0 ⇒ lim yqd (t ) = 0 . Nh− vËy α lµ
t →∞ t →∞
®iÓm kh¶o s¸t lµ mèc thêi gian t0 = 0 b¾t ®Çu hÖ sè ®Æc tr−ng cho sù t¾t dÇn cña dao ®éng,
tõ lóc gÆp chÊn ®éng ®ét ngét mµ kh«ng cßn thµnh phÇn ®iÒu hßa cos β t ®Æc tr−ng
chÞu ¶nh h−ëng cña c¸c chÊn ®éng tr−íc/sau cho kh¶ n¨ng dao ®éng cña hÖ.
®ã. Khi xÐt nhiÒu qu¸ tr×nh chång chÐo nhau
(ch¼ng h¹n khi gÆp liªn tiÕp c¸c “æ gµ” trªn D−íi ®©y lµ mét sè d¹ng ®å thÞ cña y(t)
®−êng) th× cã phøc t¹p h¬n nh−ng kÕt qu¶ trong mét sè tr−êng hîp kh¸c nhau cña c¸c
hoµn toµn nh− nhau. V× bµi b¸o chØ mang hÖ sè ®Æc tr−ng α , β :
tÝnh giíi thiÖu nªn chØ xÐt c¸c tr−êng hîp
gi¶n ®¬n. H¬n n÷a c¸c kÕt qu¶ cña lý thuyÕt

TAÄP SAN TOAÙN HOÏC - 2007 91


TRÖÔØNG THPT CHUYEÂN HOAØNG VAÊN THUÏ - HOØA BÌNH
PhÇn II - lÞch sö vµ øng dông to¸n häc

chän lß xo cã ®é cøng c vµ bé gi¶m tèc cã


hÖ sè d sao cho ∆ = d 2 − 4mc ≥ 0 th× hÖ sÏ
t¾t dÇn mµ kh«ng cã dao ®éng (Xem h×nh vÏ
4).

TÊt nhiªn bé gi¶m chÊn nãi trªn chØ lµ


mét vÝ dô ®¬n gi¶n vÒ mét hÖ thèng mµ ta
cÇn lµm “mÒm hãa” qu¸ tr×nh qu¸ ®é, lµ qu¸
tr×nh mµ ta kh«ng mong muèn. Trong thùc tÕ
nãi chung vµ lÜnh vùc Tù ®éng hãa nãi riªng,
c¸c hÖ thèng lµ hÕt søc phøc t¹p. C¸c hÖ
H×nh 2. §å thÞ ®é lón y(t) khi ∆ < 0, α > 0 thèng cÇn t¸c ®éng yÕu tè ®iÒu khiÓn ®Ó thu
®−îc ®Æc tÝnh nh− mong muèn ®−îc gäi lµ
®èi t−îng ®iÒu khiÓn. Muèn can thiÖp vµo
®èi t−îng ®iÒu khiÓn th× chóng ta cÇn cã
mét sù hiÓu biÕt nhÊt ®Þnh vÒ ®èi t−îng ®ã.
C«ng viÖc t×m hiÓu vµ biÓu diÔn th«ng tin
d−íi mét d¹ng thèng nhÊt ®−îc gäi lµ m«
h×nh hãa hÖ thèng. Trong ®ã m« h×nh hãa
to¸n häc cña hÖ thèng lµ biÓu diÔn c¸c th«ng
tin cã ®−îc d−íi d¹ng mét m« h×nh to¸n häc,
ch¼ng h¹n nh− ph−¬ng tr×nh vi ph©n (*) ë
trªn. §©y lµ mét viÖc lµm hÕt søc quan träng
cña qu¸ tr×nh tæng hîp hÖ thèng. V× sù m«
h×nh hãa cµng chÝnh x¸c th× qu¸ tr×nh t¸c
®éng cña chóng ta còng sÏ chÝnh x¸c h¬n.
H×nh 3. §å thÞ ®é lón y(t) khi ∆ < 0, α < 0
Mét trong nh÷ng nhiÖm vô cña ng−êi Kü
s− Tù ®éng hãa lµ lùa chän thiÕt bÞ/cµi ®Æt
c¸c th«ng sè (thiÕt kÕ hÖ thèng) ®Ó hÖ thèng
m¸y mãc thiÕt bÞ æn ®Þnh ( y (t ) → 0 ), ch¼ng
h¹n chØnh ®Þnh ®Ó α > 0 . Tïy tõng chØ tiªu
chÊt l−îng cô thÓ ®Æt ra, b»ng kinh nghiÖm
hoÆc b»ng kü thuËt chuyªn nghµnh, ng−êi kü
s− sÏ ®Þnh h−íng ®−îc ®Ó chän lùa c¸c th«ng
sè phï hîp./.

H×nh 4. §å thÞ ®é lón y(t) khi ∆ ≥ 0, α > 0 §¬n gi¶n hãa, ®¬n gi¶n nh− cã thÓ, nh−ng
kh«ng thÓ ®¬n gi¶n h¬n
HÖ gi¶m chÊn ta ®ang kh¶o s¸t xÈy ra ë
-Albert Einstein-
tr−êng hîp cã α > 0 . Vµ nh− ®H ®Ò cËp,
mong muèn cña ta lµ y (t ) → 0 cµng nhanh
cµng tèt. V× vËy nÕu chän c¸c thiÕt bÞ cã hÖ
sè c, d hîp lý, ta sÏ cã d¹ng ®å thÞ y(t) víi
tèc ®é t¾t dÇn nh− mong muèn. Ch¼ng h¹n,

TAÄP SAN TOAÙN HOÏC - 2007 92


TRÖÔØNG THPT CHUYEÂN HOAØNG VAÊN THUÏ - HOØA BÌNH
PhÇn II - lÞch sö vµ øng dông to¸n häc

Muèn truyÒn th«ng tin th× nhiÖm vô ®Çu


tiªn bªn phÝa ph¸t lµ mH hãa d÷ liÖu d−íi
d¹ng mét gãi tin theo mét quy t¾c mµ tÊt
Duøng Ña Thöùc Ñeå Phaùt Hieän nhiªn, bªn phÝa nhËn còng ph¶i biÕt ®−îc

Loãi Ñöôøng Truyeàn

NguyÔn L©m TuyÒn


Chuyªn To¸n K99 – 02
Sv. Líp §iÒu KhiÓn Tù §éng 1 - K47
§H B¸ch Khoa Hµ Néi

Mét c¸ch mL hãa bit ®¬n gi¶n


N gµy nay khi c«ng nghÖ th«ng tin quy t¾c ®ã. §ã lµ c«ng ®o¹n ®ãng gãi d÷
(CNTT) ®ang ph¸t triÓn m¹nh mÏ h¬n bao liÖu, nã gièng nh− viÖc bá mét l¸ th− vµo
giê hÕt th× nh÷ng kh¸i niÖm nh− m¸y vi tÝnh, trong b× th− vµ d¸n tem, ghi ®Þa chØ tr−íc khi
m¹ng internet, hÖ vi xö lý … ®H kh«ng cßn göi. Bªn nhËn sau khi thu ®−îc gãi tin d−íi
qu¸ xa l¹ víi chóng ta. CNTT ®H x©m nhËp d¹ng mét chuçi sè nhÞ ph©n th× tiÕn hµnh
vµo mäi n¬i, mäi lÜnh vùc, ®H ®Õn víi vïng t¸ch ra ®Ó thu ®−îc th«ng tin cÇn thiÕt. VÝ
s©u, vïng xa. dô, ký tù “A” ®−îc mH hãa thµnh mét sè nhÞ
ph©n 8 bit lµ 10101010, c«ng ®o¹n mH hãa
Cã lÏ ai còng ®H biÕt tíi vai trß cña To¸n gãi tin ng−êi ta thªm vµo mét sè bit cÇn thiÕt
häc ë khÝa c¹nh cung cÊp thuËt to¸n trong kü ®Ó phôc vô cho môc ®Ých riªng, ch¼ng h¹n
thuËt lËp tr×nh còng nh− c¸ch thøc tæ chøc thªm 2 bit ®Çu vµ cuèi ®Ó ph¸t hiÖn ®iÓm
d÷ liÖu, mH lÖnh thùc thi ch−¬ng tr×nh. Bµi khëi ®Çu vµ kÕt thóc cña mét gãi tin ®−îc
viÕt nµy xin ®−îc giíi thiÖu mét vÊn ®Ò mµ truyÒn/nhËn. Cô thÓ sÏ cã mét gãi tin gåm
cã lÏ cßn nhiÒu b¹n ch−a biÕt ®Õn, ®ã lµ c¸ch 10 bit sÏ ®−îc truyÒn ®i lµ 1101010100. Khi
mH hãa d÷ liÖu ®Ó ph¸t hiÖn vµ söa lçi trong nhËn, hai bit ®Çu vµ cuèi sÏ ®−îc t¸ch ra, vµ
lý thuyÕt truyÒn tin. dùa vµo chuçi bit võa nhËn, tra b¶ng mH
Cã hai d¹ng biÓu diÔn th«ng tin th«ng ng−êi ta biÕt ®−îc ký tù võa nhËn lµ ký tù
dông lµ tÝn hiÖu t−¬ng tù (Analog) vµ tÝn “A”.
hiÖu sè (Digital), trong ®ã tÝn hiÖu sè ®−îc Trong kü thuËt truyÒn dÉn th«ng tin , mÆc
sö dông phæ biÕn h¬n nhê mét sè tÝnh n¨ng dï ®H sö dông kü thuËt sè nh−ng do t¸c ®éng
®Æc tr−ng riªng cña nã, tuy nhiªn “sè” hay cña nhiÔu vµ do chÊt l−îng m«i tr−êng
“t−¬ng tù” thùc chÊt còng chØ lµ mét quy −íc truyÒn dÉn mµ th«ng tin ®−îc truyÒn t¶i
vµ chóng ®Òu truyÒn dùa trªn tÝn hiÖu chuÈn kh«ng tr¸nh khái bÞ sai lÖch. VÊn ®Ò ®Æt ra lµ
lµ dßng ®iÖn hoÆc ®iÖn ¸p. Ch¼ng h¹n, sau lµm thÕ nµo ®Ó h¹n chÕ lçi còng nh− khi lçi
®©y lµ mét c¸ch mH hãa “bit” ®¬n gi¶n trong ®H xÈy ra th× ph¶i cã biÖn ph¸p kh¾c phôc.
lý thuyÕt truyÒn tin. ë ®ã, ng−êi ta quy −íc Cã mét sè d¹ng lçi lµ: Lçi ph¸t hiÖn ®−îc
bit “0” øng víi møc ®iÖn ¸p tõ 0 – 0,5 V, nh−ng kh«ng söa ®−îc, lçi ph¸t hiÖn ®−îc
trong khi ®ã bit “1” øng víi møc ®iÖn ¸p tõ söa ®−îc vµ lçi kh«ng ph¸t hiÖn ®−îc.
4,5 – 5 V. Kho¶ng tõ 0,5 – 4,5 V lµ “d¶i BiÖn ph¸p kh¾c phôc thø nhÊt lµ sö dông
chÕt”, hay vïng kh«ng x¸c ®Þnh (don’t c¸c thiÕt bÞ phÇn cøng cao cÊp vµ c¸c biÖn
care). Khi ®iÖn ¸p r¬i vµo kho¶ng nµy th× ph¸p bäc lãt ®−êng truyÒn ®Ó gi¶m t¸c ®éng
ng−êi ta coi nh− lµ kh«ng cã tÝn hiÖu (sè). cña nhiÔu. Song ®©y chØ lµ mét biÖn ph¸p
Khi thu, ph¸t tÝn hiÖu th× c¸c bÝt tÝn hiÖu sè h¹n chÕ chø kh«ng thÓ lo¹i trõ hoµn toµn kh¶
®−îc ph¸t vµ ®−îc nhËn biÕt tu©n theo quy n¨ng bÞ lçi. MÆt kh¸c gi¸ thµnh cao còng lµ
t¾c trªn. mét yÕu tè c¶n trë ®Õn viÖc thùc hiÖn trong
thùc tÕ.

TAÄP SAN TOAÙN HOÏC - 2007 93


TRÖÔØNG THPT CHUYEÂN HOAØNG VAÊN THUÏ - HOØA BÌNH
PhÇn II - lÞch sö vµ øng dông to¸n häc

BiÖn ph¸p thø hai lµ b¶o toµn d÷ liÖu, tøc


lµ xö lý giao thøc (mH hãa hay quy −íc) ®Ó VÝ dô. Th«ng tin cÇn truyÒn I = 110101. §a
ph¸t hiÖn lçi, trong ®ã ph¸t hiÖn lçi ®ãng vai thøc quy −íc G = 1011 ( x3 + x + 1 ). Thªm 3
trß hµng ®Çu. V× khi ®H ph¸t hiÖn ®−îc lçi th× bit “0” vµo th«ng tin nguån I, ta cã P =
cã thÓ cã c¸ch kh«i phôc gi÷ liÖu, hoÆc ®¬n 110101000. Chia P cho G theo kiÓu nhÞ
gi¶n lµ bªn nhËn yªu cÇu göi l¹i d÷ liÖu. ph©n:
Chóng t«i xin ®−îc giíi thiÖu víi c¸c b¹n
mét ph−¬ng ph¸p ph¸t hiÖn lçi lµ ph−¬ng
ph¸p sö dông ®a thøc.

§©y lµ ph−¬ng ph¸p ®−îc sö dông trong


hÇu hÕt c¸c hÖ thèng truyÒn th«ng. ý t−ëng
ë ®©y lµ th«ng tin kiÓm lçi (checksum) ph¶i
®−îc tÝnh b»ng mét thuËt to¸n thÝch hîp,
trong ®ã gi¸ trÞ mçi bit cña th«ng tin nguån
®Òu ®−îc tham gia nhiÒu lÇn vµo qu¸ tr×nh
tÝnh to¸n.

§Ó tÝnh to¸n th«ng tin checksum ®ã


ng−êi ta dïng mét ®a thøc ph¸t G
(Generator polynomial) cã d¹ng nhÞ ph©n DHy bit ®−îc truyÒn ®i D = P + R =
®Æc biÖt, c¸c hÖ sè cña nã chØ cã gi¸ trÞ b»ng 110101111. Gi¶ sö d÷ liÖu nhËn ®−îc lµ D’
“0” hoÆc “1” t−¬ng øng víi c¸c ch÷ sè trong = 110101111. Chia ®a thøc D’ cho G ®−îc
mét dHy bit. sè d− lµ 0000. Do ®ã x¸c suÊt kh«ng cã lçi
lµ rÊt cao.
VÝ dô. D¹ng ®a thøc G = 1011, tøc lµ Ng−îc l¹i, nÕu d÷ liÖu ta thu ®−îc lµ D’’
x3 + x + 1 . Gi¶ sö ®a thøc G cã bËc n. DHy = 110101111 th× D’’ chia cho G d− 0001 th×
ch¾c ch¾n D” nhËn ®−îc lµ bÞ lçi.
bit mang th«ng tin nguån I (Infomation)
®−îc thªm vµo n bit “0” vµ coi nh− 1 ®a thøc Ph−¬ng ph¸p nµy cã vÎ nh− rÊt phøc t¹p,
nhÞ ph©n P. §a thøc P ®−îc chia cho ®a thøc nh−ng sù thùc lµ viÖc thùc hiÖn nã l¹i rÊt ®¬n
G dùa vµo quy t¾c ®¬n gi¶n cña phÐp trõ cã gi¶n. PhÐp chia ®a thøc nhÞ ph©n ë ®©y ®−îc
nhí nh− sau: thùc hiÖn thuÇn tóy bëi c¸c phÐp trõ kh«ng
i) 1 – 1 = 0. cã nhí (chÝnh lµ phÐp XOR bit). Bªn c¹nh
ii) 0 – 0 = 0. ®ã, ®Ó kiÓm tra, chØ cÇn phÐp so s¸nh vµ sao
iii) 1 – 0 = 1. chÐp bit th«ng th−êng. Nhê vËy mµ qu¸ tr×nh
iv) 0 – 1 = 1. tÝnh to¸n cña vi xö lý lµ rÊt nhanh.
Nh− vËy, víi quy −íc nµy, râ rµng lµ phÐp
céng (kh«ng nhí) vµ phÐp trõ (kh«ng nhí) Cßn rÊt nhiÒu vÊn ®Ò chuyªn ngµnh mµ
®Òu cho cïng mét kÕt qu¶. trong ph¹m vi h¹n hÑp cña TËp san nµy
i’) 1 + 1 = 0. kh«ng ®ñ ®Ó ®Ò cËp tíi. V× vËy, nÕu c¸c b¹n
ii’) 0 + 0 = 0. thËt sù quan t©m ®Õn th× c¸c chuyªn ngµnh
iii’) 1 + 0 = 1. C«ng nghÖ Th«ng tin, §iÖn tö – Tù ®éng hãa
iv’) 0 + 1 = 1. … trong c¸c tr−êng §¹i häc Kü thuËt ®ang
Do ®ã, nÕu ®a thøc P chia cho G cã phÇn chµo ®ãn c¸c b¹n. Xin chóc c¸c b¹n thùc
d− lµ R, nghÜa lµ P – R chia hÕt cho G, th× P hiÖn ®−îc nh÷ng −íc m¬ cña m×nh!
+ R (b»ng P - R) còng chia hÕt cho G. ThËt
ra, vÒ b¶n chÊt, quan hÖ “céng”, “trõ” hay
“chia hÕt” ë ®©y ch¼ng qua chØ lµ mét quy
−íc. Nã cho phÐp ta thu ®−îc mét kÕt qu¶
duy nhÊt sau mét phÐp tÝnh, cô thÓ ë ®©y lµ
phÐp chia, nhê ®ã ng−êi ta cã thÓ kiÓm tra
®−îc sè bÞ chia cã bÞ thay ®æi hay kh«ng dùa
vµo viÖc kiÓm tra sè d− khi mµ sè chia
kh«ng ®æi.

TAÄP SAN TOAÙN HOÏC - 2007 94


TRÖÔØNG THPT CHUYEÂN HOAØNG VAÊN THUÏ - HOØA BÌNH
PhÇn II - lÞch sö vµ øng dông to¸n häc

Caáu Truùc
Töï nhieân
NguyÔn th¸I ngäc
Chuyªn to¸n k99-02
Sv. Líp §T8 – 48, Khoa §iÖn tö ViÔn Th«ng, §H B¸ch Khoa Hµ Néi

Con ng−êi chóng ta th−êng sö dông hÖ Cã tån t¹i mét c¸ch biÓu diÔn sè kh¸c
trong m¸y tÝnh kh«ng? T×m hÖ ®Õm tèi −u ®Ó
®Õm c¬ sè 10 ®Ó biÓu diÔn c¸c con sè. Tuy
nhiªn trong m¸y tÝnh, chóng ta thÊy r»ng c¸c cã thÓ thiÕt kÕ mét bé nhí m¸y tÝnh cã dung
tr¹ng th¸i th−êng chØ tån t¹i ë 2 d¹ng ®èi lËp l−îng M sao cho sè l−îng thiÕt bÞ lµ tèi
nhau, ch¼ng h¹n tr¹ng th¸i bËt c«ng t¾c víi thiÓu.
t¾t c«ng t¾c, tr¹ng th¸i cã dßng ®iÖn víi §Ó gi¶i quyÕt bµi to¸n nµy, ta gäi r lµ c¬
kh«ng cã dßng ®iÖn, hay tr¹ng th¸i cã ®iÖn sè cña hÖ ®Õm, p lµ sè cét biÓu diÔn sè.
¸p víi kh«ng cã ®iÖn ¸p, v.v… Do ®ã, ®Ó l−u Lý luËn t−¬ng tù nh− víi tr−êng hîp r=2
tr÷ th«ng tin trong m¸y tÝnh, ng−êi ta sö lµ hÖ c¬ sè tù nhiªn, ta cã hÖ ph−¬ng tr×nh
dông hÖ ®Õm c¬ sè 2 (t−¬ng øng víi 2 tr¹ng sau:
th¸i) mµ kh«ng sö dông hÖ ®Õm c¬ sè 10.  M = rp
Quy −íc cho c¸ch biÓu diÔn hÖ ®Õm c¬ sè 2  ( *)
 N = α rp
lµ hai ký tù “0” vµ “1”. ViÖc biÓu diÔn c¸c
ký hiÖu kh¸c mµ con ng−êi mong muèn sÏ Tõ (*) ta rót N theo M sÏ ®−îc ph−¬ng
trë thµnh mét chuçi bit 0, 1. VÝ dô, ký tù tr×nh sau:
“A” biÓu diÔn d−íi d¹ng 8 bit trong b¶ng mH  r 
ASCII lµ 01000001. N = α ln M  .
Tõ ®ã con ng−êi x©y dùng nªn v« sè c¸c  ln r 
ký hiÖu kh¸c nhau. D÷ liÖu sÏ ®−îc l−u tr÷ ë ®©y M lµ cè ®inh. Chóng ta kh¶o s¸t sù
d−íi d¹ng c¸c « nhí. Ch¼ng h¹n mét « nhí biÕn thiªn cña N theo r vµ sÏ t×m ®−îc N ®¹t
cã p cét biÓu diÔn sè. Th× dung l−îng cña « gi¸ trÞ Min khi r=e.
nhí ®ã lµ 2 p t−¬ng øng víi 2 p tr¹ng th¸i Nh− vËy lµ: sè l−îng thiÕt bÞ biÓu diÔn «
kh¸c nhau mµ « nhí cã thÓ biÓu diÔn ®−îc. nhí sÏ lµ nhá nhÊt khi chóng ta sö dông hÖ
VÝ dô: p = 10 th× dung l−îng cña « nhí sÏ ®Õm e.
lµ 210 t−¬ng ®−¬ng víi 1Kb.
Tuy nhiªn, mét vÊn ®Ò rÊt khã kh¨n cña
Ng−êi ta thÊy r»ng sè l−îng thiÕt bÞ N ®Ó con ng−êi ®ã lµ t×m ra nh÷ng phÇn tö c¬ b¶n
biÓu diÔn mét « nhí nh− vËy sÏ tû lÖ thuËn cña cÊu tróc tù nhiªn cã hÖ ®Õm lµ c¬ sè e vµ
víi sè cét biÓu diÔn sè. Tøc lµ: N = α 2 p . ®©y vÉn chØ lµ vÊn ®Ò cña t−¬ng lai. NhiÒu
nhµ nghiªn cøu cho r»ng, tæ chøc bé nhí
Trong ®ã sè 2 lµ sè tr¹ng th¸i cã thÓ cã cña
trong nHo con ng−êi lµ ë d¹ng c¬ sè e. §ã lµ
mét cét biÓu diÔn sè.
cÊu tróc tù nhiªn mµ con ng−êi ®ang h−íng
®Õn!
Cã mét bµi to¸n ®Æt ra:

TAÄP SAN TOAÙN HOÏC - 2007 95


TRÖÔØNG THPT CHUYEÂN HOAØNG VAÊN THUÏ - HOØA BÌNH
PhÇn iiI - HäC TO¸N Vµ NGO¹I NG÷

PhÇn iII

Häc To¸n vµ Ngo¹i ng÷

TAÄP SAN TOAÙN HOÏC - 2007 96


TRÖÔØNG THPT CHUYEÂN HOAØNG VAÊN THUÏ - HOØA BÌNH
PhÇn iiI - HäC TO¸N Vµ NGO¹I NG÷

HOÏC TOAÙN VAØ NGOAÏI NGÖÕ


Ng« thµnh long
Líp 12a to¸n, k01- 04, khèi ptct – tin, §h khtn, ®hqg hµ néi
Sv. Khoa CÇu ®−êng, §H Mosscow.

Chaøo c¸c b¹n, nh÷ng ng−êi ®i t×m kiÕm a1 ≥ 72 (this time, the largest and second – largest in
each row, and the elements of the row coutaining a0
vÎ ®Ñp to¸n häc. T«i còng muèn gia nhËp may exceed a1).
vµo “ ®éi t×m kiÕm” ®ã b»ng mét vÊn ®Ò rÊt Hence: a0 + … + a9 ≥ 80 +72 + (a9 + 7) + (a9 +
®¸ng ph¶i quan t©m ®èi víi c¸c b¹n häc 6) + … + a9 = 8a9 + 180.
sinh giái to¸n. Qua lêi t©m sù cña nh÷ng Mean while, the row containing a9 has sum at
ng−êi b¹n t«i ®H tõng tham gia c¸c kú thi most: 100 + 99 + a9 + … + (a9 – 7) = 8a9 + 171
When is iess than the sum of the ay .
Quèc tÕ th× so víi c¸c n−íc, chóng ta kh«ng
thua vÒ kiÕn thøc mµ thua vÒ ngo¹i ng÷. Problem 4. The set M cousists of n point, in the
§iÒu nµy thÓ hiÖn râ qua c¸c cuéc giao l−u plane, no three lyning on a line . For eace triagle
gi÷a c¸c ®oµn víi nhau. V× vËy, qua ®©y t«i with wertices in M, count the nuber of points of M
chØ muèn nhÊn m¹nh tÇm quan träng cña lying in is merior Prave that the arithmetic mean
viÖc häc ngo¹i ng÷ ®èi víi c¸c b¹n häc sinh these numbers does not exceed π/4.
giái to¸n. T«i xin giíi thiÖu vÒ mét sè ®Ò Solution. It suffices to show that if p1, p2, p3, p4 are
four then randomly shosen points of the set, then the
to¸n vµ lêi gi¶i b»ng tiÕng Anh ®Ó c¸c b¹n probaboliti that p4 lies in p1p2p3 is at most 1/4. In fact,
tham kh¶o. at most one these four point lies inside the triargle
fromed by the other three, from each resulf follows
Petersburg City
Mathematical Olimpiad(Russia) Problem 5. Show that for any natural number n, bet
wen n2 and (n + 1)2 one can find three oistinct
Problem 1. In how many zeroes can the number1n + natural numbers a, b, c such that a2 + b2 divisible by
2n +3n 4n end for n ∈ N? c.
Solution. There can no zeroes (i, e, n = 4), one Zero Solution (We must as sume n > 1 )
(n =1) or two zeroes (n = 2). In fact, for n ≥ 3, 2n and Take: a = n2 + 2 , b = n2 + n +1, c = n2 + 1 then:
4n are divisible by 8, write 1n + 3n is congnient to 2 or a2 + b2 = (2n2 + 2n +5).c
4 mod 8. Thus the sum cannot end in 3 or more
zeroes. Problem 6. A country contains 1998 cities, any two
joined by a direct flight. The ticket prices on each of
Problem 2. The diagonals of parallelogram ABCD these flights are different. Is it possible that any two
meet at O. The circumcicle of triangle ABO meets AD trips visiting each city of origin have different to tal
at E and the circumcicle of DOE meets BE at F. Show prices ?
that ∠BCA=∠FCD. Solution. Yes. Choose the prices to be distinet
Solution. We use directed angles.From cycle powers of 2, then every subset of flights has a
quadrilaterals, ∠EDF = ∠EOD = ∠DAB = ∠BCD so different to tal price.
BFDC is algo cyclic. Thus∠BCF = ∠BDF = ∠OEF =
∠OAB =∠ACD whence ∠BCA = ∠FCD.

Problem 3. In a 10x10 table are writer the numbers Cuèi cïng, ®Ó kÕt thóc bµi viÕt t«i xin
from 1 to 100. From each row we select the third chóc tê b¸o cña c¸c b¹n thµnh c«ng ®Ó tiÕp
largest number . Show that the sum of these numberes tôc ph¸t huy truyÒn thèng vèn cã cña khèi
is not less than the sum of the numbers in some row . chuyªn To¸n THPT chuyªn Hoµng V¨n Thô!
Solution. Let a0 > … > a9 be the numbers Selected.
Then at most 20 number exceed a0 (the largest and
second – largest in each row) show a0 ≥ 80. Semilarly

TAÄP SAN TOAÙN HOÏC - 2007 97


TRÖÔØNG THPT CHUYEÂN HOAØNG VAÊN THUÏ - HOØA BÌNH
PhÇn iiI - HäC TO¸N Vµ NGO¹I NG÷

BBT. Ngày nay tiếng Anh là công cụ quan at a point called the radical center of the three
trọng cho tất cả mọi người.Với học sinh circles. (This is because the intersection point of any
two of these radical axes has equal power with
chuyên Toán ñiều ấy cũng không phải ngoại respect to all three circles, hence it is on the third
lệ. Chính vì thế BBT quyết ñịnh giới thiệu radical axis too.) If two circles C1 and C2 intersect,
một số bài báo Tiếng Anh về Toán sơ cấp their radical axis is the line through the intersection
của các tác giả nước ngoài cho bạn ñọc. Hai point(s) perpendicular to the line of the centers. (This
bài báo dưới ñây trích từ tờ Mathematical is because the intersection point(s) have 0 power with
respect to both circles, hence they are on the radical
Excalibur của ñại học Hong Kong, của bạn axis.) If the two circles do not intersect, their radical
Lưu Như Hòa gửi tới Ban biên tập. axis can be found by taking a third circle C3
intersecting both C1 and C2 . Let the radical axis of
Phương tích của ñiểm với C1 , C3 intersect the radical axis of C2 , C3 at P.
ñường tròn. Then the radical axis of C1 , C2 is the line through P
perpendicular to the line of centers of C1 , C2 .
Power of Points Respect to We will illustrate the usefulness of the
intersecting chords theorem, the concepts of power of
Circles a point, radical axis and radical center in the
Kin-Yin Li following examples.
Example 1. (1996 St. Petersburg City Math
Intersecting Chords Theorem. Let two lines Olympiad)
through a point P not on a circle intersect the inside Let BD be the angle bisector of angle B in triangle
ABC with D on side AC. The circumcircle of triangle
of the circle at chords AA' and BB', then PA ⋅ PA' =
BDC meets AB at E, while the circumcircle of
PB ⋅PB'. (When P is outside the circle, thelimiting
triangle ABD meets BC at F.
case A = A' refers to PA tangent to the circle.)
Prove that AE = CF.
This theorem follows from the observation that
Solution. By the intersecting chords theorem,
triangles ABP and A'B'P are similar and the
corresponding sides are in the same ratio. In the case AE × AB = AD × AC and
P is inside the circle, the product PA ⋅ PA' can be AE AD BC
determined by taking the case the chord AA' passes
CF × CB = CD × CA , so = × .
CF CD AB
through P and the center O. This gives
AB AD
PA × PA ' = r 2 − d 2 , where r is the radius of the However, = by the angle bisector
circle and d = OP. In the case P is outside the circle, CB CD
the product PA ⋅ PA' can be determined by taking the theorem. So AE = CF.
limiting case PA is tangent to the circle. Then Example 2. (1997 USA Math Olympiad)
Let ABC be a triangle, and draw isosceles
PA × PA ' = d 2 − r 2 . The power of a point P with triangles BCD, CAE, ABF externally to ABC, with
respect to a circle is the number d − r as
2 2
BC, CA, AB as their respective bases. Prove the lines
mentioned above. (In case P is on the circle, we may through A, B, C, perpendicular to the lines EF, FD,
define the power to be 0 for convenience.) For two DE, respectively, are concurrent.
Solution. Let C1 be the circle with center D and
circles C1 and C2 with different centers O1 and O2
radius BD, C2 be the circle with center E and radius
, the points whose power with respect to C1 and CE, and C3 be the circle with center F and radius AF.
The line through A perpendicular to EF is the radical
C2 are equal form a line perpendicular to line O1O2 . axis of C2 , C3 , the line through B perpendicular to
(This can be shown by setting coordinates with line FD is the radical axis of C3 , C1 and the line through
O1 O2 as the x-axis.) This line is called the radical C perpendicular to DE is the radical axis of C1 , C2 .
axis of the two circles. In the case of the three circles These three lines concur at the radical center of the
C1 , C2 , C3 with noncollinear centers O1 , O2 , O3 the three circles.
three radical axes of the three pairs of circles intersect

TAÄP SAN TOAÙN HOÏC - 2007 98


TRÖÔØNG THPT CHUYEÂN HOAØNG VAÊN THUÏ - HOØA BÌNH
PhÇn iiI - HäC TO¸N Vµ NGO¹I NG÷

Example 3. (1985 IMO) A circle with center O


passes through vertices A and C of triangle ABC and Phép Nghịch ñảo
intersects side AB at K and side BC at N. Let the
circumcircles of triangles ABC and KBN intersect at
B and M. Prove that OM is perpendicular to BM.
Solution. For the three circles mentioned, the Inversion
radical axes of the three pairs are lines AC, KN and Kin Y. Li
BM. (The centers are noncollinear because two of
them are on the perpendicular bisector of AC, but not In algebra, the method of logarithm transforms
the third.) So the axes will concur at the radical tough problems involving multiplications and
 = BKN
center P. Since PMN = NCA , it follows divisions into simpler problems involving additions
that P, M, N, C are concyclic. By power of a point, and subtractions. For every positive number x, there
is a unique real number log x in base 10. This is a
BM × BP = BN × BC = BO 2 − r 2 and one-to-one correspondence between the positive
PM × PB = PN × PK = PO 2 − r 2 , where r is numbers and the real numbers.
the radius of the circle through A, C, N, K. Then In geometry, there are also transformation
PO 2 − BO 2 = BP ( PM − BM ) = PM 2 − BM 2 methods for solving problems. In this article, we will
discuss one such method called inversion. To present
This implies OM is perpendicular to BM. (See this, we will introduce the extended plane, which is
remarks below.) the plane together with a point that we would like to
Remarks. By coordinate geometry, it can be think of as infinity. Also, we would like to think of
shown that the locus of points X such that all lines on the plane will go through this point at
PO 2 − BO 2 = PX 2 − BX 2 is the line through O infinity! To understand this, we will introduce the
perpendicular to line BP. This is a useful fact. stereographic projection, which can be described as
Example 4. (1997 Chinese Math Olympiad) follow.
Let quadrilateral ABCD be inscribed in a circle. Consider a sphere sitting on a point O of a plane.
Suppose lines AB and DC intersect at P and lines AD If we remove the north pole N of the sphere, we get a
and BC intersect at Q. From Q, construct the tangents punctured sphere. For every point P on the plane, the
QE and QF to the circle, where E and F are the points line NP will intersect the punctured sphere at a
of tangency. Prove that P, E, F are collinear. unique point Sp . So this gives a one-to-one
Solution. Let M be a point on PQ such that
correspondence between the plane and the punctured
=
CMP ADC . Then D,C,M,Q are concyclic and sphere. If we consider the points P on a circle in the
also, B,C,M,P are concyclic. Let r1 be the radius of plane, then the Sp points will form a circle on the
the circumcircle C1 of ABCD and O1be the center of punctured sphere.However, if we consider the points
C1 . By power of a point,
P on any line in the plane, then the Sp points will
PO12 − r12 = PC × PD = PM × PQ and
form a punctured circle on the sphere with N as the
QO12 − r12 = QC × QB = QM × PQ Then point removed from the circle. If we move a point P
PO12 − QO12 = ( PM − QM ) PQ = PM 2 − QM 2 on any line on the plane toward infinity, then Sp
which implies O1M ⊥ PQ . The circle C2 with will go toward the same point N! Thus,in this model,
all lines can be thought of as going to the same
QO1 as diameter passes through M, E,F and intersects infinity.
C1 at E, F. If r2 is the radius of C2 and O2 is the Now for the method of inversion, let O be a point
center of C2 , then on the plane and r be a positive number. The
inversion with center O and radius r is the function
PO − r = PM × PQ = PO − r .So P lies on
2 2 2 2
1 1 2 2
on the extended plane that sends a point X ≠ O to
the radical axis of C1 , C2 , which is the line EF.
the image point X ' on the ray OX such that
OX ⋅ OX ' = r 2 When X = O, X ' is taken to be
the point at infinity. When X is infinity, X ' is taken
to be O. The circle with center O and

TAÄP SAN TOAÙN HOÏC - 2007 99


TRÖÔØNG THPT CHUYEÂN HOAØNG VAÊN THUÏ - HOØA BÌNH
PhÇn iiI - HäC TO¸N Vµ NGO¹I NG÷

radius r is called the circle of inversion. Solution. Consider the inversion with center D
and any radius r. By fact (4), the circumcircle of
The method of inversion is based on the △ ABC is sent to the line through A ', B ', C ' .Since
following facts.
A ' B '+ B ' C ' = A ' C ' , we have by fact (7) that
(1) The function sending X to X ' described
above is a one-to-one correspondence between the r2 r2 r2
AB + BC = AC
extended plane with itself. (This follows from AD ⋅ BD BD ⋅ CD AD ⋅ CD
checking ( X ') ' = X ) AD ⋅ BD ⋅ CD
Multiplying by ,we get the desired
(2) If X is on the circle of inversion, then r2
X ' = X If X is outside the circle of inversion, then equation.
X ' is the midpoint of the chord formed by the Remarks. The steps can be reversed to get the
tangent points T1, T2 of the tangent lines from X to the converse statement that if
circle of inversion. (This follows AB ⋅ CD + AD ⋅ BC = AC ⋅ BD then A,B,C,D are
from OX ⋅ OX ' = ( r sec∠TOX
1 )( r cos∠TOX
1 ) = r2 ). concyclic.
Example 2. (1993 USAMO)
(3) A circle not passing through O is sent to a Let ABCD be a convex quadrilateral such that
circle not passing through O. In this case, the images diagonals AC and BD intersect at right
of concyclic points are concyclic. The point O, the angles, and let O be their intersection point. Prove
centers of the circle and the image circle are that the reflections of O across AB, BC, CD, DA are
collinear. However, the center of the circle is not sent concyclic.
to the center of the image circle! Solution. Let P, Q, R, S be the feet of
(4) A circle passing through O is sent to a line perpendiculars from O to AB, BC, CD, DA,
which is not passing through O and is parallel to the respectively. The problem is equivalent to showing P,
tangent line to the circle at O. Conversely, a line not Q, R, S are concyclic (since they are the midpoints of
passing through O is sent to a circle passing through O to its reflections). Note OSAP, OPBQ, OQCR,
O with the tangent line at O parallel to the line. ORDS are cyclic quadrilaterals. Let their
(5) A line passing through O is sent to itself.
(6) If two curves intersect at a certain angle at a circumcircles be called C A , CB , CC , CD ,
point P ≠ O , then the image curves will also respectively.
intersect at the same angle at P ' . If the angle is a Consider the inversion with center O and any
right angle, the curves are said to be orthogonal. So radius r. By fact (5), lines AC and BD are sent to
in particular, orthogonal curves at P are sent to themselves. By fact (4), circle C A is sent to a line
orthogonal curves at P’. A circle
LA parallel to BD, circle CB is sent to a line LB
orthogonal to the circle of inversion is sent to
itself. Tangent curves at P are sent to tangent curves parallel to AC, circle CC is sent to a line LC parallel
at P’.
(7) If points A, B are different from O and points to BD, circle C D is sent to a line LD parallel to AC.
O, A, B are not collinear, then the equation Next C A intersects C B at O and P.This implies
OA OB '
OA ⋅ OA ' = r 2 = OB ⋅ OB ' implies = . LA intersects LB at P ' . Similarly, LB intersects LC
OB OA '
at Q ' , LC intersects LD at R ' and LD intersects
Along with ∠AOB = ∠B ' OA ' they imply
∆OAB = ∆OB ' A ' are similar. Then LA at S ' .
A' B' OA' r 2
r 2 Since AC ⊥ BD , P ' Q ' R ' S ' is a rectangle,
= = so that A' B' = AB hence cyclic. Therefore, by fact (3), P, Q,
AB OB OAOB . OAOB
.
R, S are concyclic.
The following are some examples that illustrate
Example 3. (1996 IMO)
the powerful method of inversion. In each example,
Let P be a point inside triangle ABC such
when we do inversion, it is often that we take the
point that plays the most significant role and where that ∠APB − ∠ACB = ∠APC − ∠ABC .
many circles and lines intersect. Let D, E be the incenters of triangles APB, APC,
Example 1. (Ptolemy’s Theorem) respectively. Show that AP, BD, CE
For coplanar points A, B, C, D, if they are meet at a point.
concyclic, then AB ⋅ CD + AD ⋅ BC = AC ⋅ BD

TAÄP SAN TOAÙN HOÏC - 2007 100


TRÖÔØNG THPT CHUYEÂN HOAØNG VAÊN THUÏ - HOØA BÌNH
PhÇn iiI - HäC TO¸N Vµ NGO¹I NG÷

Solution. Let lines AP, BD intersect at X, lines Example 5. (1995 Russian Math Olympiad)
AP, CE intersect at Y. We have to Given a semicircle with diameter AB and center O
show X = Y. By the angle bisector and a line, which intersects the semicircle at C and D
BA XA CA YA and line AB at M (MB < MA, MD < MC). Let K be
theorem, = . Similarly, = . As X, Y the second point of intersection of the circumcircles
BP XP CP YP of triangles AOC and DOB. Prove that
CA BA ∠MKO = 90
are on AP, we get X = Y if and only if = .
CP BP Solution. Consider the inversion with center O
Consider the inversion with center A and any and radius r = OA. By fact (2), A, B, C, D are sent to
radius r. By fact (7), △ ABC ,△ AC ' B ' are similar, themselves. By fact (4), the circle through A, O, C is
sent to line AC and the circle through D, O, B is sent
△ APB,△ AB ' P ' are similar and
to line DB. Hence, the point K is sent to the
△ APC ,△ AC ' P ' are similar. Now intersection K ' of lines AC with DB and the point M
∠B ' C ' P ' = ∠AC ' P '− ∠AC ' B ' is sent to the intersection M ' of line AB with the
= ∠APC − ∠ABC = circumcircle of △OCD . Then the line MK is sent to
∠APB − ∠ACB = ∠AB ' P − ∠AB ' C ' = the circumcircle of OM ' K ' .
∠C ' B ' P ' To solve the problem, note by fact (7),
So ∆B ' C ' P ' is isosceles and P ' B ' = P ' C ' . ∠MKO = 90 if and only if ∠K ' M ' O = 90
From ∆APB, ∆AB ' P ' similar and Since BC ⊥ AK ' , AD ⊥ BK ' and O is the
∆APC , ∆AC ' P ' similar, we get midpoint of AB, so the circumcircle of △OCD is the
nine-point circle of △ ABK ' which intersects side AB
BA P ' A P ' A CA
= = = Therefore, X = Y. again at the foot of perpendicular from K ' to AB.
BP P ' B ' P ' C ' CP This point is M ' .So ∠K ' M ' O = 90 and we are

Example 4. (1995 Israeli Math Olympiad)
done.
Let PQ be the diameter of semicircle H. Circle O
Example 6. (1995 Iranian Math Olympiad)
is internally tangent to H and tangent to PQ at C. Let
Let M, N and P be points of intersection of the
A be a point on H and B a point on PQ such that
incircle of triangle ABC with sides AB, BC and CA
AB ⊥ PQ and is tangent to O. Prove that AC respectively. Prove that the orthocenter
bisects ∠PAB . of △ MNP , the incenter of △ ABC and the
Solution. Consider the inversion with center C circumcenter of △ ABC are collinear.
and any radius r. By fact (7), △CAP,△CP ' A ' Solution. Note the incircle of △ ABC is the
similar and △CAB,△CB ' A ' similar. So AC bisects circumcircle of △ MNP . So the first two points are
PAB if and only if ∠CAP = ∠CAB if and only if on the Euler line of △ MNP . Consider inversion with
∠CP ' A ' = ∠CB ' A ' . respect to the incircle of △ ABC with center I. By
By fact (5), line PQ is sent to itself. Since circle O fact (2), A, B, C are sent to the midpoints
passes through C, circle O is sent to a line A ', B ', C ' of PM, MN, NP, respectively. The
O ' parallel to PQ. By fact (6), since H is tangent to circumcenter of △ A ' B ' C ' is the center of the nine
circle O and is orthogonal to line PQ, H is sent to the
point circle of △ MNP ,which is on the Euler line of
semicircle H ' tangent to line O ' and has
△ MNP .By fact (3), the circumcircle of △ ABC is
diameter P ' Q ' . Since segment AB is tangent to
also on the Euler line of △ MNP .
circle O and is orthogonal to PQ, segment AB is sent
to 
A ' B ' on the semicircle tangent to line O ' and has
diameter CB ' .Now observe that  A ' Q ' and A 'C
are symmetrical with respect to the perpendicular
bisector of CQ ' so we get ∠CP ' A ' = ∠CB ' A ' .
In the solutions of the next two examples, we will
consider the nine-point circle and the Euler line of a
triangle. Please consult Vol. 3, No. 1 of Mathematical
Excalibur for discussion if necessary.

TAÄP SAN TOAÙN HOÏC - 2007 101


TRÖÔØNG THPT CHUYEÂN HOAØNG VAÊN THUÏ - HOØA BÌNH
PhÇn IV – Nh÷ng bµi to¸n hay vµ c¸c bµi to¸n tù s¸ng t¹o.

PhÇn iV

Nh÷ng bµi to¸n hay vµ


C¸c bµi to¸n tù s¸ng t¹o

TAÄP SAN TOAÙN HOÏC 2007 102


TRÖÔØNG THPT CHUYEÂN HOAØNG VAÊN THUÏ - HOØA BÌNH
PhÇn IV – Nh÷ng bµi to¸n hay vµ c¸c bµi to¸n tù s¸ng t¹o.

BBT. Trong chuyªn môc nµy, xin ®−îc i) Tr−êng hîp α = 0 .


Khi ®ã ta cã P ( x).P (2 x 2 ) = P ( x) hay
giíi thiÖu víi b¹n ®äc nh÷ng bµi to¸n ®−îc P ( x) ( P (2 x 2 ) − 1) = 0 víi mäi x ∈ R . Do ®ã,
®¸nh gi¸ lµ hay, s©u s¾c vÒ mÆt to¸n häc vµ
®éc ®¸o vÒ lêi gi¶i. Bªn c¹nh ®ã cßn cã víi mäi x∈R th× P( x) = 0 hoÆc lµ
nh÷ng bµi to¸n do chÝnh c¸c t¸c gi¶ ®Ò xuÊt P (2 x ) = 1 . NÕu P ( x) kh¸c h»ng sè th×
2

s¸ng t¹o nªn. ThËt khã ®Ó ®−a ra mét ®Þnh P ( x) = 0 hoÆc P ( x) = 1 t¹i v« h¹n gi¸ trÞ cña
nghÜa chÝnh x¸c thÕ nµo lµ mét bµi to¸n
hay/mét lêi gi¶i hay, v× ®iÒu ®ã cßn phô x. §iÒu nµy kh«ng thÓ xÈy ra, vËy P ( x) lµ
thuéc vµo quan ®iÓm vµ së thÝch cña mçi mét ®a thøc h»ng. NghÜa lµ P ( x) ≡ 0 hoÆc
ng−êi yªu to¸n. “Mét bµi to¸n hay lµ bµi P( x) ≡ 1 .
to¸n cã nhiÒu lêi gi¶i”, “Lêi gi¶i mét bµi ii) Tr−êng hîp 2: 0 < α < 2 . Víi ®a thøc
to¸n ®−îc gäi lµ hay khi sö dông Ýt nhÊt c¸c P ( x) lµ h»ng, P ( x) = a , ta cã a 2 = a nªn
d÷ kiÖn cña ®Ò bµi”, “Nh÷ng bµi to¸n cã lêi
a = 0 hoÆc a = 1 . Ta thÊy P ( x) ≡ 0 hoÆc
gi¶i s¬ cÊp nhÊt míi lµ nh÷ng bµi to¸n hay”,
“Mét bµi to¸n hay lµ mét bµi to¸n mµ sau P ( x) ≡ 1 tháa mHn bµi ra.
khi gi¶i ®−îc, chóng ta ‘µ’ lªn mét tiÕng khe XÐt khi P ( x) kh¸c h»ng sè
khÏ!”, v.v … P ( x) = a0 x n + a1 x n −1 + ... + an −1 x + an , (a0 ≠ 0) ,
§ã lµ mét vµi ®Þnh nghÜa trong v« sè c¸c
quan ®iÓm kh¸c nhau. Nh−ng nh÷ng ®Þnh n∈ N* .
nghÜa ®ã lµ kh«ng thËt quan träng, ®iÒu Víi x = 0, ta thu ®−îc P (0) = an = 0 hoÆc
®¸ng chó ý lµ nh÷ng bµi to¸n d−íi ®©y ®Òu lµ P (0) = an = 1 . NÕu an = 0 th×
nh÷ng bµi to¸n chän läc, ®H tõng xuÊt hiÖn
P ( x) = x m .Q ( x), Q (0) ≠ 0, m ∈ N *
trong c¸c kú thi Olympic, trªn c¸c t¹p chÝ
To¸n häc, nh÷ng bµi to¸n do chÝnh c¸c b¹n Thay vµo ®iÒu kiÖn bµi ra, ta thu ®−îc
chuyªn To¸n x©y dùng nªn trong qu¸ tr×nh (2x )
2 m
.Q ( x).Q ( x 2 ) = (α x 2 + 1) m Q (α x 3 + x)
mµy mß, suy nghÜ vµ cã c¶ nh÷ng sù t×nh cê
vµ Q ( x) = 0 , tr¸i víi gi¶ thiÕt. VËy nªn
... Kh«ng ai cã thÓ kh¼ng ®Þnh r»ng “T«i sÏ
nghÜ ra 1 bµi to¸n ngay b©y giê !” mµ an = 1 . §ång nhÊt hÖ sè bËc cao nhÊt hai vÕ
nh÷ng bµi to¸n míi sÏ chØ ®−îc h×nh thµnh α 
n

trªn nÒn t¶ng lý thuyÕt s©u s¾c, mét sù ®µo cña (1) ta ®−îc a0 =   . Tõ ®iÒu kiÖn bµi
s©u suy nghÜ cÇn thiÕt. Vµ ch¾c ch¾n, ®èi víi 2
c¸c b¹n yªu to¸n, ai còng ®H cã ®−îc nh÷ng ra ta thÊy nÕu P ( x0 ) = 0 th× P (α x03 + x0 ) = 0
bµi to¸n cña riªng m×nh … XÐt dHy sè xn +1 = α xn3 + xn , (n = 0, 1, 2,
C¸c bµi to¸n tù s¸ng t¹o …), x0 ≠ 0 . Ta thÊy { xn } lµ dHy ®¬n ®iÖu
t¨ng khi x0 > 0 vµ lµ dHy ®¬n ®iÖu gi¶m khi
Sau ®©y lµ nh÷ng bµi to¸n do t¸c gi¶
x0 < 0 . Tõ ®ã suy ra nÕu P ( x) cã mét
NguyÔn L©m TuyÒn ®Ò xuÊt:
nghiÖm thùc kh¸c 0 th× nã sÏ cã v« sè
Bµi T1/2007. T×m tÊt c¶ c¸c ®a thøc P ( x) nghiÖm thùc. §iÒu nµy kh«ng thÓ xÈy ra.
VËy P ( x) ≠ 0 víi mäi x ∈ R . Suy ra P ( x)
cã hÖ sè thùc tháa mHn ®iÒu kiÖn
P ( x).P (2 x 2 ) = P (α x 3 + x), ∀x ∈ R (1) chØ cã nghiÖm phøc z1 , z2 ,..., zn vµ
Trong ®ã α lµ sè thùc thuéc ®o¹n [ 0; 2] .
n
an  2 
z1.z2 ...zn = ( −1) . = − 
n

Lêi gi¶i. a0  α 

TAÄP SAN TOAÙN HOÏC 2007 103


TRÖÔØNG THPT CHUYEÂN HOAØNG VAÊN THUÏ - HOØA BÌNH
PhÇn IV – Nh÷ng bµi to¸n hay vµ c¸c bµi to¸n tù s¸ng t¹o.

n m−n
2 xnm n n  2003 
⇒ z1 . z2 ... zn =   . VËy nªn tån t¹i zk + ≥  
α  x1 + x2 + ... + xn −1 n − 1  n 
n n n

2 trong ®ã c¸c sè m, n nguyªn, xi (i = 1, 2, …,


víi zk ≥ . §iÒu nµy dÉn ®Õn
α n) d−¬ng tháa mHn ®iÒu kiÖn m ≥ n > 1 vµ
2
2 4 x1n + x2n + ... + xnn ≥ 2003 .
α z + 1 ≥ α z −1 ≥ α   −1 =
2 2
−1 > 1
k k
α  α Lêi gi¶i. Do vai trß ®èi xøng cña bÊt ®¼ng
(do 0 < α < 2 ) ⇒ α zk3 + zk > zk . Ta thu thøc (B§T) trªn, kh«ng mÊt tÝnh tæng qu¸t ta
gi¶ thiÕt x1 ≤ x2 ≤ ... ≤ xn .
®−îc P ( x) cã v« sè nghiÖm. §iÒu nµy
kh«ng thÓ xÈy ra. Nh− vËy trong tr−êng hîp §Æt S = x1n + x2n + ... + xnn . Ta cã
nµy còng chØ cã 2 ®a thøc h»ng tháa mHn bµi x1m x2m xnm
ra lµ P ( x) ≡ 0 hoÆc P ( x) ≡ 1 . ≤ ≤ ... ≤ vµ
S − x1n S − x2n S − xnn
iii) Tr−êng hîp α = 2 . Ta cã
S − x1n ≥ S − x2n ≥ ... ≥ S − xnn , do ®ã theo B§T
P ( x).P (2 x 2 ) = P (2 x3 + x), ∀x ∈ R .
Trªb−sep cho hai dHy ®¬n ®iÖu ng−îc chiÒu
§©y lµ bµi to¸n quen thuéc, vµ nh− chóng
ta ®H biÕt, kÕt qu¶ trong tr−êng hîp nµy lµ  x1m x2m xnm 
 + + ... + ×
 S − x1 S − x2 S − xnn 
n n

P ( x) = 0, ∀x ∈ R
× ( S − x1n + S − x2n + ... + S − xnn ) ≥

 P ( x) = 1, ∀x ∈ R
 ≥ n ( x1m + x2m + ... + xnm ) . Tõ B§T nµy suy ra:
 P ( x) = ( x 2 + 1) , ∀x ∈ R
n


(C¸c b¹n cã thÓ xem chøng minh chi tiÕt  x1m x2m xnm 
 + + ... + ≥
 S − x1 S − x2 S − xnn 
n n
trong bµi b¸o “øng dông cña mét bµi to¸n
tæng qu¸t cña t¸c gi¶ NguyÔn Hµ ThuËt, n x1m + x2m + ... + xnm
trang 57 – PhÇn I – S¸ng t¹o to¸n häc”). ≥ . (1)
n − 1 x1n + x2n + ... + xnn
NhËn xÐt. Víi x > 0, ¸p dông B§T C«-si ta cã:
1). Cã thÓ tæng qu¸t bµi to¸n trªn nh− sau: nx m + (m − n) ≥ mx n
Cho c¸c sè thùc k , α sao cho 0 ≤ α ≤ k . n
T×m tÊt c¶ c¸c ®a thøc P ( x) cã hÖ sè thùc Thay x = xi n , i = 1, 2,... råi céng tõng vÕ:
S
tháa mHn ®iÒu kiÖn m
 n
P ( x).P (kx 2 ) = P (α x 3 + x), ∀x ∈ R n  n  (x
m
1 + x2m + ... + xnm ) + n(m − n) ≥ mn
2). Hai bµi to¸n nãi trªn lµ mét minh häa  S
cho h−íng tæng qu¸t c¸c bµi to¸n t×m ®a thøc m
 n
tháa mHn mét ®¼ng thøc. Do ®ã x + x + ... + x ≥ n  n  (*).
m
1
m
2
m
n
3). Bµi to¸n T1/2007 trong tr−êng hîp  S
α = 1 chÝnh lµ bµi T10/348 t¹p chÝ THTT Bëi vËy
th¸ng 6/2006 do t¸c gi¶ ®Ò xuÊt. §©y lµ d¹ng m
to¸n khã v× cã liªn quan ®Õn nghiÖm sè phøc x1m + x2m + ... + xnm n  n n 
≥ .  =
vµ c¸c tÝnh chÊt cña ®a thøc. x1n + x2n + ... + xnn S  S 
m
−1 m−n m−n
Bµi T2/2007. Chøng minh r»ng:  S n S  2003 
=  = n   ≥n  (2)
x1m x2m n n  n 
+ +…
x2n + x3n + ... + xnn x1n + x3n + ... + xnn Tõ (1) vµ (2) ta nhËn ®−îc B§T cÇn chøng
minh. §¼ng thøc xÈy ra

TAÄP SAN TOAÙN HOÏC 2007 104


TRÖÔØNG THPT CHUYEÂN HOAØNG VAÊN THUÏ - HOØA BÌNH
PhÇn IV – Nh÷ng bµi to¸n hay vµ c¸c bµi to¸n tù s¸ng t¹o.

S n 2003 BÊt ®¼ng thøc (*) ë trªn kh«ng ph¶i lµ


⇔ x1 = x2 = ... = xn = n = . mét kÕt qu¶ qu¸ xa l¹. C¸c b¹n cã thÓ t×m
n n thÊy trong c¸c tµi liÖu vÒ bÊt ®¼ng thøc, hoÆc
NhËn xÐt. trong cuèn “TuyÓn tËp ®Ò thi v« ®Þch 19
Bµi to¸n trªn xuÊt sø tõ bµi to¸n tr−êng n−íc trong ®ã cã ViÖt Nam”. Tuy nhiªn theo
hîp riªng sau ®©y: chøng minh trªn lµ mét lêi gi¶i kh¸ míi mÎ
Cho c¸c sè thùc d−¬ng a, b, c tháa mHn cho bµi to¸n nµy b»ng c¸ch vËn dông bÊt
®iÒu kiÖn a 2b 2 + b 2 c 2 + c 2 a 2 ≥ a 2b 2 c 2 . Chøng ®¼ng thøc C«-si.
minh r»ng
a 2b2 b2 c 2 c2 a2 3 Bµi T3/2007. KÝ hiÖu T lµ tËp hîp tÊt c¶ c¸c
+ + ≥ .
c ( a + b ) a (b + c ) b (c + a ) 2
3 2 2 3 2 2 3 2 2 ®a thøc bËc 2007 cã ®óng 11 nghiÖm thùc kÓ
c¶ nghiÖm béi.Víi mçi P ( x ) ∈ T ®Æt
§©y lµ mét bµi to¸n t−¬ng ®èi c¬ b¶n vµ
hay. Nã cã nhiÒu lêi gi¶i kh¸c nhau, trong Q p ( x ) = ( x 20 + 1) P ( x ) − P ' ( x )
®ã cã lêi gi¶i rÊt ®Ñp sau ®©y:
vµ gäi S p lµ sè nghiÖm thùc cña ®a thøc
3 3 3
§Æt x = ,y= ,z = . Q p ( x ) . T×m Min S p .
a b c P∈T

Ta cã x 2 + y 2 + z 2 ≥ 3 vµ B§T cÇn chøng Lêi gi¶i. KÝ hiÖu x1 < x2 < ... < xk lµ c¸c
minh cã d¹ng nghiÖm thùc béi ti t−¬ng øng cña ®a thøc
x3 y3 z3 3 P ( x ) . Khi ®ã ta cã t1 + t2 + ... + tk = 11 vµ
+ + ≥ (**).
y +z
2 2
z +x
2 2
x +y
2 2
2
P ( x ) = G ( x ) ∏ ( x − xi ) i , trong ®ã ®a thøc
t

¸p dông B§T C«-si ta cã 2 x 3 + 1 ≥ 3 x 2 . 1≤i ≤ k


MÆt kh¸c, v× x 2 + y 2 + z 2 ≥ 3 , suy ra G ( x ) v« nghiÖm thùc.
2 x3 1 8 x2 Sö dông mét tÝnh chÊt quen thuéc cña ®a
+ ≥ .
y2 + z2 3 3 y2 + z2 thøc: NÕu xi lµ nghiÖm béi ti cña ®a
T−¬ng tù: thøc P ( x ) th× xi lµ nghiÖm béi ti − 1 cña ®a
2 y3 1 8 y2
+ ≥ . , thøc P ' ( x ) . Tõ ®ã suy xi còng lµ nghiÖm
z 2 + x2 3 3 z 2 + x2
2 z3 1 8 z2 béi ti − 1 cña ®a thøc
+ ≥ .
Q p ( x ) = ( x 20 + 1) P ( x ) − P ' ( x )
.
x2 + y 2 3 3 x2 + y2
Céng 3 B§T trªn theo tõng vÕ vµ ¸p dông víi i = 1, 2,..., k .
x2 y2 z2 3 x 21
B§T Nesbit 2 + 2 + 2 ≥ XÐt hµm sè f ( x ) = e
− −x
. P ( x ) . NhËn
y +z z +x x +y
2 2 2 21
2
ta thu ®−îc (**). thÊy f ( xi ) = 0 víi i = 1, 2,..., k nªn theo ®Þnh
Bµi T2/2007 lµ bµi to¸n T10/232 trªn lÝ Roll, f ( x) ph¶i cã Ýt nhÊt 1 nghiÖm øng
THTT 6/2003 do t¸c gi¶ ®Ò xuÊt. Nã b¾t
nguån tõ bµi to¸n nãi trªn. víi mçi kho¶ng ( x j ; x j +1 ), j = 1, k − 1 . Nh−
Nh− t¸c gi¶ còng ®H ®Ò cËp ®Õn trong c¸c vËy ®a thøc Q p ( x ) cã Ýt nhÊt
bµi viÕt vÒ bÊt ®¼ng thøc. Khi gi¶i c¸c bµi
to¸n vÒ bÊt ®¼ng thøc, chóng ta th−êng ®Æt  
Èn phô sao cho c¸c biÕn míi khi xÈy ra dÊu  ∑ ( ti − 1)  + ( k − 1) = 10
 1≤i ≤ k 
b»ng ®Òu lµ c¸c gi¸ trÞ ®Æc biÖt, th−êng lµ cè
t×nh lÊy sao cho gi¸ trÞ ®ã b»ng 1. Khi ®ã, nghiÖm trªn [ x1 ; xk ] .
bµi to¸n ®ì r¾c rèi vµ còng dÔ tr×nh bµy h¬n.

TAÄP SAN TOAÙN HOÏC 2007 105


TRÖÔØNG THPT CHUYEÂN HOAØNG VAÊN THUÏ - HOØA BÌNH
PhÇn IV – Nh÷ng bµi to¸n hay vµ c¸c bµi to¸n tù s¸ng t¹o.

Q ( x) Lêi gi¶i. Gi¶ sö uk lµ sè h¹ng nhá nhÊt


TiÕp theo, xÐt hµm sè h ( x ) = trªn
P ( x) trong dHy ( un ) . Khi ®ã nÕu k ≥ 1 th× tõ gi¶
P '( x) thiÕt ta cã un > u uu , tr¸i víi ®iÒu gi¶ sö.
( xk ; +∞ ) , ta cã: h ( x ) = x 20 + 1 − = k −1

P ( x) VËy u0 lµ sè h¹ng nhá nhÊt vµ duy nhÊt cña


G '( x) ti dHy ( un ) .
= x 20 + 1 − −∑ .
G ( x) 1≤i ≤ k x − xi NhËn xÐt: Sè h¹ng nhá thø k ( k = 1, 2, 3.,
Râ rµng: khi x → +∞ th× h ( x ) → +∞ vµ khi …) trong dHy ( un ) lµ u k −1 vµ duy nhÊt.
x → xk + th× h ( x ) → −∞ nªn theo ®Þnh lý ThËt vËy, víi k = 1 nhËn xÐt ®óng theo
chøng minh trªn. Gi¶ sö nhËn xÐt ®óng víi k
B«nxan«-C«si th× h( x) cã Ýt nhÊt mét
= 1, 2, …, m. V× mäi sè h¹ng cña dHy ( un )
nghiÖm thùc trong ( xk ; +∞ ) . VËy Q p ( x ) cã
®Òu lµ sè tù nhiªn vµ do tÝnh duy nhÊt cña uk
Ýt nhÊt 11 nghiÖm hay S p ≥ 11 .
nªn uk ≥ k víi mäi k = 1, 2, …, m. (*)
B©y giê ta chØ ra mét ®a thøc P ( x) mµ
Gäi u j ( j ≥ m + 1) lµ sè h¹ng nhá nhÊt thø
S P = 1 . XÐt líp c¸c ®a thøc P ( x ) d¹ng
m+1. Theo gi¶ thiÕt ta cã
Pk ( x ) = k .x11 + x 2007 ∈ T (k ∈ N * ) . u j > uuu ⇒ uuu ∈ {u0 , u1 ,...., um −1}
j −1 j −1
Ta cã:
⇒ uu j −1 ∈ {0,1, 2,..., m − 1}
QP ( x ) = ( x 20 + 1)( k .x11 + x 2007 ) −
KÕt hîp víi (*) suy ra
− (11k .x10 + 2007 x 2006 ) = x10 .R ( x ) , trong ®ã u j −1 ≤ m ≤ um ⇒ j − 1 ≤ m ⇔ j ≤ m + 1
®a thøc R ( x ) = VËy j = m + 1. NhËn xÐt ®−îc chøng
= x 2017 + x1997 − 2007 x1996 − kx 21 + kx − 11k minh. Tõ nhËn xÐt trªn suy ra dHy ( un ) t¨ng
cã nghiÖm v× R(x) bËc lÎ. ngÆt. Do ®ã tõ gi¶ thiÕt un +1 > uuu vµ tõ (*)
Ngoµi ra R ' ( x ) = 2017 x 2016 + 1997 x1996 −
n

suy ra n + 1 > uun ≥ un ≥ n. Nh−ng v× un ∈ N


−2007.1996 x1995 + 21kx 20 + k bËc ch½n nªn
víi mäi n nªn un = n .
hµm cã cùc tiÓu, dÉn ®Õn víi k ®ñ lín th×
R '( x ) > 0 ⇒ R ( x ) cã duy nhÊt 1 nghiÖm. Thö l¹i ta thÊy dHy un = n (n = 0, 1, 2, ...)
Tõ ®ã suy ra ®a thøc Q p ( x ) cã ®óng 11 tho¶ mHn mäi yªu cÇu cña bµi to¸n.
nghiÖm. Bµi T5/2007. T×m tÊt c¶ c¸c hµm sè
VËy Min S P = 11. f ( x ) x¸c ®Þnh, cã ®¹o hµm liªn tôc t¹i mäi
P∈T
NhËn xÐt. ®iÓm trªn [a, b] vµ tho¶ mHn c¸c ®iÒu kiÖn
C¸i phøc t¹p cña bµi to¸n trªn lµ chØ ra sau:
®−îc nghiÖm thø 11 vµ ®a thøc P(x) ®Ó biÓu i) Ph−¬ng tr×nh f (α ) + f ' (α ) = 0 v« nghiÖm
thøc SP ®¹t gi¸ trÞ nhá nhÊt. Cßn viÖc ¸p
dông ®Þnh lý Roll ®Ó chøng minh SP ≥ 10, lµ trªn [a, b] .
mét t− duy quen thuéc cña c¸c b¹n chuyªn ii) Ph−¬ng tr×nh f(x) = 0 cã v« sè nghiÖm
To¸n khi gi¶i c¸c bµi to¸n liªn quan ®Õn trªn [a, b] .
nghiÖm cña ®a thøc. Lêi gi¶i. Gi¶ sö hµm sè f ( x) tho¶ mHn c¸c
Bµi T4/2007. T×m tÊt c¶ c¸c dHy sè ( un ) ®iÒu kiÖn cña bµi bµi to¸n. Khi ®ã tõ ®iÒu
(n = 0, 1, 2, ….) sao cho un lµ sè tù nhiªn vµ kiÖn ii) suy ra tån t¹i dHy sè v« h¹n (Wn )
un +1 > uuu víi mäi n = 0, 1, 2, … (n = 1, 2, 3, ...) mµ f (Wn ) = 0 vµ
n

TAÄP SAN TOAÙN HOÏC 2007 106


TRÖÔØNG THPT CHUYEÂN HOAØNG VAÊN THUÏ - HOØA BÌNH
PhÇn IV – Nh÷ng bµi to¸n hay vµ c¸c bµi to¸n tù s¸ng t¹o.

a ≤ W1 ≤ W2 ≤ ... ≤ Wn ≤ ... ≤ b . ( )
f − k = ( k 3 + a2 k 2 + a4 k + a6 ) −
DÔ thÊy dHy (Wn ) t¨ng vµ bÞ chÆn nªn tån
− k ( a1k 2 + a3 k + a5 ) = − ( a1k 2 + a3 k + a5 ) −
t¹i giíi h¹n lim Wn = α ∈ [ a; b ] .
n→+∞ − k ( a1k 2 + a3 k + a5 )
Theo ®Þnh lÝ Roll: víi mçi i, tån t¹i
Ci ∈ (Wi ;Wi+1 ) sao cho f '(Ci ) = 0 , mÆt kh¸c (
= −1 − k )(a k + a k + a )
1
2
3 5

do Wi ≤ Ci ≤ Wi+1 nªn lim Cn = α . ⇒ f ( k ) f (− k ) =


n→+∞

Cuèi cïng, v× f ( x) vµ f '( x) lµ c¸c hµm = (1 − k ) ( a1k 2 + a3 k + a5 ) ≤ 0 ( do k ≥ 1 )


2

liªn tôc nªn:


⇒ Ph−¬ng tr×nh f ( x ) = 0 cã nghiÖm thùc
( n→+∞
)
f (α ) = f lim Wn = lim f (Wn ) = 0
n→+∞ ⇒ ®pcm.
( n→+∞
)
f ' (α ) = f ' lim Cn = lim f ' ( Cn ) = 0
n→+∞ Bµi T7/2007. Cho 2 dHy sè xn vµ un (n = 0,
1, 2, …) ®−îc x¸c ®Þnh theo c¸ch sau :
⇒ f (α ) + f ' (α ) = 0 , m©u thuÉn víi i). VËy

kh«ng tån t¹i hµm sè nµo tho¶ mHn ®Ò bµi. 
 u1 > 0
Bµi T6/2007. Gi¶ sö r»ng ph−¬ng tr×nh  1 2007 −i
1 + ( a1 + a2 ) x + ( a3 + a4 ) x 2 + ( a5 + a6 ) x3 = 0(1) u n +1 = u n + k + ∑ u n , ∀ n ≥ 1
 u n i =1
cã mét nghiÖm thuéc ®o¹n [0,1]. Chøng  2007

minh r»ng ph−¬ng tr×nh:  2006.u n2006


xn =
x 6 + a1 x 5 + a2 x 4 + a3 x 3 + a4 x 2 + + a5 x + a6 = 0  n +1
cã nghiÖm thùc. T×m tÊt c¶ c¸c sè thùc d−¬ng k sao cho
Lêi gi¶i. Gi¶ sö x0 ∈ [ 0,1] lµ mét nghiÖm dHy xn dÇn tíi 2007 khi n dÇn tíi v« h¹n.
tho¶ mHn ®iÒu kiÖn ®Ò bµi cña ph−¬ng tr×nh Lêi gi¶i. §Æt b = Max {− k ; −i} . Ta cã
1≤ i ≤ 2007
1 2007 2007
(1). Ta cã nhËn xÐt lµ x0 ≠ 0 vµ lµ 2006.u 2006
u 2006
2006n
x0 xn = n
= . n
(*)
nghiÖm cña ph−¬ng tr×nh: n +1 n n +1
x3 + ( a1 + a2 ) x 2 + ( a3 + a4 ) x + ( a5 + a6 ) = 0 ¸p dông bµi to¸n vÝ dô 3 – trang 44 phÇn
 uα 
⇔ k 3 + a2 k 2 + a4 k + a6 = − ( a1k 2 + a3k + a5 ) (2) 1 th× dHy sè  n  cã giíi h¹n h÷u h¹n kh¸c
n
§Æt f ( x ) = x 6 + a1 x 5 + a2 x 4 + a3 x 3 + 0 khi vµ chØ khi α = b , h¬n n÷a lóc ®ã ta
+ a4 x 2 + a5 x + a6 . Ta cÇn chøng minh uα
cßn cã lim n = 1 − b (1). Tõ (*) ta thÊy, do
ph−¬ng tr×nh f ( x ) = 0 cã nghiÖm. n →∞ n

ThËt vËy, dùa vµo (2) ta cã: 2006n


lim = 2006 nªn ®Ó lim xn = 2007 th×
( )
f k = ( k 3 + a2 k 2 + a4 k + a6 ) +
n →∞ n + 1
2007
n →∞

+ k ( a1k + a3 k + a5 ) =
2 u 2006 2007
ph¶i cã lim n = (2). VËy tõ (1) vµ
n →∞
= − ( a1k 2 + a3 k + a5 ) + k ( a1k 2 + a3 k + a5 ) =
n 2006
2007 1
(2) ta suy ra 1 − b = ⇔ b=− ⇒
(
= −1 − k )(a k 1
2
+ a3 k + a5 ) 2006 2006

TAÄP SAN TOAÙN HOÏC 2007 107


TRÖÔØNG THPT CHUYEÂN HOAØNG VAÊN THUÏ - HOØA BÌNH
PhÇn IV – Nh÷ng bµi to¸n hay vµ c¸c bµi to¸n tù s¸ng t¹o.

k=
1
. Vµ ®©y còng lµ tÊt c¶ c¸c gi¸ trÞ 6) d ≥ 3d ' (d vµ d’ lµ c¸c kho¶ng c¸ch gi÷a
2006 t©m ®−êng trßn néi tiÕp vµ ngo¹i tiÕp c¸c
cÇn t×m cña k. tam gi¸c ABC vµ A’B’C’)
a 'b 'c ' abc
Cßn rÊt nhiÒu nh÷ng bµi to¸n míi do t«i 7) 3 ≥ 2R2 + .
a '+ b '+ c ' a+b+c
t×nh cê ph¸t hiÖn ra hoÆc tõ sù tæng qu¸t hãa,
thay ®æi d÷ kiÖn ®Ò bµi mµ t«i kh«ng cã ®iÒu Bµi T9/2007. Cho tam gi¸c ABC bÊt kú.
kiÖn tr×nh bµy lêi gi¶i ®Ó c¸c b¹n tiÖn theo
Chøng minh r»ng:
dâi. T«i xin ®−îc nªu ra ®Ò bµi mét vµi trong sin A sin B sin C
sè nh÷ng bµi to¸n nh− thÕ. Mêi c¸c b¹n  3  3  3
  +  +  ≥
tham kh¶o. Vµ nÕu cã ®iÒu kiÖn, rÊt vui khi
 2   2   2 
®−îc trao ®æi víi c¸c b¹n!
3
 32
Bµi T8/2007. Cho tam gi¸c ABC nhän ≥ 3   ≥
(®iÒu kiÖn chÆt h¬n lµ c¸c gãc cña tam gi¸c  2 
ABC ®Òu nhá h¬n hoÆc b»ng 3 3 3

3 −1 ≥ ( sin A ) 2 + ( sin B ) 2 + ( sin C ) 2


4ar cos − π ≈ 940 ) néi tiÕp ®−êng trßn
2 Bµi T10/2007. Cho tam gi¸c nhän ABC.
t©m O, b¸n kÝnh R. Nèi A víi t©m ®−êng Chøng minh r»ng:
trßn néi tiÕp I c¾t ®−êng trßn t©m O lÇn n÷a 3 3

t¹i A’. T−¬ng tù ta cã B’ vµ C’. KÝ hiÖu a, b,  3 2


( sin A ) + ( sin B ) + ( sin C ) ≤ 
sin B sin C sin A

c; Ra , Rb , Rc lµ c¸c c¹nh; c¸c b¸n kÝnh ®−êng  2 
trßn bµng tiÕp øng víi c¸c ®Ønh A, B, C vµ
a’, b’, c’; Ra' , Rb' , Rc' lµ c¸c c¹nh; c¸c b¸n Bµi T11/2007. Cho hµm sè f(x) x¸c ®Þnh, cã
kÝnh ®−êng trßn bµng tiÕp øng víi c¸c ®Ønh ®¹o hµm tíi cÊp 2 vµ tho¶ mHn ®iÒu kiÖn sau
A’, B’, C’. Khi ®ã ta cã c¸c bÊt ®¼ng thøc víi mäi x :
19
sau: 2007  f ( x ) + f '' ( x )  ≥ 5 x +
1) 3(IA.IB + IB.IC + IC.IA) ≥ 2
≥ 4 ( r 2 + 4 Rr ) = 2(ab + bc + ca) – Chøng minh r»ng:
19 + 5π + 1890 x
− ( a 2 + b 2 + c 2 ) > 0. f (189 x ) + f (π + 189 x ) ≥
2007
9  A B C víi mäi sè thùc x.
2) R ≥ 3R  sin + sin + sin  ≥
2  2 2 2
Bµi T12/2007. Cho tø diÖn ABCD néi tiÕp
≥ Ra + Rb + Rc .
mét mÆt cÇu t©m O vµ gäi G lµ träng t©m cña
 A B C tø diÖn. LÊy ®iÓm M n»m bªn trong hoÆc
3) 3  sin + sin + sin  ≥
 2 2 2 trªn mÆt cña h×nh cÇu ®−êng kÝnh OG. C¸c
®−êng th¼ng MA, MB, MC, MD c¾t mÆt cÇu
 A B C
≥ ( sin A + sin B + sin C )  tg + tg + tg  t©m O lÇn n÷a t¹i c¸c ®iÓm A1, B1, C1, D1
 2 2 2 theo thø tù. Chøng minh r»ng
4)
27
R ≥ 3 ( Ra '+ Rb '+ Rc ') ≥ V(A1B1C1D1) ≥ V(ABCD)
2 trong ®ã kÝ hiÖu V chØ thÓ tÝch cña tø diÖn.
≥ 9 R + Ra + Rb + Rc ≥ 3( Ra + Rb + Rc )
XuÊt sø: §©y lµ bµi to¸n t−¬ng tù trong
5) 3r’ ≥ R + r ( ≥ 3r)
kh«ng gian cña mét bµi to¸n h×nh häc ph¼ng
®H cã trªn t¹p chÝ To¸n häc vµ tuæi trÎ. Nã

TAÄP SAN TOAÙN HOÏC 2007 108


TRÖÔØNG THPT CHUYEÂN HOAØNG VAÊN THUÏ - HOØA BÌNH
PhÇn IV – Nh÷ng bµi to¸n hay vµ c¸c bµi to¸n tù s¸ng t¹o.

®−îc h×nh thµnh sau khi t«i gi¶i ®−îc bµi Nh÷ng bµi to¸n hay
to¸n vÒ tû lÖ thÓ tÝch tø diÖn sau ®©y:
V ( A1 B1C1 D1 ) MA1.MB1MC1.MD1 Bµi T17/2007. (Bïi Lª Vò).
=
V ( ABCD ) MA.MB.MC.MD Cho dHy sè thùc (xn) ®−îc x¸c ®Þnh bëi
Tuy nhiªn, l−u ý lµ ®¼ng thøc trªn ®óng víi x0, x1 ∈ R vµ c«ng thøc:
mäi ®iÓm M trong kh«ng gian. Bµi T12/2007 1 + x n +1
xn+2 = , ∀n = 0, 1, 2 ...
còng chÝnh lµ néi dung bµi T12/333 trªn t¹p xn
chÝ To¸n häc vµ Tuæi trÎ do t«i ®Ò xuÊt. T×m x1998.
( V« ®Þch Ireland 98 - 99)
Bµi T13/2007. Cho a, b, c lµ 3 c¹nh cña
∆ABC . Chøng minh r»ng: Lêi gi¶i. DÔ dµng kiÓm tra tÝnh ®óng ®¾n
A B C cña c¸c biÓu thøc sau:
2 tg tg tg
abc ≥
4r
. 2 2 2 . 1 + x1 1 + x1 + x0 1 + x0
x2 = , x3 = , x4 =
3

3 tg A tg B tg C x0 x1 x0 x1
2 2 2 1 + x4
DÊu ®¼ng thøc x¶y ra khi nµo? x5 = = x0 ⇒ x 6 = x1 ⇒ ...
a+b+c x3
Ngoµi ra nÕu ®Æt p = , hHy chøng ⇒ x n +5 = x n , ∀n = 0;1;2...
2
minh r»ng 1 + x1 + x 0
VËy x1998 = x3 = .
abc ≥ .( ( p−a)( p−b) +( p−b)( p−c) +( p−c)( p−a) ) .
3 4 x1 x0
3
Bµi T18/2007. (Bïi ViÕt Léc).
Bµi T14/2007. Cho c¸c sè thùc a, b tháa T×m tÊt c¶ bé 4 sè tù nhiªn (x, y, z, t) tháa
a, b ∈ [ 0,1] ( x + y )( y + z )( z + x ) = txyz
mHn: 
mHn ®iÒu kiÖn  3 3 .
a + b >1  ( x, y ) = ( y , z ) = ( z , x ) = 1
Chøng minh r»ng: (Romania - 95-96)
a 6 + b 6 + 1 + 3a 2b 2 ≥ 2a 3 + 2b3 + 2a 3b3 .
Lêi gi¶i. Ta cã txyz = (x+y)(y+z)(z+x) =
Bµi T15/2007. Cho n (n > 1) sè thùc = x(y + z)(z + x) + xy(y + z) + yz(y + z)
⇒ yz(y+z) ⋮ x.
x1 , x2 ,..., xn . §Æt S k = ∑ xi1 xi2 … xik , Do x, y, z ®«i mét nguyªn tè cïng nhau
1≤i1 < i2 <...<ik ≤ n nªn (y +z) ⋮ x ⇒ (x + y + z) ⋮ x
víi mäi k = 1, 2, 3, …, n. Víi mçi sè T−¬ng tù, ta cã (x + y + z) ⋮ y vµ
nguyªn k mµ 1 < k < n, chøng minh r»ng: (x + y + z) ⋮ z. VÉn do x, y, z ®«i mét
(n − k − 1) S k +1S k −1 ≤ (n − k ) S k2 nguyªn tè cïng nhau nªn (x + y + z) ⋮ xyz
⇒ (x + y + z) ≥ xyz. C«ng viÖc cßn l¹i lµ
Bµi T16/2007. Gi¶ sö A, B, C lµ ba gãc cña ®¬n gi¶n, xin dµnh cho b¹n ®äc.
mét tam gi¸c. T×m gi¸ trÞ nhá nhÊt cña biÓu
thøc: Bµi T19/2007. (Bïi Lª Vò).
 A B  B C  C A Cho xi ∈ R, víi i = 1, 2, ..., n tháa mHn:
T =  tg + tg  tg + tg  tg + tg  . n n
 2 2  2 2  2 2
∑x
i =1
i = 1, ∑ x i = 0 .
i =1
n
xi 1 1
Chøng minh r»ng ∑
i =1 i
≤ −
2 2n
.

TAÄP SAN TOAÙN HOÏC 2007 109


TRÖÔØNG THPT CHUYEÂN HOAØNG VAÊN THUÏ - HOØA BÌNH
PhÇn IV – Nh÷ng bµi to¸n hay vµ c¸c bµi to¸n tù s¸ng t¹o.

Lêi gi¶i. §Æt A = {i / xi > 0}, B = {i / xi < 0} 1 2


§¼ng thøc x¶y ra khi y = ,z = .
2 5
Ta cã: ∑ xi + ∑ xi = 0 , ∑ xi − ∑ xi = 1 VËy MaxP = 13.
i∈ A i∈B i∈ A i∈B

1 Bµi T21/2007. (NguyÔn L©m TuyÒn).


⇒ ∑x
i∈ A
i = − ∑ xi =
i∈B 2 T×m tÊt c¶ c¸c hµm sè f(x) x¸c ®Þnh trªn
n xj tËp c¸c sè thùc R tháa mHn hÖ thøc:
xi xi 1 1
⇒ ∑ i
= ∑ +∑
i j∈B j
≤ −
2 2n
. f(y - f(x)) = f(x2002- y) - 2001yf(x), víi
i =1 i∈ A mäi sè thùc x, y.
(VietNamB - 2002)
Bµi T20/2007. (NguyÔn L©m TuyÒn).
XÐt c¸c sè thùc d−¬ng x, y, z tháa mHn hÖ Lêi gi¶i. LÇn l−ît thay y = f(x), y = x2002 vµo
®iÒu kiÖn: ph−¬ng tr×nh hµm ban ®Çu ta ®−îc:
2 f(0) = f(x2002 - f(x)) - 2001f2(x), víi mäi
 5 ≤ z ≤ min { x; y} (1) x ∈ R (1)
 f(x2002 - f(x)) = f(0) - 2001x2002f(x), víi mäi
 4 x ∈ R (2)
 xz ≥ (2)
 15 Céng (1) vµ (2) theo tõng vÕ, ta ®−îc:
 1 x f(x) = - f2(x) ≤ 0 ⇒ f ( x) ≤ 0 víi mäi
2002

 yz ≥ (3)
 5 x ∈ R vµ víi x = 0 th× f(0) = 0.
T×m gi¸ trÞ lín nhÊt cña biÓu thøc VËy tõ (1) ta cã:
1 2 3 2001f2(x) = 2001f2(x) + f(0) = f(x2002 -
P(x, y, z) = + + f(x)) ≤ 0 ⇒ f(x) = 0, ∀x ∈ R .
x y z
(VietNamB - 2001) Thö l¹i thÊy hµm sè nµy tháa mHn. VËy
f(x) ≡ 0, ∀x ∈ R .
Lêi gi¶i.
2 Bµi T22/2007. (NguyÔn L©m TuyÒn).
* Tõ (1) ta cã x ≥ z ≥ . Cho hai n - gi¸c ®Òu b»ng nhau (n ≥ 3)
5 A1A3...A2n-1 ®−îc t« mµu xanh vµ A2A4...A2n
2 1 1 3 5 ®−îc t« mµu ®á. Hai n - gi¸c ®Òu ®ã ®−îc
NÕu x > ⇒ + < + = 4
3 x z 2 2 xÕp chång lªn nhau sao cho phÇn chung lµ
2 2 mét 2n - gi¸c B1B2....B2n. Chøng minh r»ng
NÕu ≤x≤ tæng ®é dµi c¸c c¹nh ®−îc t« mµu xanh vµ
5 3
tæng ®é dµi c¸c c¹nh ®−îc t« mµu ®á cña 2n
 2  2
⇒ 15  x −  x −  ≤ 0 - gi¸c ®ã b»ng nhau.
 5  3 (Olympic To¸n APMO - 2001)
15 x 1
⇔ 15 x 2 − 16 x + 4 ≤ 0 ⇔ + ≤4
4 x Lêi gi¶i. Víi chó ý ta dùng c¸c ®iÓm Bi sao
cho mäi h×nh chiÕu cña Ai+1 ®Òu thuéc ®o¹n
1 1 15 x 1 BiBi+1. Ký hiÖu c¸c tam gi¸c BiAIBi+1 lµ Ti
VËy + ≤ + ≤4
x z 4 x ( ∀i = 1, n ), víi quy −íc T1 ≡ Tn+1. V×
2 2 A1A3...A2n-1 vµ A2A4...A2n lµ c¸c n - gi¸c ®Òu
§¼ng thøc x¶y ra khi x = ,z = .
3 5 nªn c¸c tam gi¸c Ti ®«i mét ®ång d¹ng. Gäi
1 1 9 hi lµ ®é dµi ®−êng cao h¹ tõ c¸c ®Ønh Ai
T−¬ng tù, ta cã + ≤ . xuèng c¸c c¹nh xi t−¬ng øng cña tam gi¸c Ti.
y z 2

TAÄP SAN TOAÙN HOÏC 2007 110


TRÖÔØNG THPT CHUYEÂN HOAØNG VAÊN THUÏ - HOØA BÌNH
PhÇn IV – Nh÷ng bµi to¸n hay vµ c¸c bµi to¸n tù s¸ng t¹o.

Theo tÝnh chÊt cña tam gi¸c ®ång d¹ng, ta Bµi T24/2007. (Bïi Lª Vò).
h h h T×m tÊt c¶ c¸c ®a thøc P(x) víi hÖ sè thùc
cã: k = 1 = 2 = .... = 2 n = tháa mHn: P(x).P(x-1) = P(x2).
x1 x 2 x2n
(Ai x¬ len - 1996)
h + h3 + ... + h2 n −1 h2 + h4 + ... + h2 n
= 1 =
x1 + x3 + ... + x 2 n −1 x 2 + x 4 + ... + x 2 n Lêi gi¶i. Gi¶ sö a lµ nghiÖm cña P(x), tõ gi¶
§Ó ý lµ x1+x3+...+x2n-1 vµ x2+x4+...+x2n lµ thiÕt suy ra : a2, a4, a8, ... còng lµ nghiÖm cña
tæng c¸c c¹nh t« mµu xanh, ®á t−¬ng øng P(x) ∈ N. Tån t¹i n ∈ N* sao cho an = 1
cña 2n - gi¸c B1B2....B2n. V× vËy, viÖc chøng 2π 2π
⇒ a = cos + i sin
minh bµi to¸n quy vÒ viÖc chøng minh n n
h1+h3+...+h2n-1= h2+h4+...+h2n. Gäi Si lµ T−¬ng tù, tõ P(a) = 0 ta cã : a+1, (a+1)2,
diÖn tÝch cña c¸c tam gi¸c (a+1)4, ... còng lµ nghiÖm cña P(x)
Ai-1AiAi+1, vµ S lµ diÖn tÝch 2n - gi¸c 2π 2π
A1A2...A2n, P lµ diÖn tÝch cña hai n - gi¸c ®Òu ⇒ a+1 = cos + i sin ,m ∈ N *
m m
A1A3...A2n-1 vµ A2A4...A2n. Khi ®ã S - P = 2π 2π 2π 2π
= S1+S3+...+S2n-1 ⇒ cos = cos − 1; sin = sin
= S2+S4+...+S2n. n m n m
Do vËy h1+h3+...+h2n-1 = h2+h4+...+h2n. 2π 1 2π 3
⇒ cos = − ; sin =
Bµi to¸n ®−îc chøng minh. n 2 n 2
1 3
Bµi T23/2007. (Bïi Lª Vò). ⇒a = − +i
2 2
Cho hµm f ( x ) kh¶ vi trªn [ 0;1] vµ ⇒ a lµ nghiÖm cña P1(x) = x2 + x +1.
k > 0 ®Ó f ' ( x ) ≤ k f ( x ) , ∀x ∈ [ 0;1] . DÔ thÊy P1(x) bÊt kh¶ quy nªn suy ra
P(x) = (x2 + x +1)n.
Chøng minh r»ng nÕu f ( 0 ) = 0 th× Thö l¹i thÊy tháa mHn.
f ( x ) = 0, ∀x ∈ [ 0;1] .
Bµi T25/2007. (Bïi Lª Vò).
Mçi ®iÓm trong mÆt ph¼ng ®−îc t« bëi
Lêi gi¶i.
mét mµu ®en hoÆc ®á. CMR ta cã thÓ t×m
Chia ®o¹n [ 0;1] thµnh n ®o¹n [ xi ; xi +1 ] sao ®−îc 3 ®iÓm cïng mµu mµ mçi cÆp ®iÓm cã
1
cho x1 − x0 = x2 − x1 = ... = xn − xn −1 < kho¶ng c¸ch b»ng 1 hoÆc 3 .
k (China - 1996)
Do f ( x ) liªn tôc trªn [ 0;1] nªn tån t¹i
Lêi gi¶i. DÕ thÊy cã v« sè ®iÓm ®−îc t« mµu
mét sè a n»m gi÷a 0 vµ xi sao cho : ®en, v« sè ®iÓm ®−îc t« mµu ®á(1)
f ( a ) = Max f ( x ) . NÕu f ( a ) ≠ 0 th× theo Ta gäi 2 ®iÓm A, B lµ cã tÝnh chÊt (*) nÕu
[0;1] chóng tháa mHn hai ®iÒu kiÖn sau:
®Þnh lý Lagr¨ng ta cã tån t¹i mét sè c n»m +) Hai ®iÓm ®−îc t« bëi hai mµu kh¸c nhau.
gi÷a 0 vµ a sao cho: +) AB = 2
f ( a ) − f ( 0) f ( a ) Ta chøng minh tån t¹i 2 ®iÓm cã tÝnh chÊt
f '(c) = = (*). ThËt vËy, gi¶ sö ng−îc l¹i th× ta cã : víi
a a
mäi 2 ®iÓm A, B thuéc mÆt ph¼ng mµ AB =
f (a)
⇒ f '(c) = > kf ( c ) . M©u thuÉn. 2 th× ®Òu cïng mµu.
a Tõ nhËn xÐt (1) suy ra tån t¹i 2 ®iÓm C,D
VËy ta cã ®iÒu ph¶i chøng minh. kh¸c mµu mµ CD > 2. VÏ ®−êng trßn
(C;2),(D;2) c¾t [CD] t¹i C1, D1, ... TiÕp tôc

TAÄP SAN TOAÙN HOÏC 2007 111


TRÖÔØNG THPT CHUYEÂN HOAØNG VAÊN THUÏ - HOØA BÌNH
PhÇn IV – Nh÷ng bµi to¸n hay vµ c¸c bµi to¸n tù s¸ng t¹o.

nh− vËy, ta thu ®−îc c¸c ®iÓm Ci, Di cã tÝnh c¸ phÇn tö lÎ còng ph¶i lµ sè ch½n. Ta chia
chÊt: c¸c tËp nµy thµnh tõng ®«i mét (theo mét thø
+) C¸c ®iÓm n»m trªn (Ci; 2) th× cïng tù tïy ý). Víi c¸c tËp hîp cña mçi cÆp ta ¸p
mµu víi C dông thuËt to¸n nh− ®H nªu trªn bµi to¸n, cã
+) C¸c ®iÓm n»m trªn (Di; 2) th× cïng nghÜa lµ tõ tËp hîp lín sang tËp hîp nhá h¬n.
mµu víi D. Khi ®ã, tÊt c¶ sè phÇn tö cña c¸c tËp ®Òu lµ
Tõ c¸ch dùng trªn, dÔ thÊy lu«n tån t¹i i, j ch½n. XÐt c¸c tËp mµ sè phÇn tö cña nã
sao cho (Ci; 2) vµ (D; 2) kh«ng rêi nhau. Gäi kh«ng chia hÕt cho 4. V× tæng sè c¸c phÇn tö
P lµ mét ®iÓm chung cña hai ®−êng trßn nµy, chia hÕt cho 4 nªn ta suy ra sè c¸c tËp con
ta cã : nh− vËy còng ph¶i lµ sè ch½n. Ta l¹i chia c¸c
+) P cïng mµu víi C tËp nµy thµnh tæng cÆp mét c¸ch tïy ý vµ víi
+) P cïng mµu víi D ( m©u thuÉn ) mçi mét cÆp ta l¹i ¸p dông thuËt to¸n ®H nªu
VËy nhËn ®Þnh ®−îc chøng minh. trªn. Lóc nµy, sè c¸c phÇn tö ë mçi tËp con
C«ng viÖc cßn l¹i lµ ®¬n gi¶n, xin dµnh ®Òu chia hÕt cho 4. T−¬ng tù , ta lËp l¹i c¸c
cho b¹n ®äc. b−íc nµy sao cho c¸ phÇn tö cña mçi tËp
Chó ý : ta cã thÓ më réng bµi to¸n thay v× chia hÕt cho 8, 16, …Khi mµ tæng sè c¸c
1 bëi v« sè vµ 1, 3 bëi c¸c sè a, b tháa phÇn tö cña tËp bÊt k× chia hÕt cho 2n , th× tÊt
mHn ®iÒu kiÖn nµo ®ã . c¶ 2n phÇn tö sÏ n»m trong mét tËp hîp. §ã
lµ ®iÒu ph¶i chøng minh.
Bµi T26/2007. (Lª B¶o Kh¸nh).
Cho a, b lµ 2 sè nguyªn d−¬ng ph©n biÖt Bµi T28/2007. (Bïi Lª Vò).
tháa mHn: ab(a+b) ⋮ (a2 + ab +b2) Tån t¹i hay kh«ng hµm f: R → R tháa
Chøng minh: a − b > 3 ab mHn :
i) f(1) = 1
ii) f giíi néi
Lêi gi¶i. §Æt (a,b) = d ⇒ a = du, b = dv.
 1  1
Trong ®ã (u,v) = 1 iii) f  x + 2  = f ( x) + f 2  ∀x ≠ 0
⇒ duv(u+v) ⋮ (u2+uv+v2)  x   x
Do (u,v) = 1 nªn ta cã:( u2+uv+v2; Lêi gi¶i. Gi¶ sö tån t¹i hµm sè f tháa mHn ®Ò
uv(u+v) ) = 1 ⇒ d ⋮ (u2+uv+v2) bµi. X©y dùng dHy (xn) nh− sau:
⇒ d ≥ u2+uv+v2 > uv 
⇒ ®pcm. 
 x1 = 2

Bµi T27/2007. (Bïi Lª Vò).  1
 xn +1 = xn2 + ,
TËp hîp gåm 2n phÇn tö ®−îc chia thµnh  xn
c¸c tËp con ®«i mét kh«ng giao nhau. XÐt   
mét thuËt to¸n chuyÓn mét sè phÇn tö cña  xn +1 = xn + 1 , f  1  ≥ 1
mét tËp con nµy vµo tËp con kh¸c. Ngoµi ra,  xn2
 xn 
sè phÇn tö ®−îc chuyÓn b»ng sè phÇn tö cña DÔ dµng quy n¹p ®−îc f(n) > n, ∀n ∈ N
tËp con thø hai (tËp hîp nµy ph¶i cã sè phÇn ⇒ f kh«ng giíi néi (m©u thuÉn).
tö kh«ng lín h¬n tËp ®Çu tiªn). Chøng minh VËy kh«ng tån t¹i hµm f tháa mHn ®iÒu kiÖn
r»ng sau mét sè h÷u h¹n lÇn c¸c phÐp ®Ò bµi.
chuyÓn nh− vËy, ta cã thÓ nhËn ®−îc mét sè
tËp con trïng nhau víi tËp hîp lóc ban ®Çu
(gåm 2n phÇn tö).
Mathematics is an art, as an art choses the
Lêi gi¶i. XÐt c¸c tËp con (nÕu cã thÓ cã) mµ
beauty and freedom
chóng chøa mét sè lÎ c¸c phÇn tö. V× tæng
-P.Morse-
c¸c phÇn tö lµ ch½n, nªn sè c¸c tËp cã v« sè

TAÄP SAN TOAÙN HOÏC 2007 112


TRÖÔØNG THPT CHUYEÂN HOAØNG VAÊN THUÏ - HOØA BÌNH
PhÇn IV – Nh÷ng bµi to¸n hay vµ c¸c bµi to¸n tù s¸ng t¹o.

TAÄP SAN TOAÙN HOÏC 2007 113


TRÖÔØNG THPT CHUYEÂN HOAØNG VAÊN THUÏ - HOØA BÌNH

You might also like